Solution: Case I: Assume Statement I Is True

You might also like

You are on page 1of 844

Exam Reports http://testfunda.com/LMS/Student/NewReports.

aspx

Section I

1. Saira, Mumtaz and Zeenat have a ball, a pen and a pencil, and each girl has
3 Marks
just one object in hand. Among the following statements, only one is true and
the other two are false.

I. Saira has a ball.


II. Mumtaz does not have the ball.
III. Zeenat does not have the pen.

Who has the ball?

1) Saira
2) Mumtaz
3) Zeenat
4) Indeterminate

Solution:
Case I: Assume statement I is true.

In such a case, statements II and III have to be false.

Thus, if Saira has the ball, Mumtaz cannot have the ball.

This makes statement II true as well, which is not possible.

Thus, statement I cannot be true.

Case II: Assume statement II is true.

In such a case, statements I and III have to be false.

If Mumtaz does not have the ball, then one of Saira or Zeenat has to have the
ball.

Since statement I is false, Saira does not have the ball.

Since Saira does not have the ball, Zeenat has to have the ball.

Thus, Zeenat cannot have the pen.

Thus, this makes statement III true, which is not possible.

Thus, statement II cannot be true.

1 of 28 9/16/2011 10:18 AM
Exam Reports http://testfunda.com/LMS/Student/NewReports.aspx

Thus, only statement III is true.

Thus, Zeenat who does not have the pen, has either the ball or the pencil.

Since statement II is false, Mumtaz has to have the ball.

Hence, Zeenat has the pencil and Saira has the pen.

Hence, option 2.

2. Albert, David, Jerome and Tommy were plucking mangoes in a grove to earn
3 Marks
some pocket money during the summer holidays. Their earnings were directly
related to the number of mangoes plucked and had the following relationship:

Jerome got less money than Tommy. Jerome and Tommy together got the
same amount at Albert and David taken together. Albert and Tommy together
got less than David and Jerome taken together.

Who earned the most pocket money? Who plucked the least number of
mangoes?

1) David, Jerome
2) David, Albert
3) Jerome, Tommy
4) Jerome, Albert

Solution:
Let the money with Jerome, Tommy, David and Albert be J, T, D and A
respectively.

(1) Jerome got less money than Tommy.

J<T ... (i)

(2) Jerome and Tommy together got the same amount at Albert and David
taken together.

J+T=A+D ... (ii)

Albert and Tommy together got less than David and Jerome taken together.

A+T<D+J ... (iii)

Adding (ii) and (iii), you get J + A + 2T < A + 2D + J

∴ 2T < 2D

2 of 28 9/16/2011 10:18 AM
Exam Reports http://testfunda.com/LMS/Student/NewReports.aspx

∴T<D ... (iv)

From (i) and (iv), you get J < T < D

Subtracting (iii) from (ii), you get J – A > A – J

∴ 2J > 2A

∴J>A ... (v)

The final order is A < J < T < D

Thus, Albert plucked the least number of mangoes and David earned the
most pocket money.

Hence, option 2.

Group Question
Answer the following questions based on the information given below.

Kya-Kya is an obscure island which is inhabited by two types of people: the ‘Yes’
type and the ‘No’ type. Natives of the ‘Yes’ type ask only those questions which
have the right answer as ‘Yes’, while those of the ‘No’ type ask only those
questions which have the right answer as ‘No’. For example: The ‘Yes’ type people
will ask questions like “Is 2 plus 2 equal to 4?” while the ‘No’ type will ask
questions like “Is 2 plus 2 equal to five?” The following questions are based on
your visit to the island of Kya-Kya.

3. If an islander asks, “Do I belong to the ‘No’ type?”, which of the following
3 Marks
is correct?

1) He is a ‘No’.
2) He is a ‘Yes’.
3) It is impossible for him to have asked such a question.
4) His type cannot be identified.

Solution:
Since there are only two types of people on this island, the person has
to belong to one of the two types.

Case I: Assume that the person belongs to the ‘Yes’ type.

If the above assumption is true, the answer to the question should be


‘Yes’ (as per the definition of the type) but the actual answer to the

3 of 28 9/16/2011 10:18 AM
Exam Reports http://testfunda.com/LMS/Student/NewReports.aspx

question will be ‘No’.

Thus, a person of the ‘Yes’ type can never ask this question.

Case II: Assume that the person belongs to the ‘No’ type.

If the above assumption is true, the answer to the question should be


‘No’ (as per the definition of the type) but the actual answer to the
question will be ‘Yes’.

Thus, a person of the ‘No’ type can never ask this question.

Since these are the only type of people on this island, no one from the
island can ask this question.

Thus, it is impossible for the islander to have asked this question.

Hence, option 3.

4. Ram and Laxman are brothers from the island. Laxman asks you, “Is at
3 Marks
least one from our pair of the ‘No’ type?” You can conclude that:

1) Ram is ‘No’, Laxman is ‘Yes’.


2) Both are ‘Yes’.
3) Ram is ‘Yes’, Laxman is ‘No’.
4) Both are ‘No’.

Solution:
There are 4 cases possible here, 2 where both brothers are of the same
type and 2 where both brothers are of different types.

Case I : Both brothers are of the type ‘Yes’.

If both brothers are of the type ‘Yes’, the answer to Laxman's question
(as per the definition of the type) should be ‘Yes’ while the actual
answer will be ‘No’.

Thus, both brothers cannot be of the type ‘Yes’.

Case II : Both brothers are of the type ‘No’.

If both brothers are of the type ‘No’, the answer to Laxman's question
(as per the definition of the type) should be ‘No’ while the actual answer
will be ‘Yes’.

4 of 28 9/16/2011 10:18 AM
Exam Reports http://testfunda.com/LMS/Student/NewReports.aspx

Thus, both brothers cannot be of the type ‘No’.

Hence, options 2 and 4 can be eliminated.

Case III: Laxman is the ‘No’ type and Ram is the ‘Yes’ type.

If Laxman is of the ‘No’ type, the answer to his question (as per the
definition of the type) should be ‘No’ while the actual answer will be
‘Yes’.

Thus, Laxman cannot be of the ‘No’ type.

Hence, option 3 can be eliminated.

Case IV: Laxman is the ‘Yes’ type and Ram is the ‘No’ type.

If Laxman is of the ‘Yes’ type, the answer to his question (as per the
definition of the type) should be ‘Yes’ while the actual answer will also
be ‘Yes’ (because Ram is of the ‘No’ type).

Thus, Laxman is the ‘Yes’ type and Ram is the ‘No’ type.

Hence, option 1.

5. You are approached by one of the islanders and asked, “Am I of


3 Marks
the ‘Yes’ type?” You can infer that …

1) he is a ‘No’ type.
2) he is a ‘Yes’ type.
3) such a question is impossible to ask.
4) no conclusion is possible.

Solution:
The islander can either be a ‘Yes’ type person or a ‘No’ type person.

Case I: The islander is a ‘Yes’ type person.

For such a person, the answer to the question will be ‘Yes’ as per the
definition of the type and the actual answer will also be ‘Yes’.

Thus, the person could be a ‘Yes’ type person.

Case II: The islander is a ‘No’ type person.

For such a person, the answer to the question will be ‘No’ as per the

5 of 28 9/16/2011 10:18 AM
Exam Reports http://testfunda.com/LMS/Student/NewReports.aspx

definition of the type and the actual answer will also be ‘No’.

Thus, the person could be a ‘No’ type person.

Thus, the person could be any islander whose type cannot be


determined.

Hence, no conclusion is possible.

Hence, option 4.

6. Each of three friends knows whether the other two have passed or failed in an
3 Marks
examination, but does not know his own result. The teacher comes and says,
“At least one student has failed”. If all three friends still do not know their own
results which of the following is true?

1) One student has failed.


2) Two students have failed.
3) Two or more students have failed.
4) All three have failed.

Solution:

Let the three students be A, B and C.

Their results can have 8 possibilities as shown in the table above.

It is known that even after the teacher says “At least one student has failed”,
no one knows his result.

Consider that exactly one student has failed.

This is seen in cases IV, VI and VII.

6 of 28 9/16/2011 10:18 AM
Exam Reports http://testfunda.com/LMS/Student/NewReports.aspx

In case IV, A knows that B and C have passed, but doesn't know his own
result.

Now, if the teacher says that at least one student has failed, it becomes clear
to A that he is the one who has failed.

Thus, A knows his result now, which is contradictory with the information in
the question.

Thus, only A cannot fail.

Using a similar logic, only B or only C cannot fail.

Thus, only one student cannot fail.

Hence, option 1 can be eliminated.

Now consider that exactly two students have failed.

This is seen in cases II, III and V.

In case II, C knows that A and B have failed, but doesn't know his own result.

Now, if the teacher says that at least one student has failed, it may mean that
A and B have failed while C has passed, or that all three have failed.

Thus, C still does not know his result.

This can be proved for case III and V as well.

In general, it is possible that exactly two students failed.

Finally, consider that all three students failed (as seen in case I).

Every person knows that the other two students failed.

Now, if the teacher says that at least one student has failed, it may mean
that two have failed while the third has passed, or that all three have failed.

Thus, it is possible that all three students have failed.

Thus, two or more students have failed.

Hence, option 3.

7. Amar, Akbar and Anthony come from the same public school in the
3 Marks
Himalayas. Every boy in that school either fishes for trout or plays Frisbee. All
fishermen like snow while no Frisbee player likes rain. Amar dislikes whatever

7 of 28 9/16/2011 10:18 AM
Exam Reports http://testfunda.com/LMS/Student/NewReports.aspx

Akbar likes and likes whatever Akbar dislikes. Akbar likes rain and snow.
Anthony likes whatever the other two like. Who is a fisherman but not a
Frisbee player?

1) Amar
2) Akbar
3) Anthony
4) More than one of the three boys

Solution:
All fishermen like snow while no Frisbee player likes rain.

Akbar likes rain and snow.

Therefore, Akbar is a fisherman but not a Frisbee player.

Amar dislikes both rain and snow.

Therefore, Amar is a Frisbee player but not a fisherman.

Since Anthony likes what the other two like, he cannot like anything. This is
because Amar dislikes rain and snow.

Therefore, Anthony is also a Frisbee player but not a fisherman.

Thus, only Akbar is a fisherman but not a Frisbee player.

Hence, option 2.

Group Question
Answer the following questions based on the information given below.

A leading socialite decided to organise a dinner and invited some of her friends.
Only the host and hostess were sitting at opposite ends of a rectangular table,
with three people on each side. The seating arrangement was such that
every person had at least on person of the opposite sex next to him/her. Maqbool
is opposite Shoba, who is not the hostess. Ratan has a woman on his right and is
sitting opposite a woman. Monisha is sitting to the hostess’s right, next to
Dhirubhai. One person is seated between Madhuri and Urmila, who is not the
hostess. The men were Maqbool, Ratan, Dhirubhai and Jackie, while the women
were Madhuri, Urmila, Shoba and Monisha.

8. Jackie must be ...


3 Marks

I. the host.

8 of 28 9/16/2011 10:18 AM
Exam Reports http://testfunda.com/LMS/Student/NewReports.aspx

II. seated to Shoba’s right.


III. seated opposite Urmila.

1) I only
2) III only
3) I and II only
4) II and III only

Solution:

Neither Shoba nor Urmila is the hostess. Also, Monisha is sitting to the
right of the hostess.

Thus, only Madhuri can be the hostess.

Thus, if one starts from Madhuri, Monisha is to her immediate right and
Dhirubhai is to Monisha's right.

Also, there is one person between Madhuri and Urmila.

Thus, Urmila has to be at the second place from left, starting from
Madhuri. This means that Urmila and Dhirubhai sit opposite each other.

Now, Shoba and Maqbool sit opposite each other. The only place where
two seats opposite each other are available is to the immediate right and
left of the host.

This means that Maqbool cannot be the host. Also, one of Ratan and
Jackie has to be the host.

Thus, there is one empty seat between Dhirubhai and the host (who is a
male).

Since every person must have at least one person of the opposite sex
next to him/her, the person between Dhirubhai and the host cannot be a
male.

9 of 28 9/16/2011 10:18 AM
Exam Reports http://testfunda.com/LMS/Student/NewReports.aspx

Since the only female left is Shoba, she has to be to the immediate right
of Dhirubhai and Maqbool is opposite her (i.e. to the immediate right
of the host).

Since Ratan has a woman to his right, he cannot be the host (due to
Maqbool on his right).

Therefore, Jackie is the host and Ratan is between Madhuri and Urmila.

Thus, the final arrangement is as shown in the figure above.

Statement I is true as Jackie is the host.

Statement II is also true as Jackie is to Shoba's right.

Statement III is false as Dhirubhai, and not Jackie, is seated opposite


Urmila.

Thus, only statements I and II are true.

Hence, option 3.

9. Which of the following persons is definitely not seated next to a person


3 Marks
of the same sex?

1) Maqbool
2) Madhuri
3) Jackie
4) Shoba

Solution:
From the arrangement obtained in the solution to the first question, it
can be seen that Shoba is sitting between two men (Jackie and
Dhirubhai).

Thus, Shoba is not seated next to a person of the same sex.

Hence, option 4.

10. If Ratan had exchanged seats with the person four places to his left,
3 Marks
which of the following would have been true after the exchange?

I. No one was seated between two persons of the opposite sex. (e.g. no man was
seated between two women)

10 of 28 9/16/2011 10:18 AM
Exam Reports http://testfunda.com/LMS/Student/NewReports.aspx

II. One side of the table consisted entirely of persons of the same sex.
III. Either the host or the hostess changed seats.

1) I only
2) II only
3) I and II only
4) II and III only

Solution:
Consider the final arrangement obtained in the solution to the first
question.

If Ratan had exchange seats with the person four places to his left, he
would have had to change places with Shoba.

No one else would need to change places.

Thus, neither the host nor the hostess would have changed their place.

Thus, statement III is false.

Hence, option 4 can be eliminated.

The side to Madhuri's right would have consisted of Monisha, Dhirubhai


and Ratan i.e. female, male and male.

The side to Madhuri's left would have consisted of Shoba, Urmila and
Maqbool i.e. female, female and male.

Thus, no side of the table would consist of people from the same sex.

Thus, statement II is false.

Hence, options 2 and 3 can be eliminated.

As per the new arrangement, Shoba and Ratan would be seated


between two people of the same sex.

No one would have been seated between two people of the opposite
sex.

Thus, statement I is true.

Hence, option 1.

11 of 28 9/16/2011 10:18 AM
Exam Reports http://testfunda.com/LMS/Student/NewReports.aspx

11. If each person is placed directly opposite his/her spouse, which of the
3 Marks
following pairs must be married?

1) Ratan and Monisha


2) Madhuri and Dhirubhai
3) Urmila and Jackie
4) Ratan and Madhuri

Solution:
Refer to the final arrangement obtained in the solution to the first
question.

Among the pairs given in the answer options, only Ratan and Monisha
are seated opposite each other.

Hence, Ratan and Monisha must be married.

Hence, option 1.

Group Question
Answer the following questions based on the information given below.

Five of India’s leading models are posing for a photograph promoting “y’know,
world peace and understanding”. But then, Rakesh Shreshtha, the photographer,
is having a tough time getting them to stand in a straight line, because Aishwarya
refuses to stand next to Sushmita because Sushmita had said something about
her in a leading gossip magazine. Rachael and Anu want to stand together
because they are “such good friends, y’know”. Manpreet on the other hand cannot
get along well with Rachael, because there is some talk about Rachael scheming
to get a contract already awarded to Manpreet. Anu believes her friendly
astrologer who has asked her to stand at the extreme right for all group
photographs. Finally, Rakesh managed to pacify the girls and got a beautiful
picture of five beautiful girls smiling beautifully in a beautiful straight line,
promoting world peace.

12. If Aishwarya is standing to the extreme left, which girl is standing in the
3 Marks
middle?

1) Manpreet
2) Sushmita
3) Rachael
4) Cannot say

12 of 28 9/16/2011 10:18 AM
Exam Reports http://testfunda.com/LMS/Student/NewReports.aspx

Solution:
Anu stands to the extreme right and Rachael wants to stand next to
Anu.

Thus, Rachael is second from right.

Now, Manpreet and Rachael cannot stand next to each other.

Thus, Manpreet has to be at the extreme left or at the second position


from the left.

If Manpreet is at the extreme left, Aishwarya and Sushmita have to


occupy the second and third position from the left.

However, Aishwarya and Sushmita cannot stand together.

Thus, Manpreet cannot be at the extreme left and so has to be second


from left.

Thus, one of Aishwarya and Sushmita is at the extreme left and the
other is at the centre.

The final arrangement is as shown below:

Now, if Aishwarya stands to the extreme left, then Sushmita would have
to stand in the middle.

Hence, option 2.

13. If Aishwarya stands to the extreme left, who stands second from left?
3 Marks

1) Cannot say
2) Sushmita
3) Rachel
4) Manpreet

Solution:
Refer to the arrangement obtained in the solution to the first question.

13 of 28 9/16/2011 10:18 AM
Exam Reports http://testfunda.com/LMS/Student/NewReports.aspx

Therefore, if Aishwarys stands to the extreme left, Manpreet stands


second from left.

Hence, option 4.

14. If Anu’s astrologer tells her to stand second from left and Aishwarya
3 Marks
decides to stand second from right, which girl stands on the extreme
right?

1) Rachel
2) Sushmita
3) Cannot say
4) Manpreet

Solution:
Anu is second from left and Aishwarya is second from right.

Since Aishwarya is second from right, Sushmita cannot be at the centre


or at the extreme right.

Therefore, Sushmita is at the extreme left.

Since Rachael is next to Anu, Rachael has to be at the centre.

Thus, the only place available for Manpreet is at the extreme right.

Thus, Manpreet stands on the extreme right as shown below.

Hence, option 4.

15. In each of the following sentences the main statement is followed by four
3 Marks sentences each. Select a pair of sentences that relate logically with the given
statement.
Either Sam is ill; or he is drunk.

A. Sam is ill.
B. Sam is not ill.
C. Sam is drunk.
D. Sam is not drunk.

14 of 28 9/16/2011 10:18 AM
Exam Reports http://testfunda.com/LMS/Student/NewReports.aspx

1) AB
2) DA
3) AC
4) CD

Solution:
This logical connective is of the following type

Either P or Q

The conclusions for the above type of logical connective are as follows

1. ~P ⇒ Q
2. ~Q ⇒ P

Thus, if Sam is not ill, then he is drunk (BC) or if Sam is not drunk, then he is
ill (DA).

BC is not given in any of the answer options.

Thus, the pair DA relates logically with the given statement.

Hence, option 2.

16. In each of the following sentences the main statement is followed by four
3 Marks sentences each. Select a pair of sentences that relate logically with the given
statement.
Whenever Ram hears of a tragedy, he loses sleep.

A. Ram heard of a tragedy.


B. Ram did not hear of a tragedy.
C. Ram lost sleep.
D. Ram did not lose sleep.

1) CA
2) BD
3) DB
4) AD

Solution:
This logical connective is of the following type

15 of 28 9/16/2011 10:18 AM
Exam Reports http://testfunda.com/LMS/Student/NewReports.aspx

Whenever P … Q which is the same as “If P then Q”

The conclusions for the above type of logical connective are as follows

1. P ⇒ Q
2. ~Q ⇒ ~P

Thus, if Ram heard a tragedy, he lost sleep (AC) or, if Ram did not lose sleep,
he had not heard of a tragedy (DB).

AC is not given in any of the answer options.

Therefore, DB relates logically with the given statement.

Hence, option 3.

17. In each of the following sentences the main statement is followed by four
3 Marks sentences each. Select a pair of sentences that relate logically with the given
statement.
Either the train is late; or it has derailed.

A. The train is late.


B. The train is not late.
C. The train is derailed.
D. The train is not derailed.

1) AB
2) DB
3) CA
4) BC

Solution:
This logical connective is of the following type

Either P or Q

The conclusions for the above type of logical connective are as follows

1. ~P ⇒ Q
2. ~Q ⇒ P

Thus, if the train is not late, it has been derailed (BC) or, if the train is not
derailed, it is late (DA).

16 of 28 9/16/2011 10:18 AM
Exam Reports http://testfunda.com/LMS/Student/NewReports.aspx

DA is not mentioned in any of the answer options.

Thus, the BC pair relates logically with the given statement.

Hence, option 4.

18. In each of the following sentences the main statement is followed by four
3 Marks sentences each. Select a pair of sentences that relate logically with the given
statement.
When I hear a horror story I have a nightmare.

A. I heard a story.
B. I did not hear a horror story.
C. I did not have a nightmare.
D. I had a nightmare.

1) CB
2) AD
3) BC
4) AC

Solution:
This logical connective is of the following type

When P … Q which is the same as “If P then Q”

The conclusions for the above type of logical connective are as follows

1. P ⇒ Q
2. ~Q ⇒ ~P

Thus, if I heard a horror story, I had a nightmare or if I did not have a


nightmare, I had not heard a horror story (CB).

Thus, the CB pair relates logically with the given statement.

Hence, option 1.

19. In each of the following sentences the main statement is followed by four
3 Marks sentences each. Select a pair of sentences that relate logically with the given
statement.
When I eat berries I get rashes.

A. I ate berries.

17 of 28 9/16/2011 10:18 AM
Exam Reports http://testfunda.com/LMS/Student/NewReports.aspx

B. I did not get rashes.


C. I did not eat berries.
D. I got rashes.

1) DA
2) BC
3) CB
4) AB

Solution:
This logical connective is of the following type

When P … Q which is the same as “If P then Q”.

The conclusions for the above type of logical connective are as follows

1. P ⇒ Q
2. ~Q ⇒ ~P

Thus, if I ate berries, I got rashes (AD) or if I did not get rashes, I did not eat
berries (BC).

AD is not mentioned in any of the answer options.

Thus, the BC pair relates logically with the given statement.

Hence, option 2.

Group Question
Answer the following questions based on the information given below.

Amar, Akbar, Anthony are three friends. Only three colours are available for their
shirts, viz., red, green and blue. Amar did not wear a red shirt. Akbar did not wear
a green shirt. Anthony did not wear a blue shirt.

20. If Akbar and Anthony wear the same colour, then which of the following
3 Marks
is not true?

1) Amar wears blue and Akbar wears green.


2) Amar wears green and Akbar wears red.
3) Amar wears blue and Akbar does not wear blue.
4) Anthony wears red.

18 of 28 9/16/2011 10:18 AM
Exam Reports http://testfunda.com/LMS/Student/NewReports.aspx

Solution:
Let the red, blue and green shirts be denoted by R, B and G
respectively.

Given conditions:

i. Amar does not wear R, hence only G or B.


ii. Akbar does not wear G, hence only R or B.
iii. Anthony does not wear B, hence only R or G.

Since Akbar and Anthony wear the same colour, both have to wear red.

As per option 1, Akbar wears green, which is not possible.

Hence, the statement in option 1 is not true.

All the other statements are possible.

Hence, option 1.

21. If two of them wear the same colour then how many of the following
3 Marks
statements must be false?

I. Amar wears blue and Akbar does not wear green.


II. Amar does not wear blue and Akbar wears blue.
III. Amar does not wear blue and Akbar does not wear blue.
IV. Amar wears green. Akbar does not wear red. Anthony does not wear green.

1) None
2) One
3) Two
4) Three

Solution:
Given that any two wear the same colour, only the following 3 cases
satisfy this condition:

Case 1: Amar = Akbar = B, Anthony = R or G

Case 2: Amar = Anthony = G, Akbar = R or B

Case 3: Akbar = Anthony = R, Amar = B or G

Consider the given statements:

19 of 28 9/16/2011 10:18 AM
Exam Reports http://testfunda.com/LMS/Student/NewReports.aspx

Statement 1: Amar = B, Akbar ≠ G

This combination is possible in cases 1 and 3. Hence, this statement is


not definitely false.

Statement 2: Amar ≠ B, Akbar = B

This combination is possible in case 2. Hence, this statement is not


definitely false.

Statement 3: Amar ≠ B, Akbar ≠ B

This combination is possible in cases 2 and 3. Hence, this statement is


not definitely false.

Statement 4: Amar = G, Akbar ≠ R, Anthony ≠ G

This combination is not possible in any of the 3 cases listed above.


Hence, this statement is definitely false.

Thus, there is only one statement that is definitely false.

Hence, option 2.

Group Question
Answer the following questions based on the information given below.

Mr. Bankatlal acted as a judge for the beauty contest. There were four participants,
viz., Ms. Andhra Pradesh, Ms. Uttar Pradesh, Ms. West Bengal and Ms.
Maharashtra. Mrs. Bankatlal, who was very anxious about the results, asked him
about it as soon as he was back home. Mr. Bankatlal just said that the one who was
wearing the yellow saree had won the contest. When Mrs. Bankatlal pressed for
further details, he elaborated as follows:

All of them were sitting in a row.


All of them wore sarees of different colours, viz., Green, Yellow, White and Red.
There was only one runner-up and she was sitting beside Ms. Maharashtra.
The runner-up was wearing the Green saree.
Ms. West Bengal was not sitting at the ends and was not the runner-up.
The winner and the runner-up were not sitting adjacent to each other.
Ms. Maharashtra was wearing a white saree.
Ms. Andhra Pradesh was not wearing the Green saree.
The participants wearing the Yellow saree and the White saree were at the ends.

22. Who wore the Red saree?

20 of 28 9/16/2011 10:18 AM
Exam Reports http://testfunda.com/LMS/Student/NewReports.aspx

3 Marks
1) Ms. Andhra Pradesh
2) Ms. West Bengal
3) Ms. Uttar Pradesh
4) Ms. Maharashtra

Solution:
Consider the following statements:

1. All of them were sitting in a row.


2. All of them wore sarees of different colours, viz., Green, Yellow, White and Red.
3. There was only one runner-up and she was sitting beside Ms. Maharashtra.
4. The runner-up was wearing the Green saree.
5. Ms. West Bengal was not sitting at the ends and was not the runner-up.
6. The winner and the runner-up were not sitting adjacent to each other.
7. Ms. Maharashtra was wearing a white saree.
8. Ms. Andhra Pradesh was not wearing the Green saree.
9. The participants wearing the Yellow saree and the White saree were at the ends.
10. The winner was wearing Yellow saree (This is obtained from the facts that the
runner-up was sitting next to Ms. Maharashtra and that the winner was not seated
next to the runner up. Thus, the winner would have to be at the other end. The
person at the other end was wearing a yellow saree).

Using (1), (2), (7), (9) and (10), we have

Using (3) and (4), we have

Using (5) and (8), we have the final arrangement as:

Thus, Ms. West Bengal wore the red saree.

Hence, option 2.

21 of 28 9/16/2011 10:18 AM
Exam Reports http://testfunda.com/LMS/Student/NewReports.aspx

23. Ms. West Bengal was sitting adjacent to …


3 Marks

1) Ms. Andhra Pradesh and Ms. Maharashtra


2) Ms. Uttar Pradesh and Ms. Maharashtra
3) Ms. Andhra Pradesh and Ms. Uttar Pradesh
4) Ms. Uttar Pradesh only

Solution:
From the final table obtained in the solution to the first question, Ms. West
Bengal was sitting adjacent to Ms. Uttar Pradesh and Ms. Andhra
Pradesh.

Hence, option 3.

24. Which saree was worn by Ms. Andhra Pradesh?


3 Marks

1) Yellow
2) Red
3) Green
4) White

Solution:
From the final table obtained in the solution to the first question, Ms.
Andhra Pradesh wore the Yellow saree.

Hence, option 1.

25. Who was the runner-up?


3 Marks

1) Ms. Andhra Pradesh


2) Ms. West Bengal
3) Ms. Uttar Pradesh
4) Ms. Maharashtra

Solution:
From the final table obtained in the solution to the first question, Ms. Uttar
Pradesh was the runner-up of the contest.

Hence, option 3.

22 of 28 9/16/2011 10:18 AM
Exam Reports http://testfunda.com/LMS/Student/NewReports.aspx

26. Abraham, Border, Charlie, Dennis and Elmer and their respective wives dined
3 Marks
together and were seated at a circular table. The seats were so arranged that
men and women alternated and each woman was three places distant from
her husband. Mrs. Charlie sat to the left of Mr. Abraham. Mrs. Elmer sat two
places to the right of Mrs. Border. Who sat to the right of Mr. Abraham?

1) Mrs. Dennis
2) Mrs. Elmer
3) Mrs. Border
4) Mrs. Border or Mrs. Dennis

Solution:
It is given that men and women alternated in their seating arrangement.
Hence, each man is between two women.

Identify the women who cannot be seated to the right of Mr. Abraham.

Since each woman was three places away from her husband, Mrs. Abraham
cannot sit next to Mr. Abraham.

It is known that Mrs. Charlie is to the left of Mr. Abraham, so she cannot be at
the right side simultaneously.

Since Mrs. Elmer is two places to the right of Mrs. Border, Mrs. Elmer cannot
be sitting to the right of Mr. Abraham.

So, either Mrs. Border or Mrs. Dennis could sit to the right of Mr. Abraham.

Hence, option 4.

Group Question
Answer the following questions based on the information given below.

Seven university cricket players are to be honoured at a special luncheon. The


players will be seated on the dais along one side of a single rectangular table.

A and G have to leave the luncheon early and must be seated at the extreme right end of
the table, which is closest to the exit.
B will receive the Man of the Match award and must be in the center chair.
C and D who are bitter rivals for the position of wicket keeper, dislike one another and
should be seated as far apart as possible.
E and F are best friends and want to sit together.

27. Which of the following may not be seated at either end of the table?
3 Marks

23 of 28 9/16/2011 10:18 AM
Exam Reports http://testfunda.com/LMS/Student/NewReports.aspx

1) C
2) D
3) G
4) F

Solution:
Consider the blank seat from left to right as shown below.

(Left) __ __ __ __ __ __ __ (Right)

B will be at the centre. Simultaneously, A and G will occupy the two


seats at the extreme right.

Thus, the arrangement now is:

__ __ __ B __ A/G G/A

Since C and D have to be as far away as possible from each other, one
of them has to be to the immediate right of B while the other has to be
at the extreme left seat.

Now, E and F have to be together. They anyways have only the second
and third seat from the left available.

Hence, they occupy those seats.

Thus, the final arrangement is as shown below.

D/C E/F F/E B C/D A/G G/A

Thus, F may not be seated at either end of the table.

Hence, option 4.

28. Which of the following pairs may not be seated together?


3 Marks

1) E and A
2) B and D
3) C and F
4) G and D

24 of 28 9/16/2011 10:18 AM
Exam Reports http://testfunda.com/LMS/Student/NewReports.aspx

Solution:
From the solution to the first question, it can be seen that B and D, C
and F, as well as G and D can be seated together.

Only E and A cannot never be seated together.

Hence, option 1.

29. Each question has a main statement followed by four statements labelled A,
3 Marks B, C and D. Choose the ordered pair of statements where the first statement
implies the second, and the two statements are logically consistent with the
main statement.
Either the orangutan is not angry, or he frowns upon the world.

A. The orangutan frowns upon the world.


B. The orangutan is not angry.
C. The orangutan does not frown upon the world.
D. The orangutan is angry.

1) CB only
2) DA only
3) AB only
4) CB and DA

Solution:
This logical connective is of the following type

Either P or Q

The conclusions for the above type of logical connective are as follows

1. ~P ⇒ Q
2. ~Q ⇒ P

Thus, if the orangutan is angry, he frowns upon the world (DA) or if the
orangutan does not frown upon the world, the orangutan is not angry (CB).

Thus, both pairs (DA and CB) are logically consistent with the main
statement.

Hence, option 4.

30. Each question has a main statement followed by four statements labelled A,
3 Marks

25 of 28 9/16/2011 10:18 AM
Exam Reports http://testfunda.com/LMS/Student/NewReports.aspx

B, C and D. Choose the ordered pair of statements where the first statement
implies the second, and the two statements are logically consistent with the
main statement.
Either Ravana is a demon, or he is a hero.

A. Ravana is a hero.
B. Ravana is a demon.
C. Ravana is not a demon.
D. Ravana is not a hero.

1) CD only
2) BA only
3) CD and BA
4) DB and CA

Solution:
This logical connective is of the following type

Either P or Q

The conclusions for the above type of logical connective are as follows

1. ~P ⇒ Q
2. ~Q ⇒ P

Thus, if Ravana is not a demon, then he is a hero (CA) or if Ravana is not a


hero, then he is a demon (DB).

Thus, both pairs (DB and CA) are logically consistent with the main
statement.

Hence, option 4.

31. Each question has a main statement followed by four statements labelled A,
3 Marks B, C and D. Choose the ordered pair of statements where the first statement
implies the second, and the two statements are logically consistent with the
main statement.
Whenever Rajeev uses the internet, he dreams about spiders.

A. Rajeev did not dream about spiders.


B. Rajeev used the internet.
C. Rajeev dreamt about spiders.
D. Rajeev did not use the internet.

26 of 28 9/16/2011 10:18 AM
Exam Reports http://testfunda.com/LMS/Student/NewReports.aspx

1) AD
2) DC
3) CB
4) DA

Solution:
This logical connective is of the following type

Whenever P then Q which is the same as “If P then Q”

The conclusions for the above type of logical connective are as follows

1. P ⇒ Q
2. ~Q ⇒ ~P

Thus, if Rajeev used the internet, he dreamt about spiders (BC) or if Rajeev
did not dream about spiders, he did not use the internet (AD).

The BC pair is not given in the options.

Therefore, the AD pair is logically consistent with the main statement.

Hence, option 1.

32. Each question has a main statement followed by four statements labelled A,
3 Marks B, C and D. Choose the ordered pair of statements where the first statement
implies the second, and the two statements are logically consistent with the
main statement.
If I talk to my professors, then I do not need to take a pill for headache.

A. I talked to my professors.
B. I did not need to take a pill for headache.
C. I needed to take a pill for headache.
D. I did not talk to my professors.

1) AB only
2) DC only
3) CD only
4) AB and CD

Solution:
This logical connective is of the following type

27 of 28 9/16/2011 10:18 AM
Exam Reports http://testfunda.com/LMS/Student/NewReports.aspx

If P then Q

The conclusions for the above type of logical connective are as follows

1. P ⇒ Q
2. ~Q ⇒ ~P

Thus, if I talked to my professors, I did not need to take a pill for headache
(AB) or if I needed to take a pill for my headache, I did not talk to my
professors (CD).

Thus, both pairs (AB and CD) are logically consistent with the main
statement.

Hence, option 4.

28 of 28 9/16/2011 10:18 AM
Exam Reports http://testfunda.com/LMS/Student/NewReports.aspx

Section I

Group Question
Answer the following questions based on the information given below.

Five women decided to go shopping to M.G. Road, Bangalore. They arrived at the
designated meeting place in the following order: 1. Archana, 2. Chellamma, 3.
Dhenuka, 4. Helen, and 5. Shahnaz. Each woman spent at least Rs. 1000. Below
are some additional facts about how much they spent during their shopping
spree.

i. The woman who spent Rs. 2234 arrived before the lady who spent Rs. 1193.
ii. One woman spent Rs. 1340 and she was not Dhenuka.
iii. One woman spent Rs. 1378 more than Chellamma.
iv. One woman spent Rs. 2517 and she was not Archana.
v. Helen spent more than Dhenuka.
vi. Shahnaz spent the largest amount and Chellamma the smallest.

[CAT 2003 Leaked Test]

1. What was the amount spent by Helen?


3 Marks

1) Rs. 1193
2) Rs. 1340
3) Rs. 2234
4) Rs. 2517

Solution:
The amounts are Rs. 2,234, Rs. 1,193, Rs. 1,340 and Rs. 2,517. The
fifth amount is not given.

∵ One woman’s amount = Chellamma’s amount + Rs. 1,378

∵ All the women spent more than Rs. 1,000.

∴ Chellamma’s amount

= Rs. 2,517 – Rs. 1,378

= Rs. 1,139

∴ The biggest amount of Rs. 2,517 is spent by Shahnaz.

∵ It is given that Helen spent more than Dhenuka and Dhenuka did not

1 of 23 9/16/2011 10:28 AM
Exam Reports http://testfunda.com/LMS/Student/NewReports.aspx

spend Rs. 1,340.

∴ Dhenuka spent Rs. 1,193.

We also know that the women who spent Rs. 2,234 arrived before the
lady who spent 1193.

∴ Archana spent Rs. 2,234 and Helen spent Rs. 1,340.

The amount spent by Helen was Rs. 1,340.

Hence, option 2.

2. Which of the following amounts was spent by one of them?


3 Marks

1) Rs. 1139
2) Rs. 1378
3) Rs. 2571
4) Rs. 2718

Solution:
Rs. 1,139 was one of the amounts which was spent by Chellamma.

Hence, option 1.

3. The woman who spent Rs. 1193 is


3 Marks

1) Archana
2) Chellamma
3) Dhenuka
4) Helen

Solution:

2 of 23 9/16/2011 10:28 AM
Exam Reports http://testfunda.com/LMS/Student/NewReports.aspx

The woman who spent Rs. 1,193 was Dhenuka.

Hence, option 3.

Group Question
Answer the following questions based on the information given below.

Five friends meet every morning at Sree Sagar restaurant for an idli-vada
breakfast. Each consumes a different number of idlis and vadas. The number of
idlis consumed are 1, 4, 5, 6, and 8, while the number of vadas consumed are 0,
1, 2, 4, and 6. Below are some more facts about who eats what and how much?

i. The number of vadas eaten by Ignesh is three times the number of vadas consumed by
the person who eats four idlis.
ii. Three persons, including the one who eats four vadas, eat without chutney.
iii. Sandeep does not take any chutney.
iv. The one who eats one idli a day does not eat any vadas or chutney. Further, he is not
Mukesh.
v. Daljit eats idli with chutney and also eats vada.
vi. Mukesh, who does not take chutney, eats half as many vadas as the person who eats twice
as many idlis as he does.
vii. Bimal eats two more idlis than Ignesh, but Ignesh eats two more vadas than Bimal.

[CAT 2003 Leaked Test]

4. Which one of the following statements is true?


3 Marks

1) Daljit eats 5 idlis.


2) Ignesh eats 8 idlis.
3) Bimal eats 1 idli.
4) Bimal eats 6 idlis.

Solution:
From (i),

Number of vadas eaten by Ignesh is 6 and the person eating 4 idlis eats
2 vadas.

From (vii),

Number of vadas eaten by Bimal = 4

Number of idlis eaten by Bimal = 8

3 of 23 9/16/2011 10:28 AM
Exam Reports http://testfunda.com/LMS/Student/NewReports.aspx

Number of idlis eaten by Ignesh = 6

Ignesh, Mukesh, Daljit and Bimal eat at least 1 vada and 1 idli.

∴ Sandeep doesn’t eat any vadas but eats 1 idli.

From (vi),

Number of vadas eaten by Mukesh = 2

Number of idlis eaten by Mukesh = 4

Daljit eats 5 idlis.

Hence, option 1.

5. Which of the following statements is true?


3 Marks

1) Sandeep eats 2 vadas.


2) Mukesh eats 4 vadas.
3) Ignesh eats 6 vadas.
4) Bimal eats 4 vadas.

Solution:
Ignesh eats 6 vadas and Bimal eats 4 vadas.

There seems to be some problem with this question as there are two
options which are correct.

Hence, option 3 or option 4.

6. Which of the following statements is true?


3 Marks

1) Mukesh eats 8 idlis and 4 vadas but no chutney.

4 of 23 9/16/2011 10:28 AM
Exam Reports http://testfunda.com/LMS/Student/NewReports.aspx

2) The person who eats 5 idlis and 1 vada does not take
chutney.
3) The person who eats equal number of vadas and idlis also
takes chutney.
4) The person who eats 4 idlis and 2 vadas also takes chutney.

Solution:
Ignesh eats 6 vadas and 6 idlis with chutney.

Hence, option 3.

Group Question
Answer the following questions based on the information given below.

Four families decided to attend the marriage ceremony of one of their colleagues.
One family has no kids, while the others have at least one kid each. Each family
with kids has at least one kid attending the marriage. Given below is some
information about the families, and who reached when to attend the marriage.

The family with 2 kids came just before the family with no kids.
Shanthi who does not have any kids reached just before Sridevi’s family.
Sunil and his wife reached last with their only kid.
Anil is not the husband of Joya.
Anil and Raj are fathers.
Sridevi’s and Anita’s daughters go to the same school.
Joya came before Shanthi and met Anita when she reached the venue.
Raman stays the farthest from the venue.
Raj said his son could not come because of his exams.

[CAT 2003 Re-Test]

7. Which woman arrived third?


3 Marks

1) Shanthi
2) Sridevi
3) Anita
4) Joya

Solution:
Let’s organize the information given in a tabular form.

5 of 23 9/16/2011 10:28 AM
Exam Reports http://testfunda.com/LMS/Student/NewReports.aspx

Using first two statements, we have,

Anil, Raj and Sunil have kids.

∴ Raman is the husband of Shanthi.

∵ Joya came before Shanthi and met Anita on the Venue.

∴ Anita reached the venue first, Joya reached second, Shanthi reached
third and Sridevi reached last.

As Sunil reached the last with his only kid, Sunil is the husband of
Sridevi.

∵ Anil is not the husband of Joya.

∴ Raj is Joya’s husband and Anil is Anita’s husband.

From the table we can see that Shanthi arrived third.

Hence, option 1.

8. Name the correct pair of husband and wife.


3 Marks

6 of 23 9/16/2011 10:28 AM
Exam Reports http://testfunda.com/LMS/Student/NewReports.aspx

1) Raj and Shanthi


2) Sunil and Sridevi
3) Anil and Sridevi
4) Raj and Anita

Solution:
Referring the solution given in the first question of the set we get that,

Out of the given choices, Sunil and Sridevi is the correct pair.

Hence, option 2.

9. Of the following pairs, whose daughters go to the same school?


3 Marks

1) Anil and Raman


2) Sunil and Raman
3) Sunil and Anil
4) Raj and Anil

Solution:
The daughters of Anita and Sridevi go to the same school.

Referring the table given in the solution of the first question of the set
we get that,

Anita’s husband is Anil and Sridevi’s husband is Sunil.

∴ Sunil and Anil is the correct answer.

Hence, option 3.

10. Whose family is known to have more than one kid for certain?
3 Marks

1) Raman’s
2) Raj’s
3) Anil’s
4) Sunil’s

Solution:

7 of 23 9/16/2011 10:28 AM
Exam Reports http://testfunda.com/LMS/Student/NewReports.aspx

Raj’s family consists of two kids for certain.

Anil has at least one kid, and hence, could be having exactly one kid
also.

Hence, option 2.

Group Question
Answer the following questions based on the information given below.

The plan above shows an office block for six officers, A, B, C, D, E and F. Both B
and C occupy offices to the right of corridor (as one enters the office block) and A
occupies an office to the left of the corridor. E and F occupy offices on opposite
sides of the corridor but their offices do not face each other. The offices of C and
D face each other. E does not have a corner office. F’s office is further down the
corridor than A’s, but on the same side.

[CAT 2003 Re-Test]

11. If E sits in his office and faces the corridor, whose office is to his left?
3 Marks

1) A
2) B
3) C
4) D

Solution:

8 of 23 9/16/2011 10:28 AM
Exam Reports http://testfunda.com/LMS/Student/NewReports.aspx

From the data given we can figure out that A, F and D have offices on
the left side of the corridor and B, E and C have offices on the right side
of the corridor.

∵ E cannot occupy the corner office.

∴ E occupies an office in the middle of the right corridor.

∵ F cannot sit opposite E and A has to occupy an office before F.

∴ F occupies an office at far end of the left side of the corridor.

∴ D and C have to be at the beginning of the corridor.

∴ A has to be in the middle of the left corridor and B has to be at the


far end of the right corridor.

The arrangement of offices will be as shown in the figure above :

If E sits in his office and faces the corridor. C’s office is to his left.

Hence, option 3.

12. Whose office faces A’s office?


3 Marks

1) B
2) C
3) D
4) E

Solution:
From the arrangement given in the solution of the first question of the
set we get that,

E’s office faces A’s office.

Hence, option 4.

9 of 23 9/16/2011 10:28 AM
Exam Reports http://testfunda.com/LMS/Student/NewReports.aspx

13. Who is/are F’s neighbour(s)?


3 Marks

1) A only
2) A and D
3) C only
4) B and C

Solution:
Only A is F’s neighbour.

Hence, option 1.

14. D was heard telling someone to go further down the corridor to the last
3 Marks
office on the right. To whose room was he trying to direct that person?

1) A
2) B
3) C
4) F

Solution:
D was trying to direct the person to B’s office as B’s office is last on the
right side of the corridor.

Hence, option 2.

Group Question
Answer the following questions based on the information given below.

Seven faculty members at a management institute frequent a lounge for strong


coffee and stimulating conversation. On being asked about their visit to the
lounge last Friday we got the following responses.

JC : I came in first, and the next two persons to enter were SS and SM. When
I left the lounge, JP and VR were present in the lounge. DG left with me.
JP : When I entered the lounge with VR, JC was sitting there. There was
someone else, but I cannot remember who it was.
SM : I went to the lounge for a short while, and met JC, SS and DG in the
lounge on that day.
SS : I left immediately after SM left.
DG : I met JC, SS, SM, JP and VR during my first visit to the lounge. I went
back to my office with JC. When I went to the lounge the second time, JP and VR

10 of 23 9/16/2011 10:28 AM
Exam Reports http://testfunda.com/LMS/Student/NewReports.aspx

were there.
PK : I had some urgent work, so I did not sit in the lounge that day, but just
collected my coffee and left. JP and DG were the only people in the lounge while
I was there.
VR : No comments.

[CAT 2003 Re-Test]

15. Based on the responses, which of the two JP or DG, entered the lounge
3 Marks
first?

1) JP
2) DG
3) Both entered together
4) Cannot be deduced

Solution:

The information can be organised in a tabular form as shown above:

JC arrives first in the lounge, followed by SS and SM.

∴ DG must have followed them as SM met him before he left.

SM leaves after DG comes followed by SS.

JP and VR enter to find JC and DG sitting in the lounge.

JC leaves with DG, after meeting JP and VR.

DG comes back on his second visit to the lounge finding JP and VR still
sitting in the lounge.

11 of 23 9/16/2011 10:28 AM
Exam Reports http://testfunda.com/LMS/Student/NewReports.aspx

VR leaves the lounge, and only JP and DG are left behind in the
lounge.

PK enters the lounge and finds only JP and DG sitting in the lounge.

PK leaves, which means that JP and DG must be the last two faculty
members to leave the lounge.

Between JP and DG, DG entered the lounge first.

Hence, option 2.

16. Who was sitting with JC when JP entered the lounge?


3 Marks

1) SS
2) SM
3) DG
4) PK

Solution:
DG was sitting with JC when JP entered the lounge.

Hence, option 3.

17. How many of the seven members did VR meet on Friday in the lounge?
3 Marks

1) 2
2) 3
3) 4
4) 5

Solution:
VR met JC, DG and JP in the lounge on Friday.

Hence, option 2.

18. Who were the last two faculty members to leave the lounge?
3 Marks

1) JC and DG
2) PK and DG

12 of 23 9/16/2011 10:28 AM
Exam Reports http://testfunda.com/LMS/Student/NewReports.aspx

3) JP and PK
4) JP and DG

Solution:
JP and DG were the last two faculty members to leave the lounge.

Hence, option 4.

Group Question
Answer the following questions based on the information given below.

K, L, M, N, P, Q, R, S, U and W are the only ten members in a department. There


is a proposal to form a team from within the members of the department, subject
to the following conditions:

A team must include exactly one among P, R, and S.


A team must include either M or Q, but not both.
If a team includes K, then it must also include L, and vice versa.
If a team includes one among S, U, and W, then it must also include the other two.
L and N cannot be members of the same team.
L and U cannot be members of the same team.
The size of a team is defined as the number of members in the team.

[CAT 2006]

19. What could be the size of a team that includes K?


3 Marks

1) 2 or 3
2) 2 or 4
3) 3 or 4
4) Only 2
5) Only 4

Solution:
As K is included, L is included. So, N and U cannot be included. As U is
not included, S and W are not included. One out of M and Q and one
out of P and R will be included.

Thus, the team will include: K, L, (M or Q) and (P or R).

Hence, option 5.

13 of 23 9/16/2011 10:28 AM
Exam Reports http://testfunda.com/LMS/Student/NewReports.aspx

20. In how many ways a team can be constituted so that the team includes
3 Marks
N?

1) 2
2) 3
3) 4
4) 5
5) 6

Solution:
If the team includes N, it does not include L and K.

One out of M and Q can be included and one out of P, S and R can be
included.

If S is a member, so are U and W.

Thus the possible teams are:

1. N, M, P
2. N, M, R
3. N, Q, P
4. N, Q, R
5. N, M, S, U, W
6. N, Q, S, U, W

Hence, option 5.

21. What would be the size of the largest possible team?


3 Marks

1) 8
2) 7
3) 6
4) 5
5) Cannot be determined

Solution:
If S is not included, the team can have P or R, M or Q, K and L.

If S is included, the team will have S, U, W, M or Q, N.

This is the largest possible team.

14 of 23 9/16/2011 10:28 AM
Exam Reports http://testfunda.com/LMS/Student/NewReports.aspx

Hence, option 4.

22. Who can be a member of a team of size 5?


3 Marks

1) K
2) L
3) M
4) P
5) R

Solution:
If K or L are included, N, U, S and W are excluded. One out of P and R
and one out of M and Q are included. Thus the team has only 4
members.

If P or R are included, the team can have M or Q, K and L. This team


also has 4 members.

A team having M can have S, U, W and N i.e., 5 members.

Hence, option 3.

23. Who cannot be a member of a team of size 3?


3 Marks

1) L
2) M
3) N
4) P
5) Q

Solution:
A team sized 3 has to have M or Q and P or R. The only other member
that can be selected all alone is N.

L cannot be selected as K has to be selected with him.

Hence, option 1.

Group Question

15 of 23 9/16/2011 10:28 AM
Exam Reports http://testfunda.com/LMS/Student/NewReports.aspx

Answer the following questions based on the information given below.

Answer the following questions based on the statements given below:

i. There are three houses on each side of the road.


ii. These six houses are labeled as P, Q, R, S, T and U.
iii. The houses are of different colours, namely, Red, Blue, Green, Orange, Yellow and White.
iv. The houses are of different heights.
v. T, the tallest house, is exactly opposite to the Red coloured house.
vi. The shortest house is exactly opposite to the Green coloured house.
vii. U, the Orange coloured house, is located between P and S.
viii. R, the Yellow coloured house, is exactly opposite to P.
ix. Q, the Green coloured house, is exactly opposite to U.
x. P, the White coloured house, is taller than R, but shorter than S and Q.

[CAT 2008]

24. What is the colour of the tallest house?


3 Marks

1) Red
2) Blue
3) Green
4) Yellow
5) None of these

Solution:
We have to arrange six houses on opposite sides of a road.

From condition (vii), we can say that P, U and S lie on one side of the
road as follows:

From condition (viii) and (ix) we can further complete the arrangement
as follows. We have also used the color of the house P from statement
(x).

16 of 23 9/16/2011 10:28 AM
Exam Reports http://testfunda.com/LMS/Student/NewReports.aspx

The only left house is definitely T. From conditions (v) and we can
complete the arrangement as follows.

From condition (vi) it can be deduced that U is the shortest house. Also
from the last condition it can be deduced that P is the fourth tallest, R is
the fifth tallest and S and Q are second and third tallest not in that
order.

Filling all this data we can see the arrangement as follows:

The color of the tallest house (T) is Blue.

Hence, option 2.

25. What is the colour of the house diagonally opposite to the Yellow
3 Marks
coloured house?

1) White
2) Blue
3) Green
4) Red
5) None of these

17 of 23 9/16/2011 10:28 AM
Exam Reports http://testfunda.com/LMS/Student/NewReports.aspx

Solution:
The house diagonally opposite to the Yellow coloured house is S which
has red colour.

Hence, option 4.

26. Which is the second tallest house?


3 Marks

1) P
2) S
3) Q
4) R
5) Cannot be determined

Solution:
The second tallest house can be either S or Q. We cannot determine for
sure which of them is the second tallest.

Hence, option 5.

Group Question
Answer the following questions based on the information given below.

Answer the following questions based on the information given below:

In a sports event, six teams (A, B, C, D, E and F) are competing against each
other. Matches are scheduled in two stages. Each team plays three matches in
Stage-I and two matches in Stage-II. No team plays against the same team more
than once in the event. No ties are permitted in any of the matches. The
observations after the completion of Stage-I and Stage-II are as given below.

Stage-I:

One team won all the three matches.


Two teams lost all the matches.
D lost to A but won against C and F.
E lost to B but won against C and F.
B lost at least one match.
F did not play against the top team of Stage-I.

Stage-II:

18 of 23 9/16/2011 10:28 AM
Exam Reports http://testfunda.com/LMS/Student/NewReports.aspx

The leader of Stage-I lost the next two matches.


Of the two teams at the bottom after Stage-I, one team won both matches, while the
other lost both matches.
One more team lost both matches in Stage-II.

[CAT 2008]

27. The team(s) with the most wins in the event is (are):
3 Marks

1) A
2) A&C
3) F
4) E
5) B&E

Solution:
Let the bold letters denote the teams that have lost.

From condition 3 of stage I,

D lost to A.

D won against C.

D won against F.

These can be represented as:

D -- A

D -- C

D -- F

Similarly, condition 4 of stage I can be represented as:

E -- B

E -- C

E -- F

Since D and E have participated in three matches in stage I, they would


not be involved in any other match in stage I.

From the above representations it is clear that all other teams except A

19 of 23 9/16/2011 10:28 AM
Exam Reports http://testfunda.com/LMS/Student/NewReports.aspx

have lost at least one match.

∴ From condition 1, of stage I, only A has won all the three matches in
stage I.

Also, A will participate in 2 more matches as every team participates in 3


matches in stage I.

∴ A will win in 2 of the remaining 3 matches.

Also A is the top team as it wins all matches in stage I.

From condition 6 of stage I,

F did not play against A.

∴ A won against B and C which can be represented as:

B -- A

A -- C

The only 2 teams which have not won even a single match so far is C
and F.

From statement 6 of stage I, F loses in the remaining match against B,


which can be

represented as:

F -- B

Stage I can be represented as:

D -- A B -- A

D -- C A -- C

D -- F F -- B

E -- B

E -- C

E -- F

From condition 1 of stage II,

20 of 23 9/16/2011 10:28 AM
Exam Reports http://testfunda.com/LMS/Student/NewReports.aspx

A lost both matches in stage II.

Also, since no team plays against the same team more than once in the
event, A plays matches against E and F.

A -- E

A -- F

Since one of the two teams at the bottom after stage I won both
matches in stage II, F is the team which has won both the matches in
stage II.

Also C lost both matches in stage II.

F -- C

B -- C

The last condition states that one more team lost both matches in stage
II.

∴ D lost both matches in stage II.

D -- B

D -- E

Stage II can be represented as:

A -- E

A -- F

F -- C

B -- C

D -- B

D -- E

Now, we can calculate the number of times each team has won.

21 of 23 9/16/2011 10:28 AM
Exam Reports http://testfunda.com/LMS/Student/NewReports.aspx

It can be observed from the above table that B and E have most wins in
the event.

Hence, option 5.

28. The two teams that defeated the leader of Stage-I are:
3 Marks

1) F&D
2) E&F
3) B&D
4) E&D
5) F&D

Solution:
E and F defeated A.

Hence, option 2.

29. The only team(s) that won both the matches in Stage-II is (are):
3 Marks

1) B
2) E&F
3) A, E & F
4) B, E & F
5) B&F

Solution:
B, E and F are the three teams that won both matches in stage II.

22 of 23 9/16/2011 10:28 AM
Exam Reports http://testfunda.com/LMS/Student/NewReports.aspx

Hence, option 4.

30. The teams that won exactly two matches in the event are:
3 Marks

1) A, D & F
2) D&E
3) E&F
4) D, E & F
5) D&F

Solution:
From the table it is clear that the team that won excatly two matches in
the event are D and F.

Hence, option 5.

23 of 23 9/16/2011 10:28 AM
Exam Reports http://testfunda.com/LMS/Student/NewReports.aspx

Section I

Group Question
Answer the following questions based on the information given below.

The following graph shows the value of liquor supplied by 5 states in 1996 and
the excise duty rates in each state.

The amount of liquor supplied in 5 distilleries in Tamil Nadu i.e. A, B, C, D, E (from


bottom to top) in lakh litres is shown in the graph below.

1. What is the lowest percentage difference in the excise duty rates for any
3 Marks
two states?

1 of 23 9/16/2011 10:10 AM
Exam Reports http://testfunda.com/LMS/Student/NewReports.aspx

1) 13
2) 15
3) 20
4) Indeterminate

Solution:
The approximate excise duty rate for each state is:

TN : 25%

AP : 60%

Maharashtra : 50%

MP : 40%

Delhi : 80%

The lowest percentage difference will be possible only when any two
consecutive values are considered.

The excise duty rates, arranged in ascending order, are as follows:

25 - 40 - 50 - 60 - 80

The corresponding approximate percentage difference in each case is:

60%, 25%, 20%, 33.33%

Thus, the lowest percentage difference is 20%.

Hence, option 3.

2. Which of the five states manufactured liquor at the lowest cost?


3 Marks

1) Tamil Nadu
2) Delhi
3) The states which has the lowest value for (Wholesale price –
Excise duty) per litre
4) Indeterminate

2 of 23 9/16/2011 10:10 AM
Exam Reports http://testfunda.com/LMS/Student/NewReports.aspx

Solution:
The answer to this question cannot be found.

This is because the total quantity of liquor produced in each state is not
known.

Even if that quantity is known, one would have to assume that the total
cost of liquor comes only from excise duty.

Therefore, it is impossible to determine the state that produces liquor at


the lowest cost.

Hence, option 4.

3. If Excise duty is levied before the goods leave the factory (on the value
3 Marks
of the liquor), then which of the following choices shows the distilleries
in ascending order of the excise duty paid by them for the year 1996?
(Assume the total liquor in TN is supplied by only these 5 distilleries).

1) ECABD
2) ADEBC
3) DCEBA
4) Indeterminate

Solution:
This question can be answered by simple observation.

Only the 5 distilleries given supply liquor in TN.

Also, the excise duty is levied on the goods (based on the value of the
liquor)

∴ Excise Duty ∝ Value of Liquor

From the bar chart for 1996, it is obvious that distillery D supplies the
minimum amount (and hence value) of liquor and distillery A supplies
the maximum.

Thus, the order starts from D and ends in A.

Hence, options 1,2 and 4 can be eliminated.

By observation, it is clear that the correct order is DCEBA.

3 of 23 9/16/2011 10:10 AM
Exam Reports http://testfunda.com/LMS/Student/NewReports.aspx

Hence, option 3.

4. If the Tamil Nadu distillery, with the least average simple annual growth
3 Marks
in amount of liquor supplied in the given period had shown the same
rate of growth as the one which grew fastest, what would that distillery’s
supply have been in 1998, in lakh litres?

1) 13
2) 15.4
3) 130
4) Indeterminate

Solution:
Observe that distillery E remains relatively constant throughout the
period while distillery C grows quite drastically in the given period.

Thus, E is the slowest growing distillery while C is the fastest growing


one in the given period.

The value in 1996 for distillery E is approximately 2.5

The rate of growth between 1996 and 1998 for distillery C

= (11 − 1.5)/1.5 × 100

= 633.33% (approx)

If distillery E had grown at this rate, it supply in 1998 would have been

2.5 × 6.33 = 15.8 lakh litres (approx).

Hence, option 2.

Group Question
Answer the following questions based on the information given below.

The graph below shows the market value of 4 shares at the end of each month in
a period from January to June.

4 of 23 9/16/2011 10:10 AM
Exam Reports http://testfunda.com/LMS/Student/NewReports.aspx

5. Which share showed the greatest percentage increase in market value


3 Marks
in any month during the entire period?

1) A
2) B
3) C
4) D

Solution:
Observe that C grows with a very grentle slope throughout the period
and so can be ruled out.

The three possible options are : D (between Feb and Mar), B (between
Mar and Apr) and A (between Jan and Feb).

A increased from 40 to 50 between Jan and Feb. This is a growth of


25%.

B increased from 55 to 60 between Feb and Mar and from 60 to 65


between Mar and Apr. The first is a growth of 9.09% while the second is
a growth of 8.33%.

D increased from 95 to 115 between Feb and Mar. This is a growth of


approximately 17%.

Thus, A showed the maximum percentage increase.

Hence, option 1.

6. In which month was the greatest absolute change in market value for

5 of 23 9/16/2011 10:10 AM
Exam Reports http://testfunda.com/LMS/Student/NewReports.aspx

3 Marks
any share recorded?

1) March
2) April
3) May
4) June

Solution:
The market value of D in the month of March increased to 115 from 95 in
the month of February.

There is no other absolute change greater than this during the given
period.

Hence, option 1.

7. In which month was the greatest percentage increase in market value


3 Marks
for any share recorded?

1) February
2) March
3) April
4) May

Solution:
The market value of A increased from 40 to 50 in February.

This is the highest percentage increase in the given period.

Hence, option 1.

8. An individual wishes to sell 1 share of C and 1 share of D to buy 1 share


3 Marks
of A at the end of a month. At the end of which month would the
individual’s profit/loss from this decision, due to share value changes,
be the most?

1) February
2) March

6 of 23 9/16/2011 10:10 AM
Exam Reports http://testfunda.com/LMS/Student/NewReports.aspx

3) April
4) June

Solution:
The profit/loss in the month of February = 90 + 72 – 50 = 112

The profit/loss in the month of March = 115 + 74 – 50 = 139

The profit/loss in the month of April = 105 + 75 – 40 = 140

The profit/loss in the month of June = 110 + 80 – 45 = 145

Thus, among the options given, the maximum profit was made in the
month of June.

Hence, option 4.

9. An individual decides to sell 1 share of A and 1 share of B to buy 1


3 Marks
share of D at the end of the month. What can be the individual’s
greatest gain from this decision, due to share value changes?

1) 5
2) 10
3) 15
4) None

Solution:
The person earns money on shares A and B and spends money on
share D.

The gain/loss in the month of Jan = 40 + 60 – 100 = 0

The gain/loss in the month of Feb = 50 + 55 – 95 = 10

The gain/loss in the month of Mar = 50 + 60 – 115 = –5

The gain/loss in the month of April = 40 + 65 – 105 = 0

The gain/loss in the month of May = 45 + 60 – 100 = 5

The gain/loss in the month of June = 45 + 55 – 110 = –10

Thus, the maximum possible gain can be 10.

7 of 23 9/16/2011 10:10 AM
Exam Reports http://testfunda.com/LMS/Student/NewReports.aspx

Hence, option 2.

Group Question
Answer the following questions based on the information given below.

Study the following graph and answer questions that follow. The x-axis denotes
the years from 1983 to 1991.

10. The sum of food and fertilizer production has shown a constant value
3 Marks
for how many years at a stretch?

1) None of the years


2) 3
3) 4
4) 5

Solution:

8 of 23 9/16/2011 10:10 AM
Exam Reports http://testfunda.com/LMS/Student/NewReports.aspx

The sum of food and fertilizer production has been constant at 8.5
units from 1984 to 1988 i.e.5 consecutive years and then at 8 units from
1990 to 1991 i.e. 2 consecutive years.

2 years is not given in any of the options.

Thus, the sum has shown a constant value for 5 years at a stretch.

Hence, option 4.

11. If in 1988, the sum of the food and fertilizer production was 170 million
3 Marks
tons, the value of food production must have been (approximately, in
million tons) _____.

1) 90
2) 70
3) 100
4) Insufficient data

Solution:
The sum of food and fertilizer production in 1988 as per the graph is 8.5
units, and food production contributes 5 units to this.

The actual total production is 170 million tons.

Thus, the food production should be

Hence, option 3.

12. From its apparent behaviour, the food production in year 1992 can be
3 Marks
expected to …

1) go up.
2) go down.
3) remain the same as in the previous year.
4) Nothing can be said.

Solution:
The food production follows the pattern shown below:

9 of 23 9/16/2011 10:10 AM
Exam Reports http://testfunda.com/LMS/Student/NewReports.aspx

Decrease - Constant - Increase - Constant - Decrease - Constant, post


which it repeats.

From 89 to 91, the pattern has been Increase-Constant.

Hence, as per the pattern, the food production in 1992 should go down.

Hence, option 2.

13. Going by the previous trends, one can say that fertilizer production has
3 Marks
shown an anomalous behaviour in which year?

1) 1985
2) 1984
3) 1991
4) 1989

Solution:
Observe that the fertilizer production was the same in 1984 and 1985.

Similarly, the fertiliser production in 1986 is the same as the production


in 1987. The same is observed even in 1990 and 1991.

Also, the value in each set of 2 years described above is different.

Going by this logic, the fertilizer production in 1989 and 1988 should
have been the same and it should have been different from the 1986-87
period as well as the 1990-91 period.

Hence, as per the previous trends, the anomalous behaviour is in 1989.

Hence, option 4.

14. A scholar observed that if the production of fertilizers in 1989 had been
3 Marks
the same as that in 1988, then the total fertilizer production for all the
given years would have been 450 million tons. Using this information,
and knowing that the food production has been plotted on the same
scale, one may say that the food production in 1983 was _____
(approximately, in million tons).

1) 80
2) 130
3) 105

10 of 23 9/16/2011 10:10 AM
Exam Reports http://testfunda.com/LMS/Student/NewReports.aspx

4) Indeterminate

Solution:
Based on the condition given, the fertilizer production in 1989 should
have been 3.5 units

The total production of fertilizer = 2.5 + 3.5 + 3.5 + 2 + 2 + 3.5 + 3.5 + 1


+ 1 = 22.5 units

The actual total production of fertilizers = 450 million tons.

The production of food in 1983 = 6.5 units.

Therefore, the actual food production = (6.5 × 450)/22.5

= 130 million tonnes

Hence, option 2.

Group Question
Answer the following questions based on the information given below.

The following table shows the number of households in a country during the
period from 1970 to 1990. The subsequent pie-charts show the distribution of
households for the years 1970 and 1990 based on the number of children.

11 of 23 9/16/2011 10:10 AM
Exam Reports http://testfunda.com/LMS/Student/NewReports.aspx

15. Which of the following statements about the households is true?


3 Marks

1) There were more households with children in 1970 than in


1990.
2) There were more households with 3 or more children in 1970
than in 1990.
3) The number of households with two children decreased
slightly from 1970 to 1990.
4) None of the above

12 of 23 9/16/2011 10:10 AM
Exam Reports http://testfunda.com/LMS/Student/NewReports.aspx

Solution:
Statement 1:

The number of households with children in 1970 = 65% of 120 = 78


million

The number of households with children in 1990 = 60% of 160 = 96


million

Thus, there were more households with children in 1990 than in 1970.

Thus, statement 1 is false.

Statement 2:

The number of households with 3 or more children in 1970 = 25% of


120 = 30 million

The number of households with 3 or more children in 1990 = 20% of


160 = 32 million

Thus, there were more households with 3 or more children in 1990 than
in 1970.

Thus, statement 2 is false.

Statement 3:

The number of households with 2 children in 1970 = 20% of 120 = 24


million

The number of households with 2 children in 1990 = 18% of 160 = 28.8


million

Thus, the number of households with 2 children increased from 1970 to


1990.

Thus, statement 3 is false.

Thus, none of the statements are true.

Hence,option 4.

16. The average number of children per household in 1990 was _____.
3 Marks

1) 1.18

13 of 23 9/16/2011 10:10 AM
Exam Reports http://testfunda.com/LMS/Student/NewReports.aspx

2) 1.35
3) 2.0
4) Indeterminate

Solution:
This question could have been answered using weighted averages, had
the last category of the pie chart been “3 children”.

Since the category is “more than 3 children”, the maximum number of


children in a family can be anything.

Hence, the average number of children per household cannot be found.

Hence, option 4.

17. The simple annual growth rate of the number of households from 1970
3 Marks
to 1990 was_____.

1) 1.52%
2) 1.65%
3) 2.10%
4) 3.00%

Solution:
The number of households in 1970 and 1990 is 120 and 160 million
respectively.

Hence, option 2.

18. Assuming that the average number of children per household in 1970
3 Marks
was 3, how many households were without children in 1970?

1) 24 million
2) 42 million
3) 14 million
4) Indeterminate

14 of 23 9/16/2011 10:10 AM
Exam Reports http://testfunda.com/LMS/Student/NewReports.aspx

Solution:
The number of households that were without children is independent of
the average number of children.

In 1970, the number of households without children is 35% of 120, i.e.


42 million.

Hence, option 2.

Group Question
Answer the questions based on the following graph.

19. In which year is the profit per rupee of equity the highest?
3 Marks

1) 1991
2) 1992
3) 1993
4) 1990

Solution:

15 of 23 9/16/2011 10:10 AM
Exam Reports http://testfunda.com/LMS/Student/NewReports.aspx

The values of the sales, expenditure, profit and equity can be plotted as
shown in the first table above.

The profit per rupee is as calculated in the second table above.

From the table, it is clear that the profit per rupee of equity was the
highest in the year 1991.

Hence, option 1.

Alternatively,

Observe that the profit is less than the equity in each given year.

Thus, the profit per rupee of equity will be highest where the profit is as
close as possible to the equity figure.

The profit in 1991 is very close to the corresponding equity figure.

Hence, the profit per rupee is highest in 1991.

Hence, option 1.

20. In which year is the sales per rupee of equity the highest?
3 Marks

1) 1990
2) 1991
3) 1992
4) 1994

Solution:
Refer to the first table in the solution to the first question.

16 of 23 9/16/2011 10:10 AM
Exam Reports http://testfunda.com/LMS/Student/NewReports.aspx

The sales per equity for each year is shown below:

1990: 80/10 = 8

1991: 92/10 = 9.2

1992: 106/25 = 4.24

1993: 128/25 = 5.12

Thus, the sales per rupee of equity is the highest in 1991.

Hence, option 2.

21. The simple annual growth rate in sales was the highest between the
3 Marks
years:

1) 1990-91
2) 1991-92
3) 1992-93
4) 1990-92

Solution:
The growth rate for sales in 1991

The growth rate for sales in 1992

The growth rate for sales in 1993

Thus, the simple annual growth rate was the highest between 1992 and
1993.

Hence, option 3.

Group Question

17 of 23 9/16/2011 10:10 AM
Exam Reports http://testfunda.com/LMS/Student/NewReports.aspx

Answer the questions based on the following graph.

Note: The Y-axis denotes Solubility in kg/litre of water.

22. Which of the following salts has the greatest solubility?


3 Marks

1) Potassium Chlorate at 80°C


2) Potassium Chloride at 35°C
3) Potassium Nitrate at 39°C
4) Sodium Chloride at 85°C

Solution:
The solubility of each salt under the given conditions is:

Potassium Chlorate at 80°C = 0.4 kg/litre (approx)

Potassium Chloride at 35°C = 0.4 kg/litre (approx)

Potassium Nitrate at 39°C = 0.45 kg/litre (approx)

18 of 23 9/16/2011 10:10 AM
Exam Reports http://testfunda.com/LMS/Student/NewReports.aspx

Sodium Chloride at 85°C = 0.4 kg/litre

Thus, Potassium Nitrate at 39°C has the highest solubility.

Hence, option 3.

23. Approximately how many kg of Potassium Nitrate can be dissolved in 10


3 Marks
litres of water at 30°C?

1) 0.04
2) 0.4
3) 4
4) 0.35

Solution:
At 30°C, the solubility of Potassium Nitrate is approximately 0.36 kg/litre
of water.

Therefore, in 10 litres of water, the amount of soluble Potassium Nitrate


is 0.36 × 10 = 3.6 kg

4 kg is the closest answer.

Hence, option 3.

24. By what percent is the solubility of Potassium Chlorate in water


3 Marks
increased as the water is heated from 30°C to 80°C?

1) 100
2) 200
3) 250
4) 150

Solution:
At 30°C, solubility of Potassium Chlorate = 0.15 kg/litre (approx)

At 80°C, the solubility of Potassium Chlorate = 0.4 kg/litre

∴ The percentage increase = (0.25/0.15) × 100 = 167% (approx).

Hence, option 4.

19 of 23 9/16/2011 10:10 AM
Exam Reports http://testfunda.com/LMS/Student/NewReports.aspx

25. If 1 mole of Potassium Chloride weighs 0.07456 kg, approximately, how


3 Marks
many moles of Potassium Chloride can be dissolved in 100 litres of
water at 36°C?

1) 700
2) 650
3) 480
4) 540

Solution:
At 36°C, the solubility of Potassium Chloride is 0.4 kg/litre (approximate)

So, in 100 litres of water, the amount of soluble Potassium Chloride


is 0.4 × 100 = 40 kg

1 mole weighs 0.07456 kg.

Hence, the required number of moles is 40/0.07456 i.e. 536.5 ≈ 540


moles.

Hence, option 4.

26. Which of the salts has the greatest change in solubility in kg/litre of
3 Marks
water between 15°C and 25°C?

1) Potassium Chlorate
2) Potassium Nitrate
3) Sodium Chlorate
4) Sodium Nitrate

Solution:
This can be directly solved by observation.

The salt that has the steepest curve between 15°C and 25°C between
those two temperatures will have the greatest change in solubility.

It is clear from the curve that, among the options given, Sodium
Chlorate has the steepest curve and hence, the greatest change in
solubility.

Hence, option 3.

20 of 23 9/16/2011 10:10 AM
Exam Reports http://testfunda.com/LMS/Student/NewReports.aspx

Group Question
Answer the following questions based on the information given below.

Refer to the pie-charts given below and answer the questions that follow.

27. What fraction of Ghosh Babu’s weight consists of muscular and skin
3 Marks
protein?

1)

2)

21 of 23 9/16/2011 10:10 AM
Exam Reports http://testfunda.com/LMS/Student/NewReports.aspx

3)

4) Indeterminate

Solution:
15% of Ghosh Babu's body consists of protein.

Out of this, (8 + 25 = 33%) comes from skin and muscular protein.

So, the required fraction is 33% of 15% of the body weight

i.e. 0.33 × 0.15 × Body Wieght

i.e. 0.05 × Body Wieght

Thus, 5% (or 1/20) of Ghosh Babu's weight consists of muscular and


skin protein.

Hence, option 3.

28. The ratio of distribution of protein in muscle to the distribution of protein


3 Marks
in skin is _____.

1) 3:1
2) 3 : 10
3) 1:3
4) 3.5 : 1

Solution:
The ratio of the distribution of protein in muscle to the protein
distribution in skin = 25% of total protein : 8% of total protein

i.e. 25 : 8 or 3 : 1 (approx).

Hence, option 1.

29. What percent of Ghosh Babu’s body weight is made up of skin?


3 Marks

1) 0.15
2) 10
3) 1.2

22 of 23 9/16/2011 10:10 AM
Exam Reports http://testfunda.com/LMS/Student/NewReports.aspx

4) Indeterminate

Solution:
The occurrence of protein in the skin is given.

But the weight of the skin is not known.

Hence, the skin percent in Ghosh Babu's weight cannot be found.

Hence, option 4.

30. In terms of total body weight, the proportion of material other than water
3 Marks
and protein is closest to _____.

1)

2)

3)

4)

Solution:
The percentage of material other than water and protein in the total
body weight is 15%.

So, the fraction of such material in the total body weight

Hence, option 1.

23 of 23 9/16/2011 10:10 AM
Exam Reports http://testfunda.com/LMS/Student/NewReports.aspx

Section I

Group Question

Answer the questions based on the following graph.

The bar graph shows the breakup of engineering students in different categories of
engineering colleges in India. the number of students given in the graph is in thousands.

1. What was the total number of engineering students in 1989-90?


3 Marks

1) 28500
2) 4400
3) 4200
4) 420000

Solution:

The number of students in each category of college for each year can be as
shown in the table above.

1 of 18 9/16/2011 10:12 AM
Exam Reports http://testfunda.com/LMS/Student/NewReports.aspx

As can be seen from the table, the total number of engineering students in
1989-90 is 415000.

The closest answer option is 420000.

Hence, option 4.

2. The growth rate of students of Government Engineering Colleges is _____


3 Marks
compared to that of Private Engineering Colleges between 1988-89 and
1989-90.

1) more
2) less
3) equal
4)

Solution:
The growth rate of Government Engineering College students between the
years 1988-89 and 1989-90 is

The growth rate of Private Engineering College students between the years
1988-89 and 1989-90 is

Thus, the two growth rates are equal.

Hence, option 3.

3. The total number of Engineering Students in 1991-92, assuming a 10%


3 Marks
reduction in the number over the previous year, is _____.

1) 5700
2) 57000
3) 44800
4) None of these

Solution:
The total number of engineering students in 1990-91 is 560000.

2 of 18 9/16/2011 10:12 AM
Exam Reports http://testfunda.com/LMS/Student/NewReports.aspx

If there is a decrease of 10% in the next year, then the total number of
engineering students in the year 1991-92

= 0.9 × 560000

= 504000

Hence, option 4.

4. In 1990-91, what percent of Engineering Students were studying at the IITs?


3 Marks

1) 16
2) 15
3) 14
4) 12

Solution:
The percentage of engineering students that were studying in the IITs in
1990-91

Hence, option 3.

Group Question

Answer the following questions based on the information given below.

Refer to the following pie chart and answer the questions that follow.

3 of 18 9/16/2011 10:12 AM
Exam Reports http://testfunda.com/LMS/Student/NewReports.aspx

5. The operation profit in 1991-92 increased over that in 1990-91 by _____.


3 Marks

4 of 18 9/16/2011 10:12 AM
Exam Reports http://testfunda.com/LMS/Student/NewReports.aspx

1) 23%
2) 22%
3) 25%
4) 24%

Solution:
The percentage increase in the operation profit in 1991-92 over 1990-91

Hence, option 1.

6. The interest burden in 1991-92 was higher than that in 1990-91 by _____.
3 Marks

1) 50%
2) Rs. 25 lakhs
3) 90%
4) More than one of the above

Solution:
The interest in the year 1990-91 = 30% of 130 = Rs. 39 lakhs.

The interest in the year 1991-92 = 40% of 160 = Rs. 64 lakhs.

The difference in the interests = 64 – 39 = Rs. 25 lakhs.

Thus, the interest burden was higher by Rs. 25 lakhs.

This value when expressed as a percentage is (25/39) × 100 which is more than
50%.

Hence, option 2.

7. If, on an average, a 20% rate of interest was charged on borrowed funds, then
3 Marks
the total borrowed funds used by this company in the given two years amounted
to _____.

1) Rs. 221 lakhs


2) Rs. 195 lakhs
3) Rs. 368 lakhs
4) Rs. 515 lakhs

5 of 18 9/16/2011 10:12 AM
Exam Reports http://testfunda.com/LMS/Student/NewReports.aspx

Solution:
The interest in the year 1990-91 = 30% of 130 = Rs. 39 lakhs.

The interest in the year 1991-92 = 40% of 160 = Rs. 64 lakhs.

Thus, the total interest amount paid = 39 + 64 = Rs. 103 lakhs.

It is given that 103 lakhs is 20% of the total borrowed funds.

Hence, option 4.

8. The retained profit in 1991-92, as compared to that in 1990-91 was _____.


3 Marks

1) higher by 2.5%
2) higher by 1.5%
3) lower by 2.5%
4) lower by 1.5%

Solution:

Hence, option 4.

9. The equity base of these companies remained unchanged. Then the total
3 Marks
dividend earning (lakh rupees) by the share holders in 1991-92 was _____.

1) Rs. 104 lakhs


2) Rs. 9 lakhs
3) Rs. 12.8 lakhs
4) Rs. 15.6 lakhs

Solution:
The total dividend earning by the share holders in 1991-92 is 8% of 160 lakhs,
i.e. 12.8 lakhs.

6 of 18 9/16/2011 10:12 AM
Exam Reports http://testfunda.com/LMS/Student/NewReports.aspx

Hence, option 3.

Group Question

Answer the following questions based on the information given below.

The bar graph below shows the revenue obtained by a publishing house by selling books,
magazines and journals (in Rs. lakhs).

10. Which year shows the highest change in revenue obtained from journals?
3 Marks

1) 1989
2) 1990
3) 1991
4) 1992

Solution:

The revenue obtained from each type of publication in each year is as shown in
the table above.

The change in revenue from journals for each year is +1, −2 and −1.

Thus, the highest change in revenue obtained from journals is in 1991.

Hence, option 3.

7 of 18 9/16/2011 10:12 AM
Exam Reports http://testfunda.com/LMS/Student/NewReports.aspx

11. In 1992, what percent of the total revenue came from books?
3 Marks

1) 45
2) 55
3) 35
4) 25

Solution:
The total revenue in 1992 is Rs. 173 lakhs and the revenue from books is Rs. 79
lakhs.

Hence, the required percentage

Hence, option 1.

12. The number of years in which there was an increase in revenue from at least
3 Marks
two categories is _____.

1) 1
2) 2
3) 3
4) 4

Solution:
For each year, the number of categories from which there was an increase in
revenue is:

1990: 3 (All categories)

1991: 2 (magazines and books)

1992: 1 (magazines)

Thus, in two different years, there was an increase in revenue from at least two
categories.

Hence, option 2.

13. If 1993 were to show the same percentage growth as 1992 over 1991, the
3 Marks
revenue in 1993 must be _____.

1) Rs. 194 lakhs


2) Rs. 187 lakhs

8 of 18 9/16/2011 10:12 AM
Exam Reports http://testfunda.com/LMS/Student/NewReports.aspx

3) Rs. 172 lakhs


4) Rs. 177 lakhs

Solution:
The revenue in 1991 and 1992 is Rs. 169 lakhs and Rs. 173 lakhs respectively
i.e. a growth of Rs. 4 lakhs.

The percentage increase in 1992 over 1991

1993 has to show the same percentage growth in revenue.

So, the total revenue in 1993 = 1.0231 × 173 = Rs. 177 lakhs

Hence, option 4.

14. The growth in total revenue from 1989 to 1992 is _____.


3 Marks

1) 21%
2) 28%
3) 15%
4) 11%

Solution:
The total revenue in the years 1989 and 1992 is Rs. 150 lakhs and Rs. 173
lakhs respectively.

So, the required percentage growth

Hence, option 3.

Group Question

Answer the questions based on the following graph.

A manufacturer can choose from any of the three types of tests available for checking the
quality of his product. The graph that follows gives the relative costs for each of these
tests for a given percentage of defective pieces (p).

9 of 18 9/16/2011 10:12 AM
Exam Reports http://testfunda.com/LMS/Student/NewReports.aspx

15. If p is equal to 0.2, then which test will be feasible?


3 Marks

1) Either 1 or 2
2) 2 only
3) 3 only
4) Either 2 or 3

Solution:
From the graph, it can be seen that for p = 0.2, the relative cost is the same for
test 2 and 3 and is the least.

Hence, either one of test 2 or 3 is feasible.

Hence, option 4.

16. When will Test-3 be feasible?


3 Marks

1) p > 0.2
2) 0.1 < p < 0.2
3) 0.05 < p < 0.2
4) p < 0.05

Solution:
From the graph, it is clear that test 3 will be feasible only when p > 0.2.

Hence, option 1.

17. Adopting Test-2 will be feasible if the percentage of defective pieces ‘p’ lies in
3 Marks
the range :

10 of 18 9/16/2011 10:12 AM
Exam Reports http://testfunda.com/LMS/Student/NewReports.aspx

1) 0.10 to 0.20
2) 0.20 to 0.30
3) 0.05 to 0.20
4) 0.00 to 0.05

Solution:
From the graph, observe that test 2 is the cheapest in the range between p =
0.05 and p = 0.2

Thus, test 2 is feasible when 0.05 < p < 0.2.

Hence, option 3.

18. When is Test-1 feasible?


3 Marks

1) p < 0.05
2) 0.0 < p < 0.2
3) 0.1 < p < 0.2
4) 0.05 to 0.2

Solution:
From the graph, observe that test 1 is feasible only when p < 0.05.

Hence, option 1.

19. If p < 0.2, then the best alternative will be _____.


3 Marks

1) Conduct Test-2
2) Conduct Test-3
3) Conduct Test-1
4) Do not conduct Test-3

Solution:
If p < 0.2, then either of test 1 or test 2 proves to be cheaper.

Hence, if p < 0.2, it is best not to conduct test 3.

Hence, option 4.

Group Question

Answer the questions based on the following graph.

11 of 18 9/16/2011 10:12 AM
Exam Reports http://testfunda.com/LMS/Student/NewReports.aspx

All figures are in Rs. lakhs

20. The average revenue collected in the given period is approximately______.


3 Marks

1) Rs. 164 lakhs


2) Rs. 168 lakhs
3) Rs. 171 lakhs
4) Rs. 175 lakhs

Solution:

The revenue, expenditure and profit for each year can be plotted in a single table
as shown above.

The total revenue in the given period = 120 + 135 + 145 + 165 + 185 + 200 + 220
= Rs. 1170 lakhs

Hence, option 2.

21. The total expenditure for the given period forms what percent of the revenues
3 Marks
during the same period?

1) 75%
2) 67%
3) 62%

12 of 18 9/16/2011 10:12 AM
Exam Reports http://testfunda.com/LMS/Student/NewReports.aspx

4) 83%

Solution:
From the solution to the first question, the total revenue for the given period = Rs.
1170 lakhs.

The total expenditure for the given period = 100 + 110 + 115 + 125 + 135 + 140 +
150 = Rs. 875 lakhs

Hence, option 1.

22. Which year showed the greatest percentage increase in profit as compared to the
3 Marks
previous year?

1) 1993
2) 1994
3) 1990
4) 1992

Solution:
The percentage increase in profit in the years 1990, 1992, 1993 and 1994 is

1990: (5/20) × 100 = 25%

1992: (10/30) × 100 = 33.33%

1993: (10/40) × 100 = 25%

1994: (10/50) × 100 = 20%

Thus, 1992 showed the greatest percentage increase.

Hence, option 4.

23. In which year was the growth in expenditure as compared to the previous year the
3 Marks
maximum?

1) 1993
2) 1995
3) 1990
4) 1992

13 of 18 9/16/2011 10:12 AM
Exam Reports http://testfunda.com/LMS/Student/NewReports.aspx

Solution:
Consider the years mentioned in the answer options.

The absolute growth in expenditure in each of these years is Rs. 10 lakhs.

In such a case, the growth in expenditure is maximum for the fraction that has the
lowest denominator i.e. the inital year which has the least expenditure.

The least initial value of expenditure is in 1989 (Rs. 100 lakhs) and so
the greatest percentage growth in expenditure is 10%.

Hence, option 3.

24. If the profit in 1996 shows the annual rate of growth that it had shown in 1995 over
3 Marks
the previous year, then what approximately what will be the profit in 1996?

1) Rs. 72 lakhs
2) Rs. 82 lakhs
3) Rs. 93 lakhs
4) Rs. 78 lakhs

Solution:

If the profit in 1996 shows the same growth rate, the profit in 1996 = 1.167 × 70 = Rs.
81.69 lakhs

Hence, option 2.

Group Question
Answer the following questions based on the information given below.

14 of 18 9/16/2011 10:12 AM
Exam Reports http://testfunda.com/LMS/Student/NewReports.aspx

25. In which year was the increase in raw material the maximum?
3 Marks

1) 1992
2) 1993
3) 1994
4) 1995

Solution:

The actual values under each head are shown in the table above.

The amount of raw-material in the years 1991, 1992, 1993, 1994 and 1995 is
respectively 60, 50, 65, 75 and 80 units.

Thus, the maximum increase in the amount of raw material is in 1993.

Hence, option 2.

26. For which period was the change in profit the maximum?
3 Marks

1) 1991-92
2) 1992-93
3) 1993-94
4) 1994-95

15 of 18 9/16/2011 10:12 AM
Exam Reports http://testfunda.com/LMS/Student/NewReports.aspx

Solution:
The change in profit is the maximum in the period 1993 to 1994, because the
profit in 1993 was 20 units which reduced to –30 units in 1994.

Thus, the profit changed by 20 − (−30) = 50 units.

Hence, option 3.

27. Which component of the cost of production has remained more or less constant
3 Marks
over the period?

1) Interest
2) Overheads
3) Wages
4) Raw material

Solution:
This question can be solved directly by observation.

From the graph, observe that only the interest component remains more or less
constant throughout the given period.

Hence, option 1.

28. In which year were the overheads, as a percentage of the raw material, the
3 Marks
maximum?

1) 1995
2) 1994
3) 1992
4) 1993

Solution:
The overheads, as a percentage of the raw material, for the years given in the
options is :

1992 : (20/50) × 100 = 40%

1993 : (15/65) × 100 = 23.07%

1994 : (25/75) × 100 = 33.33%

1995 : (20/80) × 100 = 25%

Thus, the overheads as a percentage of the raw material were the highest in
1992.

16 of 18 9/16/2011 10:12 AM
Exam Reports http://testfunda.com/LMS/Student/NewReports.aspx

Hence, option 3.

29. Over the period, the profits formed what approximate percent of the total cost?
3 Marks

1) 3%
2) 5%
3) 8%
4) 11%

Solution:
From 1991 to 1995, the total amount spent on raw-materials, wages, overheads,
interest is respectively 330, 290, 90 and 260 units.

Thus, the total cost in the given period is 330 + 290 + 90 + 260 = 970 units.

In the same period, the total profit is 45 units.

Thus, the required percentage is (45/970) × 100 = 4.64% ≈ 5%.

Hence, option 2.

30. If the interest component is not included in the total cost calculation, which year
3 Marks
would show the maximum profit per unit cost?

1) 1991
2) 1992
3) 1993
4) 1995

Solution:
Excluding the interest component, the total cost in 1991, 1992, 1993, 1994 and
1995 is respectively 115, 125, 140, 165 and 165 units.

The profit in 1991, 1992, 1993, 1994 and 1995 is respectively 15, 25, 20, –30
and 15 units.

Start with the simplest calculation:

The profit in 1992 is (25/125) × 100 i.e. 20% or 1/5th.

For all the other years, the same ratio is less than 1/5.

Hence, the maximum profit is seen in 1992.

Hence, option 2.

17 of 18 9/16/2011 10:12 AM
Exam Reports http://testfunda.com/LMS/Student/NewReports.aspx

18 of 18 9/16/2011 10:12 AM
Exam Reports http://testfunda.com/LMS/Student/NewReports.aspx

Section I

Group Question
Answer the questions based on the following graph.

1. What is the average value of the contract secured during the years shown in the
3 Marks
diagram?

1) Rs. 103.48 crores


2) Rs. 105 crores
3) Rs. 100 crores
4) Rs.125.2 crores

Solution:

Hence, option 1.

2. Compared to the performance in 1985 (i.e. taking it as the base), what can you
3 Marks
say about the performances in the years ’84, ’85, ’86, ’87, ’88 respectively, in
percentage terms?

1) 150, 100, 211, 216, 97


2) 100, 67, 141,144, 65
3) 150, 100, 200, 215, 100
4) 120, 100, 220, 230, 68

1 of 23 9/16/2011 9:55 AM
Exam Reports http://testfunda.com/LMS/Student/NewReports.aspx

Solution:
If you consider the performance in the year 1985 as the base, the performance
for the years 1984, 85, 86, 87 and 88 is respectively

Thus, the performance for the years 1984, 85, 86, 87 and 88 in percentage
terms is respectively 150, 100, 211, 216 and 97.

Hence, option 1.

Alternatively,

This question can also be solved by considering the answer options.

Consider option 2:

The values given for each year in option 2 indicate the actual performance for
these 5 years. The actual values are unrelated in the sense that they are not
calculated using the 1985 value as a base. When the performance in the year
1985 is taken as a base, it implies that given the current performance in any
year, what would have been the performance in that year had the performance
in the year 1985 been 100. Therefore, the values given in option 2 cannot be
the performance figures considering 1985 as the base year.

Hence, option 2 can be eliminated.

Consider option 4.

Since 1985 is the base year, the performance in 1985 is taken to be 100.

Observe that the actual performance in 1988 is marginally less than the actual
performance in 1985 (approximately 3% less).

However, as per the values given in option 4, the performance in 1988 is 2/3 of
the performance in 1985 (i.e. approximately 32% less).

Therefore, the values given in option 4 cannot be the required performance


figures.

Hence, option 4 can be eliminated.

Consider option 3.

As per the values in option 3, the performance in 1986 is exactly double the
performance in 1985.

2 of 23 9/16/2011 9:55 AM
Exam Reports http://testfunda.com/LMS/Student/NewReports.aspx

However, the actual performance is 141 in 1986 and 67 in 1985 i.e. more
than double.

Therefore, the values given in option 3 cannot be the required performance


figures.

Hence, option 3 can be eliminated.

Hence, option 1.

3. In which year is the highest percentage decline seen in the value of the
3 Marks
contracts secured compared to the preceding year?

1) 1985
2) 1988
3) 1984
4) 1986

Solution:
Since the question mentions percentage decline and not percentage change,
all the years mentioned need not be considered.

Observe that, with respect to the previous year, the value of the contracts
secured decreases only in 1985 and in 1988.

Therefore, they can be the only possible options.

Hence, options 3 and 4 can be eliminated.

The percentage decline in 1985 = [(100.5 − 67)/100.5] × 100

= 33.33%

The percentage decline in 1988 = [(143.9 − 65)/143.9] × 100

= 55% (approximately)

Thus, the highest percentage decline is in 1988.

Hence, option 2.

4. Each question is followed by two statements, A and B. Select the correct option
3 Marks based on the following instructions:

Mark (1) if the question can be answered by one of the statements, but not by the
other.
Mark (2) if the question can be answered by using either statement, independently
of the other.

3 of 23 9/16/2011 9:55 AM
Exam Reports http://testfunda.com/LMS/Student/NewReports.aspx

Mark (3) if the question can be answered by using both statements together, but
not by either statement alone.
Mark (4) if the question cannot be answered by either of the statements.
What is the distance x between two cities A and B (in km)? x is an integer.

B. x ≤ 10 km

1) 1
2) 2
3) 3
4) 4

Solution:
Using Statement A alone:

Thus, a unique value of x cannot be found.

Thus, the question cannot be answered using statement A alone.

Using Statement B alone:

Assuming that A and B are two different cities, x can take any value from 1 to 9.

Thus, a unique value of x cannot be found.

Thus, the question cannot be answered using statement B alone.

Using both the statements together:

When both the statements are combined, the only valid value of x is x = 4.

Thus, the distance between the two cities is 4 km.

Thus, the question can be answered using both the statements together but not by
using either statement alone.

Hence, option 3.

Group Question

Answer the following questions based on the information given below.

4 of 23 9/16/2011 9:55 AM
Exam Reports http://testfunda.com/LMS/Student/NewReports.aspx

Prakash has to decide whether or not to test a batch of 1000 widgets before sending
them to the buyer. In case he decides to test, he has two options: (1) Use test I; (2) Use
test II. Test I costs Rs. 2 per widget. However, the test is not perfect. It sends 20% of
the bad widgets to the buyer as good. Test II costs Rs. 3 per widget. It identifies all the
bad widgets. A defective widget identified before sending can be corrected at a cost of
Rs. 25 per widget. All defective widgets are identified at the buyer’s end and a penalty
of Rs. 50 per defective widget has to be paid by Prakash.

5. Prakash should not test if the number of bad widgets in the lot …
3 Marks

1) is less than 100.


2) is more than 200.
3) is between 120 and 190.
4) Indeterminate

Solution:
The total number of widgets = 1000

Let the total number of bad widgets be x.

The charges incurred under test I:

Testing charges = 1000 × 2 = Rs. 2,000

Penalty = 0.2x × 50 = Rs. 10x

Repair charges = 0.8x × 25 = Rs. 20x

∴ Total cost incurred under test I = 2000 + 30x

The charges incurred under test II

Testing charges = 1000 × 3 = Rs. 3,000

Repair charges = Rs. 25x

∴ Total cost incurred under test II = 3000 + 25x

When there is no test conducted:

If there is no test, then the only cost incurred is the penalty at the buyer's
end.

The penalty charged at buyer’s end = x × 50 = Rs. 50x

It is beneficial not to conduct a test when the cost without the test is less than
the cost with the test.

Consider Test I:

5 of 23 9/16/2011 9:55 AM
Exam Reports http://testfunda.com/LMS/Student/NewReports.aspx

i.e. 50x < 2000 + 30x

∴ 20x < 2000

∴ x < 100

Thus, when the number of bad widgets is less than 100, it is better to not
conduct a test than to conduct test I.

Consider Test II:

i.e. 50x < 3000 + 25x

∴ 25x < 3000

∴ x < 120

Thus, when the number of bad widgets is less than 120, it is better to not
conduct a test than to conduct test II.

Thus, in any case, when the number of bad widgets is less than 100, it is
better not to conduct a test.

Hence, option 1.

6. If there are 120 defective widgets in the lot, Prakash …


3 Marks

1) should either use Test I or not test at all.


2) should either use Test II or not test at all.
3) should use either Test I or Test II.
4) should use Test I only.

Solution:
The total number of bad widgets i.e. x = 120

Referring to the solution in the first question,

Total cost incurred under test I = 2000 + 30x = 2000 + (30 × 120) = Rs. 5,600

Total cost incurred under test II = 3000 + 25x = 3000 + (25 × 120) = Rs. 6,000

If there is no test, then the penalty charged at buyer’s end = 50 × 120 = Rs.
6,000

Since test I is the cheapest, Prakash should use test I only.

Hence, option 4.

6 of 23 9/16/2011 9:55 AM
Exam Reports http://testfunda.com/LMS/Student/NewReports.aspx

7. If the number of defective widgets in the lot is between 200 and 400, Prakash
3 Marks

1) may use Test I or Test II.


2) should use Test I only.
3) should use Test II only.
4) cannot decide.

Solution:
Let the total number of bad widgets be x.

Find the number of defective widgets for which one test is cheaper than the
other.

Total cost of Test I: 2000 + 30x

Total cost of Test II: 3000 + 25x

First find the value of x for which test II is cheaper than test I.

∴ 2000 + 30x > 3000 + 25x

∴ 5x > 1000

∴ x > 200

Thus, for any number of defective widgets greater than 200, it would be
cheaper to go for test II than test I.

Now find the value of x for which test I is cheaper than test II.

∴ 2000 + 30x < 3000 + 25x

∴ 5x < 1000

∴ x < 200

Thus, for any number of defective widgets less than 200, it would be cheaper
to go for test I than test II.

Hence, option 3.

8. If Prakash is told that the lot has 160 defective widgets, he should …
3 Marks

1) use Test I only.


2) use Test II only.

7 of 23 9/16/2011 9:55 AM
Exam Reports http://testfunda.com/LMS/Student/NewReports.aspx

3) do no testing.
4) either use Test I or do no test.

Solution:
The total number of bad widgets i.e. x = 160

Total cost incurred under test I = 2000 + 30x = 2000 + (30 × 160) = Rs. 6,800

Total cost incurred under test II = 3000 + 25x = 3000 + (25 × 160) = Rs. 7,000

If there is no test, then the penalty charged at buyer’s end = 50 × 160 = Rs.
8,000

Thus, test I is the least expensive and so Prakash should go for test I.

Hence, option 1.

9. If there are 200 defective widgets in the lot, Prakash …


3 Marks

1) may use either Test I or Test II.


2) should use Test I or not use any test.
3) should use Test II or not use any test.
4) cannot decide.

Solution:
The total number of bad widgets i.e. x = 200

Total cost incurred under test I = 2000 + 30x = 2000 + (30 × 200) = Rs. 8,000

Total cost incurred under test II = 3000 + 25x = 3000 + (25 × 200) = Rs. 8,000

If there is no test, then the penalty charged at buyer’s end = 50 × 200 = Rs.
10,000

Since the amount spent for test I or test II is the same, Prakash can
use either test I or test II.

Hence, option 1.

10. In a six-node network, two nodes are connected to all the other nodes. Of the
3 Marks
remaining four, each is connected to four nodes. What is the total number of links
in the network?

1) 13

8 of 23 9/16/2011 9:55 AM
Exam Reports http://testfunda.com/LMS/Student/NewReports.aspx

2) 15
3) 17
4) 26

Solution:
Let the 6 nodes be A, B, C, D, E and F.

2 nodes are connected to all the other nodes.

Let these 2 nodes be A and B.

Thus A is connected to C, D, E and F, forming 4 links i.e. AC, AD, AE and AF.

Similarly, B also forms four links through the nodes C, D, E and F i.e. BC, BD, BE
and BF.

Finally, A and B are connected to each other thereby forming a link AB (which is
the same as BA).

Thus, 4 + 4 + 1 i.e. 9 links are formed.

Now, each of the four nodes i.e. C, D, E and F is connected to four nodes.

For each of these nodes, two connections are with A and B each.

Thus, each node is connected to two other nodes.

Assume that C is connected to D and E, thus forming two links CD and CE.

Now, D is already connected to A, B and C (AD, BD and CD).

Thus, D can only be connected to one more node, say F, forming the link DF.

Similarly, E can now only be connected only to F, forming EF.

Finally, all the four links for F have already been formed i.e. AF, BF, EF and DF.

Thus, no new link for F can be formed.

Thus, the number of new links is 4 (CD, CE, DF and EF)

∴ The total number of links = 9 + 4 = 13 links

Hence, option 1.

Group Question

Answer the following questions based on the information given below.

9 of 23 9/16/2011 9:55 AM
Exam Reports http://testfunda.com/LMS/Student/NewReports.aspx

A company produces five types of shirts - A, B, C, D, E - using cloth of three qualities -


High, Medium and Low as well as using dyes of three qualities - High, Medium and
Low. One shirt requires exactly 1.5 m of cloth. The following table gives respectively:

1. The number of shirts (of each category) produced, in thousands.


2. The percentage distribution of cloth quality in each type of shirt.
3. The percentage distribution of dye quality in each type of shirt.

11. What is the total requirement of cloth?


3 Marks

1) 150000 m
2) 200000 m
3) 225000 m
4) 250000 m

Solution:
Total requirement of cloth = Total number of shirts × Cloth required per shirt

The total number of shirts produced (in thousands) = 20 + 30 + 30 + 10 + 10


= 100

Amount of cloth required per shirt = 1.5 m

∴ Total requirement of cloth = 100000 × 1.5 = 150000 m.

Hence, option 1.

12. How many metres of low-quality cloth are consumed?


3 Marks

1) 22500 m
2) 46500 m
3) 60000 m
4) 40000 m

Solution:
No Type A shirt uses low quality cloth.

10 of 23 9/16/2011 9:55 AM
Exam Reports http://testfunda.com/LMS/Student/NewReports.aspx

The total cloth required for Type B shirts is 30000 × 1.5 = 45000 m.

30% of this cloth is of low quality.

Hence, first find the total cloth required for each type of shirt.

The total cloth required for Type C, D and E shirts is 45000 m, 15000 m and
15000 m respectively.

Type C, D and E shirts use 30%, 40% and 90% low quality cloth respectively.

∴ Total amount of low quality cloth required = (0.3 × 45000) + (0.3 ×


45000) + (0.4 × 15000) + (0.9 × 15000)

= 13500 + 13500 + 6000 + 13500

= 46500 m.

Thus, 46500 metres of low quality cloth are consumed.

Hence, option 2.

13. How many metres of high quality cloth are consumed by Type A shirts?
3 Marks

1) 8000 m
2) 112000 m
3) 24000 m
4) 30000 m

Solution:
The number of Type A shirts made = 20000

∴ The total amount of cloth required for Type A shirts = 20000 × 1.5 = 30000

Each type A shirt uses 80% high quality cloth.

∴ The total amount of high quality cloth required for Type A shirts = 0.8 ×
30000 = 24000 m.

Thus, 24000 metres of high quality cloth are consumed by Type A shirts.

Hence, option 3.

14. What is the ratio of the three qualities of dyes in high-quality cloth?
3 Marks

1) 2:3:5

11 of 23 9/16/2011 9:55 AM
Exam Reports http://testfunda.com/LMS/Student/NewReports.aspx

2) 1:2:5
3) 7 : 9 : 10
4) Indeterminate

Solution:
The relationship between the proportion of dyes and the proportion of a
certain quality of cloth is not known.

One cannot assume that the proportion of dyes applicable to a certain shirt
type is also applicable to the cloth of different quality levels within the same
shirt type.

Hence, even though the total amount of cloth required and the proportion of
dyes used in each type of shirt can be found, the proportion of dyes in the
cloth of a certain quality within a specific shirt type cannot be calculated.

Hence, the required ratio cannot be found.

Hence, option 4.

15. What is the ratio of low-quality dye used for Type C shirts to low-quality
3 Marks
dye used for Type D shirts?

1) 3:2
2) 2:1
3) 1:2
4) 2:3

Solution:
Number of Type C shirts made = 30000

Each Type C shirt uses 40% low quality dye.

∴ Amount of low-quality dye required for Type C shirts = 0.4 × 30000 =


12000 units

Number of Type D shirts made = 10000

Each Type D shirt uses 60% low quality dye.

∴ Amount of low-quality dye required for Type D shirts = 0.6 × 10000 = 6000
units

Thus, the required ratio = 12000 : 6000 = 2 : 1

12 of 23 9/16/2011 9:55 AM
Exam Reports http://testfunda.com/LMS/Student/NewReports.aspx

Hence, option 2.

16. Each question is followed by two statements, A and B. Select the correct option
3 Marks based on the following instructions:

Mark (1) if the question can be answered by one of the statements, but not by the
other.
Mark (2) if the question can be answered by using either statement, independently
of the other.
Mark (3) if the question can be answered by using both statements together, but
not by either statement alone.
Mark (4) if the question cannot be answered by either of the statements.
What is the ratio of the volume of the given right circular cone to the one obtained
from it?

A. The smaller cone is obtained by passing a plane parallel to the base and dividing the
original height in the ratio 1 : 2.
B. The height and the base of the new cone are one-third those of the original cone.

1) 1
2) 2
3) 3
4) 4

Solution:
Using statement A alone:

Since the smaller cone is obtained by cutting the original cone in a plane parallel
to the base , the height, circumference as well as radius of the new cone are
proportional to the old cone.

∴ Height of new cone : Height of old cone = Radius of new cone : Radius of old
cone = 1 : 3

∴ Volume of new cone : Volume of old cone = (rnew : rold)3 = (1 : 3)3 = 1 : 27

Thus, the question can be answered using statement A alone.

Using statement B alone:

Since the base of the new cone is one-third of the original cone, the radius of the
new cone is also one-third of the original cone.

∴ Volume of new cone : Volume of old cone = (rnew : rold)3 = (1 : 3)3 = 1 : 27

Thus, the question can be answered using statement B alone.

13 of 23 9/16/2011 9:55 AM
Exam Reports http://testfunda.com/LMS/Student/NewReports.aspx

Thus, the question can be answered using any one statement independently.

Hence, option 2.

Group Question
Answer the following questions based on the information given below.

The questions that follow are based on the price fluctuations of four commodities -
Arhar, Pepper, Sugar and Gold - during the period February 1999 - July 1999 as shown
in the figures below. In the graphs, A2 implies the second week of April, JN2 implies the
second week of June and so on.

14 of 23 9/16/2011 9:55 AM
Exam Reports http://testfunda.com/LMS/Student/NewReports.aspx

17. “Price change” of a commodity is defined as the absolute difference in


3 Marks
the final and initial price of that commodity expressed as a percentage of the
first price of that commodity. Which commodity shows the highest price
change?

1) Arhar
2) Pepper
3) Sugar
4) Gold

Solution:
For Arhar: Initial price = Rs 1,700, Final Price = Rs 2,200.

∴ Price Change = [(2200 − 1700)/1700] × 100 = 500/17 ≈ 30%

For Pepper: Initial price = Rs 18,500, Final Price = Rs 19,400.

∴ Price Change = [(19400 − 18500)/18500] × 100 = 900/185 ≈ 5%

15 of 23 9/16/2011 9:55 AM
Exam Reports http://testfunda.com/LMS/Student/NewReports.aspx

For Sugar: Initial price = Rs 1,435, Final Price = Rs 1,410.

∴ Absolute difference in the final and initial price of sugar = Rs. 25

∴ Price Change = [25/1435] × 100 = 500/287 i.e. < 2%

For Gold: Initial price = Rs 4,250, Final Price = Rs 3,725.

∴ Absolute difference in the final and initial price of sugar = Rs. 525

∴ Price Change = [525/4250] × 100 = 210/17 ≈ 12.3%

Thus, Arhar shows the highest price change.

Hence, option 1.

18. Price volatility (PV) of a commodity is defined as follows:


3 Marks

What is the commodity with the lowest price volatility?

1) Arhar
2) Pepper
3) Sugar
4) Gold

Solution:

Thus, it can be seen that Sugar has the lowest PV.

Hence, option 3.

19. Mr. X, a funds manager with an investment company invested 25% of his
3 Marks
funds in each of the four commodities at the beginning of the period. He sold

16 of 23 9/16/2011 9:55 AM
Exam Reports http://testfunda.com/LMS/Student/NewReports.aspx

the commodities at the end of the period. His investments in the commodities
resulted in:

1) 17% profit
2) 5.5% loss
3) No profit, no loss
4) 4.3% profit

Solution:
Let the profit/loss percentage on Arhar, Pepper, Sugar and Gold be P1, P2,
P3 and P4 respectively.

If the person has invested W 1, W 2, W 3 and W 4 proportion of his investment


in Arhar, Pepper, Sugar and Gold respectively, the total profit/loss is nothing
but the weighted average of the individual profit/loss.

∴ Total Profit/Loss = (P1W 1 + P2W 2 + P3W 3 + P4W 4)/(W 1 + W 2 + W 3 + W 4)

Since Mr. X invested 25% of hits total investments in each of the four
commodities, W 1 = W 2 = W 3 = W 4 = 0.25

∴ Total Profit/Loss = 0.25 × (P1 + P2 + P3 + P4) = (P1 + P2 + P3 + P4)/4

For Arhar: P1 = [(2200 − 1700)/1700] × 100 = 29.4 % profit

For Pepper: P2 = [(19400 − 18500)/18500] × 100 = 4.9 % profit

For Sugar: P3 = [(1410 − 1435)/1435] × 100 = 1.7 % loss = −1.7 % profit

For Gold: P4 = [(3720 − 4250)/4250] × 100 = 12.5 % loss = −12.5 % profit

∴ Total Profit/Loss = (29.4 + 4.9 − 1.7 − 12.5)/4 = 20.1%/4 = 5%


(approximately)

Since the closest answer is given in option 4, that is marked as the correct
answer.

Hence, option 4.

20. The price volatility of the commodity with the highest PV during the
3 Marks
February-July period is approximately equal to:

1) 3%
2) 40%
3) 20%

17 of 23 9/16/2011 9:55 AM
Exam Reports http://testfunda.com/LMS/Student/NewReports.aspx

4) 12%

Solution:
Arhar had the highest PV during the Februrary-July period and the value of
this PV was approximately 40%.

Hence, option 2.

Group Question

Answer the following questions based on the information given below.

The primitive tribes on the island of Lexicophobos have recently developed a language
for themselves which has a very limited vocabulary. In fact, the words can be classified
into only three types: the “Bingoes”, the “Cingoes” and the “Dingoes”.

The Bingoes type of words are: Grumbs, Harrumphs, Ihavitoo


The Cingoes type of words are: Ihavitoo, Jingongo, Koolodo
The Dingoes type of words are: Lovitoo, Metoo, Nana

They have also devised some rules of grammar:

I. Every sentence must have only five words.


II. Every sentence must have two Bingoes, one Cingo and two Dingoes.
III. If Grumbs is used in a sentence, Ihavitoo must also be used and vice versa.
IV. Koolodo can be used in a sentence only if Lovitoo is used.

21. Which choice of words in a sentence is not possible, if no rules of grammar


3 Marks
are to be violated?

1) Grumbs and Harrumphs as the Bingoes and Ihavitoo as the Cingo.


2) Harrumphs and Ihavitoo as the Bingoes.
3) Grumbs and Ihavitoo as the Bingoes and Lovitoo and Nana as the
Dingoes.
4) Metoo and Nana as the Dingoes.

Solution:
Consider Option 1:

According to condition III. Grumbs and Ihavitoo have to be part of the same
sentence.

Ihavitoo is a Bingo as well as a Cingo.

The condition does not specify whether Ihavitoo is to be used as a Bingo or


as a Cingo.

18 of 23 9/16/2011 9:55 AM
Exam Reports http://testfunda.com/LMS/Student/NewReports.aspx

Thus, if Grumbs and Harrumphs are used as Bingoes and Ihavitoo is used as
a Cingo, no condition is violated.

Thus, the choice of words in option 1 is possible.

Hence, option 1 can be eliminated.

Consider Option 3:

Using Grumbs and Ihavitoo as the Bingoes and Lovitoo and Nana as the
Dingoes ensures that the relevant conditions (II and III) are satisfied.

Thus, the choice of words in option 3 is possible.

Hence, option 3 can be eliminated.

Consider Option 4:

The only relevant condition here is that there have to be 2 Dingoes.

Since Metoo and Nana are not connected to any other word, they can be
used together.

Thus, the choice of words in option 4 is possible.

Hence, option 4 can be eliminated.

Consider Option 2:

Ihavitoo and Grumps have to be together in a sentence.

However, if Harrumphs and Ihavitoo as the Bingoes are part of the same
sentence, the number of slots available for Bingoes is exhausted. Thus,
Grumps cannot be used in the same sentence.

This violates condition III.

The only way Ihavitoo, Grumps and Harrumphs can be used in the same
sentence is if Ihavitoo is used as a Cingo while the other two are used as
Bingoes.

Thus, the choice of words in option 2 is not possible.

Hence, option 2.

22. If Grumbs and Harrumphs are the Bingoes in a sentence, and no rule of
3 Marks
grammar is violated, which of the following is/are necessarily true?

I. Ihavitoo is the Cingo.


II. Lovitoo is the Dingo.

19 of 23 9/16/2011 9:55 AM
Exam Reports http://testfunda.com/LMS/Student/NewReports.aspx

III. Either Lovitoo or Metoo must be one of - or both - the Dingoes.

1) I only
2) II only
3) III only
4) I and III only

Solution:
Consider Statement I:

Grumbs and Ihavitoo have to be part of the same sentence.

Since Grumbs and Harrumphs are already used as Bingoes, Ihavitoo cannot
be used as a Bingo.

Therefore, Ihavitoo has to be used as a Cingo.

Since, only one Cingo is required, Ihavitoo is the only Cingo.

Thus, Statement I is definitely true.

Hence, options 2 and 3 can be eliminated.

Consider Statement II:

With the given combination, it is possible that the Dingoes are Metoo and
Nana. This does not violate any condition.

On the other hand, having Lovitoo as a Dingo also does not violate any
condition.

Thus, Statement II may or may not be true.

Consider Statement III:

Any two of the the three possible Dingoes can be chosen without violating
any given condition.

Hence, statement III is true.

Thus, statements I and III are true.

Hence, option 4.

23. Which of the following is a valid sentence if no grammar rule is to be


3 Marks
violated?

20 of 23 9/16/2011 9:55 AM
Exam Reports http://testfunda.com/LMS/Student/NewReports.aspx

1) Grumbs Harrumphs Ihavitoo Lovitoo Metoo.


2) Grumbs Harrumphs Ihavitoo Jingongo Lovitoo.
3) Harrumphs Ihavitoo Jingongo Lovitoo Metoo.
4) Grumbs Ihavitoo Koolodo Metoo Nana.

Solution:
According to condition II, there should be exactly one Cingo in each
sentence.

Consider option 2:

Since Grumbs and Harrumphs are Bingoes, Ihavitoo has been used as a
Cingo.

At the same time, Jingongo is also a Cingo.

Since there cannot be two Cingoes in a single sentence, this combination is


invalid.

Hence, option 2 can be eliminated.

Consider option 3:

According to condition III, Ihavitoo and Grumbs have to be part of the same
sentence.

Since Ihavitoo is part of option 3 and Grumbs is not, this is not a valid
combination.

Hence, option 3 can be eliminated.

Consider option 4:

According to condition IV, Koolodo can be used in a sentence only if Lovitoo


is used.

Since Koolodo is used in option 4 but Lovitoo is not, this is not a valid
combination.

Hence, option 4 can be eliminated.

Consider option 1:

The sentence satisfies all the given conditions.

Hence, it is a valid combination.

21 of 23 9/16/2011 9:55 AM
Exam Reports http://testfunda.com/LMS/Student/NewReports.aspx

Hence, option 1.

24. If in a sentence Grumps is the Bingo and no rule of grammar is violated,


3 Marks
which of the following cannot be true?

1) Harrumphs must be a Bingo.


2) Ihavitoo must be a Bingo.
3) Lovitoo may be used.
4) All three Bingoes are used.

Solution:
If Grumps is the Bingo, any one of Harrumphs or Ihavitoo can be the other
Bingo.

If Ihavitoo is the other Bingo, one from Jingongo or Koolodo can be the
Cingo.

If Harrumphs is the other Bingo, Ihavitoo has to be the Cingo.

In either case, a valid combination can be formed.

Hence, Ihavitoo need not necessarily be a Bingo.

Hence, the statement in option 2 is not true.

Hence, option 2.

25. Each question is followed by two statements, A and B. Select the correct option
3 Marks based on the following instructions:

Mark (1) if the question can be answered by one of the statements, but not by the
other.
Mark (2) if the question can be answered by using either statement, independently
of the other.
Mark (3) if the question can be answered by using both statements together, but
not by either statement alone.
Mark (4) if the question cannot be answered by either of the statements.
What is the average weight of the 3 new members who have recently been
included into the team?

A. The average weight of the team increases by 20 kg.


B. The 3 new members substitute three other members whose weights are 64 kg, 75 kg and 66
kg.

1) 1
2) 2

22 of 23 9/16/2011 9:55 AM
Exam Reports http://testfunda.com/LMS/Student/NewReports.aspx

3) 3
4) 4

Solution:
Using statement A alone:

It has not been mentioned whether these new members have been added to the
team or have replaced a few members. Depending on the case, the calculations for
average changes.

Thus, the average weight cannot be found.

Thus, the question cannot be answered using statement A alone.

Using statement B alone:

The 3 new members replace 3 other members whose total weight is 205 kg.

However, the number of members in the team as well as the impact of the
replacement on the average weight is not known.

Thus, the average weight cannot be found.

Thus, the question cannot be answered using statement B alone.

Using both the statements together:

Since these new members replace three old members, the total number of
members remains the same.

Since the number of members in the team is not known, the average weight cannot
be found.

Thus, the question cannot be answered on the basis of the two statements.

Hence, option 4.

23 of 23 9/16/2011 9:55 AM
Exam Reports http://testfunda.com/LMS/Student/NewReports.aspx

Section I

1. Each question is followed by two statements, A and B. Select the correct


3 Marks option based on the following instructions:

Mark (1) if the question can be answered by one of the statements, but not by
the other.
Mark (2) if the question can be answered by using
either statement, independently of the other.
Mark (3) if the question can be answered by using both statements together,
but not by either statement alone.
Mark (4) if the question cannot be answered by either of the statements.
A and B work at digging a ditch alternately for a day each. If A can dig a ditch
in ‘a ’ days and B can dig it in ‘b ’ days, will the work get done faster if A
begins the work?

A.

B. b > a

1) 1
2) 2
3) 3
4) 4

Solution:
Using statement A alone:

A can dig ‘1/a’ part of the ditch in one day while B can dig ‘1/b’ part of the
ditch in one day.

Irrespective of who starts first, the total work done at the end of 2 days is
always [(1/a) + (1/b)]

Similarly, if the total number of days taken to complete the work is an even
number, the total amount of work done is 2n × [(1/a) + (1/b)]

Thus, if the total amount of work done is of the form given above, the total
time taken to complete the work is the same irrespective of who starts.

Had the work been completed in say, 3 days, the total amount of work done
would have been [(2/a) + (1/b)] or [(1/a) + (2/b)]

Thus, in general, had the work taken an odd number of days for completion,

1 of 21 9/16/2011 9:56 AM
Exam Reports http://testfunda.com/LMS/Student/NewReports.aspx

the total amount of work would have been [(2n/a) + ((2n − 1)/b)] or [((2n −
1)/a) + (2n/b)]

Since the total amount of work done, as per the given equation, falls in the
first category, the work gets completed in an even number of days.

Since the work gets completed in an even number of days, it takes the same
time irrespective of who starts.

Thus, the question can be answered using statement A alone.

Using statement B alone:

Since b > a, B does less work per day compared to A.

Assume that A can do 5 units of work per day while B can do 2 units of work
per day.

Also, assume that the total work consists of 18 units.

If A starts first, the work gets completed in 5 days.

If B starts first, the work gets completed in 6 days.

Thus, the work gets completed faster if A starts.

Now, assume that the total work consists of 20 units.

If A starts first, the work gets completed in 6 days.

If B starts first, the work gets completed in 6 days.

Now, the work does not get completed faster if A starts.

Thus, the work may or may not get completed faster if A starts, depending on
the amount of work.

Thus, the question cannot be answered using statement B alone.

Thus, the question can be answered by one of the statements but not by the
other.

Hence, option 1.

Group Question
Answer the following questions based on the information given below.

A, B, C and D are to be seated in a row. However, C and D cannot be seated

2 of 21 9/16/2011 9:56 AM
Exam Reports http://testfunda.com/LMS/Student/NewReports.aspx

together. Also, B cannot be at the third place.

2. Which of the following must be definitely false?


3 Marks

1) A is at the first place.


2) A is at the second place.
3) A is at the third place.
4) A is at the fourth place.

Solution:
The given data does not mention whether the third place is from the
right or from the left.

As such, assign the first place to the seat at the extreme right or
extreme left and proceed from there. Assume that the seat at the
extreme left is the first place.

Given conditions:

(i) B cannot be at the third place (ii) C and D cannot sit together.

Since B cannot be at the third place, B can possibly take any of the
other three places. Assume B to be at each place and see the
combinations possible.

All the possible cases are listed below:

Case 1: When B is at seat 1

Case 2: When B is at seat 2

Case 3: When B is at seat 2

Case 4: When B is at seat 4

3 of 21 9/16/2011 9:56 AM
Exam Reports http://testfunda.com/LMS/Student/NewReports.aspx

From the 4 cases listed above, it can be seen that A can never be at the
first place.

Hence, option 1.

3. If A is not at the third place, then C can occupy which of the following
3 Marks
places?

1) The first place only.


2) The third place only.
3) Any one of the the first and third place.
4) Any of the four places.

Solution:
If A is not at the third place, there are two possible candidates for the
third place i.e. C and D.

If any of them is at the third place, the other cannot occupy the two
places adjacent to it (i.e. second and fourth place). The other place has
to mandatorily occupy the first place.

If C is at the third place, D has to be at the first place while if D is at the


third place, C has to be at the first place.

Thus, C can occupy any one of the first and third place.

Hence, option 3.

4. If A and B are together then which of the following must necessarily be


3 Marks
true?

1) C is not at the first place.


2) A is at the third place.
3) D is at the first place.
4) C is at the first place.

Solution:
From the solution to the first question, observe that A and B can be

4 of 21 9/16/2011 9:56 AM
Exam Reports http://testfunda.com/LMS/Student/NewReports.aspx

together only if B is at the second place and A is at the third place.

Thus, A has to necessarily be at the third place.

Hence, option 2.

Group Question
Answer the following questions based on the information given below.

The table below shows the amount of money invested (in Rs. crore) in the core
infrastructural areas of two districts, Chittoor and Khammam, in Andhra Pradesh.

5. By what percent was the total investment in the two districts more in
3 Marks
1996 as compared to that in 1995?

1) 14%
2) 21%
3) 24%
4) 18%

Solution:
The total investment in the year 1995 = 2923.1 + 7081.6 = 10004.7
crores

The total investment in the year 1996 = 3489.5 + 8352 = 11841.5 crores

∴ The required percentage = [(11841.5 − 10004.7)/10004.7] × 100

≈ 1836.8/100

≈ 18.4%

Hence, option 4.

5 of 21 9/16/2011 9:56 AM
Exam Reports http://testfunda.com/LMS/Student/NewReports.aspx

6. The investment in Electricity and Thermal Energy in 1995 in these two


3 Marks
districts formed what percent of the total investment made in that year?

1) 41%
2) 47%
3) 52%
4) 55%

Solution:
The total investment in Electricity and Thermal Energy in the year 1995
= 815.2 + 632.4 + 2065.8 + 1232.7 = 4746.10 crores

The total investment made in the year 1995 = 2923.10 + 7081.6 =


10004.70 crores

Hence, option 2.

7. In Khammam district, the investment in which area in 1996 showed the


3 Marks
least percentage increase over the investment in that area in 1995?

1) Electricity
2) Chemical
3) Solar
4) Nuclear

Solution:
The percentage increase for each area mentioned in the options is:

Electricity : [(2365.1 − 2065.8)/2065.8] × 100 ≈ 14.5%

Chemical : [(986.4 − 745.3)/745.3] × 100 ≈ 32.5%

Solar : [(1792.1 − 1363.5)/1363.5] × 100 ≈ 31.4%

Nuclear : [(2182.1 − 1674.3)/1674.3] × 100 ≈ 30.3%

Thus, Electricity shows the least percentage improvement over the


previous year.

Hence, option 1.

6 of 21 9/16/2011 9:56 AM
Exam Reports http://testfunda.com/LMS/Student/NewReports.aspx

Alternatively,

Such problems can often be solved faster using approximations and


calculating the value in terms of a range rather than a precise value. If
all the calculations fall within the same range, one can go into further
detail.

Consider each option here:

Electricity : There is an approximate increase of 300 over an initial value


of approximately 2000. This implies a percentage increase of roughly
15%.

Chemical : There is an approximate increase of 240 over an initial value


of approximately 740. Since 240 is 33.33% of 720, the percentage here
has to be marginally less than 33.33% but definitely greater than 30%.

Solar : There is an approximate increase of 430 over an initial value of


approximately 1350. 405 is 30% of 1350. Hence, the percentage here
has to be close to 30%.

Nuclear : There is an approximate increase of 500 over an initial value of


approximately 1700. This implies a percentage increase of roughly 30%.

Thus, it is evident that the lowest percentage increase is for Electricity.

However, note that the percentage increase for the other three areas are
very cloe to each other. Hence, if the maximum percentage increase
had been asked, Electricity could have been eliminated but the other
three values would have to be calculated in a little more detail.

Hence, option 1.

8. The total investment in Khammam was approximately how many times


3 Marks
the total investment in Chittoor in the two given years?

1) 2.8
2) 2.0
3) 2.4
4) 1.7

Solution:
The total investment in Chittoor in the years 1995 and 1996 is = 2923.10

7 of 21 9/16/2011 9:56 AM
Exam Reports http://testfunda.com/LMS/Student/NewReports.aspx

+ 3489.50 = 6412.60 crores

The total investment in Khammam in the years 1995 and 1996 is =


7081.6 + 8352 = 15433.60 crores

Hence, option 3.

9. If the total investment in Khammam shows the same rate of increase in


3 Marks
1997, as it had shown from 1995 to 1996, what would be the total
investment in Khammam (approximately) in 1997 (in Rs. crore)?

1) 9,850
2) 10,020
3) 9,170
4) 8,540

Solution:
The percentage increase in the investment in Khammam from the year
1995 to 1996 is

So, the investment in 1997 in Khammam = 1.1794 × 8352 = Rs. 9,850


crores (approximately).

Hence, option 1.

10. Each question is followed by two statements, A and B. Select the correct
3 Marks option based on the following instructions:

Mark (1) if the question can be answered by one of the statements, but not by
the other.
Mark (2) if the question can be answered by using
either statement, independently of the other.
Mark (3) if the question can be answered by using both statements together,
but not by either statement alone.
Mark (4) if the question cannot be answered by either of the statements.
What is the price of tea?

A. Price of coffee is Rs. 5 more than that of tea.


B. Price of coffee was Rs. 5 less than the price of a cold drink which costs three times

8 of 21 9/16/2011 9:56 AM
Exam Reports http://testfunda.com/LMS/Student/NewReports.aspx

the price of tea.

1) 1
2) 2
3) 3
4) 4

Solution:
Let the price of coffee and tea be Rs. x and Rs. y respectively.

Using statement A alone:

x=y+5

Since this is a single equation in two variables, a unique solution cannot be


found.

Thus, the question cannot be answered using statement A alone.

Using statement B alone:

Let the price of a cold drink be Rs. z

∴ x = z − 5 and z = 3y

∴ x = 3y − 5

Since this is a single equation in two variables, a unique solution cannot be


found.

Thus, the question cannot be answered using statement B alone.

Using both the statements together:

x = y + 5 and x = 3y − 5

∴ y + 5 = 3y − 5

∴y=5

Thus, the price of tea is Rs. 10.

Thus, the question can be answered using both the statements together but
not by using either statement alone.

9 of 21 9/16/2011 9:56 AM
Exam Reports http://testfunda.com/LMS/Student/NewReports.aspx

Hence, option 3.

Group Question
Answer the following questions based on the information given below.

In a game played by two people there are initially N match sticks kept on the
table. A move in the game consists of a player removing either one or two
matchsticks from the table. The one who takes the last matchstick loses. Players
make moves alternately. The player who makes the first move is A. The other
player is B.

11. The smallest value of N (greater than 5) that ensures a win for B is
3 Marks
_____.

1) 7
2) 6
3) 10
4) 8

Solution:
Let N = 1. Since there is only one matchstick, A picks it up and loses
the game.

Let N = 2. Here A picks 1 match stick and B loses the game.

Let N = 3. Here A picks 2 match sticks and B loses the game.

Let N = 4. If A picks 1 match stick, B picks 2 match sticks and A loses


the game. If A picks 2 match sticks, B picks 1 match sticks and A still
loses the game.

Thus, when there are exactly 1, 4, 7.. matchsticks left just before A has
to pick them up, A definitely loses the game.

Consider the given options:

Let N = 6. Here A picks 2 match sticks. If B picks 1 matchstick, A now


picks 2 and B loses the game. If B picks 2 match sticks, A picks 1 and B
still loses the game.

Let N = 7.

Assume that A picks 2 match sticks.

The sequence of picking up is as follows:

10 of 21 9/16/2011 9:56 AM
Exam Reports http://testfunda.com/LMS/Student/NewReports.aspx

A: 2
B:1
A: 1/2
B : 2/1

Thus, 1 matchstick is left for A and so A loses the game.

Now, assume that A picks 1 match stick.

The sequence of picking up is as follows:

A: 1
B:2
A: 1/2
B : 2/1

Thus, 1 matchstick is again left for A and so A loses the game.

Thus, there have to be 7 matchsticks left for B to definitely win the


game.

Hence, option 1.

12. The largest of N (less than 50) that ensures a win for B is _____.
3 Marks

1) 46
2) 47
3) 48
4) 49

Solution:
As seen in the solution to the previous question, if the number of match
sticks left just before A's turn are 1, 4, 7 and so on, A loses the game.

If one continues with this series, A loses the game if there are either 46
or 49.

Since, 49 is the largest number just less than 50, the largest number
less than 50 that ensures a win for B is 49.

Hence, option 4.

Group Question

11 of 21 9/16/2011 9:56 AM
Exam Reports http://testfunda.com/LMS/Student/NewReports.aspx

Answer the following questions based on the information given below.

Ghosh Babu stays at Ghosh Housing Society, Aghosh Colony, Dighospur ,


Kolkata. In Ghosh Housing Society, 6 people daily read Ganashakti and 4 people
read Anand Bazar Patrika. In Ghosh Housing Society there is no person who
reads both the newspapers. The total number of people who read either of these
two newspapers in Aghosh Colony and Dighospur is 52 and 200 respectively. The
number of people who read Ganashakti in Aghosh Colony and Dighospur is 33
and 121 respectively, while the number of people who read Anand Bazar Patrika
in Aghosh Colony and Dighospur is 32 and 117 respectively.

13. The number of people in Dighospur who read only Ganashakti is _____.
3 Marks

1) 121
2) 83
3) 79
4) 127

Solution:
In Dighospur, the number of people who read Ganashakti is 121 while
the number of people who read Anand Bazaar Patrika is 117.
Also, the number of people who read either of these two newspapers is
200.

Thus, the number of people who read both the newspapers


in Dighospur = 121 + 117 – 200 = 38

Thus, the number of people who read only Ganashakti in Dighospur =


121 – 38 = 83

Hence, option 2.

14. The number of people in Aghosh Colony who read both these
3 Marks
newspapers is _____.

1) 13
2) 20
3) 19
4) 14

Solution:
In Agosh Colony, the number of people who read Ganashakti is 33 and
the number of people who read Anand Bazaar Patrika is 32.

12 of 21 9/16/2011 9:56 AM
Exam Reports http://testfunda.com/LMS/Student/NewReports.aspx

The total number of people who read either of these two newspapers is
52.

Thus, the number of people who read both the newspapers in Agosh
Colony = 33 + 32 – 52 = 13

Hence, option 1.

15. The number of people in Aghosh Colony who read only one paper is
3 Marks
_____.

1) 29
2) 19
3) 39
4) 20

Solution:
The number of people in Aghosh colony who read at least one of the
two newspapers = 52

From the solution to the previous question, the number of people in


Aghosh colony who read both the newspapers = 13

∴ Number of people in Aghosh colony who read exactly one


newspaper = 52 − 13 = 39

Hence, option 3.

Group Question
Answer the following questions based on the information given below.

13 of 21 9/16/2011 9:56 AM
Exam Reports http://testfunda.com/LMS/Student/NewReports.aspx

16. When was the per capita production of milk the least?
3 Marks

1) 1990
2) 1992
3) 1994
4) 1996

Solution:

Per capita production of milk = (Total production of milk)/(Female


population + Male population)

For 1990: Per capita production = 5/(34 + 36) = 5/70 = 1/14

For 1992: Per capita production = 7/(35 + 37) = 7/72

For 1994: Per capita production = 8/(37 + 39) = 8/76 = 2/19

For 1996: Per capita production = 6/(40 + 43) = 6/83

Now,

14 of 21 9/16/2011 9:56 AM
Exam Reports http://testfunda.com/LMS/Student/NewReports.aspx

1990: 5/70 = 1/14

1992: 7/72 = 1/(72/7) = 1/10.28

1994: 2/19 = 1/(19/2) = 1/9.5

1996: 6/83 = 1/(83/6) = 1/13.82

Since each fraction has the same numerator, the fraction with the
largest denominator has the smallest value.

Thus, 1/14 is the smallest value.

Thus, the per capita production was the least in 1990.

Hence, option 1.

17. When was the per capita production of food-grains the most?
3 Marks

1) 1992
2) 1993
3) 1994
4) 1995

Solution:
The per capita production of food-grains for years 1992, 1993, 1994 and
1995 is

1992: 20/72 = 1/(72/20) = 1/3.6

1993: 21/74 = 1/(74/21) = 1/3.52

1994: 25/76 = 1/(76/25) = 1/3.04

1995: 31/79 = 1/(79/31) = 1/2.55

Since each fraction has the same numerator, the fraction with the
smallest denominator has the largest value.

Thus, 31/79 has the largest value.

15 of 21 9/16/2011 9:56 AM
Exam Reports http://testfunda.com/LMS/Student/NewReports.aspx

Thus, the per capita production of food grains was highest in 1995.

Hence, option 4.

18. In which year was the difference between the increase in the production
3 Marks
of food-grains and milk the maximum?

1) 1993
2) 1994
3) 1995
4) 1996

Solution:
This question can be solved by direct observation.

It is evident that the difference between the increase in the production of


food grains and milk was maximum in 1995.

Hence, option 3.

19. If milk contains 320 calories and food-grains contain 160 calories, in
3 Marks
which year was the per capita consumption of calories the highest?

1) 1993
2) 1994
3) 1995
4) 1996

Solution:
The per capita consumption of calories in the years 1993, 1994, 1995
and 1996 is respectively,

1993: 5920/74 = 80

16 of 21 9/16/2011 9:56 AM
Exam Reports http://testfunda.com/LMS/Student/NewReports.aspx

1994: 6560/76 = 86.32

1995: 7200/79 = 91.14

1996: 6240/83 = 75.18

Thus, the per capita consumption of calories is the most in 1995.

Hence, option 3.

20. If one gallon of milk contains 120 gm of a particular nutrient and one ton
3 Marks
of food-grains contains 80 gm of the same nutrient, in which year was
the availability of this nutrient the maximum?

1) 1993
2) 1994
3) 1995
4) 1996

Solution:
The availability of the concerned nutrient for each given year is:

1993: (120 × 8) + (80 × 21) = 960 + 1680 = 2640 million gms

1994: (120 × 8) + (80 × 25) = 960 + 2000 = 2960 million gms

1995: (120 × 7) + (80 × 31) = 840 + 2480 = 3320 million gms

1996: (120 × 6) + (80 × 27) = 720 + 2160 = 2880 million gms

Thus, the availability of the nutrient was maximum in 1995.

Hence, option 3.

21. Referring to the above question, in which year was the per capita
3 Marks
consumption of this nutrient the highest?

1) 1993
2) 1994
3) 1995
4) 1996

17 of 21 9/16/2011 9:56 AM
Exam Reports http://testfunda.com/LMS/Student/NewReports.aspx

Solution:
The per capita consumption of the nutrient for each of the given years is
:

1993: 2640/(36 + 38) = 2640/74 = 35.67

1994: 2960/(37 + 39) = 2960/76 = 38.95

1995: 3320/(39 + 40) = 3320/79 = 42.03

1996: 2880/(40 + 43) = 2880/83 = 34.7

Thus, the per capita consumption was the highest in 1995.

Hence, option 3.

22. Each question is followed by two statements, A and B. Select the correct
3 Marks option based on the following instructions:

Mark (1) if the question can be answered by one of the statements, but not by
the other.
Mark (2) if the question can be answered by using
either statement, independently of the other.
Mark (3) if the question can be answered by using both statements together,
but not by either statement alone.
Mark (4) if the question cannot be answered by either of the statements.
What is the value of p + q?

A.

B.

1) 1
2) 2
3) 3
4) 4

Solution:
Using statement A alone:

Since this is a single linear equation in three variables, a unique solution

18 of 21 9/16/2011 9:56 AM
Exam Reports http://testfunda.com/LMS/Student/NewReports.aspx

cannot be found for this equation.

Thus, the question cannot be answered using statement A alone.

Using statement B alone:

Since this is a single linear equation in three variables, a unique solution


cannot be found for this equation.

Thus, the question cannot be answered using statement B alone.

Using both the statements together:

Even when the two statements are combined, we have two equations and
three unknowns. Therefore, a unique solution cannot be found.

The only conclusion obtained on solving these two equations is p + 7q = 0.

Thus, the value of p + q cannot be found.

Thus, the question cannot be answered by either of the statements.

Hence, option 4.

Group Question
Answer the following questions based on the information given below.

A, B, C, D, E and F are a group of friends from a club. There are two doctors, one
lecturer, one architect, one accountant and one lawyer in the group. There are
two married couples in the group. The lawyer is married to D who is a doctor. No
lady in the group is either an architect or an accountant. C, the accountant, is
married to F who is a lecturer. A is married to D and E is not a doctor.

23. What is E's profession?


3 Marks

1) Lawyer
2) Architect
3) Lecturer
4) Accountant

Solution:
There are 6 people (A, B, C, D, E and F) who have the following
occupations (D1, D2, Ac, Ar, Lc and Lw) in no particular order.

The lawyer is married to D, who is a doctor. At the same time, A is

19 of 21 9/16/2011 9:56 AM
Exam Reports http://testfunda.com/LMS/Student/NewReports.aspx

married to D.

Thus, A (Lw) is married to D(D1).

C (Ac) is married to F(Lc).

Since E is not a doctor, E can only be the architect while B is the other
doctor.

Thus, E is the architect.

Hence, option 2.

24. How many members of the group are male?


3 Marks

1) 2
2) 3
3) 4
4) Cannot be determined

Solution:
From the solution to the previous question, it is clear that C being the
accountant, cannot be a lady. Hence, F is a lady.

Also, one out of A and D is a lady.

Since E is the architect, E cannot be a lady.

However, B (the other doctor) can be male or female.

Thus, the total number of females cannot be found.

Hence, the total number of men cannot be found.

Hence, option 4.

25. Each question is followed by two statements, A and B. Select the correct
3 Marks option based on the following instructions:

Mark (1) if the question can be answered by one of the statements, but not by
the other.
Mark (2) if the question can be answered by using
either statement, independently of the other.
Mark (3) if the question can be answered by using both statements together,
but not by either statement alone.

20 of 21 9/16/2011 9:56 AM
Exam Reports http://testfunda.com/LMS/Student/NewReports.aspx

Mark (4) if the question cannot be answered by either of the statements.


If a, b, c are integers, is (a − b + c) > (a + b − c)?

A. b is negative.
B. c is positive.

1) 1
2) 2
3) 3
4) 4

Solution:
Is (a – b + c) > (a + b – c)?

Subtracting a from both sides, the question gets converted to a comparison


between (–b + c) and (b – c)

The same question can now be written as : Is c > b?

Using statement A alone:

b is negative, but there is no mention of c.

Thus, the question cannot be answered using statement A alone.

Using statement B alone:

c is positive, but there is no mention of b.

Thus, the question cannot be answered using statement B alone.

Using both the statements together:

Since c is positive and b is negative, c > b.

Thus, the question can be answered by using both statements together but
not by using either statement alone.

Hence, option 3.

21 of 21 9/16/2011 9:56 AM
Exam Reports http://testfunda.com/LMS/Student/NewReports.aspx

Section I

1. Each question is followed by two statements, A and B. Select the correct


3 Marks option based on the following instructions:

Mark (1) if the question can be answered by one of the statements, but not by
the other.
Mark (2) if the question can be answered by using
either statement, independently of the other.
Mark (3) if the question can be answered by using both statements together,
but not by either statement alone.
Mark (4) if the question cannot be answered by either of the statements.
What is the area bounded by the two lines and the co-ordinate axes in the
first quadrant?

A. The lines intersect at a point which also lies on the lines 3x − 4y = 1 and 7x − 8y = 5.
B. The lines are perpendicular, and one of them intersects the y-axis at an intercept of
4.

1) 1
2) 2
3) 3
4) 4

Solution:
Using Statement A alone:

The lines intersect at the point of intersection of 3x − 4y = 1 and 7x − 8y = 5.

On solving these equations you get, x = 3 and y = 2.

Thus, the given lines intersect at the point (3, 2).

However, the point of intersection of these two lines with the x-axis and the
y-axis is not known.

Therefore, the required area cannot be found.

Thus, the question cannot be answered using statement A alone.

Using statement B alone:

One of the lines passes through the point (0, 4).

1 of 14 9/16/2011 9:57 AM
Exam Reports http://testfunda.com/LMS/Student/NewReports.aspx

If the co-ordinates of any other point through which this line passes are
known, one can find the equation of this line.

If this equation is known, the equation of the other line and its x-intercept can
be found.

In such a case, the area can be calculated.

Since no other co-ordinate is known, the above calculations are not possible.

Therefore, the required area cannot be found.

Thus, the question cannot be answered using statement B alone.

Using both the statements together:

Line L1 passes through (0, 4) as well as (3, 2).

Hence, the slope of this line can be found. This slope is −2/3.

Since the other line i.e. L2 is perpendicular to this line, the slope of L2 is 3/2.

Using this slope and the co-ordinates (3, 2), the equation of line L2 can be
found.

Once this equation is known, the x-intercept of L2 can be found by


substituting y = 0 in L2.

Thus, all four co-ordinates of the bounded area are now known.

Therefore, the area of the bounded region can be calculated.

Thus, the question can be answered using both the statements together, but
not by using either statement alone.

Hence, option 3.

2. A cube is made into a number of smaller cubes by dividing each edge into
3 Marks
four equal parts. What is the minimum number of similar additional small
cubes that would be required to create a bigger cube which would completely
enclose the original cube?

1) 152
2) 261
3) 61
4) 148

2 of 14 9/16/2011 9:57 AM
Exam Reports http://testfunda.com/LMS/Student/NewReports.aspx

Solution:
When a cube is cut into four equal parts on each edge, the total number of
smaller cubes created is 4 × 4 × 4 = 64 (Length × Breadth × Height)

When a bigger cube encloses the original cube using small cubes similar to
the ones just created, it implies that one small cube gets added to the length
on each side.

Thus, the length of the bigger cube consists of 6 small cubes.

Similarly, the breadth as well as height of the bigger cube will consist of 6
small cubes each.

Thus, the bigger cube consists of 6 × 6 × 6 = 216 cubes

So, the number of additional smaller cubes = 216 – 64 = 152

Hence, option 1.

Group Question
Answer the following questions based on the information given below.

The bar chart below shows the total sales, total expenditure and share capital of
four companies, A, B, C and D. ‘Profits’ is defined as the difference between total
sales and total expenditure.

3. Which company has the lowest profit per rupee of sales?


3 Marks

1) A
2) B

3 of 14 9/16/2011 9:57 AM
Exam Reports http://testfunda.com/LMS/Student/NewReports.aspx

3) C
4) D

Solution:

Profit = Sales − Expenditure

Profit per rupee of sales = Profit/Sales

These figures can be calculated as shown in the table above.

It can be seen from the table that the profit per rupee of sales is the
least for the company C (0.0833).

Hence, option 3.

4. Which company has the lowest profit per rupee of the share capital?
3 Marks

1) A
2) B
3) C
4) D

Solution:
Profit per rupee of share capital = Profit/Share Capital

The profit per rupee of share capital for each company is given below:

A : 5/10 = 0.5

B : 1/0.5 = 2

C : 1/8 = 0.125

D : 3/5 = 0.6

Thus, company C has the lowest profit per rupee of the share capital.

4 of 14 9/16/2011 9:57 AM
Exam Reports http://testfunda.com/LMS/Student/NewReports.aspx

Hence, option 3.

5. Which company has the highest sales per rupee of the share capital?
3 Marks

1) A
2) B
3) C
4) D

Solution:
Sales per rupee of share capital = Sales/Share Capital

This ratio for each company can be calculated from the table in the
solution to the first question and is given below.

A : 20/10 = 2

B : 4/0.5 = 8

C : 12/8 = 1.5

D : 15/5 = 3

Thus, company B has the highest sales per rupee of the share capital.

Hence, option 2.

6. Which company has the lowest profit per rupee of the total expenditure?
3 Marks

1) A
2) B
3) C
4) D

Solution:
Profit per rupee of the expenditure = Profit/Expenditure

This ratio can be calculated for each company from the table in the
solution to the first question and is given below.

A : 5/15 = 0.33

5 of 14 9/16/2011 9:57 AM
Exam Reports http://testfunda.com/LMS/Student/NewReports.aspx

B : 1/3 = 0.33

C : 1/11 = 0.09

D : 3/12 = 0.25

Thus, company C has the lowest profit per rupee of the total
expenditure.

Hence, option 3.

7. Each question is followed by two statements, A and B. Select the correct


3 Marks option based on the following instructions:

Mark (1) if the question can be answered by one of the statements, but not by
the other.
Mark (2) if the question can be answered by using
either statement, independently of the other.
Mark (3) if the question can be answered by using both statements together,
but not by either statement alone.
Mark (4) if the question cannot be answered by either of the statements.
A train started from Station A, developed engine trouble and reached Station
B, 40 minutes late. What is the distance between Stations A and B?

A. The engine trouble developed after travelling 40 km from Station A and the speed
reduced to 1/4 of the original speed.
B. The engine trouble developed after travelling 40 km from station A in two hours and
the speed reduced to 1/4 of the original speed.

1) 1
2) 2
3) 3
4) 4

Solution:
Using Statement A alone:

Since the time in which the train travelled the 40 kms is not known, the
distance cannot be found.

Thus, the question cannot be answered using statement A alone.

Using Statement B alone:

6 of 14 9/16/2011 9:57 AM
Exam Reports http://testfunda.com/LMS/Student/NewReports.aspx

The train travelled 40 km in 2 hours, and then the engine trouble started.

Since the speed reduces to 1/4 of the original speed, the time taken if 4 times
the original time.

If the time taken to travel the balance distance at 40 km is t minutes, then the
time taken to travel the same distance at the revised speed is 4t minutes.

Thus, the additionl time taken is 4t − t = 3t minutes.

The train reached 40 minutes late.

∴ 3t = 40

∴ t = 40/3

Once this time, the balance distance travelled and the total distance can be
calculated.

Thus, the question can be answered using statement B alone.

Thus, the question can be answered by one of the statements but not by the
other.

Hence, option 1.

Group Question
Answer the following questions based on the information given below.

Recently, Ghosh Babu spent his winter vacation on Kyakya Island. During the
vacation, he visited the local casino where he came across a new card game. Two
players, using a normal deck of 52 playing cards, play this game. One player is
called the “Dealer” and the other is called the “Player”. First, the Player picks a
card at random from the deck. This is called the “base card”. The amount in
rupees equal to the face value of the base card is called the “base amount”. The
face values of Ace, King, Queen and Jack are ten. For other cards, the face value
is the number on the card. Once the Player picks a card from the deck, the Dealer
pays him the base amount. Then the Dealer picks a card from the deck and this
card is called the “top card”. If the top card is of the same suit as the base card,
the player pays twice the base amount to the Dealer. If the top card is of the same
colour as the base card (but not the same suit) then the Player pays the base

7 of 14 9/16/2011 9:57 AM
Exam Reports http://testfunda.com/LMS/Student/NewReports.aspx

amount to the Dealer. If the top card happens to be of a different colour than the
base card, the Dealer pays the base amount to the Player.

Ghosh Babu played the game 4 times. In the first game, he picked the eight of
clubs and the Dealer picked the queen of clubs. In the second, he picked the ten
of hearts and the Dealer picked the two of spades. Next time, Ghosh Babu picked
the six of diamonds and the Dealer picked the ace of hearts. In the finally
game, he picked the eight of spades and the Dealer picked jack of
spades. Answer the following questions based on the information given
about these four games.

8. If Ghosh Babu stopped playing the game when his gain was the
3 Marks
maximum, the gain in Rs. would have been:

1) 12
2) 20
3) 16
4) 4

Solution:
Assume that the Player gets an amount n from the Dealer at the
beginning of each game.

The amount with the Player at the end of the game is as shown below:

Case I : If the Dealer picks a card from the same suit, the Player pays
2n to the Dealer and is left with −n

Case II : If the Dealer picks a card of the same colour but not from the
same suit, the Player pays n to the Dealer and is left with 0

Case III : If the Dealer picks a card from a different colour, the
Player gets n from the Dealer and is left with 2n

Game I :

Ghosh Babu picked the eight of clubs and so the Dealer gave him Rs.
8. The Dealer picked the queen of clubs (same suit card) and Ghosh
Babu gave him Rs. 16.

Thus, Ghosh Babu had a loss of Rs. 8 at the end of the first game.

In the second game, Ghosh Babu picked the ten of hearts and so the
Dealer gave him Rs. 10. The Dealer picked the two of spades (different
colour card) and gave Ghosh Babu Rs. 10.

8 of 14 9/16/2011 9:57 AM
Exam Reports http://testfunda.com/LMS/Student/NewReports.aspx

Thus, the amount with Ghosh Babu at the end of the second game
(Ghosh Babu's gain) was −8 + 10 + 10 = Rs. 12.

In the third game, Ghosh Babu picked the six of diamonds and so the
Dealer gave him Rs. 6. The Dealer picked the ace of hearts (same
colour card) and so Ghosh Babu gave him the base amount i.e. Rs. 6.
Thus, there was no profit and no loss.

Thus, Ghosh Babu's gain at the end of the third game was again Rs.
12.

In the final game, Ghosh Babu picked the eight of spades and got Rs. 8
from the Dealer. The Dealer then picked the jack of spades (same suit
card) and so Ghosh Babu gave him Rs. 16.

Thus, at the end of the fourth game, the amount with Ghosh Babu (his
gain) was 12 + 8 − 16 = Rs. 4

Thus, the maximum gain was at the end of either Game II or Game III.

Therefore, Ghosh Babu would have stopped here.

The gain at this stage was Rs. 12.

Hence, option 1.

9. The initial amount that Ghosh Babu had (before the beginning of the
3 Marks
game sessions) was Rs. X. At no point did he have to borrow any
money. What is the minimum possible value of X?

1) 16
2) 8
3) 100
4) 24

Solution:
As seen from the solution to the previous question, Ghosh Babu
incurred a loss of Rs. 8 at the end of the first game.

Thus, if he already had at least Rs. 8 before the beginning of the game
sessions, he would have got Rs. 8 from the Dealer at the neginning of
the first game and would have paid Rs. 16 to the Dealer at the end of
the first game. He would therefore have a zero balance, but would not
need to borrow money.

9 of 14 9/16/2011 9:57 AM
Exam Reports http://testfunda.com/LMS/Student/NewReports.aspx

Thus, the minimum possible value of X is 8.

Hence, option 2.

10. If the final amount of money that Ghosh Babu had with him was Rs.
3 Marks
100, what was initial amount that he had?

1) 120
2) 8
3) 4
4) 96

Solution:
From the solution to the first question, Ghosh Babu had a gain of Rs. 4
at the end of four games.

Thus, if he had Rs. 100 at the end of the fourth game, he should have
had Rs. (100 – 4) = Rs. 96 with him at the start of the session.

Hence, option 4.

11. Each question is followed by two statements, A and B. Select the correct
3 Marks option based on the following instructions:

Mark (1) if the question can be answered by one of the statements, but not by
the other.
Mark (2) if the question can be answered by using
either statement, independently of the other.
Mark (3) if the question can be answered by using both statements together,
but not by either statement alone.
Mark (4) if the question cannot be answered by either of the statements.
What is the first term of an arithmetic progression of positive integers?

A. The sum of the squares of the first and second term is 116.
B. The fifth term is divisible by 7.

1) 1
2) 2
3) 3
4) 4

10 of 14 9/16/2011 9:57 AM
Exam Reports http://testfunda.com/LMS/Student/NewReports.aspx

Solution:
Using Statement A alone:

The sum of the squares of the first and second term is 116.

The arithmetic progression is of positive integers.

The only positive integers that add up to 116 and are both perfect squares are
100 and 16.

Thus, the squares of the first and second term are 16 and 100.

Thus, the first term is one from 4 and 10.

If the first term is 10, 4 becomes the second term and the third term becomes
−2.

This is not possible as the arithmetic progression is of positive integers.

Therefore, the first term is 4.

Thus, the question can be answered using statement A alone.

Using statement B alone.

The fifth term is divisible by 7.

∴ a + 4d = 7n

Since this is a linear equation in 3 variables, the value of a cannot be found.

Therefore, the first term cannot be found.

Thus, the question cannot be answered using statement B alone.

Thus, the question can be answered by one of the statements but not by the
other.

Hence, option 1.

12. Each question is followed by two statements, A and B. Select the correct
3 Marks option based on the following instructions:

Mark (1) if the question can be answered by one of the statements, but not by
the other.
Mark (2) if the question can be answered by using
either statement, independently of the other.

11 of 14 9/16/2011 9:57 AM
Exam Reports http://testfunda.com/LMS/Student/NewReports.aspx

Mark (3) if the question can be answered by using both statements together,
but not by either statement alone.
Mark (4) if the question cannot be answered by either of the statements.
What are the values of the 3 integers a,b and c?

A. ab = 8
B. bc = 9

1) 1
2) 2
3) 3
4) 4

Solution:
Using statement A alone:

ab = 8 ... (i)

The combinations possible are (1, 8), (2, 4), (4, 2), (8, 1), (−1, −8), (−2, −4),
(−4, −2) and (−8, −1)

Thus, a unique value of a and b cannot be found.

Thus, the question cannot be answered using statement A alone.

Using statement B alone:

bc = 9 ... (ii)

The combinations possible are (1, 9), (3, 3), (9, 1), (−1, −9), (−3, −3) and (−8,
−1)

Thus, a unique value of b and c cannot be found.

Thus, the question cannot be answered using statement B alone.

Using both the statements together:

This is possible if a = 8 and b = 9 or if a = −8 and b = −9.

In the first case, c = 1, while in the second case, c = −1.

12 of 14 9/16/2011 9:57 AM
Exam Reports http://testfunda.com/LMS/Student/NewReports.aspx

Thus, a = 8, b = 9 and c = 1 or a = −8, b = −9 and c = −1.

Again, a unique value of a, b and c cannot be found.

Thus, the question cannot be answered by either of the statements.

Hence, option 4.

13. In each of the following sentences the main statement is followed by four
3 Marks sentences each. Select a pair of sentences that relate logically with the given
statement.
Either Sita is sick or she is careless.

A. Sita is not sick.


B. Sita is not careless.
C. Sita is sick.
D. Sita is careless.

1) AB
2) AD
3) BA
4) DA

Solution:
This logical connective is of the following type

Either P or Q

This implies that if P does not occur then Q occurs or if Q does not occur
then P occurs.

The conclusions for this type of logical connective are as follows

1. ~P ⇒ Q
2. ~Q ⇒ P

Thus, if Sita is not sick, then she is careless (i.e. AD), or , if Sita is not
careless then she is sick. (i.e. BC).

However, BC is not part of any option.

Thus, AD relates logically with the given statement.

Hence, option 2.

13 of 14 9/16/2011 9:57 AM
Exam Reports http://testfunda.com/LMS/Student/NewReports.aspx

14. In each of the following sentences the main statement is followed by four
3 Marks sentences each. Select a pair of sentences that relate logically with the given
statement.
Ram gets a swollen nose whenever he eats hamburgers.

A. Ram gets a swollen nose.


B. Ram does not eat hamburgers.
C. Ram does not get a swollen nose.
D. Ram eats hamburgers.

1) AB
2) DC
3) AC
4) CB

Solution:
This logical connective is of the following type

Q whenever P

This connective is of the same type as : If P then Q.

The conclusions for the above type of logical connective are as follows

1. P ⇒ Q
2. ~Q ⇒ ~P

Thus, if Ram eats hamburgers, he gets a swollen nose (i.e. DA) or if Ram
does not get a swollen nose, he does not eat hamburgers (i.e. CB).

DA is not part of the options.

Therefore, CB relates logically with the given statement.

Hence, option 4.

15. In each of the following sentences the main statement is followed by four
3 Marks sentences each. Select a pair of sentences that relate logically with the given
statement.
Either the employees have no confidence in the management or they are
hostile by nature.

A. They are hostile by nature.


B. They are not hostile by nature.
C. They have confidence in the management.
D. They have no confidence in the management.

14 of 14 9/16/2011 9:57 AM
Exam Reports http://testfunda.com/LMS/Student/NewReports.aspx

Section I

1. Each question is followed by two statements, A and B. Select the correct


3 Marks option based on the following instructions:

Mark (1) if the question can be answered by one of the statements, but not by
the other.
Mark (2) if the question can be answered by using
either statement, independently of the other.
Mark (3) if the question can be answered by using both statements together,
but not by either statement alone.
Mark (4) if the question cannot be answered by either of the statements.
If R is an integer between 1 and 9, P − R = 2370, what is the value of R?

A. P is divisible by 4.
B. P is divisible by 9.

1) 1
2) 2
3) 3
4) 4

Solution:
Using Statement A alone:

P is divisible by 4 and R is an integer between 1 and 9.

P can be divisble by 4 if P = 2372 or P = 2376

For these values of P, R = 2 or R = 6.

Thus, a unique value of R cannot be found.

Thus, the question cannot be answered using statement A alone.

Using Statement B alone:

P is divisible by 9 and R is an integer between 1 and 9.

P can be divisible by 9 if P = 2376.

For this value of P, R = 6.

Thus, the only possible value of (P, R) is (2376, 6).

1 of 39 9/16/2011 10:20 AM
Exam Reports http://testfunda.com/LMS/Student/NewReports.aspx

Thus, the question can be answered using statement B alone.

Thus, the question can be answered using statement B alone but not by
using statement A alone.

Thus, the question can be answered using one of the statements, but not by
using the other statement.

Hence, option 1.

2. Each question is followed by two statements, A and B. Select the correct


3 Marks option based on the following instructions:

Mark (1) if the question can be answered by one of the statements, but not by
the other.
Mark (2) if the question can be answered by using
either statement, independently of the other.
Mark (3) if the question can be answered by using both statements together,
but not by either statement alone.
Mark (4) if the question cannot be answered by either of the statements.
A man distributed 43 chocolates to his children. How many of his children are
more than five years old?

A. A child older than five years gets 5 chocolates.


B. A child less than or equal to 5 years gets 6 chocolates.

1) 1
2) 2
3) 3
4) 4

Solution:
Using Statement A alone:

A child older than five years gets 5 chocolates.

Since there are 43 chocolates, there can be a maximum of 8 children who are
older than 5 years.

However, the actual number of children who are older than five years cannot
be found.

Thus, the question cannot be answered using statement A alone.

2 of 39 9/16/2011 10:20 AM
Exam Reports http://testfunda.com/LMS/Student/NewReports.aspx

Using Statement B alone:

A child less than or equal to 5 years gets 6 chocolates.

The actual number of children who are older than five years cannot be found.

Thus, the actual number of children who are older than five years cannot be
found.

Thus, the question cannot be answered using statement B alone.

Using both the statements together:

Let the number of children who are less than or equal to 5 years be x and the
number of children who are older than five years be y.

∴ 5x + 6y = 43

Since x and y stand for number of people, they should have integral values.

Therefore, the only possible value of (x, y) is (5, 3)

Thus, 3 children are more than 5 years old.

Thus, the question can be answered using both statements together, but not
by using either statement alone.

Hence, option 3.

3. Each question is followed by two statements, A and B. Select the correct


3 Marks option based on the following instructions:

Mark (1) if the question can be answered by one of the statements, but not by
the other.
Mark (2) if the question can be answered by using
either statement, independently of the other.
Mark (3) if the question can be answered by using both statements together,
but not by either statement alone.
Mark (4) if the question cannot be answered by either of the statements.
Ramu went by car from Kolkata to Trichy via Chennai, without any stoppages.
The average speed for the entire journey was 40 kmph. What was the
average speed from Chennai to Trichy?

A. The distance from Chennai to Trichy is 0.3 times the distance from Kolkata to
Chennai.
B. The average speed from Chennai to Trichy was twice the average speed
from Kolkata to Chennai.

3 of 39 9/16/2011 10:20 AM
Exam Reports http://testfunda.com/LMS/Student/NewReports.aspx

1) 1
2) 2
3) 3
4) 4

Solution:
Using Statement A alone:

The average speed for the entire journey was 40 kmph.

Let the distance between Kolkata and Chennai be 10 km. Therefore, the
distance between Chennai and Trichy is 3 km.

The average speed from Chennai to Trichy cannot be found from this data.

Thus, the question cannot be answered using statement A alone.

Using Statement B alone:

Let the average speed from Chennai to Trichy be 2x kmph, then the average
speed from Kolkata to Chennai is x kmph.

The average speed for the whole journey is given as:

40 = Total Distance Travelled/Total Time Taken

∴ 40 = [10 + 3]/[(10/x) + (3/2x)]

∴ 40 = (13 × 2x)/23

∴ x = (40 × 23)/26

∴ 2x = (40 × 23)/13 kmph

This is the average speed from Chennai to Trichy.

Thus, the question can be answered using both statements together, but not
by using either statement alone.

Hence, option 3.

4. Each question is followed by two statements, A and B. Select the correct


3 Marks option based on the following instructions:

Mark (1) if the question can be answered by one of the statements, but not by

4 of 39 9/16/2011 10:20 AM
Exam Reports http://testfunda.com/LMS/Student/NewReports.aspx

the other.
Mark (2) if the question can be answered by using
either statement, independently of the other.
Mark (3) if the question can be answered by using both statements together,
but not by either statement alone.
Mark (4) if the question cannot be answered by either of the statements.
x, y and z are three positive odd integers. Is x + z divisible by 4?

A. y − x = 2
B. z − y = 2

1) 1
2) 2
3) 3
4) 4

Solution:
Using Statement A alone:

y−x=2

Since the relationship between x and z is not known, and the value of x is not
known, the value of x + z cannot be found.

Thus, the question cannot be answered using statement A alone.

Using Statement B alone:

z−y=2

Since the relationship between x and z is not known, and the value of z is not
known, the value of x + z cannot be found.

Thus, the question cannot be answered using statement B alone.

Using both the statements together:

Adding the two equations, we get z – x = 4

∴z=x+4

Now, z + x = x + 4 + x = 2x + 4

Since x is an odd integer, z + x = 2(2k + 1) + 4

5 of 39 9/16/2011 10:20 AM
Exam Reports http://testfunda.com/LMS/Student/NewReports.aspx

= 4k + 6

Thus, x + z is not divisible by 4.

Thus, the question can be answered using both the statements together but
not by using either statement alone.

Hence, option 3.

5. Each question is followed by two statements, A and B. Select the correct


3 Marks option based on the following instructions:

Mark (1) if the question can be answered by one of the statements, but not by
the other.
Mark (2) if the question can be answered by using
either statement, independently of the other.
Mark (3) if the question can be answered by using both statements together,
but not by either statement alone.
Mark (4) if the question cannot be answered by either of the statements.
The unit price of product P1 is non-increasing and that of product P2 is
decreasing. Which product will be costlier 5 years hence?

A. Current unit price of P1 is twice that of P2.


B. 5 years ago, the unit price of P2 was twice that of P1.

1) 1
2) 2
3) 3
4) 4

Solution:
The unit price of P1 either decreases or remains constant. The unit price of
P2 keeps on decreasing.

Using Statement A alone:

The current price of P1 is twice that of P2.

P1 and P2 can both follow a decreasing trend.

However, the rate of decrease is not known.

Thus, if the rate of decrease for P1 is much higher than P2, P2 can be
costlier 5 years hence.

6 of 39 9/16/2011 10:20 AM
Exam Reports http://testfunda.com/LMS/Student/NewReports.aspx

P1 can also remain constant.

If P1 remains constant, P1 will be costlier 5 years hence.

Thus, the question cannot be answered using statement A alone.

Using Statement B alone:

5 years ago, the unit price of P2 was twice that of P1.

Depending on the rate of decrease of P1 and P2, one of them will be


costlier 5 years from now.

Thus, the question cannot be answered using statement B alone.

Using both the statements together:

When both the statements are combined, it seems that the rate of decrease
of P2 is much more than the rate of decrease of P1.

However, the question does not say that the rate of decrease for a particular
product is constant throughout.

Hence, the costlier product cannot be determined.

Thus, the question cannot be answered using either of the statements.

Hence, option 4.

6. Each question is followed by two statements, A and B. Select the correct


3 Marks option based on the following instructions:

Mark (1) if the question can be answered by one of the statements, but not by
the other.
Mark (2) if the question can be answered by using
either statement, independently of the other.
Mark (3) if the question can be answered by using both statements together,
but not by either statement alone.
Mark (4) if the question cannot be answered by either of the statements.
X is older than Y, Z is younger than W and V is older than Y. Is Z younger
than X?

A. W may not be older than V.


B. W is not older than V.

1) 1
2) 2

7 of 39 9/16/2011 10:20 AM
Exam Reports http://testfunda.com/LMS/Student/NewReports.aspx

3) 3
4) 4

Solution:
X > Y, Z < W, V > Y.

To find the relationship between X and Z, Z needs to be compared to either X


or Y.

Using Statement A alone:

W may not be older than V.

From this data, no relationship can be identified between X and Z.

Thus, the question cannot be answered using statement A alone.

Using Statement B alone:

W is not older than V.

Again, from this data, no relationship can be identified between X and Z.

Thus, the question cannot be answered using statement B alone.

Using both the statements together:

Even when both the statements are combined, no relationship can be


established.

Thus, the question cannot be answered by either of the statements.

Hence, option 4.

7. Each question is followed by two statements, A and B. Select the correct


3 Marks option based on the following instructions:

Mark (1) if the question can be answered by one of the statements, but not by
the other.
Mark (2) if the question can be answered by using
either statement, independently of the other.
Mark (3) if the question can be answered by using both statements together,
but not by either statement alone.
Mark (4) if the question cannot be answered by either of the statements.
How long did Mr. X take to cover 5000 km. journey with 10 stopovers?

8 of 39 9/16/2011 10:20 AM
Exam Reports http://testfunda.com/LMS/Student/NewReports.aspx

A. The ith stopover lasted i2 minutes.


B. The average speed between any two stopovers was 66 kmph.

1) 1
2) 2
3) 3
4) 4

Solution:
Using Statement A alone:

The 1st stopover lasted 12 minutes, the 2nd stopover lasted 22 minutes and so
on. The 10th stopover lasted 102 minutes.

Therefore,

However, the actual travelling time is not known.

Hence, the total time cannot be calculated.

Thus, the question cannot be answered using statement A alone.

Using Statement B alone:

However, the stopover time is not known.

Hence, the total time cannot be calculated.

Thus, the question cannot be answered using statement B alone.

Using both the statements together:

The stopover time is obtained from statement A while the travelling time is
obtained from statement B.

From this data, the total time can be calculated.

Thus, the question can be answered using both statements together, but not
by using either statement alone.

9 of 39 9/16/2011 10:20 AM
Exam Reports http://testfunda.com/LMS/Student/NewReports.aspx

Hence, option 3.

8. Each question is followed by two statements, A and B. Select the correct


3 Marks option based on the following instructions:

Mark (1) if the question can be answered by one of the statements, but not by
the other.
Mark (2) if the question can be answered by using
either statement, independently of the other.
Mark (3) if the question can be answered by using both statements together,
but not by either statement alone.
Mark (4) if the question cannot be answered by either of the statements.

A. x + y > 0
B. x and y are positive integers and each is greater than 2.

1) 1
2) 2
3) 3
4) 4

Solution:
Using Statement A alone:

x + y > 0.

Consider x = y = 2

The value of the expression becomes 2 i.e. > 1

Consider x = y = 1

The value of the expression becomes 1.

Thus, the question cannot be answered using statement A alone.

Using Statement B alone:

When the expression of the left hand side is simplified, you get [x2y + xy2]/(x2
+ y2)

10 of 39 9/16/2011 10:20 AM
Exam Reports http://testfunda.com/LMS/Student/NewReports.aspx

Since x and y are positive integers, this expression will be positive.

Since the numerator consists of x2 and y2, each multiplied by a positive


number greater than 2, the numerator will be definitely larger than the
denominator.

Hence, the value of the expression is greater than 1.

Thus, the question can be answered using statement B alone but not by
using statement A alone.

Thus, the question can be answered by one of the statement but not by the
other.

Hence, option 1.

9. Each question is followed by two statements, A and B. Select the correct


3 Marks option based on the following instructions:

Mark (1) if the question can be answered by one of the statements, but not by
the other.
Mark (2) if the question can be answered by using
either statement, independently of the other.
Mark (3) if the question can be answered by using both statements together,
but not by either statement alone.
Mark (4) if the question cannot be answered by either of the statements.
Is it more profitable for Company M to produce Q?

A. Product R sells at a price four times that of Q.


B. One unit of Q requires 2 units of labour, while one unit of R requires 5 units of
labour. There is no other constraint on production.

1) 1
2) 2
3) 3
4) 4

Solution:
Using Statement A alone:

Though the selling price of Q and R can be compared, the cost and effort
involved in producing Q and R is not known.

Hence, it cannot be said whether it is profitable for Company M to produce Q.

11 of 39 9/16/2011 10:20 AM
Exam Reports http://testfunda.com/LMS/Student/NewReports.aspx

Thus, the question cannot be answered using statement A alone.

Using Statement B alone:

Though the effort involved in producing Q and R is known, the selling price of
Q and R cannot be compared.

Hence, it cannot be said whether it is profitable for Company M to produce Q.

Thus, the question cannot be answered using statement B alone.

Using both the statements together:

Since there is no other constraint, except labour, the selling price can be
assumed to be directly proportional to the amount of labour involved.

Since R sells at a price four times that of Q, R should ideally require 8 units
of labour for every 2 units of labour needed by Q.

However, R actually requires only 5 units of labour.

Thus, R uses less labour than required and gets a higher selling price.

Hence, it is more profitable for the company to produce R and not Q.

Thus, the question can be answered using both statements together, but not
by using either statement alone.

Hence, option 3.

10. Each question is followed by two statements, A and B. Select the correct
3 Marks option based on the following instructions:

Mark (1) if the question can be answered by one of the statements, but not by
the other.
Mark (2) if the question can be answered by using
either statement, independently of the other.
Mark (3) if the question can be answered by using both statements together,
but not by either statement alone.
Mark (4) if the question cannot be answered by either of the statements.
What is the value of the prime number x?

A. x2 + x is a two-digit number greater than 50.


B. x3 is a three-digit number.

1) 1

12 of 39 9/16/2011 10:20 AM
Exam Reports http://testfunda.com/LMS/Student/NewReports.aspx

2) 2
3) 3
4) 4

Solution:
Using Statement A alone:

x2 + x > 50

∴ x(x + 1) > 50

For x = 5; x(x + 1) = 5 × 6 = 30

For x = 7; x(x + 1) = 7 × 8 = 56

For x = 11; x(x + 1) = 11 × 12 = 132

Thus, the only prime number that satisfies the twin conditions of x2 + x being
a two-digit number that is greater than 50 is 7.

Thus, the question can be answered using statement A alone.

Using Statement B alone:

100 ≤ x3 ≤ 999

For x = 5; x3 = 125

For x = 7; x3 = 343

For x = 11; x3 = 1331

Thus, both x = 5 and x = 7 are valid values.

Thus, a unique value of x cannot be found.

Thus, the question cannot be answered using statement B alone.

Thus, the question can be answered using statement A alone but not by
using statement B alone.

Thus, the question can be answered by one of the statements but not by the
other.

13 of 39 9/16/2011 10:20 AM
Exam Reports http://testfunda.com/LMS/Student/NewReports.aspx

Hence, option 1.

11. Each question is followed by two statements, A and B. Select the correct
3 Marks option based on the following instructions:

Mark (1) if the question can be answered by one of the statements, but not by
the other.
Mark (2) if the question can be answered by using
either statement, independently of the other.
Mark (3) if the question can be answered by using both statements together,
but not by either statement alone.
Mark (4) if the question cannot be answered by either of the statements.
The average of three unequal quotations for a particular share is Rs. 110. If
all the amounts are quoted in integral rupee values, does the highest
quotation exceed Rs. 129?

A. The lowest quotation is Rs. 100.


B. One of the quotations is Rs. 115.

1) 1
2) 2
3) 3
4) 4

Solution:
Using Statement A alone:

The sum of the value of the three quotations is 3 × 110 = Rs. 330

The lowest quotation is Rs. 101 and no two quotations are equal.

The maximum possible value of any quotation can be obtained when the
second highest quotation is Rs. 101

In such a case, the maximum possible value of a quotation = 330 − (100 +


101) = Rs. 129.

Thus, the highest quotation does not exceed Rs. 129.

Thus, the question can be answered using statement A alone.

Using Statement B alone:

One of the quotations is Rs. 115.

14 of 39 9/16/2011 10:20 AM
Exam Reports http://testfunda.com/LMS/Student/NewReports.aspx

Therefore, the sum of the other two quotations is Rs. 215

Since the lowest quotation value is not known, the highest quotation value
can be anything.

Thus, the question cannot be answered using statement B alone.

Thus, the question can be answered using statement A alone but not by
using statement B alone.

Thus, the question can be answered by one of the statements but not by the
other.

Hence, option 1.

12. Each question is followed by two statements, A and B. Select the correct
3 Marks option based on the following instructions:

Mark (1) if the question can be answered by one of the statements, but not by
the other.
Mark (2) if the question can be answered by using
either statement, independently of the other.
Mark (3) if the question can be answered by using both statements together,
but not by either statement alone.
Mark (4) if the question cannot be answered by either of the statements.
How many people (from a group surveyed) read both Indian Express and
Times of India?

A. Out of a total of 200 readers, 100 read Indian Express, 120 read Times of India and 50
read Hindu.
B. Out of a total of 200 readers, 100 read Indian Express, 120 read Times of India and 50
read neither.

1) 1
2) 2
3) 3
4) 4

Solution:
Using Statement A alone:

100 people read Indian Express, 120 read Times of India and 50 read Hindu.

Since this involves three newspapers, the number of people who read both
indian Express and Times of India cannot be found.

15 of 39 9/16/2011 10:20 AM
Exam Reports http://testfunda.com/LMS/Student/NewReports.aspx

Thus, the question cannot be answered using statement A alone.

Using Statement B alone:

If the survey comprises only two newspapers (Indian Express and Times of
India), the number of people who read at least one newspaper is 150.

Therefore, the number of people who read both newspapers is 100 + 120
− 150 = 70

If the survey comprises three newspapers (including Hindu), the 50 who read
neither Times of India nor Indian Express consists of those who read only
Hindu and those who do not read all three newspapers.

Since this breakup is not known, the number of people who read both
newspapers cannot be found.

Thus, the question cannot be answered using statement B alone.

Using both the statements together:

Even when the data is combined, the number of people who read both
newspapers cannot be found.

Thus, the question cannot be answered by either statement.

Hence, option 4.

13. Each question is followed by two statements, A and B. Select the correct
3 Marks option based on the following instructions:

Mark (1) if the question can be answered by one of the statements, but not by
the other.
Mark (2) if the question can be answered by using
either statement, independently of the other.
Mark (3) if the question can be answered by using both statements together,
but not by either statement alone.
Mark (4) if the question cannot be answered by either of the statements.
X says to Y, “I am 3 times as old as you were 3 years ago”. How old is X?

A. Y’s age 17 years from now would be the same as X’s present age.
B. X’s age nine years from now is 3 times Y’s present age.

1) 1
2) 2

16 of 39 9/16/2011 10:20 AM
Exam Reports http://testfunda.com/LMS/Student/NewReports.aspx

3) 3
4) 4

Solution:
Let the current ages of X and Y be x and y years respectively.

∴ x = 3(y − 3)

∴ 3y – x = 9 …(i)

Using Statement A alone:

y + 17 = x

∴ x – y = 17 …(ii)

Solving (i) and (ii), you get x = 30 and y = 13

Thus, X is 30 years old.

Thus, the question can be answered using statement A alone.

Using Statement B alone:

x + 9 = 3y

∴ 3y − x = 9 …(iii)

Since equations (iii) and (i) are the same, the value of x and y cannot be
found.

Thus, the question cannot be answered using statement B alone.

Thus, the question can be answered using statement A alone but not by
using statement B alone.

Thus, the question can be answered by one of the statements, but not by the
other.

Hence, option 1.

14. Each question is followed by two statements, A and B. Select the correct
3 Marks option based on the following instructions:

Mark (1) if the question can be answered by one of the statements, but not by
the other.

17 of 39 9/16/2011 10:20 AM
Exam Reports http://testfunda.com/LMS/Student/NewReports.aspx

Mark (2) if the question can be answered by using


either statement, independently of the other.
Mark (3) if the question can be answered by using both statements together,
but not by either statement alone.
Mark (4) if the question cannot be answered by either of the statements.
What is the area under the lines GHI and JKL in the given quadrilateral
OPQR, if it is known that all the small spaces are squares of the same area?

A. Length ABCDEQ is greater than or equal to 60.


B. Area OPQR is less than or equal to 1512.

1) 1
2) 2
3) 3
4) 4

Solution:
Let the side of each small square be a units.

Thus, the area under the lines GHI and JKL is (3a × 2a) + (a × 2a) = 8a2

Using Statement A alone:

Length ABCDEQ is greater than or equal to 60

∴ 10a ≥ 60

∴a≥6

Since the exact value of a is not known, the required area cannot be found.

Thus, the question cannot be answered using statement A alone.

18 of 39 9/16/2011 10:20 AM
Exam Reports http://testfunda.com/LMS/Student/NewReports.aspx

Using Statement B alone:

Area OPQR is less than or equal to 1512.

∴ 42a2 ≤ 1512

∴ a2 ≤ 36

∴a≤6

Since the exact value of a is not known, the required area cannot be found.

Thus, the question cannot be answered using statement B alone.

Using both the statements together:

When both the statements are combined, the only possible value of a is 6.

Hence, the required area is 8 × 62 = 288

Thus, the question can be answered using both the statements together but
not by using either statement alone.

Hence, option 3.

15. Each question is followed by two statements, A and B. Select the correct
3 Marks option based on the following instructions:

Mark (1) if the question can be answered by one of the statements, but not by
the other.
Mark (2) if the question can be answered by using
either statement, independently of the other.
Mark (3) if the question can be answered by using both statements together,
but not by either statement alone.
Mark (4) if the question cannot be answered by either of the statements.
What is the radius of the circle?

A. The ratio of its area to its circumference is greater than 7.


B. Diameter of the circle ≤ 32

1) 1
2) 2
3) 3

19 of 39 9/16/2011 10:20 AM
Exam Reports http://testfunda.com/LMS/Student/NewReports.aspx

4) 4

Solution:
Using Statement A alone:

(πr2)/(2πr) > 7

∴ r/2 > 7

∴ r > 14

Thus, the radius can be any value greater than 14, but the exact value cannot
be found.

Thus, the question cannot be answered using statement A alone.

Using Statement B alone:

d ≤ 32

∴ r ≤ 16

Thus, the radius can be any value up to 16, but the exact value cannot be
found.

Thus, the question cannot be answered using statement B alone.

Using both the statements together:

When both statements are combined, the possible integral values of r are 14
and 15.

A unique value of r can still not be found.

Thus, the question cannot be answered by either statement.

Hence, option 4.

16. Each question is followed by two statements, A and B. Select the correct
3 Marks option based on the following instructions:

Mark (1) if the question can be answered by one of the statements, but not by
the other.
Mark (2) if the question can be answered by using
either statement, independently of the other.
Mark (3) if the question can be answered by using both statements together,
but not by either statement alone.

20 of 39 9/16/2011 10:20 AM
Exam Reports http://testfunda.com/LMS/Student/NewReports.aspx

Mark (4) if the question cannot be answered by either of the statements.


What is the time difference between New York and London, if according to
GMT sun rises in London before New York?

A. The departure time of the flight at New York is exactly 9.00 a.m. local time and the
arrival time at London is 10.00 a.m. local time.
B. The flight time is 5 hours.

1) 1
2) 2
3) 3
4) 4

Solution:
Using Statement A alone:

As the duration of the journey is not known, the time difference cannot be
calculated.

Thus, the question cannot be answered using statement A alone.

Using Statement B alone:

As the time of departure and arrival is not known, the time difference cannot
be calculated.

Thus, the question cannot be answered using statement B alone.

Using both the statements together:

The flight departs at 9.00 a.m. New York time and arrives at 2 p.m. London
time.

The local time of arrival in London is 10.00 a.m.

Thus, if London is ahead of New York, the time difference is 4 hours while
London is behind New York, the time difference is 20 hours.

Since London is ahead of New York, the time difference is 4 hours.

Thus, the question can be answered using both the statements together but
not by using either statement alone.

Hence, option 3.

21 of 39 9/16/2011 10:20 AM
Exam Reports http://testfunda.com/LMS/Student/NewReports.aspx

17. Each question is followed by two statements, A and B. Select the correct
3 Marks option based on the following instructions:

Mark (1) if the question can be answered by one of the statements, but not by
the other.
Mark (2) if the question can be answered by using
either statement, independently of the other.
Mark (3) if the question can be answered by using both statements together,
but not by either statement alone.
Mark (4) if the question cannot be answered by either of the statements.
Mr. Murthy takes the morning train to his office from station A to station B, and
his colleague Mr. Rahman joins him on the way. There are three stations C, D
and E on the way, not necessarily in that sequence. What is the sequence of
stations?

A. Mr. Rahman boards the train at D.


B. Mr. Thomas, who travels between C and D has two segments of his journey in
common with Mr. Murthy but none with Mr. Rahman.

1) 1
2) 2
3) 3
4) 4

Solution:
Using Statement A alone:

Mr. Rahman boards the train at D.

This does not give any indication about the sequence of stations.

Thus, the question cannot be answered using statement A alone.

Using Statement B alone:

Since the station at which Mr. Rahman boards the train is not known, the
sequence cannot be determined.

Thus, the question cannot be answered using statement B alone.

Using both the statements together:

Mr. Thomas boards the train at C and gets down at D.

Mr. Rahman boards the train at D.

22 of 39 9/16/2011 10:20 AM
Exam Reports http://testfunda.com/LMS/Student/NewReports.aspx

Since Mr. Thomas has 2 segments of his journey common with Mr. Murthy
and none with Mr. Rahman, the two common segments have to be CE and
ED.

Hence, the order of stations has to be A-C-E-D-B.

Thus, the question can be answered using both the statements together but
not by using either statement alone.

Hence, option 3.

18. Each question is followed by two statements, A and B. Select the correct
3 Marks option based on the following instructions:

Mark (1) if the question can be answered by one of the statements, but not by
the other.
Mark (2) if the question can be answered by using
either statement, independently of the other.
Mark (3) if the question can be answered by using both statements together,
but not by either statement alone.
Mark (4) if the question cannot be answered by either of the statements.
What is the length of the line SQ which is the diagonal of a square as well as
the diameter of a circle?

A. All four vertices of the square lie on the circumference of the circle.
B. The numerical value of the area of the circle is twice the length of SQ.

1) 1
2) 2
3) 3
4) 4

Solution:
Using Statement A alone:

The data given in this statement is the same as the data given in the
question.

Hence, the length of SQ cannot be found.

Thus, the question cannot be answered using statement A alone.

Using Statement B alone:

The numerical value of the area of the circle is twice the length of SQ, where

23 of 39 9/16/2011 10:20 AM
Exam Reports http://testfunda.com/LMS/Student/NewReports.aspx

SQ is the diameter of the circle.

∴ π × SQ2/4 = 2SQ

∴ SQ = 8/π

Thus, the value of SQ can be found.

Thus, the question can be answered using statement B alone.

Thus, the question can be answered using statement B alone but not by
using statement A alone.

Thus, the question can be answered by one of the statements but not by the
other.

Hence, option 1.

19. Each question is followed by two statements, A and B. Select the correct
3 Marks option based on the following instructions:

Mark (1) if the question can be answered by one of the statements, but not by
the other.
Mark (2) if the question can be answered by using
either statement, independently of the other.
Mark (3) if the question can be answered by using both statements together,
but not by either statement alone.
Mark (4) if the question cannot be answered by either of the statements.
P, Q, R and S are four consecutive even integers. What is the value of the
largest integer among these?

A. The average of the four numbers is the first prime number greater than 10.
B. The relation between the largest and smallest number is less than 10.

1) 1
2) 2
3) 3
4) 4

Solution:
Let the numbers P, Q, R and S be x − 3, x − 1, x + 1 and x + 3 respectively.

Hence, the average of these 4 numbers is x. The largest integer among these
is x + 3.

24 of 39 9/16/2011 10:20 AM
Exam Reports http://testfunda.com/LMS/Student/NewReports.aspx

Using Statement A alone:

The average of the four numbers is the first prime number greater than 10 i.e.
11.

∴ x = 11

Therefore, the largest integer among these is 11 + 3 = 14.

Thus, the question can be answered using statement A alone.

Using Statement B alone:

The relation between the largest and smallest number is less than 10.

This does not provide any information.

Thus, the question cannot be answered using statement B alone.

Thus, the question can be answered using statement A alone but not by
using statement B alone.

Thus, the question can be answered by one of the statements but not by the
other.

Hence, option 1.

20. Each question is followed by two statements, A and B. Select the correct
3 Marks option based on the following instructions:

Mark (1) if the question can be answered by one of the statements, but not by
the other.
Mark (2) if the question can be answered by using
either statement, independently of the other.
Mark (3) if the question can be answered by using both statements together,
but not by either statement alone.
Mark (4) if the question cannot be answered by either of the statements.
What is the price of an apple?

A. The average of the price of an apple and the price of an orange is Rs. 5.
B. 24 apples and 24 oranges cost Rs. 240.

1) 1
2) 2
3) 3

25 of 39 9/16/2011 10:20 AM
Exam Reports http://testfunda.com/LMS/Student/NewReports.aspx

4) 4

Solution:
Let the cost of an apple and an orange be a and r respectively.

Using Statement A alone:

(a + r)/2 = 5

∴ a + r = 10

Since this is a single linear equation in two variables, a unique value of a and
r cannot be found.

Thus, the question cannot be answered using statement A alone.

Using Statement B alone:

24a + 24r = 240

∴ a + r = 10

Since this is a single linear equation in two variables, a unique value of a and
r cannot be found.

Thus, the question cannot be answered using statement B alone.

Using both the statements together:

Since both the statements give the same equation, there is still only one
equation in terms of two variables.

Hence, the value of a and r cannot be found.

Thus, the question cannot be answered using either of the statements.

Hence, option 4.

21. Each question is followed by two statements, A and B. Select the correct
3 Marks option based on the following instructions:

Mark (1) if the question can be answered by one of the statements, but not by
the other.
Mark (2) if the question can be answered by using
either statement, independently of the other.
Mark (3) if the question can be answered by using both statements together,
but not by either statement alone.

26 of 39 9/16/2011 10:20 AM
Exam Reports http://testfunda.com/LMS/Student/NewReports.aspx

Mark (4) if the question cannot be answered by either of the statements.


How much money does Prem have?

A. Prem has at least 100 rupees more than Jagdish.


B. The total money that both Prem and Jagdish have is not more than 500 rupees.

1) 1
2) 2
3) 3
4) 4

Solution:
Let the money with Prem and Jagdish be p and j respectively.

Using Statement A alone:

p – j ≥ 100

However, the amount with Prem cannot be determined from this inequation.

Thus, the question cannot be answered using statement A alone.

Using Statement B alone:

p + j ≤ 500

However, the amount with Prem cannot be determined from this inequation.

Thus, the question cannot be answered using statement B alone.

Using both the statements together:

Even if both the inequations are combined, the exact amount with Prem
cannot be determined.

Thus, the question cannot be answered by either of the statements.

Hence, option 4.

22. Each question is followed by two statements, A and B. Select the correct
3 Marks option based on the following instructions:

Mark (1) if the question can be answered by one of the statements, but not by
the other.
Mark (2) if the question can be answered by using

27 of 39 9/16/2011 10:20 AM
Exam Reports http://testfunda.com/LMS/Student/NewReports.aspx

either statement, independently of the other.


Mark (3) if the question can be answered by using both statements together,
but not by either statement alone.
Mark (4) if the question cannot be answered by either of the statements.
Given that x and y are non-negative. What is the value of x?

A. 2x + 2y ≤ 40
B. x − 2y ≥ 20

1) 1
2) 2
3) 3
4) 4

Solution:
x ≥ 0, y ≥ 0

Using Statement A alone:

2x + 2y ≤ 40 ...(i)

The exact value of x cannot be obtained from this inequation.

Thus, the question cannot be answered using statement A alone.

Using Statement B alone:

x − 2y ≥ 20 ...(ii)

The exact value of x cannot be obtained from this inequation.

Thus, the question cannot be answered using statement B alone.

Using both the statements together:

From (i) we get x + y ≤ 20

Subtracting 3y from both the sides of the inequality we get x − 2y ≤ 20 – 3y,


which will be an integer less than 20 as y is a non negative integer.

From (ii) we get x − 2y ≥ 20

∴ from the above argument we get 20 ≤ x − 2y ≤ 20 – 3y, this will be


possible only when y = 0 and the equality holds.

28 of 39 9/16/2011 10:20 AM
Exam Reports http://testfunda.com/LMS/Student/NewReports.aspx

Thus, x = 20

Thus, the question can be answered by using both the statements.

Hence, option 3.

23. Each question is followed by two statements, A and B. Select the correct
3 Marks option based on the following instructions:

Mark (1) if the question can be answered by one of the statements, but not by
the other.
Mark (2) if the question can be answered by using
either statement, independently of the other.
Mark (3) if the question can be answered by using both statements together,
but not by either statement alone.
Mark (4) if the question cannot be answered by either of the statements.
Is the average of the largest and the smallest of four given numbers greater
than the average of four numbers?

A. The difference between the largest and the second largest numbers is greater than
the difference between the second smallest and the smallest numbers.
B. The difference between the largest and the second largest numbers is less than the
difference between the second smallest and the smallest numbers.

1) 1
2) 2
3) 3
4) 4

Solution:
If the numbers are a, b, c and d in the descending order,

Using Statement A alone:

If (a − b) > (c − d), then a + d > b + c

∴a+d+a+d>a+d+b+c

Dividing both the sides by 4,

29 of 39 9/16/2011 10:20 AM
Exam Reports http://testfunda.com/LMS/Student/NewReports.aspx

Thus, the average of the largest and smallest number is greater than the
average of all four numbers.

Thus, the question can be answered using statement A alone.

Using Statement B alone:

If (a − b) < (c − d), then a + d < b + c

∴a+d+a+d<a+d+b+c

Dividing both the sides by 4,

Thus, the average of the largest and smallest number is less than the
average of all four numbers.

Thus, the question can be answered using statement B alone.

Thus, the question can be answered using either statement, independently of


the other.

Hence, option 2.

24. Each question is followed by two statements, A and B. Select the correct
3 Marks option based on the following instructions:

Mark (1) if the question can be answered by one of the statements, but not by
the other.
Mark (2) if the question can be answered by using
either statement, independently of the other.
Mark (3) if the question can be answered by using both statements together,
but not by either statement alone.
Mark (4) if the question cannot be answered by either of the statements.
What are the ages of three brothers?

A. The product of their ages is 21.


B. The sum of their ages is not divisible by 3.

1) 1

30 of 39 9/16/2011 10:20 AM
Exam Reports http://testfunda.com/LMS/Student/NewReports.aspx

2) 2
3) 3
4) 4

Solution:
Using Statement A alone:

The product of the brothers' ages is 21.

There are two combinations that give this value i.e. (1, 3, 7) and (1, 1, 21).

Since a unique combination cannot be identified, the ages cannot be found.

Thus, the question cannot be answered using statement A alone.

Using Statement B alone:

The sum of their ages is not divisible by 3.

The ages cannot be calculated from this data.

Thus, the question cannot be answered using statement B alone.

Using both the statements together:

For the two combinations, the sum of the ages is 11 and 23 respectively.

Both these numbers are not divisible by 3.

Thus, both combinations are still valid.

Hence, the ages cannot be found.

Thus, the question cannot be answered by either of the statements.

Hence, option 4.

25. Each question is followed by two statements, A and B. Select the correct
3 Marks option based on the following instructions:

Mark (1) if the question can be answered by one of the statements, but not by
the other.
Mark (2) if the question can be answered by using
either statement, independently of the other.
Mark (3) if the question can be answered by using both statements together,
but not by either statement alone.

31 of 39 9/16/2011 10:20 AM
Exam Reports http://testfunda.com/LMS/Student/NewReports.aspx

Mark (4) if the question cannot be answered by either of the statements.


Two types of widgets, namely type A and B, are produced on a machine. The
number of machine hours available per week is 80. How many widgets of type
A must be produced?

A. One unit of the type A widget requires 2 machine hours and one unit of the type B
widget requires 4 machine hours.
B. The widget dealer wants to supply at least 10 units of type A widget per week and he
would not accept less than 15 units of type B widget.

1) 1
2) 2
3) 3
4) 4

Solution:
Let the number of type A and type B widgets be a and b respectively.

The number of machine hours available is 80.

Using Statement A alone:

One type A widget requires 2 machine hours while one type B widget requires
4 machine hours.

∴ 2a + 4b = 80

Since this is a linear equation in two variables, the value of a and b cannot be
found.

Thus, the question cannot be answered using statement A alone.

Using Statement B alone:

a ≥ 10 and b ≥ 15

However, this does not give the exact value of A.

Thus, the question cannot be answered using statement B alone.

Using both the statements together:

If the dealer makes the minimum number of widgets of each type, the time
that will be taken is:

10 units of type A require 20 hours.

32 of 39 9/16/2011 10:20 AM
Exam Reports http://testfunda.com/LMS/Student/NewReports.aspx

15 units of type B require 60 hours.

This consumes all 80 hours available.

Thus 10 widgets of type A must be produced.

Hence, option 3.

26. Each question is followed by two statements, A and B. Select the correct
3 Marks option based on the following instructions:

Mark (1) if the question can be answered by one of the statements, but not by
the other.
Mark (2) if the question can be answered by using
either statement, independently of the other.
Mark (3) if the question can be answered by using both statements together,
but not by either statement alone.
Mark (4) if the question cannot be answered by either of the statements.
What is the area of a regular hexagon?

A. The length of the boundary of the hexagon is 36 cm.


B. The area of the hexagon is 6 times the area of an equilateral triangle formed on one
of the sides.

1) 1
2) 2
3) 3
4) 4

Solution:
Using Statement A alone:

The perimeter of the hexagon is 36 cm.

Therefore, the side of the hexagon is 6 cm.

One the side of a regular hexagon is known, the area can be easily found.

Thus, the question can be answered using statement A alone.

Using Statement B alone:

For any regular hexagon, the area is always 6 times the area of an equilateral
triangle formed on one of the sides of the hexagon.

33 of 39 9/16/2011 10:20 AM
Exam Reports http://testfunda.com/LMS/Student/NewReports.aspx

Thus, this information does not provide the area of the hexagon.

Thus, the question cannot be answered using statement B alone.

Thus, the question can be answered using statement A alone but not by
using statement B alone.

Thus, the question can be answered by one of the statements but not by the
other.

Hence, option 1.

27. Each question is followed by two statements, A and B. Select the correct
3 Marks option based on the following instructions:

Mark (1) if the question can be answered by one of the statements, but not by
the other.
Mark (2) if the question can be answered by using
either statement, independently of the other.
Mark (3) if the question can be answered by using both statements together,
but not by either statement alone.
Mark (4) if the question cannot be answered by either of the statements.
What is the price of one kg mangoes?

A. 10 kgs of mangoes and two dozen oranges cost Rs. 252.


B. 2 kgs of mangoes can be bought in exchange for one dozen oranges.

1) 1
2) 2
3) 3
4) 4

Solution:
Let the cost of one kg mangoes be x and the cost of one dozen oranges be y.

Using Statement A alone:

10x + 2y = 252

Since this is a single linear equation in two variables, the value of x and y
cannot be found.

Thus, the question cannot be answered using statement A alone.

34 of 39 9/16/2011 10:20 AM
Exam Reports http://testfunda.com/LMS/Student/NewReports.aspx

Using Statement B alone:

2x = y

This is a single linear equation in two variables and so the value of x and y
cannot be calculated.

Thus, the question cannot be answered using statement B alone.

Using both the statements together:

Substitute the second equation in the first.

∴ 10x + 2(2x) = 252

∴ 14x = 252

∴ x = 18

Thus, the cost of one kg mangoes is Rs. 18

Thus, the question can be answered using both the statements together but
not by using either statement alone.

Hence, option 3.

28. Each question is followed by two statements, A and B. Select the correct
3 Marks option based on the following instructions:

Mark (1) if the question can be answered by one of the statements, but not by
the other.
Mark (2) if the question can be answered by using
either statement, independently of the other.
Mark (3) if the question can be answered by using both statements together,
but not by either statement alone.
Mark (4) if the question cannot be answered by either of the statements.
Is the distance from the office to home less than the distance from the cinema
hall to home?

A. The time taken to travel from home to the office is as much as the time taken from
home to the cinema hall, both distances being covered without stopping.
B. The road from the cinema hall to home is bad and the speed reduces as compared to
that on the road from home to the office.

1) 1
2) 2

35 of 39 9/16/2011 10:20 AM
Exam Reports http://testfunda.com/LMS/Student/NewReports.aspx

3) 3
4) 4

Solution:
Using Statement A alone:

The time from home to office = time from home to cinema hall.

Thus, Distance ∝ Speed.

Since the relationship between the two speeds is not given, the two distances
cannot be compared.

Thus, the question cannot be answered using statement A alone.

Using Statement B alone:

Speed from cinema hall to home < speed from home to office.

Since the relationship betwen the time taken on these two stretches is not
known, the two distances cannot be compared.

Thus, the question cannot be answered using statement B alone.

Using both the statements together.

Since time is constant, and the speed from home to the cinema hall is less
than the speed from home to the office, the distance from home to the
cinema hall is less than the distance from home to office.

Thus, the question can be answered by using both the statements together
but not by using either statement alone.

Hence, option 3.

29. Each question is followed by two statements, A and B. Select the correct
3 Marks option based on the following instructions:

Mark (1) if the question can be answered by one of the statements, but not by
the other.
Mark (2) if the question can be answered by using
either statement, independently of the other.
Mark (3) if the question can be answered by using both statements together,
but not by either statement alone.
Mark (4) if the question cannot be answered by either of the statements.
If twenty sweets are distributed among some boys and girls such that each

36 of 39 9/16/2011 10:20 AM
Exam Reports http://testfunda.com/LMS/Student/NewReports.aspx

girl gets two sweets and each boy gets three sweets, what is the number of
boys and girls?

A. The number of girls is not more than five.


B. If each girl gets three sweets and each boy gets two sweets, the total number of
sweets required for the children will still be the same.

1) 1
2) 2
3) 3
4) 4

Solution:
Let the number of boys and girls be B and G respectively.

2G + 3B = 20

Using statement A alone:

G≤5

If G = 1, B = 6

If G = 4, B = 4

Thus, two combinations of G and B are possible.

Hence, the number of boys and girls cannot be found.

Thus, the question cannot be answered using statement A alone.

Using statement B alone:

3G + 2B = 20

∴ 2G + 3B = 3G + 2B

∴G=B

Substituting this in any one of the two equations, you get, G = B = 4.

Thus, there are 4 boys and 4 girls.

Thus, the question can be answered using statement B alone.

Thus, the question can be answered using statement B alone but not by

37 of 39 9/16/2011 10:20 AM
Exam Reports http://testfunda.com/LMS/Student/NewReports.aspx

using statement A alone.

Thus, the question can be answered by one the statements but not by the
other.

Hence, option 1.

30. Each question is followed by two statements, A and B. Select the correct
3 Marks option based on the following instructions:

Mark (1) if the question can be answered by one of the statements, but not by
the other.
Mark (2) if the question can be answered by using
either statement, independently of the other.
Mark (3) if the question can be answered by using both statements together,
but not by either statement alone.
Mark (4) if the question cannot be answered by either of the statements.
If the selling price were to be increased by 10%, the sales would reduce by
10%. In what ratio would the profits change?

A. The cost price remains constant.


B. The cost price increases by 10%.

1) 1
2) 2
3) 3
4) 4

Solution:
If the original selling price is S and the sales is V, the total revenue is SV.

When the selling price becomes 1.1s and the sales reduce to 0.9V, the total
revenue is 0.99SV.

Thus, the revenue decreases by 1%.

Using Statement A alone.

Let the cost price be C.

Hence, the old profit is SV − CV and the new profit is 0.99SV − 0.9CV.

However, this does not indicate the necessary profit ratio.

Thus, the question cannot be answered using statement A alone.

38 of 39 9/16/2011 10:20 AM
Exam Reports http://testfunda.com/LMS/Student/NewReports.aspx

Using Statement B alone:

In the first case, the profit = (S − C) × V

In the second case, the profit = 1.1(S − C) × 0.9V = 0.99(S − C)V

Now, the required ratio can be found.

Thus, the question can be answered using statement B alone.

Thus, the question can be answered using statement B alone but not by
using statement A alone.

Thus, the question can be answered by one of the statements but not by the
other.

Hence, option 1.

39 of 39 9/16/2011 10:20 AM
Exam Reports http://testfunda.com/LMS/Student/NewReports.aspx

Section I

1. Each question is followed by two statements, A and B. Select the correct


3 Marks option based on the following instructions:

Mark (1) if the question can be answered by one of the statements, but not by
the other.
Mark (2) if the question can be answered by using
either statement, independently of the other.
Mark (3) if the question can be answered by using both statements together,
but not by either statement alone.
Mark (4) if the question cannot be answered by either of the statements.
Is segment PQ greater than segment RS?

A. PB > RE, BQ = ES
B. B is a point on PQ and E is a point on RS.

1) 1
2) 2
3) 3
4) 4

Solution:
Using Statement A alone:

If B is a point on PQ and E is a point on RS, then

PB + BQ > RE + ES

∴ PQ > RS

Since it is not known if B and E are on those segments, the above conclusion
may or may not be true.

Thus, the question cannot be answered using statement A alone.

Using Statement B alone:

B is a point on PQ and E is a point on RS.

However, this does not provide any other useful information.

Thus, the question cannot be answered using statement B alone.

1 of 31 9/16/2011 10:21 AM
Exam Reports http://testfunda.com/LMS/Student/NewReports.aspx

Using both the statements together:

Since it is known that B is a point on PQ and E is a point on RS, PQ > RS as


seen from statament A.

Thus, the question can be answered using both the statements together, but
not by using either statement alone.

Hence, option 3.

2. Each question is followed by two statements, A and B. Select the correct


3 Marks option based on the following instructions:

Mark (1) if the question can be answered by one of the statements, but not by
the other.
Mark (2) if the question can be answered by using
either statement, independently of the other.
Mark (3) if the question can be answered by using both statements together,
but not by either statement alone.
Mark (4) if the question cannot be answered by either of the statements.
Three boys had a few Coffee Bite toffees with them. The number of toffees
with the second boy was four more than those with the first and the number
of toffees with the third boy was four more than those with the second. How
many toffees were there in all?

A. The number of toffees with each of them is a multiple of 2.


B. The first boy ate up four toffees from what he had and the second boy ate up six
toffees from what had and the third boy gave them two toffees each from what he
had and the number of toffees remaining with each of them formed a geometric
progression.

1) 1
2) 2
3) 3
4) 4

Solution:
Let the number of toffees with the 1st, 2nd and the 3rd boy be (n − 4), n and (n
+ 4) respectively.

Using Statement A alone:

This statement does not provide any useful information.

Thus, the question cannot be answered using statement A alone.

2 of 31 9/16/2011 10:21 AM
Exam Reports http://testfunda.com/LMS/Student/NewReports.aspx

Using Statement B alone:

The first, second and third boy now have (n − 8), (n − 6) and n toffees
respectively.

(n − 8), (n − 6) and n are in G.P.

∴ (n − 6)2 = n(n − 8)

∴n=9

Thus, the question can be answered using statement B alone.

Thus, the question can be answered using statement B alone but not by
using statement A alone.

Thus, the question can be answered by one the statements but not by the
other.

Hence, option 1.

3. Each question is followed by two statements, A and B. Select the correct


3 Marks option based on the following instructions:

Mark (1) if the question can be answered by one of the statements, but not by
the other.
Mark (2) if the question can be answered by using
either statement, independently of the other.
Mark (3) if the question can be answered by using both statements together,
but not by either statement alone.
Mark (4) if the question cannot be answered by either of the statements.
Little Beau Peep lost her sheep. She couldn’t remember how many were
there. She knew that in the next year she would have 400 more sheep than
the number she had last year. How many sheep were there?

A. The number of sheep last year was 20% more than the year before that and this
simple rate of increase continued to be the same for the next 10 years.
B. The increase is compounded annually.

1) 1
2) 2
3) 3
4) 4

3 of 31 9/16/2011 10:21 AM
Exam Reports http://testfunda.com/LMS/Student/NewReports.aspx

Solution:
Let the number of sheep last year be x.

Using Statement A alone:

Since the number of sheep grows by 20% every year, the number of sheep
this year is 1.2x and the number of sheep next year is 1.44x.

∴ 1.44x = x + 400

∴ x = 909

Thus, she had 1091 sheep this year.

Thus, the question can be answered using statement A alone.

Using Statement B alone:

The rate of increase is not known.

Thus, the question cannot be answered using statement B alone.

Thus, the question can be answered using statement A alone but not by
using statament B alone.

Thus, the question can be answered by one the statements but not by the
other.

Hence, option 1.

4. Each question is followed by two statements, A and B. Select the correct


3 Marks option based on the following instructions:

Mark (1) if the question can be answered by one of the statements, but not by
the other.
Mark (2) if the question can be answered by using
either statement, independently of the other.
Mark (3) if the question can be answered by using both statements together,
but not by either statement alone.
Mark (4) if the question cannot be answered by either of the statements.
What will the total cost of creating a one foot border of tiles along the inside
edges of a room be?

A. The room is 48 feet in length and 50 feet in breadth.


B. Every tile costs Rs. 10.

4 of 31 9/16/2011 10:21 AM
Exam Reports http://testfunda.com/LMS/Student/NewReports.aspx

1) 1
2) 2
3) 3
4) 4

Solution:
Using Statement A alone:

Since no cost is known, the total tiling cost cannot be found.

Thus, the question cannot be answered using statement A alone.

Using Statement B alone:

Since the dimensions of the room and that of each tile are not known, the
total tiling cost cannot be found.

Thus, the question cannot be answered using statement B alone.

Using both the statements together:

Since the dimensions of an individual tile are not known, the total tiling cost
cannot be found.

Thus, the question cannot be answered by either of the statements.

Hence, option 4.

5. Each question is followed by two statements, A and B. Select the correct


3 Marks option based on the following instructions:

Mark (1) if the question can be answered by one of the statements, but not by
the other.
Mark (2) if the question can be answered by using
either statement, independently of the other.
Mark (3) if the question can be answered by using both statements together,
but not by either statement alone.
Mark (4) if the question cannot be answered by either of the statements.
Ten boys go to a neighbouring orchard. Each boy steals a few mangoes.
What is the total number of mangoes that they steal?

A. The first boy steals 4 mangoes, the fourth boy steals 16 mangoes, the eighth boy
steals 32 mangoes and the tenth boy steals 40 mangoes.
B. The first boy stole the minimum number of mangoes and the tenth boy stole the
maximum number of mangoes.

5 of 31 9/16/2011 10:21 AM
Exam Reports http://testfunda.com/LMS/Student/NewReports.aspx

1) 1
2) 2
3) 3
4) 4

Solution:
Using Statement A alone:

From the statement, it is possible that the number of mangoes stolen by the
nth peron is 4n.

However, it is not given that the number of mangoes stolen forms a series.

Thus, the remaining 6 boys could have stolen any number of mangoes.

Thus, the question cannot be answered using statement A alone.

Using Statement B alone:

This does not provide any useful information regarding number of mangoes
stolen.

Thus, the question cannot be answered using statement B alone.

Using both the statements together:

The only additional information obtained by combining the two statements is


that the other six boys could have stolen anywhere between 5 and 39
mangoes.

However, the actual number of mangoes stolen cnanot be obtained.

Thus, the question cannot be answered by either of the statements.

Hence, option 4.

6. Each question is followed by two statements, A and B. Select the correct


3 Marks option based on the following instructions:

Mark (1) if the question can be answered by one of the statements, but not by
the other.
Mark (2) if the question can be answered by using
either statement, independently of the other.
Mark (3) if the question can be answered by using both statements together,
but not by either statement alone.

6 of 31 9/16/2011 10:21 AM
Exam Reports http://testfunda.com/LMS/Student/NewReports.aspx

Mark (4) if the question cannot be answered by either of the statements.


If x, y and z are real numbers, is z - x even or odd?

A. xyz is odd.
B. xy + yz + zx is even.

1) 1
2) 2
3) 3
4) 4

Solution:
Using Statement A alone:

Given xyz is odd, it implies that each of x, y and z is odd.

If x and z are odd, then z – x is even.

Thus, the question can be answered using statement A alone.

Using Statement B alone:

Given xy + yz + xz is even, consider the following possibility.

If x is even and y is even, then even if z is odd or even, the given sum is even.

Hence, nothing can be concluded about z – x being odd or even.

Thus, the question cannot be answered using statement B alone.

Thus, the question can be answered using statement A alone but not by
using statement B alone.

Thus, the question can be answered by one of the statements but not by the
other.

Hence, option 1.

7. Each question is followed by two statements, A and B. Select the correct


3 Marks option based on the following instructions:

Mark (1) if the question can be answered by one of the statements, but not by
the other.
Mark (2) if the question can be answered by using
either statement, independently of the other.

7 of 31 9/16/2011 10:21 AM
Exam Reports http://testfunda.com/LMS/Student/NewReports.aspx

Mark (3) if the question can be answered by using both statements together,
but not by either statement alone.
Mark (4) if the question cannot be answered by either of the statements.
What is the value of x, if x and y are consecutive positive even integers?

A. (x − y)2 = 4
B. (x + y)2 < 100

1) 1
2) 2
3) 3
4) 4

Solution:
Since x and y are consecutive positive even integers, x − y = ± 2

Using Statement A alone:

Considering the equation obtained from the data given in the question, this
statement will always be true for any x and y that satisfy the above equation.

Thus, the question cannot be answered using statement A alone.

Using Statement B alone:

(x + y)2 < 100

Consider different sets of consecutive positive integers.

(2 + 4)2 = 36 < 100

(4 + 6)2 = 100

Thus, x has to be one out of 2 and 4.

However, it has been mentioned whether x is the greater or the lesser of the
two integers.

Hence, a unique value of x cannot be found.

Thus, the question cannot be answered using statement B alone.

Using both the statements together:

8 of 31 9/16/2011 10:21 AM
Exam Reports http://testfunda.com/LMS/Student/NewReports.aspx

Since statement A does not provide any additional data, this is the same as
using statement B alone.

Hence, the value of x still cannot be found.

Thus, the question cannot be answered by either statement.

Hence, option 4.

8. Each question is followed by two statements, A and B. Select the correct


3 Marks option based on the following instructions:

Mark (1) if the question can be answered by one of the statements, but not by
the other.
Mark (2) if the question can be answered by using
either statement, independently of the other.
Mark (3) if the question can be answered by using both statements together,
but not by either statement alone.
Mark (4) if the question cannot be answered by either of the statements.
What is the profit percent?

A. The cost price is 80% of the selling price.


B. The profit is Rs. 50.

1) 1
2) 2
3) 3
4) 4

Solution:
Using Statement A alone:

Let the selling price be Rs. 100.

Hence, the cost price is Rs. 80 and the profit is Rs. 20

Therefore, the profit percent = (20/80) × 100 = 25%.

Thus, the question can be answered using statement A alone.

Using Statement B alone:

Though the profit is known, the cost price or selling price is not known.

Hence, the profit percent cannot be found.

9 of 31 9/16/2011 10:21 AM
Exam Reports http://testfunda.com/LMS/Student/NewReports.aspx

Thus, the question cannot be answered using statement B alone.

Thus, the question can be answered using statement A alone but not by
using statement B alone.

Thus, the question can be answered by one of the statements but not by the
other.

Hence, option 1.

9. Each question is followed by two statements, A and B. Select the correct


3 Marks option based on the following instructions:

Mark (1) if the question can be answered by one of the statements, but not by
the other.
Mark (2) if the question can be answered by using
either statement, independently of the other.
Mark (3) if the question can be answered by using both statements together,
but not by either statement alone.
Mark (4) if the question cannot be answered by either of the statements.
What is the area of the triangle?

A. Two sides are 41 cm each.


B. The altitude to the third side is 9 cm long.

1) 1
2) 2
3) 3
4) 4

Solution:
Using Statement A alone:

Though two sides are known, the third side or the angle between these two
sides is not known.

Hence, the area cannot be found.

Thus, the question cannot be answered using statement A alone.

Using Statament B alone:

The area of a triangle cannot be found just using the altitude to a side, unless
the triangle is equilateral in nature.

10 of 31 9/16/2011 10:21 AM
Exam Reports http://testfunda.com/LMS/Student/NewReports.aspx

Nothing of that sort is known here.

Thus, the question cannot be answered using statement B alone.

Using both the statements together:

Once both the statements are combined, the triangle can be drawn as shown
below.

In the given triangle, ∠ADB = ∠ADC = 90°.

Thus, base BC = 2 × 40 = 80 cms.

Hence,

= 360 sq. cms.

Thus, the question can be answered using both the statements together but
not by using either statement alone.

Hence, option 3.

10. Each question is followed by two statements, A and B. Select the correct
3 Marks option based on the following instructions:

Mark (1) if the question can be answered by one of the statements, but not by
the other.
Mark (2) if the question can be answered by using
either statement, independently of the other.
Mark (3) if the question can be answered by using both statements together,
but not by either statement alone.
Mark (4) if the question cannot be answered by either of the statements.

11 of 31 9/16/2011 10:21 AM
Exam Reports http://testfunda.com/LMS/Student/NewReports.aspx

What is the price of a banana?

A. With Rs. 84, I can buy 14 bananas and 35 oranges.


B. If the price of bananas is reduced by 50% then I can buy 48 bananas in Rs. 12.

1) 1
2) 2
3) 3
4) 4

Solution:
Using Statement A alone:

Let the price of one banana be Rs. x and the price of an orange be Rs. y.

∴ 14x + 35y = 84

∴ 2x + 5y = 12

It is not given that the price of the banana or orange needs to be an integer.

Hence, there can be multiple solution sets possible.

Therefore, the price of a banana cannot be found.

Thus, the question cannot be answered using statement A alone.

Using Statement B alone:

I can buy 48 bananas in Rs. 12.

Hence, one banana costs Rs. 0.25 as per the new price.

This price is half of the old price.

Thus, the price of a banana is Rs. 0.5

Thus, the question can be answered using statement B alone.

Thus, the question can be answered using statement B alone but not by
using statement A alone.

Thus, the question can be answered by one statement but not by the other.

Hence, option 1.

12 of 31 9/16/2011 10:21 AM
Exam Reports http://testfunda.com/LMS/Student/NewReports.aspx

11. Each question is followed by two statements, A and B. Select the correct
3 Marks option based on the following instructions:

Mark (1) if the question can be answered by one of the statements, but not by
the other.
Mark (2) if the question can be answered by using
either statement, independently of the other.
Mark (3) if the question can be answered by using both statements together,
but not by either statement alone.
Mark (4) if the question cannot be answered by either of the statements.
What is the perimeter of the rectangle ABCD?

A. Area of the rectangle is 48 square units.


B. Length of the diagonal is 10 units.

1) 1
2) 2
3) 3
4) 4

Solution:
Using Statement A alone:

Area = Length × Breadth = 48

Multiple combinations such as (1, 48), (2, 24), (3, 16), (4, 12), (6, 8) are
possible.

Hence, the perimeter cannot be found.

Thus, the question cannot be answered using statement A alone.

Using Statement B alone.

Since the diagonal measures 10 units, (Length)2 + (Breadth)2 = 100

There can be multiple solutions to this equation.

Hence, the perimeter cannot be found.

Thus, the question cannot be answered using statement B alone.

Using both the statements together:

Let l and b denote length and breadth of the given rectangle respectively.

13 of 31 9/16/2011 10:21 AM
Exam Reports http://testfunda.com/LMS/Student/NewReports.aspx

lb = 48 and l2 + b2 = 100

(l2) +(b2) − 2lb =(l − b)2 = 100 − (2 × 48) = 4

∴ (l − b) = 2

l 2 + b2 + 2lb = (l + b)2 = 100 + (2 × 48) = 196

∴ (l + b) = 14

∴ l = 8, b = 6

Thus, the perimeter is 28 units.

Thus, the question can be answered using both the statements together but
not by using either statement alone.

Hence, option 3.

12. Each question is followed by two statements, A and B. Select the correct
3 Marks option based on the following instructions:

Mark (1) if the question can be answered by one of the statements, but not by
the other.
Mark (2) if the question can be answered by using
either statement, independently of the other.
Mark (3) if the question can be answered by using both statements together,
but not by either statement alone.
Mark (4) if the question cannot be answered by either of the statements.
What is the number x?

A. The L.C.M. of x and 18 is 36.


B. The H.C.F. of x and 18 is 2.

1) 1
2) 2
3) 3
4) 4

Solution:
Using Statement A alone:

LCM of x and 18 is 36.

14 of 31 9/16/2011 10:21 AM
Exam Reports http://testfunda.com/LMS/Student/NewReports.aspx

The possible combinations are:

(i) LCM of 4 and 18 is 36.

(ii) LCM of 12 and 18 is 36.

(iii) LCM of 36 and 18 is 36.

Thus, there is no unique value for x.

Thus, the question cannot be answered using statement A alone.

Using Statement B alone:

HCF of x and 18 is 2.

The possible combinations are:

(i) HCF of 2 and 18 is 2.

(ii) HCF of 4 and 18 is 2.

(iii) HCF of 8 and 18 is 2.

and so on.

Thus, there is no unique value for x.

Thus, the question cannot be answered using statement B alone.

Using both the statements together:

x × 18 = 36 × 2

∴ x = 72/18 = 4

Thus, the question can be answered using both the statements together but
not by using either statement alone.

Hence, option 3.

13. Each question is followed by two statements, A and B. Select the correct
3 Marks option based on the following instructions:

Mark (1) if the question can be answered by one of the statements, but not by
the other.
Mark (2) if the question can be answered by using

15 of 31 9/16/2011 10:21 AM
Exam Reports http://testfunda.com/LMS/Student/NewReports.aspx

either statement, independently of the other.


Mark (3) if the question can be answered by using both statements together,
but not by either statement alone.
Mark (4) if the question cannot be answered by either of the statements.
Is x + y – z + t even?

A. x + y + t is even.
B. tz is odd.

1) 1
2) 2
3) 3
4) 4

Solution:
Using Statement A alone:

x + y + t is even.

Hence, x + y − z + t becomes Even − z

If z is even, this difference will be even and vice versa.

Since z is not known, nothing can be said about the difference.

Thus, the question cannot be answered using statement A alone.

Using Statement B alone:

tz is odd.

This is possible when both t and z are odd.

However, nothing is known about x and y.

Hence, nothing can be said about the difference.

Thus, the question cannot be answered using statement B alone.

Using both the statements together:

Since z is odd, x + y − z + t will become Even − Odd i.e. Odd.

Thus, the question can be answered using both the statements together but
not by using either statement alone.

16 of 31 9/16/2011 10:21 AM
Exam Reports http://testfunda.com/LMS/Student/NewReports.aspx

Hence, option 3.

14. Each question is followed by two statements, A and B. Select the correct
3 Marks option based on the following instructions:

Mark (1) if the question can be answered by one of the statements, but not by
the other.
Mark (2) if the question can be answered by using
either statement, independently of the other.
Mark (3) if the question can be answered by using both statements together,
but not by either statement alone.
Mark (4) if the question cannot be answered by either of the statements.
A tractor travelled a distance of 5 m. What is the radius of the rear wheel?

A. The front wheel rotates ‘N’ times more than the rear wheel over this distance.
B. The circumference of the rear wheel is ‘t’ times that of the front wheel.

1) 1
2) 2
3) 3
4) 4

Solution:
Using Statement A alone:

This simply means that the front wheel is smaller than the rear wheel.

Since the value of N and the radius of the front wheel is not known, the
radius of the rear wheel cannot be found.

Thus, the question cannot be answered using statement A alone.

Using Statement B alone:

Since the value of t is not known, the two wheels cannot be compared.

Thus, the question cannot be answered using statement B alone.

Using both the statements together:

Even when the two statements are combined, no numerical value is known.

Hence, the radius of the rear wheel cannot be found.

17 of 31 9/16/2011 10:21 AM
Exam Reports http://testfunda.com/LMS/Student/NewReports.aspx

Thus, the question cannot be answered by either of the statements.

Hence, option 4.

15. Each question is followed by two statements, A and B. Select the correct
3 Marks option based on the following instructions:

Mark (1) if the question can be answered by one of the statements, but not by
the other.
Mark (2) if the question can be answered by using
either statement, independently of the other.
Mark (3) if the question can be answered by using both statements together,
but not by either statement alone.
Mark (4) if the question cannot be answered by either of the statements.
What is the ratio of the two liquids A and B in the mixture finally, if these two
liquids kept in three vessels are mixed together? (The containers are of equal
volume).

A. The ratios of liquid A to liquid B in the first and second vessels is 3 : 5 and 2 : 3
respectively.
B. The ratio of liquid A to liquid B in vessel 3 is 4 : 3.

1) 1
2) 2
3) 3
4) 4

Solution:
Using Statement A alone:

There is no data regarding the third vessel.

Hence, the ratio cannot be found.

Thus, the question cannot be answered using statement A alone.

Using Statement B alone:

There is no data regarding the first and second vessel.

Hence, the ratio cannot be found.

Thus, the question cannot be answered using statement B alone.

Using both the statements together:

18 of 31 9/16/2011 10:21 AM
Exam Reports http://testfunda.com/LMS/Student/NewReports.aspx

The ratio in which the liquids from the three vessels are taken is not known.

Hence, the required ratio cannot be found.

Thus, the question cannot be answered by either of the statements.

Hence, option 4.

16. Each question is followed by two statements, A and B. Select the correct
3 Marks option based on the following instructions:

Mark (1) if the question can be answered by one of the statements, but not by
the other.
Mark (2) if the question can be answered by using
either statement, independently of the other.
Mark (3) if the question can be answered by using both statements together,
but not by either statement alone.
Mark (4) if the question cannot be answered by either of the statements.

If α, β are the roots of the equation ax2 + bx + c = 0, then what is the value of
(α2 + β2)?

A. α + β = (−b/a)
B. αβ = c/a

1) 1
2) 2
3) 3
4) 4

Solution:
Using Statement A alone:

The sum of the roots is always (−b/a)

Hence, the required value cannot be found.

Thus, the question cannot be answered using statement A alone.

Using Statement B alone:

The sum of the roots is always (c/a)

Hence, the required value cannot be found.

19 of 31 9/16/2011 10:21 AM
Exam Reports http://testfunda.com/LMS/Student/NewReports.aspx

Thus, the question cannot be answered using statement B alone.

Using both the statements together:

Thus, the question can be answered using both the statements together but
not by using either statement alone.

Hence, option 3.

17. Each question is followed by two statements, A and B. Select the correct
3 Marks option based on the following instructions:

Mark (1) if the question can be answered by one of the statements, but not by
the other.
Mark (2) if the question can be answered by using
either statement, independently of the other.
Mark (3) if the question can be answered by using both statements together,
but not by either statement alone.
Mark (4) if the question cannot be answered by either of the statements.
What is the cost price of the article?

A. After selling the article, a loss of 25% is incurred on the cost price.
B. The selling price is three-fourths of the cost price.

1) 1
2) 2
3) 3
4) 4

Solution:
Using Statement A alone:

S.P. = 0.75 × C.P.

Since neither the selling price nor the loss is known, the cost price cannot be
found.

20 of 31 9/16/2011 10:21 AM
Exam Reports http://testfunda.com/LMS/Student/NewReports.aspx

Thus, the question cannot be answered using statement A alone.

Using Statement B alone:

S.P. = 0.75 × C.P.

Since neither the selling price nor the loss is known, the cost price cannot be
found.

Thus, the question cannot be answered using statement B alone.

Using both the statements together:

Since both the statements provide the same information, the cost price
cannot be found.

Thus, the question cannot be answered by either of the statements.

Hence, option 4.

18. Each question is followed by two statements, A and B. Select the correct
3 Marks option based on the following instructions:

Mark (1) if the question can be answered by one of the statements, but not by
the other.
Mark (2) if the question can be answered by using
either statement, independently of the other.
Mark (3) if the question can be answered by using both statements together,
but not by either statement alone.
Mark (4) if the question cannot be answered by either of the statements.
What is the Selling Price of the article?

A. The profit on Sales is 20%.


B. The profit on each unit is 25% and the cost price is Rs. 250.

1) 1
2) 2
3) 3
4) 4

Solution:
Using Statement A alone:

The profit on sales is 20%, but the actual profit is not known.

21 of 31 9/16/2011 10:21 AM
Exam Reports http://testfunda.com/LMS/Student/NewReports.aspx

Thus, the question cannot be answered using statement A alone.

Using Statement B alone:

Cost Price = Rs. 250

Profit = 250/4 = Rs. 62.5

∴ Selling Price = 250 + 62.5 = Rs. 312.5

Thus, the question can be answered using statement B alone.

Thus, the question can be answered using statement B alone but not by
using statement A alone.

Thus, the question can be answered by one of the statements but not by the
other.

Hence, option 1.

19. Each question is followed by two statements, A and B. Select the correct
3 Marks option based on the following instructions:

Mark (1) if the question can be answered by one of the statements, but not by
the other.
Mark (2) if the question can be answered by using
either statement, independently of the other.
Mark (3) if the question can be answered by using both statements together,
but not by either statement alone.
Mark (4) if the question cannot be answered by either of the statements.
How many different triangles can be formed?

A. There are 16 coplanar, straight lines.


B. No two lines are parallel.

1) 1
2) 2
3) 3
4) 4

Solution:
Using Statement A alone:

The number of parallel lines among the 16 given and the number of

22 of 31 9/16/2011 10:21 AM
Exam Reports http://testfunda.com/LMS/Student/NewReports.aspx

intersection points is not known.

Hence, the number of triangles cannot be found.

Thus, the question cannot be answered using statement A alone.

Using Statement B alone:

Since the number of lines is not known, the number of triangles cannot be
found.

Thus, the question cannot be answered using statement B alone.

Using both the statements together:

There are 16 coplanar lines that are non-parallel.

There are two cases:

(i) If all lines intersect at the same point, then no triangle can be formed.

(ii) If all the lines intersect at different points, then several triangles can be
formed, the number of which depends on the number of intersection points.

Thus, the number of triangles still cannot be found.

Thus, the question cannot be answered by either of the statements.

Hence, option 4.

20. Each question is followed by two statements, A and B. Select the correct
3 Marks option based on the following instructions:

Mark (1) if the question can be answered by one of the statements, but not by
the other.
Mark (2) if the question can be answered by using
either statement, independently of the other.
Mark (3) if the question can be answered by using both statements together,
but not by either statement alone.
Mark (4) if the question cannot be answered by either of the statements.
What is the total worth of Lakhiram’s assets?

A. A compound interest at 10% on his assets, followed by a tax of 4% on the interest,


fetched him Rs. 1,500 this year.
B. The interest is compounded once every four months.

1) 1

23 of 31 9/16/2011 10:21 AM
Exam Reports http://testfunda.com/LMS/Student/NewReports.aspx

2) 2
3) 3
4) 4

Solution:
Using Statement A alone:

It is not known whether the 10% interest is on a annual or non-annual basis.

Also, it is not known if this year is the first year of the interest or not. Since
this is a case of compound interest, the interest will change every year.

Hence, the worth cannot be calculated.

Thus, the question cannot be answered using statement A alone.

Using Statement B alone:

Nothing is known about the rate, principal, tenure etc.

Hence, the worth cannot be calculated.

Thus, the question cannot be answered using statement B alone.

Using both the statements together:

Whether the interest of 10% is on a per annum or half yearly or quarter yearly
basis is still not known.

Also, it is not known whether the given year is this year or not.

Hence, the worth cannot be calculated.

Thus, the question cannot be answered by either of the statements.

Hence, option 4.

21. Each question is followed by two statements, A and B. Select the correct
3 Marks option based on the following instructions:

Mark (1) if the question can be answered by one of the statements, but not by
the other.
Mark (2) if the question can be answered by using
either statement, independently of the other.
Mark (3) if the question can be answered by using both statements together,
but not by either statement alone.

24 of 31 9/16/2011 10:21 AM
Exam Reports http://testfunda.com/LMS/Student/NewReports.aspx

Mark (4) if the question cannot be answered by either of the statements.


How old is Sachin in 1997?

A. Sachin is 11 years younger than Anil whose age will be a prime number in 1998.
B. Anil’s age was a prime number in 1996.

1) 1
2) 2
3) 3
4) 4

Solution:
Using Statement A alone:

Anil’s age could be any prime number greater than 11 (13, 17, 19, 23, 29 ...)
in 1998.

Hence, a unique value of Sachin's age cannot be found.

Thus, the question cannot be answered using statement A alone.

Using Statement B alone:

Since there is no mention of Sachin, Sachin's age cannot be found.

Thus, the question cannot be answered using statement B alone.

Using both the statements together:

Anil’s age is a prime number in 1996 and 1998, thus his age in those two
years could be 11 and 13, 17 and 19, 29 and 31 and so on.

Thus, Sachin’s age in 1997 can have multiple possible values and so cannot
be uniquely determined.

Thus, the question cannot be answered by either of the statements.

Hence, option 4.

22. Each question is followed by two statements, A and B. Select the correct
3 Marks option based on the following instructions:

Mark (1) if the question can be answered by one of the statements, but not by
the other.
Mark (2) if the question can be answered by using

25 of 31 9/16/2011 10:21 AM
Exam Reports http://testfunda.com/LMS/Student/NewReports.aspx

either statement, independently of the other.


Mark (3) if the question can be answered by using both statements together,
but not by either statement alone.
Mark (4) if the question cannot be answered by either of the statements.
What is the number of type-2 widgets produced, if the total number of
widgets produced is 20,000?

A. If the production of type-1 widgets increases by 10% and that of type-2 decreases by
6%, the total production remains the same.
B. The ratio in which type-1 and type-2 widgets are produced is 2 : 1.

1) 1
2) 2
3) 3
4) 4

Solution:
Using Statement A alone:

Since the original production of any one type of widget is not known, the
number of type-2 widgets cannot be found.

Also, it is not known if there are any types of widgets also being
manufactured.

Thus, the question cannot be answered using statement A alone.

Using Statement B alone:

It is not known if there are other types of widgets also being manufactured.

Hence, the required value cannot be found.

Thus, the question cannot be answered using statement B alone.

Using both the statements together:

Even when the statements are combined, it is not known whether there are
other types of widgets produced.

Thus, the question cannot be answered by either of the statements.

Hence, option 4.

23. Each question is followed by two statements, A and B. Select the correct
3 Marks

26 of 31 9/16/2011 10:21 AM
Exam Reports http://testfunda.com/LMS/Student/NewReports.aspx

option based on the following instructions:

Mark (1) if the question can be answered by one of the statements, but not by
the other.
Mark (2) if the question can be answered by using
either statement, independently of the other.
Mark (3) if the question can be answered by using both statements together,
but not by either statement alone.
Mark (4) if the question cannot be answered by either of the statements.

What is the value of a3 + b3?

A. a 2 + b2 = 22
B. ab = 3

1) 1
2) 2
3) 3
4) 4

Solution:
a3 + b3 = (a + b)(a2 − ab + b2)

Using Statement A alone:

Since the value of a + b and ab is not known, the required value cannot be
found.

Thus, the question cannot be answered using statement A alone.

Using Statement B alone:

Since the value of a + b and a2 + b2 is not known, the required value cannot
be found.

Thus, the question cannot be answered using statement B alone.

Using both the statements together:

Now every term apart from a + b is known.

(a + b)2 = a2 + b2 + 2ab

Thus, (a + b)2 can be found.

27 of 31 9/16/2011 10:21 AM
Exam Reports http://testfunda.com/LMS/Student/NewReports.aspx

But, (a + b) will have two values, positive and negative.

Hence, a3 + b3 will also have two values.

Thus, a unique value cannot be found.

Thus, the question cannot be answered by either of the statements.

Hence, option 4.

From either statement I alone or statement II alone, we cannot find the values
of a and b for there is only one equation in each statement and there are two
unknowns.

Combining statements I and II, we have,

a2 + b2 = 22 …(i)

ab = 3 …(ii)

Substituting the value of a from equation (ii) in (i), we get

Solving equation (iii) will give us more than one value for a.

Hence, the combination of statements I and II also will not give us a unique
solution.

Hence, option 4.

24. Each question is followed by two statements, A and B. Select the correct
3 Marks option based on the following instructions:

Mark (1) if the question can be answered by one of the statements, but not by
the other.
Mark (2) if the question can be answered by using
either statement, independently of the other.
Mark (3) if the question can be answered by using both statements together,
but not by either statement alone.
Mark (4) if the question cannot be answered by either of the statements.
Is the three digit number completely divisible by 99?

A. The number is divisible by 9 and 11 simultaneously.


B. If the digits of the number are reversed, the number is divisible by 9 and 11.

28 of 31 9/16/2011 10:21 AM
Exam Reports http://testfunda.com/LMS/Student/NewReports.aspx

1) 1
2) 2
3) 3
4) 4

Solution:
Using Statement A alone:

The prime factors of 99 are 9 and 11.

Since the number is simultaneously divisible by 9 and 11, the number is


divisible by 99.

Thus, the question can be answered using statement A alone.

Using Statement B alone:

If the number obtained by reversing the digits is divisible by 99, then the
original number is also divisible by 99.

Let us consider the original number to be xyz

If zyx has to be divisible by 9, then z + y + x = 9n.

z + y + x is the same as x + y + z.

∴ x + y + z = 9n.

Thus, xyz is divisible by 9.

If zyx has to be divisible by 11, then (z + x − y) should either be 0 or a


multiple of 11.

(z + x − y) is the same as (x + z − y).

Thus, (x + z − y) is either 0 or a multiple of 11.

Thus, xyz is divisible by 11.

Thus, xyz is divisible by 9 and 11 i.e. 99.

Thus, the question can be answered using statement B alone.

Thus, the question can be answered using either statement alone.

29 of 31 9/16/2011 10:21 AM
Exam Reports http://testfunda.com/LMS/Student/NewReports.aspx

Hence, option 2.

25. Each question is followed by two statements, A and B. Select the correct
3 Marks option based on the following instructions:

Mark (1) if the question can be answered by one of the statements, but not by
the other.
Mark (2) if the question can be answered by using
either statement, independently of the other.
Mark (3) if the question can be answered by using both statements together,
but not by either statement alone.
Mark (4) if the question cannot be answered by either of the statements.
A person is walking from Mali to Pali, which lies to its North-East. What is the
distance between Mali and Pali?

A. When the person has covered 1/3rd of the distance, he is 3 km East and 1 km North
of Mali.
B. When the person has covered 2/3rd of the distance, he is 6 km East and 2 km North
of Mali.

1) 1
2) 2
3) 3
4) 4

Solution:
Using Statement A alone:

Since the person is 3 km East and 1 km North of Mali, the distance between
this point and the starting point can be found using Pythagoras Theorem.

This distance is 1/3rd of the total distance.

Hence, the total distance can be found.

Thus, the question can be answered using statement A alone.

Using Statement B alone:

Since the person is 6 km East and 2 km North of Mali, the distance between
this point and the starting point can be found using Pythagoras Theorem.

This distance is 2/3rd of the total distance.

Hence, the total distance can be found.

30 of 31 9/16/2011 10:21 AM
Exam Reports http://testfunda.com/LMS/Student/NewReports.aspx

Thus, the question can be answered using statement B alone.

Thus, the question can be answered using either statement alone.

Hence, option 2.

31 of 31 9/16/2011 10:21 AM
Exam Reports http://testfunda.com/LMS/Student/NewReports.aspx

Section I

1. Each question is followed by two statements, A and B. Select the correct


3 Marks option based on the following instructions:

Mark (1) if the question can be answered by one of the statements, but not by
the other.
Mark (2) if the question can be answered by using
either statement, independently of the other.
Mark (3) if the question can be answered by using both statements together,
but not by either statement alone.
Mark (4) if the question cannot be answered by either of the statements.
What is the value of x and y?

A. 3x + 2y = 45
B. 10.5x + 7y = 157.5

1) 1
2) 2
3) 3
4) 4

Solution:
Using Statement A alone:

3x + 2y = 45.

Since this is a single equation in two variables, the value of x and y cannot be
found.

Thus, the question cannot be answered using statement A alone.

Using Statement B alone:

10.5x + 7y = 157.5.

Since this is a single equation in two variables, the value of x and y cannot be
found.

Thus, the question cannot be answered using statement B alone.

Using both the statements together:

The second equation, if divided by 3.5 gives the same co-efficients as the first

1 of 20 9/16/2011 10:21 AM
Exam Reports http://testfunda.com/LMS/Student/NewReports.aspx

equation.

Thus, the two equations cannot be solved to get a value for x and y.

Thus, the question cannot be answered by either of the statements.

Hence, option 4.

2. Each question is followed by two statements, A and B. Select the correct


3 Marks option based on the following instructions:

Mark (1) if the question can be answered by one of the statements, but not by
the other.
Mark (2) if the question can be answered by using
either statement, independently of the other.
Mark (3) if the question can be answered by using both statements together,
but not by either statement alone.
Mark (4) if the question cannot be answered by either of the statements.
Three friends, P, Q and R are wearing hats, either black or white. Each
person can see the hats of the other two persons. What is the colour of P’s
hat?

A. P says that he can see one black hat and one white hat.
B. Q says that he can see one white hat and one black hat.

1) 1
2) 2
3) 3
4) 4

Solution:
Using Statement A alone:

Since the number of black and white hats in all is not specified, the colour of
P's hat cannot be determined.

Thus, the question cannot be answered using statement A alone.

Using Statement B alone:

Since the number of black and white hats in all is not specified, the colour of
P's hat cannot be determined.

Thus, the question cannot be answered using statement B alone.

2 of 20 9/16/2011 10:21 AM
Exam Reports http://testfunda.com/LMS/Student/NewReports.aspx

Using both the statements together:

There are two cases possible:

Case 1: Q is wearing a white hat.

Here, both P and Q see R wearing the black hat.

Hence, the other white hat is worn by P.

Thus, P's hat is white.

Case 2: Q is wearing a black hat.

Here, both P and Q see R wearing the white hat.

Hence, the other black hat is worn by P.

Thus, P's hat is black.

Thus, the colour of P's hat can still not be determined.

Thus, the question cannot be answered by either of the statements.

Hence, option 4.

3. Each question is followed by two statements, A and B. Select the correct


3 Marks option based on the following instructions:

Mark (1) if the question can be answered by one of the statements, but not by
the other.
Mark (2) if the question can be answered by using
either statement, independently of the other.
Mark (3) if the question can be answered by using both statements together,
but not by either statement alone.
Mark (4) if the question cannot be answered by either of the statements.
What is the speed of the car?

A. The speed of the car is 10 kmph more than that of the motor-cycle.
B. The motor-cycle takes 2 hours more than the car to cover 100 km.

1) 1
2) 2
3) 3
4) 4

3 of 20 9/16/2011 10:21 AM
Exam Reports http://testfunda.com/LMS/Student/NewReports.aspx

Solution:
Let the speed of the car and the motor-cycle be c and m kmph respectively.

Using Statement A alone:

c = m + 10

Since the speed of the motor-cycle or the time required to cover a constant
distance is not known, the speed of the car cannot be calculated.

Thus, the question cannot be answered using statement A alone.

Using statement B alone:

(100/m) − (100/c) = 2

Since the speed of the motor-cycle is not known, the speed of the car cannot
be calculated.

Thus, the question cannot be answered using statement B alone.

Using both the statements together:

c = m + 10 …(i)

(100/m) − (100/c) = 2 ...(ii)

Substituting the value of m in terms of c in equation (ii),

c2 − 10c – 500 = 0

Solving the above quadratic question, you get a unique answer for c.

Thus, the question can be answered using both the statements together but
not by using either statement alone.

Hence, option 3.

4. Each question is followed by two statements, A and B. Select the correct


3 Marks option based on the following instructions:

Mark (1) if the question can be answered by one of the statements, but not by
the other.
Mark (2) if the question can be answered by using

4 of 20 9/16/2011 10:21 AM
Exam Reports http://testfunda.com/LMS/Student/NewReports.aspx

either statement, independently of the other.


Mark (3) if the question can be answered by using both statements together,
but not by either statement alone.
Mark (4) if the question cannot be answered by either of the statements.
What is the cost price of the chair?

A. The chair and the table are sold, respectively, at profits of 15% and 20%.
B. If the cost price of the chair is increased by 10% and that of the table is increased by
20%, the profit reduces by Rs.20.

1) 1
2) 2
3) 3
4) 4

Solution:
Using Statement A alone:

Though the profit percentages are known, neither the actual profit nor the
actual selling price is known.

Hence, the cost price cannot be calculated.

Thus, the question cannot be answered using statement A alone.

Using Statement B alone:

The change (if any) in the selling price is not known.

Also, the original profit is not known.

Hence, the cost price cannot be calculated.

Thus, the question cannot be answered using statement B alone.

Using both the statements together:

There is no information on the impact on selling price. The original profit is


also not known.

Hence, the cost price cannot be found.

Thus, the question cannot be answered by either of the statements.

Hence, option 4.

5 of 20 9/16/2011 10:21 AM
Exam Reports http://testfunda.com/LMS/Student/NewReports.aspx

5. Each question is followed by two statements, A and B. Select the correct


3 Marks option based on the following instructions:

Mark (1) if the question can be answered by one of the statements, but not by
the other.
Mark (2) if the question can be answered by using
either statement, independently of the other.
Mark (3) if the question can be answered by using both statements together,
but not by either statement alone.
Mark (4) if the question cannot be answered by either of the statements.
After what time will the two persons, Tez and Gati, meet while moving around
the circular track? Both of them start at the same point and at the same time.

A. Tez moves at a constant speed of 5 m/s, while Gati starts at a speed of 2 m/s and
increases his speed by 0.5 m/s at the end of every second thereafter.
B. Gati can complete one entire round in exactly 10 seconds.

1) 1
2) 2
3) 3
4) 4

Solution:
Using Statement A alone:

Since the circumference of the track is not known, the time cannot be
calculated.

Thus, the question cannot be answered using statement A alone.

Using Statement B alone:

Since the circumference of the track is not known, the time cannot be
calculated.

Thus, the question cannot be answered using statement B alone.

Using both the statements together:

The circumference can be found since Gati completes one round in 10


seconds and her speed increases by 0.5 m/s.

However, the direction in which both of them are moving is not known.

Thus, the time cannot be found.

6 of 20 9/16/2011 10:21 AM
Exam Reports http://testfunda.com/LMS/Student/NewReports.aspx

Thus, the question cannot be answered by either statement.

Hence, option 4.

6. Each question is followed by two statements, A and B. Select the correct


3 Marks option based on the following instructions:

Mark (1) if the question can be answered by one of the statements, but not by
the other.
Mark (2) if the question can be answered by using
either statement, independently of the other.
Mark (3) if the question can be answered by using both statements together,
but not by either statement alone.
Mark (4) if the question cannot be answered by either of the statements.
In the figure below, if ∠YBC = ∠CAX = ∠YOX = 90°, then find the length of
AB.

A. Radius of the arc is given


B. OA = 5

1) 1
2) 2
3) 3
4) 4

Solution:
OC is the radius of the quarter circle.

Using Statement A alone:

If the figure OACB is a square then, AB = OC.

Knowing OC we can calculate AB.

7 of 20 9/16/2011 10:21 AM
Exam Reports http://testfunda.com/LMS/Student/NewReports.aspx

If OACB is rectangle then, AB = OC.

Knowing OC we can calculate AB.

Thus, the question can be answered using statement A alone.

Using Statement B alone:

It is not known whether OACB is a square or a rectangle.

Since nothing is known about OACB, AB cannot be calculated.

Thus, the question cannot be answered using statement B alone.

Thus, the question can be answered by using one of the statement, but not
by other.

Hence, option 1.

7. Each question is followed by two statements, A and B. Select the correct


3 Marks option based on the following instructions:

Mark (1) if the question can be answered by one of the statements, but not by
the other.
Mark (2) if the question can be answered by using
either statement, independently of the other.
Mark (3) if the question can be answered by using both statements together,
but not by either statement alone.
Mark (4) if the question cannot be answered by either of the statements.
Is n odd?

A. n is divisible by 5, 7 and 9.
B. 0 < n < 400

1) 1
2) 2
3) 3
4) 4

Solution:
Using Statement A alone:

8 of 20 9/16/2011 10:21 AM
Exam Reports http://testfunda.com/LMS/Student/NewReports.aspx

Since n is divisible by 5, 7 and 9, n is also divisible by their L.C.M i.e. 315.

Hence, n can be 315, 630, 945 and so on.

Thus, n can be even or odd.

Thus, the question cannot be answered using statement A alone.

Using Statement B alone:

0 < n < 400.

Thus, n can be even or odd.

Thus, the question cannot be answered using statement B alone.

Using both the statements together:

When both the statements are combined, there can be only one possible
value of n.

∴ n = 315.

Thus, n is odd.

Thus, the question can be answered using both the statements together but
not by using either statement alone.

Hence, option 3.

8. Each question is followed by two statements, A and B. Select the correct


3 Marks option based on the following instructions:

Mark (1) if the question can be answered by one of the statements, but not by
the other.
Mark (2) if the question can be answered by using
either statement, independently of the other.
Mark (3) if the question can be answered by using both statements together,
but not by either statement alone.
Mark (4) if the question cannot be answered by either of the statements.

A.

B.

9 of 20 9/16/2011 10:21 AM
Exam Reports http://testfunda.com/LMS/Student/NewReports.aspx

1) 1
2) 2
3) 3
4) 4

Solution:
Using Statement A alone:

This result can be obtained for both a + b and b2 − a.

Thus, the relationship between a and b cannot be established.

Thus, the question cannot be answered using statement A alone.

Using Statement B alone:

The required value = (2 + 3)/2 = 2.5

Thus, the question can be answered using statement B alone.

Thus, the question can be answered using statement B alone but not by
using statement A alone.

Thus, the question can be answered by one of the statements but not by the
other.

Hence, option 1.

9. Each question is followed by two statements, A and B. Select the correct


3 Marks option based on the following instructions:

Mark (1) if the question can be answered by one of the statements, but not by
the other.
Mark (2) if the question can be answered by using
either statement, independently of the other.
Mark (3) if the question can be answered by using both statements together,
but not by either statement alone.
Mark (4) if the question cannot be answered by either of the statements.
Radha and Rani appeared for an examination. What was the total number of
questions?

A. Radha and Rani together solved 20% of the paper.


B. Radha alone solved 3/5th of the paper solved by Rani

10 of 20 9/16/2011 10:21 AM
Exam Reports http://testfunda.com/LMS/Student/NewReports.aspx

1) 1
2) 2
3) 3
4) 4

Solution:
Using Statement A alone:

Radha and Rani together solved 20% of the paper.

Since no other information is given, the total number of questions cannot be


found.

Thus, the question cannot be answered using statement A alone.

Using Statement B alone:

The number of questions solved by Radha and Rani is in the ratio 3 : 5

However, the total number of questions cannot be found.

Thus, the question cannot be answered using statement B alone.

Using both the statements together:

Radha and Rani together solved 8x questions and so the total number of
questions is 40x.

Since x is not known, the total number of questions cannot be found.

Thus, the questions cannot be answered by either of the statements.

Hence, option 4.

10. Each question is followed by two statements, A and B. Select the correct
3 Marks option based on the following instructions:

Mark (1) if the question can be answered by one of the statements, but not by
the other.
Mark (2) if the question can be answered by using
either statement, independently of the other.
Mark (3) if the question can be answered by using both statements together,
but not by either statement alone.
Mark (4) if the question cannot be answered by either of the statements.
What is the value of a?

11 of 20 9/16/2011 10:21 AM
Exam Reports http://testfunda.com/LMS/Student/NewReports.aspx

A. Ratio of a and b is 3 : 5 where b is positive.


B. Ratio of 2a and b is 12 : 10 where a is positive.

1) 1
2) 2
3) 3
4) 4

Solution:
Using Statement A alone:

Since the value of b is not known, the value of a cannot be found.

Thus, the question cannot be answered using statement A alone.

Using Statement B alone:

Since the value of b is not known, the value of a cannot be found.

Thus, the question cannot be answered using statement B alone.

Using both the statements together:

Both statements lead to the same ratio.

Now, since b is still unknown, the value of a cannot be found.

Thus, the question cannot be answered by either statement.

Hence, option 4.

11. Each question is followed by two statements, A and B. Select the correct
3 Marks option based on the following instructions:

Mark (1) if the question can be answered by one of the statements, but not by
the other.
Mark (2) if the question can be answered by using
either statement, independently of the other.
Mark (3) if the question can be answered by using both statements together,

12 of 20 9/16/2011 10:21 AM
Exam Reports http://testfunda.com/LMS/Student/NewReports.aspx

but not by either statement alone.


Mark (4) if the question cannot be answered by either of the statements.
In a group of 150 students, find the number of girls.

A. Each girl was given 50 paisa, while each boy was given 25 paisa to purchase goods
totalling Rs. 49.
B. Girls and boys were given 30 paisa each to buy goods totalling Rs. 45.

1) 1
2) 2
3) 3
4) 4

Solution:
Let the number of boys and girls be respectively b and g.

b + g = 150 ... (i)

Using Statement A alone:

0.25b + 0.5g = 49 ... (ii)

On solving (i) and (ii), g = 46.

Thus, there are 46 girls.

Thus, the question can be answered using statement A alone.

Using Statement B alone:

0.3b + 0.3g = 45

If this equation is divided by 0.3, you get (i).

Thus, now there is only one equation and two unknowns.

Hence, the number of girls cannot be found.

Thus, the question cannot be answered using statement B alone.

Thus, the question can be answered using statement A alone but not by
using statement B alone.

Thus, the question can be answered by one of the statements but not by the
other.

13 of 20 9/16/2011 10:21 AM
Exam Reports http://testfunda.com/LMS/Student/NewReports.aspx

Hence, option 1.

12. Each question is followed by two statements, A and B. Select the correct
3 Marks option based on the following instructions:

Mark (1) if the question can be answered by one of the statements, but not by
the other.
Mark (2) if the question can be answered by using
either statement, independently of the other.
Mark (3) if the question can be answered by using both statements together,
but not by either statement alone.
Mark (4) if the question cannot be answered by either of the statements.
There are four envelope E1, E2, E3, E4 in which one was supposed to put
letters L1, L2, L3, L4 meant for persons C1, C2, C3, C4 respectively but by
mistake the letters got jumbled up and went in wrong envelopes. Now if C2 is
allowed to open an envelope at random, then how will he identify the
envelope containing the letter for him?

A. L2 has been put in E 1.


B. The letter belonging to C3 has gone in the correct envelope.

1) 1
2) 2
3) 3
4) 4

Solution:
Using Statement A alone:

If L2 is put in E1, then C2 will know that his letter is in the envelop E1.

Thus, the question can be answered using statement A alone.

Using Statement B alone:

If the letter L3 is put in E3, the other three letters can go into any of the three
remaining envelopes.

Hence, C2 cannot identify the envelope.

Thus, the question cannot be answered using statement B alone.

Thus, the question can be answered using statement A alone but not by
using statement B alone.

14 of 20 9/16/2011 10:21 AM
Exam Reports http://testfunda.com/LMS/Student/NewReports.aspx

Thus, the question can be answered by one of the statements but not by the
other.

Hence, option 1.

13. Each question is followed by two statements, A and B. Select the correct
3 Marks option based on the following instructions:

Mark (1) if the question can be answered by one of the statements, but not by
the other.
Mark (2) if the question can be answered by using
either statement, independently of the other.
Mark (3) if the question can be answered by using both statements together,
but not by either statement alone.
Mark (4) if the question cannot be answered by either of the statements.
There are four racks numbered 1, 2, 3, 4 and four books numbered 1, 2, 3, 4.
If an even numbered rack has to contain an odd numbered book and an odd
numbered rack contains an even numbered book then what is the position of
book 4?

A. The second book has been put in the third rack.


B. The third book has been put in the second rack.

1) 1
2) 2
3) 3
4) 4

Solution:
An odd numbered rack has to contain an even numbered book and vice
versa.

Using Statement A alone:

Since the second book is in the third rack, the fourth book can only be in the
first rack.

Thus, the question can be answered using statement A alone.

Using Statement B alone:

The third book is in the second rack.

The fourth book cab in either the first or third rack. It's exact position cannot

15 of 20 9/16/2011 10:21 AM
Exam Reports http://testfunda.com/LMS/Student/NewReports.aspx

be found.

Thus, the question cannot be answered using statement B alone.

Thus, the question can be answered using statement A alone but not by
using statement B alone.

Thus, the question can be answered by one of the statements but not by the
other.

Hence, option 1.

14. Each question is followed by two statements, A and B. Select the correct
3 Marks option based on the following instructions:

Mark (1) if the question can be answered by one of the statements, but not by
the other.
Mark (2) if the question can be answered by using
either statement, independently of the other.
Mark (3) if the question can be answered by using both statements together,
but not by either statement alone.
Mark (4) if the question cannot be answered by either of the statements.
What is the value of X in terms of a?

A. Arithmetic mean of X and Y is a while the geometric mean is also a.

B.

1) 1
2) 2
3) 3
4) 4

Solution:
Using Statement A alone:

Solving the equations (i) and (ii), X = a

Thus, the question can be answered using statement A alone.

16 of 20 9/16/2011 10:21 AM
Exam Reports http://testfunda.com/LMS/Student/NewReports.aspx

Using Statement B alone:

a is not mentioned in all.

Thus, the question cannot be answered using statement B alone.

Thus, the question can be answered using statement A alone but not by
using statement B alone.

Thus, the question can be answered by one of the statements but not by the
other.

Hence, option 1.

15. Each question is followed by two statements, A and B. Select the correct
3 Marks option based on the following instructions:

Mark (1) if the question can be answered by one of the statements, but not by
the other.
Mark (2) if the question can be answered by using
either statement, independently of the other.
Mark (3) if the question can be answered by using both statements together,
but not by either statement alone.
Mark (4) if the question cannot be answered by either of the statements.
Two concentric circles C1 and C2 have radii r1 and r2. If C1 fully encloses C2,
what is the radius of C1?

A. The difference of their circumference is k cm.


B. The difference of their areas is m sq. cm.

1) 1
2) 2
3) 3
4) 4

Solution:
r1 > r2

Using Statement A alone:

2π(r1 − r2) = k ... (i)

The value of r1 cannot be found from this.

17 of 20 9/16/2011 10:21 AM
Exam Reports http://testfunda.com/LMS/Student/NewReports.aspx

Thus, the question cannot be answered using statement A alone.

Using Statement B alone:

π(r12 − r22) = m ... (ii)

The value of r1 cannot be found from this.

Thus, the question cannot be answered using statement B alone.

Using both the statements together:

If you divide (ii) by (i), you get an equation in terms of r1 + r2.

Taking this equation and (i), the value of r1 can be found.

Thus, the question can be answered using both the statements together but
not by using either statement alone.

Hence, option 3.

16. Each question is followed by two statements, A and B. Select the correct
3 Marks option based on the following instructions:

Mark (1) if the question can be answered by one of the statements, but not by
the other.
Mark (2) if the question can be answered by using
either statement, independently of the other.
Mark (3) if the question can be answered by using both statements together,
but not by either statement alone.
Mark (4) if the question cannot be answered by either of the statements.
A circle circumscribes a square. What is the area of the square?

A. Radius of the circle is given.


B. Length of the tangent from a point 5 cm away from the centre of the circle is given.

1) 1
2) 2
3) 3
4) 4

Solution:
Using Statement A alone:

The diameter of the circle is the same as the diagonal of the square.

18 of 20 9/16/2011 10:21 AM
Exam Reports http://testfunda.com/LMS/Student/NewReports.aspx

Since the radius of the circle is given, the diameter of the circle and
consequently, the diagonal of the square can be found.

Using this, the side of the square and the area of the square can be found.

Thus, the question can be answered using statement A alone.

Using Statement B alone:

The distance between the tangent and the centre of the circle is nothing but
the radius of the circle.

Once the radius of the circle is known, one can find the side of the square
and the area of the square.

Thus, the question can be answered using statement B alone.

Thus, the question can be answered using either statement alone.

Hence, option 2.

17. Each question is followed by two statements, A and B. Select the correct
3 Marks option based on the following instructions:

Mark (1) if the question can be answered by one of the statements, but not by
the other.
Mark (2) if the question can be answered by using
either statement, independently of the other.
Mark (3) if the question can be answered by using both statements together,
but not by either statement alone.
Mark (4) if the question cannot be answered by either of the statements.
The average weight of students in a class is 50 kg. What is the number of
students in the class?

A. The heaviest and the lightest members of the class weigh 60 kg and 40 kg
respectively.
B. Exclusion of the heaviest and the lightest members from the class does not change
the average weight of the student.

1) 1
2) 2
3) 3
4) 4

19 of 20 9/16/2011 10:21 AM
Exam Reports http://testfunda.com/LMS/Student/NewReports.aspx

Solution:
Let the number of students in the class be n. Thus, the total weight of the
students is 50n.

Using Statement A alone:

Two members weigh 100 kg together.

The weight of the remaining n − 2 students is 50n − 100. Thus, the average is
still 50.

This does not give the total number of students.

Thus the question cannot be answered using statement A alone.

Using Statement B alone:

This is the same as the information given in statement A.

Thus, the question cannot be answered using statement B alone.

Using both the statements together:

Both the statements give the same information.

Hence, the total number of students cannot be found.

Thus, the question cannot be answered by either of the statements.

Hence, option 4.

18. Each question is followed by two statements, A and B. Select the correct
3 Marks option based on the following instructions:

Mark (1) if the question can be answered by one of the statements, but not by
the other.
Mark (2) if the question can be answered by using
either statement, independently of the other.
Mark (3) if the question can be answered by using both statements together,
but not by either statement alone.
Mark (4) if the question cannot be answered by either of the statements.
A small storage tank is spherical in shape. What is the storage volume of the
tank?

A. The wall thickness of the tank is 1 cm.


B. When the empty spherical tank is immersed in a large tank filled with water, 20
litres of water overflow from the large tank.

20 of 20 9/16/2011 10:21 AM
Exam Reports http://testfunda.com/LMS/Student/NewReports.aspx

Section I

1. Choose 1; if the question can be answered by using one of the statements


3 Marks alone, but cannot be answered using the other statement alone.

Choose 2; if the question can be answered by using either statement alone.

Choose 3; if the question can be answered by using both statements together,


but cannot be answered using either statement alone.

Choose 4; if the question cannot be answered even by using both statements


together.
Consider three real numbers, X, Y and Z. Is Z the smallest of these
numbers?

A. X is greater than at least one of Y and Z.


B. Y is greater than at least one of X and Z.

[CAT 2000]

1) 1
2) 2
3) 3
4) 4

Solution:
From statement A, there are four possibilities.

X>Y>Z

X>Z>Y

Z>X>Y

Y>X>Z

∴ This statement alone is not sufficient.

From statement B, there are four possibilities.

Y>X>Z

Y>Z>X

1 of 35 9/16/2011 10:29 AM
Exam Reports http://testfunda.com/LMS/Student/NewReports.aspx

X>Y>Z

Z>Y>X

∴ This statement alone is not sufficient.

Using both statements together we get,

Either X > Y > Z or Y > X > Z.

In both the cases Z is smallest.

Hence, option 3.

2. Choose 1; if the question can be answered by using one of the statements


3 Marks alone, but cannot be answered using the other statement alone.

Choose 2; if the question can be answered by using either statement alone.

Choose 3; if the question can be answered by using both statements together,


but cannot be answered using either statement alone.

Choose 4; if the question cannot be answered even by using both statements


together.
Let X be a real number. Is the modulus of X necessarily less than 3?

A. X(X + 3) < 0
B. X(X – 3) > 0

[CAT 2000]

1) 1
2) 2
3) 3
4) 4

Solution:
From statement A

X (X + 3) < 0

∴ –3 < X < 0

∴ |X| < 3

∴ Statement A alone is sufficient.

2 of 35 9/16/2011 10:29 AM
Exam Reports http://testfunda.com/LMS/Student/NewReports.aspx

From statement B

X(X – 3) > 0

X < 0 or X > 3

This statement does not give a unique answer.

Hence, option 1.

3. Choose 1; if the question can be answered by using one of the statements


3 Marks alone, but cannot be answered using the other statement alone.

Choose 2; if the question can be answered by using either statement alone.

Choose 3; if the question can be answered by using both statements together,


but cannot be answered using either statement alone.

Choose 4; if the question cannot be answered even by using both statements


together.
How many people are watching TV programme P?

A. Number of people watching TV programme Q is 1000 and number of people watching


both the programmes, P and Q, is 100.
B. Number of people watching either P or Q or both is 1500.

[CAT 2000]

1) 1
2) 2
3) 3
4) 4

Solution:
From Statement A

Q = 1000

P ∩ Q = 100

This doesn’t gives any information regarding how many people are watching
TV programme P

From Statement B

3 of 35 9/16/2011 10:29 AM
Exam Reports http://testfunda.com/LMS/Student/NewReports.aspx

P ∪ Q = 1500

This statement alone is not sufficient.

Let’s combine the information in both the statements.

∴P∪Q=P+Q−P∩Q

∴ 1500 = P + 1000 – 100

∴ P = 600

Hence, option 3.

Alternatively

By combining both the statements together we can arrive at the above Venn
diagram which gives the number of people watching programme P.

Hence, option 3.

4. Choose 1; if the question can be answered by using one of the statements


3 Marks alone, but cannot be answered using the other statement alone.

Choose 2; if the question can be answered by using either statement alone.

Choose 3; if the question can be answered by using both statements together,


but cannot be answered using either statement alone.

Choose 4; if the question cannot be answered even by using both statements


together.
Triangle PQR has angle PRQ equal to 90 degrees. What is the value of PR +

4 of 35 9/16/2011 10:29 AM
Exam Reports http://testfunda.com/LMS/Student/NewReports.aspx

RQ?

A. Diameter of the inscribed circle of the triangle PQR is equal to 10 cm.


B. Diameter of the circumscribed circle of the triangle PQR is equal to 26 cm.

[CAT 2000]

1) 1
2) 2
3) 3
4) 4

Solution:
From statement A

PF = PR – FR = PR – OD = PR – 5

QD = QR – DR = QR – OF = QR – 5

∵ PE = PF and QE = QD

∴ PE = PR – 5 and QE = QR – 5

∴ PQ = PE + QE = PR – 5 + QR – 5

∴ PR + QR = PQ + 10

This statement alone is not sufficient to find PR + RQ.

From Statement B

5 of 35 9/16/2011 10:29 AM
Exam Reports http://testfunda.com/LMS/Student/NewReports.aspx

Diameter of the circumscribing circle = hypotenuse of the triangle

∴ PQ = 26

∵ PR2 + RQ2 = PQ2

But this does not give the value of PR + RQ.

∴ This statement alone is not sufficient.

Let’s combine both the statements.

∴ PR + QR = PQ + 10 = 26 + 10 = 36

Hence, option 3.

5. Choose 1; if the question can be answered by using one of the statements


3 Marks alone, but cannot be answered using the other statement alone.

Choose 2; if the question can be answered by using either statement alone.

Choose 3; if the question can be answered by using both statements together,


but cannot be answered using either statement alone.

Choose 4; if the question cannot be answered even by using both statements


together.
Harshad bought shares of a company on a certain day, and sold them the
next day. While buying and selling he had to pay to the broker one percent of
the transaction value of the shares as brokerage. What was the profit earned
by him per rupee spent on buying the shares?

A. The sales price per share was 1.05 times that of its purchase price.
B. The number of shares purchased was 100.

[CAT 2000]

1) 1
2) 2
3) 3
4) 4

Solution:
From Statement A

The cost of buying the shares for Harshad is

6 of 35 9/16/2011 10:29 AM
Exam Reports http://testfunda.com/LMS/Student/NewReports.aspx

CP + 0.01 CP = 1.01 CP

The cost of selling the shares for Harshad is

SP – 0.01 SP = 0.99 SP

∴ Profit = Cost of selling – Cost of buying

= 0.99 SP – 1.01 CP

∵ SP = 1.05 CP

Profit = (0.99 × 1.05 CP) – 1.01 CP

= 0.0295 CP

∴ Profit earned per rupee spent on buying the shares is Rs 0.0295.

From the number of shares given in the statement B, we cannot conclude


anything.

Hence, option 1.

6. Choose 1; if the question can be answered by using one of the statements


3 Marks alone, but cannot be answered using the other statement alone.

Choose 2; if the question can be answered by using either statement alone.

Choose 3; if the question can be answered by using both statements together,


but cannot be answered using either statement alone.

Choose 4; if the question cannot be answered even by using both statements


together.
For any two real numbers
a ⊕ b = 1 if both a and b are positive or both a and b are negative.
= –1 if one of the two numbers a and b is positive and the other
negative.
What is (2 ⊕ 0) ⊕ (–5 ⊕ –6)?

A. a ⊕ b is zero if a is zero.
B. a ⊕ b = b ⊕ a

[CAT 2000]

1) 1
2) 2

7 of 35 9/16/2011 10:29 AM
Exam Reports http://testfunda.com/LMS/Student/NewReports.aspx

3) 3
4) 4

Solution:
(2 ⊕ 0) ⊕ (–5 ⊕ –6)

= (2 ⊕ 0) ⊕ 1

From statement A

(0 ⊕ 2) = 0 but we don’t know the value of (2 ⊕ 0).

∴ Statement A alone is not sufficient to answer the question.

From statement B

(2 ⊕ 0) = (0 ⊕ 2)

∴ Statement B alone is not sufficient to answer the question.

After combining both the statements, we get,

(2 ⊕ 0) = (0 ⊕ 2) = 0

∴ (2 ⊕ 0) ⊕ (–5 ⊕ –6)

= (2 ⊕ 0) ⊕ 1

=0⊕1

=0

∴ Both the statements are required to answer the question.

Hence, option 3.

7. Choose 1; if the question can be answered by using one of the statements


3 Marks alone, but cannot be answered using the other statement alone.

Choose 2; if the question can be answered by using either statement alone.

Choose 3; if the question can be answered by using both statements together,


but cannot be answered using either statement alone.

Choose 4; if the question cannot be answered even by using both statements


together.

8 of 35 9/16/2011 10:29 AM
Exam Reports http://testfunda.com/LMS/Student/NewReports.aspx

There are two straight lines in the x-y plane with equations ax + by = c , dx +
ey = f. Do the two straight lines intersect?

A. a, b, c, d, e and f are distinct real numbers.


B. c and f are non-zero.

[CAT 2000]

1) 1
2) 2
3) 3
4) 4

Solution:
Statement A implies that a, b, c, d, e, f are distinct real numbers.

∴ Statement A alone is not sufficient.

Statement B implies that the equations are not homogenous equations.

∴ Statement B is also not sufficient.

Even after combining both the statements the above condition is not clear.

∴ We cannot be sure whether the lines are intersecting or parallel.

Hence, option 4.

8. Choose 1; if the question can be answered by using one of the statements


3 Marks alone, but cannot be answered using the other statement alone.

Choose 2; if the question can be answered by using either statement alone.

Choose 3; if the question can be answered by using both statements together,


but cannot be answered using either statement alone.

Choose 4; if the question cannot be answered even by using both statements


together.
O is the centre of two concentric circles. ae is a chord of the outer circle and it
intersects the inner circle at point; b and d. c is a point on the chord in

9 of 35 9/16/2011 10:29 AM
Exam Reports http://testfunda.com/LMS/Student/NewReports.aspx

between b and d. What is the value of ac/ce?

A. bc/cd = 1
B. A third circle intersects the inner circle at b and d and the point c is on the line
joining the centres of the third circle and the inner circle.

[CAT 2000]

1) 1
2) 2
3) 3
4) 4

Solution:
From statement A

bc = cd

If c is the midpoint of bd it would also be midpoint of ae because circles are


concentric.

∴ ac = ce

∴ The question can be answered using statement A alone.

From Statement B

10 of 35 9/16/2011 10:29 AM
Exam Reports http://testfunda.com/LMS/Student/NewReports.aspx

If c is the point on the line joining the two centres, it has to bisect the chord
bd.

∴ c will also bisect the chord ae as the circles are concentric.

∴ ac = ce

∴ The question can be answered using statement B alone also.

Hence, option 2.

9. Choose 1; if the question can be answered by using one of the statements


3 Marks alone, but cannot be answered using the other statement alone.

Choose 2; if the question can be answered by using either statement alone.

Choose 3; if the question can be answered by using both statements together,


but cannot be answered using either statement alone.

Choose 4; if the question cannot be answered even by using both statements


together.
Ghosh Babu has decided to take a non-stop flight from Mumbai to
No-man’s-land in South America. He is scheduled to leave Mumbai at 5 am,
Indian Standard Time on December 10, 2000. What is the local time at
No-man’s-land when he reaches there?

A. The average speed of the plane is 700 kilometres per hour.


B. The flight distance is 10,500 kilometres.

[CAT 2000]

1) 1
2) 2
3) 3

11 of 35 9/16/2011 10:29 AM
Exam Reports http://testfunda.com/LMS/Student/NewReports.aspx

4) 4

Solution:
By combining both the statements we can get the duration of the flight. But
for the arrival time we should have the information regarding the time zone
difference of Mumbai and No man’s land.

Hence, option 4.

10. Choose 1; if the question can be answered by using one of the statements
3 Marks alone, but cannot be answered using the other statement alone.

Choose 2; if the question can be answered by using either statement alone.

Choose 3; if the question can be answered by using both statements together,


but cannot be answered using either statement alone.

Choose 4; if the question cannot be answered even by using both statements


together.
What are the ages of two individuals, X and Y?

A. The age difference between them is 6 years.


B. The product of their ages is divisible by 6.

[CAT 2000]

1) 1
2) 2
3) 3
4) 4

Solution:
From statement A

X–Y=6

From statement B

XY is divisible by 6.

∵ There are many possible values for example (12, 6), (18, 12), (24, 18) ...

∴ The question cannot be answered even by combining both the statements.

Hence, option 4.

12 of 35 9/16/2011 10:29 AM
Exam Reports http://testfunda.com/LMS/Student/NewReports.aspx

11. Choose 1; if the question can be answered by using one of the statements
3 Marks alone, but cannot be answered using the other statement alone.

Choose 2; if the question can be answered by using either statement alone.

Choose 3; if the question can be answered by using both statements together,


but cannot be answered using either statement alone.

Choose 4; if the question cannot be answered even by using both statements


together.
What are the values of m and n?

A. n is an even integer, m is an odd integer, and m is greater than n.


B. Product of m and n is 30.

[CAT 2001]

1) 1
2) 2
3) 3
4) 4

Solution:
Statement A doesn’t provide any specific values of m and n.

∴ Statement A alone is not sufficient to answer the question.

Using statement B alone:

m × n = 30 = 1 × 30 = 2 × 15 = 3 × 10 = 5 × 6

We cannot say what the values of m and n are.

∴ Statement B alone is also not sufficient to answer the question.

If we combine both the statements A and B together, then we have only one
possibility m = 15 and n = 2

∴ We have answered the question using both the statements A and B


together.

Hence, option 3.

Note: Negative integers are not considered as Even or Odd.

13 of 35 9/16/2011 10:29 AM
Exam Reports http://testfunda.com/LMS/Student/NewReports.aspx

12. Choose 1; if the question can be answered by using one of the statements
3 Marks alone, but cannot be answered using the other statement alone.

Choose 2; if the question can be answered by using either statement alone.

Choose 3; if the question can be answered by using both statements together,


but cannot be answered using either statement alone.

Choose 4; if the question cannot be answered even by using both statements


together.
Is Country X's GDP higher than country Y's GDP?

A. GDPs of the countries X and Y have grown over the past five years at compounded
annual rate of 5% and 6% respectively.
B. Five years ago, GDP of country X was higher than that of country Y.

[CAT 2001]

1) 1
2) 2
3) 3
4) 4

Solution:
Statement A does not give us the actual values of their GDPs 5 years ago.

Using statement B alone:

We know that GDP of country X is greater than that of Y but we do not know
by how much it is greater.

Even after combining both the statements the question cannot be answered.

Hence, option 4.

13. Choose 1; if the question can be answered by using one of the statements
3 Marks alone, but cannot be answered using the other statement alone.

Choose 2; if the question can be answered by using either statement alone.

Choose 3; if the question can be answered by using both statements together,


but cannot be answered using either statement alone.

Choose 4; if the question cannot be answered even by using both statements


together.

14 of 35 9/16/2011 10:29 AM
Exam Reports http://testfunda.com/LMS/Student/NewReports.aspx

What is the value of X?

A. X and Y are unequal even integers, less than 10, and X/Y is an odd integer.
B. X and Y are even integers, each less than 10, and product of X and Y is 12.

[CAT 2001]

1) 1
2) 2
3) 3
4) 4

Solution:
Using statement A alone:

∵ Both have to be even numbers, the only possibility is X = 6 and Y = 2

∴ Statement A alone is sufficient to answer the question.

Using statement B alone:

12 = 1 × 12 = 2 × 6 = 3 × 4

∵ Both are even integers and less than 10, the possible set of values of X
and Y are (2, 6) and (6, 2).

∴ X is either 2 or 6.

∴ No unique solution can be found using statement B alone.

Hence, option 1.

Note: Negative integers are not considered as Even or Odd.

14. Choose 1; if the question can be answered by using one of the statements
3 Marks alone, but cannot be answered using the other statement alone.

Choose 2; if the question can be answered by using either statement alone.

Choose 3; if the question can be answered by using both statements together,


but cannot be answered using either statement alone.

Choose 4; if the question cannot be answered even by using both statements


together.
On a given day a boat ferried 1500 passengers across the river in twelve

15 of 35 9/16/2011 10:29 AM
Exam Reports http://testfunda.com/LMS/Student/NewReports.aspx

hours. How many round trips did it make?

A. The boat can carry two hundred passengers at any time.


B. It takes 40 minutes each way and 20 minutes of waiting time at each terminal.

[CAT 2001]

1) 1
2) 2
3) 3
4) 4

Solution:
Statement A only gives the maximum capacity of the boat.

∴ The number of trips cannot be calculated.

∴ Statement A alone is not sufficient to answer the question.

Using statement B alone:

Time required to make one round trip = 40 + 40 + 40 = 120 minutes = 2 hours

∴ The total number of trips made = 12/2 = 6

∴ Statement B alone is sufficient to answer the question.

Hence, option 1.

15. Choose 1; if the question can be answered by using one of the statements
3 Marks alone, but cannot be answered using the other statement alone.

Choose 2; if the question can be answered by using either statement alone.

Choose 3; if the question can be answered by using both statements together,


but cannot be answered using either statement alone.

Choose 4; if the question cannot be answered even by using both statements


together.
What will be the time for downloading software?

A. Transfer rate is 6 Kilobytes per second.


B. The size of the software is 4.5 megabytes.

16 of 35 9/16/2011 10:29 AM
Exam Reports http://testfunda.com/LMS/Student/NewReports.aspx

[CAT 2001]

1) 1
2) 2
3) 3
4) 4

Solution:
Statement A alone is not sufficient because it gives only the rate at which
data is transferred but it does not give the size of the software.

Statement B alone is not sufficient because it gives only the size of the
software but it does not give the rate of data transfer.

Combining both the statements A and B together, we can get the time to
download.

∵ Time required for data transfer = (Size of the software)/(Rate of data


transfer)

Hence, option 3.

16. Choose 1; if the question can be answered by using one of the statements
3 Marks alone, but cannot be answered using the other statement alone.

Choose 2; if the question can be answered by using either statement alone.

Choose 3; if the question can be answered by using both statements together,


but cannot be answered using either statement alone.

Choose 4; if the question cannot be answered even by using both statements


together.
A square is inscribed in a circle. What is the difference between the area of
the circle and that of the square?

A. The diameter of the circle is 25 cm.


B. The side of the square is 25 cm.

[CAT 2001]

1) 1
2) 2
3) 3

17 of 35 9/16/2011 10:29 AM
Exam Reports http://testfunda.com/LMS/Student/NewReports.aspx

4) 4

Solution:
Using statement A alone:

∵ Diameter of the circle = Diagonal of the square

∴ Area of the square can be computed and thus the required difference.

∴ Statement A alone is sufficient to answer the question.

Using statement B alone:

∵ Diagonal of the square = × side = Diameter of the circle

∴ Area of the circle can be computed and thus the required difference.

∴ Statement B alone is also sufficient to answer the question.

∴ We can answer the question using either statement alone.

Hence, option 2.

17. Choose 1; if the question can be answered by using one of the statements
3 Marks alone, but cannot be answered using the other statement alone.

Choose 2; if the question can be answered by using either statement alone.

Choose 3; if the question can be answered by using both statements together,


but cannot be answered using either statement alone.

Choose 4; if the question cannot be answered even by using both statements


together.
Two friends, Ram and Gopal, bought apples from a wholesale dealer. How
many apples did they buy?

A. Ram bought one-half the number of apples that Gopal bought.


B. The wholesale dealer had a stock of 500 apples.

[CAT 2001]

1) 1
2) 2
3) 3
4) 4

18 of 35 9/16/2011 10:29 AM
Exam Reports http://testfunda.com/LMS/Student/NewReports.aspx

Solution:
Statement A simply gives the ratio of apples between Ram and Gopal.

∴ Statement A is not alone sufficient to answer the question.

Statement B doesn’t provide any information related to number of apples


bought by Ram and Gopal.

Even after combining both the statements A and B together, the question
cannot be answered.

Hence, option 4.

18. Each question is followed by two statements A and B. Answer each question
3 Marks using the following instructions:

Answer (1) if the question can be solved by any one of the statements, but not
the other one.
Answer (2) if the question can be solved by using either of the two statements.
Answer (3) if the question can be solved by using both the statements
together and not by any one of them.
Answer (4) if the question cannot be solved with the help of the given data
and more data is required.
In a hockey match, the Indian team was behind by 2 goals with 5 minutes
remaining. Did they win the match?

A. Deepak Thakur, the Indian striker scored 3 goals in the last 5 minutes of the match.
B. Korea scored a total of 3 goals in the match.

[CAT 2002]

1) 1
2) 2
3) 3
4) 4

Solution:
From statement A alone, it is not clear whether Korea too scored in the last 5
minutes or not.

From statement B alone, it is not clear whether India scored in the last 5
minutes or not.

Using both the statements, there are two cases possible:

19 of 35 9/16/2011 10:29 AM
Exam Reports http://testfunda.com/LMS/Student/NewReports.aspx

Case 1:

In this case, the match ended in a draw.

Case 2:

In this case, India wins the match.

∴ Even after combining both the statements, we cannot say whether India
won the match or not.

Hence, option 4.

19. Each question is followed by two statements A and B. Answer each question
3 Marks using the following instructions:

Answer (1) if the question can be solved by any one of the statements, but not
the other one.
Answer (2) if the question can be solved by using either of the two statements.
Answer (3) if the question can be solved by using both the statements
together and not by any one of them.
Answer (4) if the question cannot be solved with the help of the given data
and more data is required.
Four students were added to a dance class. Would the teacher be able to
divide her students evenly into a dance team (or teams) of 8?

A. If 12 students were added, then the teacher could put everyone in teams of 8
without any left overs.
B. The number of students in the class is currently not divisible by 8.

[CAT 2002]

1) 1
2) 2
3) 3
4) 4

20 of 35 9/16/2011 10:29 AM
Exam Reports http://testfunda.com/LMS/Student/NewReports.aspx

Solution:
Consider statement A:

Let x be the total number of students present in the class.

∵ (x + 12) − (x + 4) = 8

∴ If (x + 12) is divisible by 8, then (x + 4) too is divisible by 8.

Statement A alone is sufficient to answer the question.

From statement B alone, we cannot conclude anything.

Hence, option 1.

20. Each question is followed by two statements A and B. Answer each question
3 Marks using the following instructions:

Answer (1) if the question can be solved by any one of the statements, but not
the other one.
Answer (2) if the question can be solved by using either of the two statements.
Answer (3) if the question can be solved by using both the statements
together and not by any one of them.
Answer (4) if the question cannot be solved with the help of the given data
and more data is required.
Is x = y?

B. (x − 50)2 = (y − 50)2

[CAT 2002]

1) 1
2) 2
3) 3
4) 4

Solution:
Consider statement A:

21 of 35 9/16/2011 10:29 AM
Exam Reports http://testfunda.com/LMS/Student/NewReports.aspx

∴ (x + y)2 = 4xy

∴ x2 + y2 + 2xy − 4xy = 0

∴ (x − y)2 = 0

∴x=y

∴ Statement A alone is sufficient.

Consider statement B:

(x – 50)2 = (y – 50)2

∴ Either (x − 50) = (y − 50) or (x − 50) = −(y − 50)

∴ Either x = y or x + y = 100, which gives infinite values for x and y.

∴ Statement B alone is not sufficient.

Hence, option 1.

21. Each question is followed by two statements A and B. Answer each question
3 Marks using the following instructions.
Mark (1) if the question can be answered by using statement A alone but not
by using statement B alone.
Mark (2) if the question can be answered by using statement B alone but not
by using statement A alone.
Mark (3) if the question can be answered by using both the statements
together but not by using either of the statements alone.
Mark (4) if the question cannot be answered on the basis of the two
statements.
A dress was initially listed at a price that would have fetched the store a profit
of 20% on the wholesale cost. What was the wholesale cost of the dress?

A. After reducing the listed price by 10% the dress was sold for a net profit of 10
dollars.
B. The dress was sold for 50 dollars.

[CAT 2002]

1) 1
2) 2
3) 3
4) 4

22 of 35 9/16/2011 10:29 AM
Exam Reports http://testfunda.com/LMS/Student/NewReports.aspx

Solution:
Let 100x be the wholesale cost of the dress.

∴ List price of the dress = 120x

Consider statement A:

Selling Price = 0.9 × List Price = 0.9 × 120x = 108x

Now, Selling Price – Cost Price = Profit

∴ (108x) − (100x) = 10

∴ x = 10/8

∴ Wholesale Cost = 100x = Rs. 125

∴ Statement A alone is sufficient.

Consider statement B:

This gives the selling price of the dress but it is not mentioned whether any
discount is provided on the list price or not.

∴ Statement B alone is not sufficient.

Hence, option 1.

22. Each question is followed by two statements A and B. Answer each question
3 Marks using the following instructions:

Answer (1) if the question can be solved by any one of the statements, but not
the other one.
Answer (2) if the question can be solved by using either of the two statements.
Answer (3) if the question can be solved by using both the statements
together and not by any one of them.
Answer (4) if the question cannot be solved with the help of the given data
and more data is required.
Is 500 the average (arithmetic mean) score of the GMAT?

A. Half of the people who take GMAT score above 500 and half of the people score
below 500.
B. The highest GMAT score is 800 and the lowest score is 200.

[CAT 2002]

23 of 35 9/16/2011 10:29 AM
Exam Reports http://testfunda.com/LMS/Student/NewReports.aspx

1) 1
2) 2
3) 3
4) 4

Solution:
Consider statement A:

This statement alone does not give sufficient information as we do not know
how much above/below 500 these students scored.

For example, it is possible that one student scored 800, one scored 750, the
third scored 450, while the last scored 200.
Then, average = (800 + 750 + 450 + 200)/4 = 550 ≠ 500

Consider statement B:

Using this statement alone, we cannot say whether 500 is the average score
of GMAT. The previous example can be used here as well.

Combining both statements together:

Even now, we cannot determine the average. (Again, the above example can
be used.)

Hence, option 4.

23. Each question is followed by two statements A and B. Answer each question
3 Marks using the following instructions:

Answer (1) if the question can be solved by any one of the statements, but not
the other one.
Answer (2) if the question can be solved by using either of the two statements.
Answer (3) if the question can be solved by using both the statements
together and not by any one of them.
Answer (4) if the question cannot be solved with the help of the given data
and more data is required.
Is |x − 2| < 1?

A. |x| > 1
B. |x − 1| < 2

[CAT 2002]

24 of 35 9/16/2011 10:29 AM
Exam Reports http://testfunda.com/LMS/Student/NewReports.aspx

1) 1
2) 2
3) 3
4) 4

Solution:
|x − 2| < 1

Consider statement A:

|x| > 1

x < −1 or x > 1

For x = 1.5, |x − 2| < 1 is true.

For x = 4, |x − 2| < 1 is false.

∴ Statement A alone is not sufficient.

Consider statement B:

|x − 1| < 2

−2 < x − 1 < 2

−1 < x < 3

For −1 < x < 1, |x − 2| < 1 is false.

For 1 < x < 3, |x − 2| < 1 is true.

∴ Statement B alone is not sufficient.

Consider both the statements:

We have, 1 < x < 3

For this range, |x − 2| < 1 is true.

∴ Both the statements combined together are sufficient to answer the


question.

Hence, option 3.

25 of 35 9/16/2011 10:29 AM
Exam Reports http://testfunda.com/LMS/Student/NewReports.aspx

24. Each question is followed by two statements A and B. Answer each question
3 Marks using the following instructions:

Answer (1) if the question can be solved by any one of the statements, but not
the other one.
Answer (2) if the question can be solved by using either of the two statements.
Answer (3) if the question can be solved by using both the statements
together and not by any one of them.
Answer (4) if the question cannot be solved with the help of the given data
and more data is required.
Members in a club either speak French or Russian or both. Find the number
of members in a club who speak only French.

A. There are 300 members in the club and the number of members who speak both
French and Russian is 196.
B. The number of members who speak only Russian is 58.

[CAT 2002]

1) 1
2) 2
3) 3
4) 4

Solution:
Let F represent the set of members speaking French and R represent the set
of members speaking Russian.

Also, let x represent the number of members speaking only French.

Consider statement A:

Here, total number of members = 300

26 of 35 9/16/2011 10:29 AM
Exam Reports http://testfunda.com/LMS/Student/NewReports.aspx

∴ Number of members who speak only Russian = 300 – 196 – x = 104 – x

However, we cannot find the value of x.

∴ Statement A alone is not sufficient.

Consider statement B:

We are given that the number of members speaking Russian = 58

∴ Statement B alone is not sufficient.

Consider both statements together:

We have, 104 – x = 58

∴ Number of members speaking only French, x = 104 – 58 = 46

∴ The question can be answered using both statements together.

Hence, option 3.

25. Each question is followed by two statements A and B. Answer each question
3 Marks using the following instructions:

Answer (1) if the question can be solved by any one of the statements, but not
the other one.
Answer (2) if the question can be solved by using either of the two statements.
Answer (3) if the question can be solved by using both the statements
together and not by any one of them.
Answer (4) if the question cannot be solved with the help of the given data
and more data is required.
A sum of Rs. 38,500 was divided among Jagdish, Punit and Girish. Who
received the minimum amount?

27 of 35 9/16/2011 10:29 AM
Exam Reports http://testfunda.com/LMS/Student/NewReports.aspx

A. Jagdish received 2/9 of what Punit and Girish together received.


B. Punit received 3/11 of what Jagdish and Girish together received.

[CAT 2002]

1) 1
2) 2
3) 3
4) 4

Solution:
Consider statement A:

Jagdish : (Punit + Girish) = 2 : 9

However, we don’t know the percentage distribution between Punit and


Girish.

∴ Statement A alone is not sufficient.

Consider statement B:

Punit : (Jagdish + Girish) = 3 : 11

However, we do not know the percentage distribution between Jagdish and


Girish.

∴ Statement B alone is not sufficient.

Consider both statements together:

Jagdish = 18.18% of the total amount

Punit = 21.4% of the total amount

Girish = 100 − 18.18 − 21.4 = 60.42% of the total amount

∴ Jagdish received the minimum amount.

Hence, option 3.

28 of 35 9/16/2011 10:29 AM
Exam Reports http://testfunda.com/LMS/Student/NewReports.aspx

26. In each question there are two statements: A and B.


3 Marks

Choose 1 if the question can be answered by one of the statements alone but
not by the other.

Choose 2 if the question can be answered by using either statement alone.

Choose 3 if the question can be answered by using both the statements


together but cannot be answered using either statement alone.

Choose 4 if the question cannot be answered even by using both the


statements A and B.
F and M are father and mother of S, respectively. S has four uncles and three
aunts. F has two siblings. The siblings of F and M are unmarried. How many
brothers does M have?

A. F has two brothers.


B. M has five siblings.

[CAT 2003 Leaked Test]

1) 1
2) 2
3) 3
4) 4

Solution:
F and M have a total of four brothers and three sisters.

Statement A states that F has two brothers, which means M should have two
brothers.

∴ Statement A is alone sufficient.

Statement B is redundant as no additional information is provided.

Hence, option 1.

27. In each question there are two statements: A and B.


3 Marks

Choose 1 if the question can be answered by one of the statements alone but
not by the other.

Choose 2 if the question can be answered by using either statement alone.

Choose 3 if the question can be answered by using both the statements

29 of 35 9/16/2011 10:29 AM
Exam Reports http://testfunda.com/LMS/Student/NewReports.aspx

together but cannot be answered using either statement alone.

Choose 4 if the question cannot be answered even by using both the


statements A and B.
A game consists of tossing a coin successively. There is an entry fee of Rs.
10 and an additional fee of Re. 1 for each toss of the coin. The game is
considered to have ended normally when the coin turns heads on two
consecutive throws. In this case the player is paid Rs. 100. Alternatively, the
player can choose to terminate the game prematurely after any of the tosses.
Ram has incurred a loss of Rs. 50 by playing this game. How many times did
he toss the coin?

A. The game ended normally.


B. The total number of tails obtained in the game was 138.

[CAT 2003 Leaked Test]

1) 1
2) 2
3) 3
4) 4

Solution:
Let the number of tosses be x.

Total amount spent by Ram after x tosses = (10 + x × 1)

= Rs. (10 + x)

We know that Ram incurs a loss of Rs. 50.

∴ We have two cases to evaluate:

i. If the game ends normally then Ram's net loss = Rs. (10 + x) − Rs. 100
ii. If he quits prematurely his loss = Rs. (10 + x)

From statement A,

Ram's net loss = (10 + x) − 100

∴ 50 = (10 + x) − 100

∴ x = 140

∴ Statement A is alone sufficient.

30 of 35 9/16/2011 10:29 AM
Exam Reports http://testfunda.com/LMS/Student/NewReports.aspx

From statement B,

Ram gets 138 tails.

If his game ends prematurely, his loss = 10 + x = 50

∴ x = 40

This is not possibe as the number of tails > 40

∴ His game must have ended normally.

∴ 10 + x – 100 = 50

∴ x = 140

∴ Statement B is also sufficient.

Hence, option 2.

28. In each question there are two statements: A and B.


3 Marks

Choose 1 if the question can be answered by one of the statements alone but
not by the other.

Choose 2 if the question can be answered by using either statement alone.

Choose 3 if the question can be answered by using both the statements


together but cannot be answered using either statement alone.

Choose 4 if the question cannot be answered even by using both the


statements A and B.
Each packet of SOAP costs Rs. 10. Inside each packet is a gift coupon
labelled with one of the letters S, O, A, and P. If a customer submits four
such coupons that make up the word SOAP, the customer gets a free SOAP
packet. Ms. X kept buying packet after packet of SOAP till she could get one
set of coupons that formed the word SOAP. How many coupons with label P
did she get in the above process?

A. The last label obtained by her was S and the total amount spent was Rs. 210.
B. The total number of vowels obtained was 18.

[CAT 2003 Leaked Test]

1) 1
2) 2
3) 3

31 of 35 9/16/2011 10:29 AM
Exam Reports http://testfunda.com/LMS/Student/NewReports.aspx

4) 4

Solution:
From Statement A,

Also the last label obtained by her is S.

But this is not sufficient to get the number of P's.

∴ Statement A alone is not sufficient.

From Statement B,

The number of O's and A's is 18.

But this is also not individually sufficient to arrive at the required answer.

∴ Statement B alone is not sufficient.

After combining both the statements A and B, we can conclude that 18 out of
21 coupons are O's and A's and that the 21st is an S.

∴This means that the remaining two are P's.

Hence, option 3.

29. In each question there are two statements: A and B.


3 Marks

Choose 1 if the question can be answered by one of the statements alone but
not by the other.

Choose 2 if the question can be answered by using either statement alone.

Choose 3 if the question can be answered by using both the statements


together but cannot be answered using either statement alone.

Choose 4 if the question cannot be answered even by using both the


statements A and B.

32 of 35 9/16/2011 10:29 AM
Exam Reports http://testfunda.com/LMS/Student/NewReports.aspx

If A and B run a race, then A wins by 60 seconds. If B and C run the same
race, then B wins by 30 seconds. Assuming that C maintains a uniform
speed, what is the time taken by C to finish the race?

A. A and C run the same race and A wins by 375 metres.


B. The length of the race is 1 km.

[CAT 2003 Leaked Test]

1) 1
2) 2
3) 3
4) 4

Solution:
From the question, if A and C participate in a race, A will win by 90 seconds.

From statement A,

∵ A beats C by 375 m or 90 sec.

But the length of the track is not known.

∴ Statement A is not sufficient to find the time taken by C to complete the


race.

From Statement B we cannot find the speed of C.

∴ Statement B alone is not sufficient.

After combining both the statements A and B, the time taken by C to


complete the race

Hence, option 3.

30. Each question is followed by two statements, A and B. Answer each question
3 Marks using the following instructions

Choose 1 if the question can be answered by using one of the statements

33 of 35 9/16/2011 10:29 AM
Exam Reports http://testfunda.com/LMS/Student/NewReports.aspx

alone but not by using the other statement alone.


Choose 2 if the question can be answered by using either of the statements
alone.
Choose 3 if the question can be answered by using both statements together
but not by either statement alone.
Choose 4 if the question cannot be answered on the basis of the two
statements.

Is a44 < b11, given that a = 2 and b is an integer?

A. b is even
B. b is greater than 16

[CAT 2003 Leaked Test]

1) 1
2) 2
3) 3
4) 4

Solution:
a44 < b11

But, a = 2

∴ a44 = 244

∴ a44 = 1611

Using statement A alone:

b is an even integer. It may be less than or greater than 16.

So, we cannot answer the question using statement A alone.

Using statement B alone:

b is greater than 16.

∴ b11 > 1611

∴ b11 > a44

So, we can answer the question using statement B alone.

34 of 35 9/16/2011 10:29 AM
Exam Reports http://testfunda.com/LMS/Student/NewReports.aspx

Hence, option 1.

35 of 35 9/16/2011 10:29 AM
Exam Reports http://testfunda.com/LMS/Student/NewReports.aspx

Section I

1. There were x pigeons and y mynahs in a cage. One fine morning p of them
3 Marks
escaped to freedom. If the bird keeper, knowing only that the value of p
was 7, was able to figure out without looking into the cage that at least one
pigeon had escaped, then which of the following does not represent a
possible (x, y) pair?

1) (10, 8)
2) (7, 2)
3) (25, 6)
4) (12, 4)

Solution:
7 birds escaped, out of which at least one was a pigeon.

For an (x, y) pair to be possible, there should be no chance that all the birds
that escaped were mynahs.

If y ≥ 7, there is a possibility that all seven birds that escaped were mynahs.

Thus, for an (x, y) pair to be valid, y has to be less than 7.

Therefore, looking at the answer options (10, 8) cannot be a possible (x, y)


pair.

This is because this pair has 8 mynahs which means that all 7 birds that
escaped can be mynahs. This violates the basic condition of the problem.

Hence, option 1.

2. Consider the following steps:


3 Marks

1. Put x = 1, y = 2
2. Replace x by xy
3. Replace y by y + 1
4. If y = 5 then go to step 6 otherwise go to step 5.
5. Go to step 2
6. Stop

Then the final value of x equals _____.

1) 1

1 of 33 9/16/2011 10:16 AM
Exam Reports http://testfunda.com/LMS/Student/NewReports.aspx

2) 24
3) 120
4) 720

Solution:
Step 1:

x = 1 × 2 = 2; y = 2 + 1 = 3

y ≠ 5. So, go to step 5.

Step 2:

x = 2 × 3 = 6; y = 3 + 1 = 4

y ≠ 5. So, go to step 5.

Step 3:

x = 6 × 4 = 24; y = 4 + 1 = 5

Since y = 5, go to step 6 and stop.

When y = 5, x = 24

Thus, the final value of x = 24.

Hence, option 2.

3. 116 people participated in a singles tennis tournament having a knock out


3 Marks
format. The players are paired up in the first round, the winners of the first
round are paired up in the second round, and so on till the final is played
between two players. If after any round, there are an odd number of players
left, one player is given a bye, i.e., he skips that round and plays the next
round with the winners. Find the total number of matches played in the
tournament.

1) 115
2) 53
3) 232
4) 116

2 of 33 9/16/2011 10:16 AM
Exam Reports http://testfunda.com/LMS/Student/NewReports.aspx

Solution:
Consider Round 1 :

Since there are 116 players, 116/2 = 58 matches are played and 58 winners
are obtained.

Consider Round 2 :

58/2 = 29 matches are played and 29 winners are obtained.

Consider Round 3 :

Since there are 29 winners, 1 person gets a bye. The remaining 28


people play 28/2 = 14 matches among themselves.

Thus, 14 winners are obtained.

Since one person has obtained a bye, 15 people go to the next round.

Consider Round 4 :

Since there are 15 winners, 1 person gets a bye. The remaining 14


people play 14/2 = 7 matches among themselves.

Thus, 7 winners are obtained.

Since one person has obtained a bye, 8 people go to the next round.

Consider Round 5 :

8/2 = 4 matches are played and 4 winners are obtained.

Consider Round 6 :

4/2 = 2 matches are played and 2 winners are obtained.

Consider Round 7 :

2/2 = 1 last match is played and the final winner is obtained.

∴ Total number of matches played = 58 + 29 + 14 + 7 + 4 + 2 + 1 = 115

Hence, option 1.

Alternatively,

The concept of a knockout tournament is that a person who loses goes out.

3 of 33 9/16/2011 10:16 AM
Exam Reports http://testfunda.com/LMS/Student/NewReports.aspx

Thus, if a person A wins the second round against B, he is considered to be


better than 3 other players (B, the peron beaten by A in the first round and
the person beaten by B in the first round).

This is equivalent to winning 3 matches.

Thus, the person winning the tournament is considered to have won once
against every other player participating in the tournament.

Therefore, if there are n players participating in the tournament, it is


equivalent to one player beating the remaining (n − 1) players exactly once.

Therefore, in a knockout tournament, if n players participate, (n – 1) matches


should be played to decide the winner.

Hence, in a knockout tournament of 116 players, the total number of matches


played is 116 – 1 = 115.

Hence, option 1.

Group Question
Answer the following questions based on the information given below.

The following questions relate to a game to be played by you and your friend. The
game consists of a 4 x 4 board (see below) where each cell contains a positive
integer. You and your friend make moves alternately. A move by any of the players
consists of splitting the current board configuration into two equal halves and
retaining one of them. In your moves you are allowed only to split the
board vertically and decide to retain either the left half or the right half. Your
friend, in his/her moves, can only split the board horizontally and can retain either
the lower half or the upper half. After two moves by each player a single cell will
remain which can no longer be split and the number in that cell will be treated as
the gain (in rupees) of the person who has started the game. A sample game is
shown below.

4 of 33 9/16/2011 10:16 AM
Exam Reports http://testfunda.com/LMS/Student/NewReports.aspx

So your gain is Re.1. With the same initial board configuration as above and
assuming that you have to make the first move, answer the following questions.

4. If you choose (retain right) (retain left) in your turns, the best move
3 Marks
sequence for your friend to reduce your gain to a minimum will be
_____.

1) (retain upper) (retain lower)


2) (retain lower) (retain upper)
3) (retain upper) (retain upper)
4) (retain lower) (retain lower)

Solution:
The initial board is:

5 of 33 9/16/2011 10:16 AM
Exam Reports http://testfunda.com/LMS/Student/NewReports.aspx

Since your first move is (retain right), the board configuration after the
move is as shown below

Since your next move is (retain left), your final profit will be one from (2,
6, 3 and 8).

If the friend's first move is (retain lower), your possible profit will be
either Rs. 3 or Rs. 8.

If the friend's first move is (retain upper), your possible profit will be
either Rs. 2 or Rs. 6.

Since his objective is to minimise your gain, he will try and limit your
gain to the least value i.e. Rs. 2.

Hence, his first move will be (retain upper).

Hence, options 2 and 4 can be eliminated.

Since you have now selected (retain left), the numbers left for your
friend are 2(upper) and 6(lower).

To minimise your gain, your friend will select (retain upper).

Your gain now will be Rs. 2.

Hence, the moves are (retain upper), (retain upper).

6 of 33 9/16/2011 10:16 AM
Exam Reports http://testfunda.com/LMS/Student/NewReports.aspx

Hence, option 3.

5. With the initial board configuration as shown in the question, if both of


3 Marks
you select your moves intelligently then at the end of the game, your
gain will be _____.

1) Rs. 4
2) Rs. 3
3) Rs. 2
4) None of these

Solution:
The initial board is:

Since both of you play intelligently, your objective is to get the maximum
possible gain while your friend's objective is to give you the minimum
possible gain.

Case I : You select (retain left).

Since your friend also plays intelligently, he will select (retain upper) so
that your gain does not cross Rs. 5. If he selects (retain lower), your
gain can possibly go up to Rs. 9

Now, you will again select (retain left) to get a maximum possible gain of
Rs. 5

Finally, in order to minimise your gain, your friend will again select
(retain upper).

This will limit your gain to Rs. 2

Thus, with intelligent moves, your maximum gain in this case is Rs. 2

Case II : You select (retain right).

7 of 33 9/16/2011 10:16 AM
Exam Reports http://testfunda.com/LMS/Student/NewReports.aspx

Now, if your friend selects (retain upper), the minimum profit you can
earn is Rs. 2 and the maximum profit you can earn is Rs. 4

On the other hand, if your friend selects (retain lower), the minimum
profit you can earn is Rs. 2 and the maximum profit you can earn is Rs.
3.

Since your friend also plays intelligently, he will select (retain lower) so
that your gain does not cross Rs. 3.

Now, you will again select (retain left) to get a maximum possible gain of
Rs. 8

Finally, in order to minimise your gain, your friend will again select
(retain upper).

This will limit your gain to Rs. 3

Thus, with intelligent moves, your maximum gain in this case is Rs. 3

Since you want to maximise your gain, your first intelligent move will be
(retain right).

As per Case II, your gain will be Rs. 3.

Hence, option 2.

6. If your first move is (retain right), then whatever moves your friend may
3 Marks
select, you can always force a gain of no less than _____.

1) Rs. 3
2) Rs. 6
3) Rs. 4
4) None of these

Solution:
The first move will be as shown below.

8 of 33 9/16/2011 10:16 AM
Exam Reports http://testfunda.com/LMS/Student/NewReports.aspx

If your friend selects (retain upper), you will now select (retain right) to
try and get a maximum gain of Rs. 7.

This means that your friend can now limit your gain to the minimum of
(4, 7) i.e. Rs. 4

If my friend selects (retain lower), you will now select (retain left) to try
and get a maximum gain of Rs. 8.

This means that your friend can now limit your gain to the minimum of
(3, 8) i.e. Rs. 3.

Thus, irrespective of your friend's move, you will never gain anything
less than Rs. 3.

Hence, option 1.

Group Question
Answer the following questions based on the information given below.

The pages of a book are numbered 0, 1, 2 … upto M, M > 0. There are four
categories of instructions that direct a person in positioning the book at a
particular page. The instruction types and their meanings are:

I. OPEN : Position the book at page No. 1


II. CLOSE : Position the book at page No. 0
III. FORWARD, n : From the current page ,move forward by n pages; if, in this process, page
number M is reached, stop at M.
IV. BACKWARD, n : From the current page, move backward by n pages; if in this process, page
number 0 is reached, stop at page number 0.

In each of the following questions, you will find a sequence of instructions formed
from the above categories. In each case, let n1 be the page number before the
instructions are executed and n2 be the page number at which the book is
positioned after the instructions are executed.

9 of 33 9/16/2011 10:16 AM
Exam Reports http://testfunda.com/LMS/Student/NewReports.aspx

7. FORWARD, 25; BACKWARD, 10. Which of the following statements is


3 Marks
definitely true?

1) n1 = n2 if M = 10 and n1 = 0
2) M = 20 provided n1 > 0
3) n1 > 30 provided M = 900
4) n1 = 37 provided M = 25

Solution:
The given instructions are: FORWARD 25, BACKWARD 10.

Option 1: Given M = 10, n1 = 0

Thus, after FORWARD 25, page number 10 will be reached (since M =


10).

Now, after BACKWARD 10, page number 0 will be reached.

∴ n2 = 0 = n1

Hence, the given statement is definitely true.

Option 2: Given M = 20 provided n1 > 0

Assume n1 = 1.

For any M > 25, FORWARD 25 will lead to page number 26 (1 + 25)
and then BACKWARD 10 will lead to page number 16 (26 − 10). This is
valid.

For any M ≤ 25, FORWARD 25 will lead to page number M and


then BACKWARD 25 will lead to page number (M − 10) or to page 0 (if
M ≤ 10). This is also valid

Thus, M may take any value.

Hence, the given statement may or may not be true.

Option 3: Given n1 > 30 provided M = 900

For n1 = 30, n2 = 30 + 25 − 10 = 45 (which is valid).

For n1 > 30 (say 40), n2 = 40 + 25 − 10 = 55 (which is also valid).

10 of 33 9/16/2011 10:16 AM
Exam Reports http://testfunda.com/LMS/Student/NewReports.aspx

Thus, it is not necessary that n1 should be greater than 30.

Hence, the given statement may or may not be true.

Option 4: Given n1 = 37 provided M = 25

Since M is the number of pages in the book and n1 is the page from
where we start, n1 ≤ M.

Here, n1 > M

Hence, the given statement is definitely false.

Hence, option 1.

8. BACKWARD, 5; FORWARD, 5. Which of the following statements is


3 Marks
definitely true about the above set of instructions?

1) n1 = n2 provided n1 ≥ 5
2) n1 = n2 provided n1 > 0
3) n2 = 5 provided M > 0
4) n1 > n2 provided M > 0

Solution:
The given instructions are: BACKWARD 5, FORWARD 5.

Option 1: Given n1 = n2 provided n1 ≥ 5

n1 ≥ 5

Thus, after BACKWARD 5, the intermediate position (n) is n = n1 – 5.

Hence, after FORWARD 5, the final position (n2) is n2 = n1 – 5 + 5 = n1.

∴ n1 = n2

Hence, the given statement is definitely true.

Option 2: Given n1 = n2 provided n1 > 0

Consider n1 = 3.

Thus, after BACKWARD 5, the intermediate position (n) is n = 0

11 of 33 9/16/2011 10:16 AM
Exam Reports http://testfunda.com/LMS/Student/NewReports.aspx

Now, after FORWARD 5, the final position (n2) is n2 = 0 + 5 = 5

∴ n1 ≠ n2

Now, consider n1 = 5.

Thus, after BACKWARD 5, the intermediate position (n) is n = 0

Now, after FORWARD 5, the final position (n2) is n2 = 0 + 5 = 5

∴ n1 = n2

Thus, the given statement may or may not be true.

Option 3: Given n2 = 5 provided M > 0

Let n1 = 2 and M = 4

Thus, after BACKWARD 5, the intermediate position (n) is n = 0

Now, after FORWARD 5, the final position (n2) is n2 = 4

∴ n2 ≠ 5

Now, let n1 = 5 and M ≥ 5

Thus, after BACKWARD 5, the intermediate position (n) is n = 0

Now, after FORWARD 5, the final position (n2) is n2 = 0 + 5 = 5

∴ n2 = 5

Thus, the given statement may or may not be true.

Option 4: Given n1 > n2 provided M > 0.

Let n1 = 1 and M = 5:

Thus, after BACKWARD 5, the intermediate position (n) is n = 0

Now, after FORWARD 5, the final position (n2) is n2 = 0 + 5 = 5

∴ n1 < n2

Thus, the given statement is false.

Hence, option 1.

12 of 33 9/16/2011 10:16 AM
Exam Reports http://testfunda.com/LMS/Student/NewReports.aspx

NOTE : The second case in option 2 and 3 have been given to illustrate
that the statement may or may not be true. Since the question asks for
a “definitely true” condition, you can eliminate that option once it is
proved that the statement has a possibility of being false. However, if
the question had asked for a “possibly true” condition, you would have
needed to check both cases as shown above.

9. FORWARD, 10; FORWARD, 10. Which of the following statements


3 Marks
about the above instructions is true?

1) n2 – n1 = 20 only if n1 = 0
2) n2 – n1 = 20 if M > 20 and n1 = 1
3) n2 – n1 = 10 if M = 21 and n1 = 0
4) n2 > n1 if M > 0

Solution:
The given instructions are: FORWARD 10, FORWARD 10.

Option 1: Given n2 – n1 = 20 only if n1 = 0

This is a restrictive condition as it means that n2 − n1 can be 20 only if


n1 = 0.

Thus, if it can be proved that n2 – n1 = 20 for n1 ≠ 0, the given statement


becomes false.

Let n1 = 1:

Thus, after FORWARD 10, the intermediate position (n) is n = 11

Now, after the second FORWARD 10, n2 = 11 + 10 = 21

∴ n2 – n1 = 21 − 1 = 20

Thus, the given statement is false.

Option 2: Given n2 – n1 = 20 if M > 20 and n1 = 1

Note that this not a restrictive condition like the one above. You just
need to prove that the equation is satisfied for the given values of n1
and M.

n1 = 1. Let M = 21

13 of 33 9/16/2011 10:16 AM
Exam Reports http://testfunda.com/LMS/Student/NewReports.aspx

Thus, after FORWARD 10, the intermediate position (n) is n = 11

Now, after the second FORWARD 10, n2 = 11 + 10 = 21

∴ n2 – n1 = 21 − 1 = 20

Thus, the given statement is true.

Option 3: Given n2 – n1 = 10 if M= 21 and n1 = 0

Again, you just need to prove that the equation is satisfied for the given
values of n1 and M.

Thus, after FORWARD 10, the intermediate position (n) is n = 0 + 10 =


10

Now, after the second FORWARD 10, n2 = 10 + 10 = 20

∴ n2 – n1 = 20 ≠ 10

Thus, this statement is false.

Option 4: Given n2 > n1 if M > 0

This statement need not be true in all cases.

For example, if n1 = 1 and M = 1:

Thus, after FORWARD 10, the intermediate position (n) is n = 1

Now, after the second FORWARD 10, n2 = 1

Here, n2 = n1.

Thus, this statement may or may not be true.

Hence, option 2.

10. FORWARD, 5; BACKWARD, 4. Which of the following statements about


3 Marks
the above instructions is definitely true?

1) n2 = n1 + 4 provided 1 < n1 < 7


2) n2 = n1 provided M < 6
3) n2 = n1 + 1 provided M – n1 > 5

14 of 33 9/16/2011 10:16 AM
Exam Reports http://testfunda.com/LMS/Student/NewReports.aspx

4) n2 – n1 < 0 provided M > 0

Solution:
The given instructions are: FORWARD 5, BACKWARD 4.

Option 1: Given n2 = n1 + 4 provided 1 < n1 < 7

This statement need not be true in all cases.

Let n1 = 2:

Thus, after FORWARD 5, the intermediate position (n) is n = 7

Now, after BACKWARD 4, n2 = 7 − 4 = 3

Here, n2 ≠ n1 + 4

Thus, the given statement is not definitely true.

Option 2: Given n2 = n1 provided M < 6

This statement need not be true in all cases.

Let n1 = 3 and M = 5:

Thus, after FORWARD 5, the intermediate position (n) is n = 5

Now, after BACKWARD 4, n2 = 5 − 4 = 1

Here, n1 ≠ n2

Thus, the given statement is not definitely true.

Option 3: Given n2 = n1 + 1 provided M – n1 > 5

If M – n1 > 5, then M > n1 + 5.

Therefore, after FORWARD 5, the intermediate position (n) is n = n1 + 5


and not n = M

Now, after BACKWARD 4, n2 = n1 + 5 – 4 = n1 + 1

Hence, the given statement is true.

Option 4: Given n2 – n1 < 0 provided M > 0

15 of 33 9/16/2011 10:16 AM
Exam Reports http://testfunda.com/LMS/Student/NewReports.aspx

This statement need not be true in all cases.

Let n1 = 1 and M = 10:

Thus, after FORWARD 5, the intermediate position (n) is n = 1 + 5 = 6

Now, after BACKWARD 4, n2 = 6 – 4 = 2

∴ n2 − n1 = 2 – 1 = 1 > 0

Thus, the given statement may or may not be true.

Hence, option 3.

11. The last time Rahul bought Diwali cards, he found that the four types of cards
3 Marks
that he liked were priced at Rs. 2.00, Rs. 3.50, Rs. 4.50 and Rs. 5.00 each.
As Rahul wanted 30 cards, he took five each of two kinds and ten each of the
other two, putting down an exact number of 10 rupees notes on the counter
payment. How many notes did Rahul give?

1) 8
2) 9
3) 10
4) 11

Solution:
In all, there are 4 varieties of cards, of a price Rs 2, Rs 3.50, Rs 4.50 and Rs 5
per card.

Rahul buys 30 cards and puts an amount which is a multiple of 10. Also, he
buys 5 cards each of two types and 10 cards each of the other two types.

Since two denominations are in terms of decimals, they will together give a
number that is an integer only if the same number of cards is bought for each
denomination.

In such a case, the total cost of the Rs. 3.5 and the Rs. 4.5 cards will be 3.5x
+ 4.5x = 8x. Here, x is the number of cards bought for each of these two
types.

Now, if Rahul buys 10 cards each of these two denominations, he will buy 5
cards each of the Rs. 2 and the Rs. 5 denomination.

Thus, the total cost will be 2(5) + 5(5) + 8(10) = 10 + 25 + 80 = Rs. 115

16 of 33 9/16/2011 10:16 AM
Exam Reports http://testfunda.com/LMS/Student/NewReports.aspx

Since the total amount has to be a multiple of 10, this is not possible.

Hence, Rahul buys 5 cards each of the Rs. 3.5 and the Rs. 4.5 denomination
and 10 cards each of the Rs. 2 and the Rs. 5 denominations.

Thus, the total cost is 2(10) + 5(10) + 8(5) = 20 + 50 + 40 = Rs. 110.

Since Rahul places all Rs. 10 notes, he places 110/10 = 11 notes in all.

Hence, option 4.

12. I happened to be the judge in the all India Essay Competition on Nylon
3 Marks
Dying, organized some time back by a dyestuff firm. Mill technicians were
eligible to enter the competition. My work was simplified in assessing the
essays, which had to be done under five heads-Language, Coherence,
Subject Matter, Machinery and Recent Developments. Marks were to be given
out of a maximum of 20 under each head. There were only five entries
resulting in a sole winner. The winner got 90 marks. Akhila got 13 in
Coherence and Divya 10 in Machinery. Bhanu’s total was less than Akhila’s.
Charulata has sent an entry. Ela had got as many marks as Divya. No one got
20 under any head.

Who was the winner?

1) Divya and Ela


2) Charulata
3) Akhila
4) Bhanu

Solution:
Consider the answer options.

Since Bhanu's total was less than Akhila's total, Bhanu cannot be the winner.

Hence, option 4 can be eliminated.

Ela and Divya had the same marks. However, there was a sole winner.

Thus, Ela and Divya could not have been combined winners.

Hence, option 1 can be eliminated.

Now, Akhila got 13 in Coherence and the winner's total marks were 90.

Thus, if Akhila was the winner, she should have scored 90 − 13 = 77 under

17 of 33 9/16/2011 10:16 AM
Exam Reports http://testfunda.com/LMS/Student/NewReports.aspx

the remaining 4 heads.

No person got 20 under any individual head. Thus, the maximum one could
have got under any head was 19.

Even if Akhila got 19 in the remaining 4 heads, she would have scored 19 × 4
= 76 in the remaining four heads.

Thus, her total marks would have been 76 + 13 = 89.

Hence, Akhil could not have been the winer.

Hence, Charulata was the winner.

Hence, option 2.

13. A player rolls a die and receives the same number of rupees as the number
3 Marks
of dots on the face that turns up. What should the player pay for each roll if
he wants to make a profit of one rupee per throw of the die in the long run?

1) Rs. 2.50
2) Rs. 2
3) Rs. 3.50
4) Rs. 4

Solution:
The possible outcome on one throw of a fair die is one from 1, 2, 3, 4, 5 and
6.

Thus, the probability of each of these outcomes is 1/6.

Now, in the long run, since each outcome has an equal chance of appearing,
the expected outcome from one roll of an ordinary (that is, fair) six-sided die

Thus, if the player wants to make a profit of one rupee per throw in the long
run, then the amount that should be paid by the player for each roll = 3.5 – 1
= Rs. 2.5

Hence, option 1.

14. A calculator has two memory locations, A and B. The value ‘1’ is initially
3 Marks
stored in both memory locations. The following sequence of steps is carried

18 of 33 9/16/2011 10:16 AM
Exam Reports http://testfunda.com/LMS/Student/NewReports.aspx

out five times:


Add 1 to B.
Multiply A to B.
Store the result in A.

What is the value stored in the memory location A at the end of


this procedure?

1) 120
2) 450
3) 720
4) 250

Solution:

The calculations and the value in memory locations A and B at the end of
each iteration are as shown in the table given above.

Thus, the value in the memory location A at the end of the 5th iteration is 720.

Hence, option 3.

15. X and Y are playing a game. There are eleven 50 paise coins on the table
3 Marks
and each player must pick up at least one coin but not more than five. The
person picking up the last coin loses. X starts. How many coins should he
pick up at the start to ensure a win no matter what strategy Y employs?

1) 4
2) 3
3) 2
4) 5

Solution:
This problem can easily be solved by working backwards.

If X has to win the game, there has to be 1 coin left on the table after his turn.

19 of 33 9/16/2011 10:16 AM
Exam Reports http://testfunda.com/LMS/Student/NewReports.aspx

This means that B canot have the second last turn and that the last turn has
to be B's.

One player can pick a maximum of 5 coins in one turn.

Thus, A has to pick x coins at the start such that there 5 + 1 + 1 = 7 coins left
on the table.

Since there are 11 coins in all, A has to pick up 4 coins at the start.

Hence, option 1.

16. The money order commission is calculated as follows. From Rs. X to be sent
3 Marks
by money order, subtract Rs. 0.01 and divide by 10. Add 1 to the quotient
obtained. If the result is Rs. Y, the money order commission is Rs. 0.5Y. If a
person sends two money orders to Aurangabad and Bhatinda for Rs. 71 and
Rs. 48 respectively, the total commission is:

1) Rs. 7.00
2) Rs. 6.50
3) Rs. 6.00
4) Rs. 7.50

Solution:

When the money order amount is Rs. 71,

X − 0.01 = 70.99

The quotient when this is divided by 10 is 7.

∴ The commission = 0.5 × (7 + 1) = Rs. 4

When the money order amount is Rs. 48,

X − 0.01 = 47.99

The quotient when this divided by 10 is 4.

20 of 33 9/16/2011 10:16 AM
Exam Reports http://testfunda.com/LMS/Student/NewReports.aspx

∴ The commission = 0.5 × (4 + 1) = Rs. 2.5

∴ The total commission = 4 + 2.5 = Rs. 6.5

Hence, option 2.

17. The auto fare in Ahmedabad is calculated using the meter reading and the
3 Marks
following process. The meter reading (in paise) is rounded up to the multiple
of 4 just greater than this reading. For instance, if the meter reading is 37
paise, it is rounded up to 40 paise. The resultant is multiplied by 12. The final
result is then rounded off to the nearest multiple of 25 paise. If 53 paise is the
meter reading what will the actual fare be?

1) Rs. 6.75
2) Rs. 6.50
3) Rs. 6.25
4) Rs. 7.50

Solution:
Since the meter reading is 53 paise, it is first rounded off to the next multiple
of 4 i.e. 56 paise.

Now, 56 × 12 = 672 paise.

The multiple of 25 nearest to this value is 675.

Thus, the amount is rounded up to 675 paise.

Hence, the actual fare is Rs. 6.75

Hence, option 1.

18. Juhi and Bhagyashree were playing simple mathematical puzzles. Juhi wrote
3 Marks
a two-digit number and asked Bhagyashree to guess what it was. Juhi also
indicated that the number was exactly thrice the product of its digits. What
was the number that Juhi wrote?

1) 36
2) 24
3) 12
4) 48

21 of 33 9/16/2011 10:16 AM
Exam Reports http://testfunda.com/LMS/Student/NewReports.aspx

Solution:
This can be solved very easily using the answer options and verifying the
product of digits.

Option 1: 36 ≠ 3 × (3 × 6), so option 1 is incorrect.

Option 2: 24 = 3 × (2 × 4), so option 2 is correct

Option 3: 12 ≠ 3 × (1 × 2), so option 3 is incorrect.

Option 4: 48 ≠ 3 × (4 × 8), so option 4 is incorrect.

Thus, Juhi wrote the number 24.

Hence, option 2.

19. After paying all your bills, you find that you have Rs. 7.20 in your pocket. You
3 Marks
have an equal number of 50 paise and 10 paise coins, but no other coins nor
any other currency notes. How many coins do you have?

1) 8
2) 24
3) 12
4) 30

Solution:
Let the number of 50 paise coins be n.

Since the number of 50 paise coins and 10 paise coins is the same, the
number of 10 paise coins is also n.

Since the total amount in your pocket is Rs. 7.20 and you have no other coins
or currency notes,

∴ 0.5n + 0.1n = 7.20

∴ 0.6n = 7.2

∴ n = 12

∴ The total number of coins = 2 × 12 = 24

Hence, option 2.

22 of 33 9/16/2011 10:16 AM
Exam Reports http://testfunda.com/LMS/Student/NewReports.aspx

Group Question
Answer the following questions based on the information given below.

Alord has received a large order for stitching school uniforms from Mayflower
School and Little Flower School. He has two cutters to cut the fabric, five tailors to
do the stitching, and two assistants to stitch the buttons and buttonholes. Each of
these nine persons work for exactly 10 hours a day. Each of the Mayflower
uniforms requires 20 minutes for cutting the fabric, one hour for stitching, and 15
minutes for stitching the buttons and buttonholes. On the other hand, each Little
Flower uniform requires 30 minutes, 1 hour, and 30 minutes, respectively for
cutting, stitching and stitching the buttons and buttonholes.

20. On a particular day, Alord decided to complete 20 Little Flower


3 Marks
uniforms. How many Mayflower uniforms can he complete on that day?

1) 30
2) 40
3) 25
4) 0

Solution:
Alord has 2 cutters who both work for 10 hours a day.

Each Little Flower uniform requires 30 minutes for cutting.

Thus, one cutter can cut 2 uniforms per hour.

Thus, both the cutters can together cut all 20 uniforms in 5 hours.

Similarly, each Little Flower uniform requires 30 minutes for stitching


buttons and buttonholes.

Alord has 2 assistants who stitch buttons and buttonholes for 10 hours
a day.

Thus, as seen above, both the assitants can together finish 20 uniforms
in 5 hours.

Finally, Alord has 5 tailors who work 10 hours a day.

Each Little Flower uniform requires 1 hour for stitching.

Thus, if all 5 tailors work together, the Little Flower uniforms can be
completed in 4 hours.

23 of 33 9/16/2011 10:16 AM
Exam Reports http://testfunda.com/LMS/Student/NewReports.aspx

Thus, at the end of 5 hours, all 20 Little Flower uniforms are complete.

Each Mayflower uniform requires 20 minutes for cutting, 1 hour for


stitching and 15 minutes for stitching the buttons and buttonholes.

Thus, 2 cutters working together can complete 30 Mayflower uniforms in


the 5 hours left.

Similarly, 2 assistants working together can complete 40 Mayflower


uniforms left with them.

Finally, 5 tailors working together can complete 30 Mayflower uniforms


in the 6 hours left with them.

Since the number of completed Mayflower uniforms is required, the


minimum of the three values above is to chosen.

Thus, Alord can complete 30 Mayflower uniforms.

Hence, option 1.

21. If Alord decides to complete 30 Little Flower uniforms only and do


3 Marks
nothing else on a particular day, how many total man-hours will be idle?

1) 20
2) 30
3) 5
4) 25

Solution:
Each Little Flower uniform requires 30 minutes, 1 hour and 30 minutes
for cutting, stitching and stitching buttons and button holes respectively.

Also, Alord has 2 cutters, 5 tailors and 2 assistants to do these three


respective activities. Each person works for 10 hours a day.

Working together, both cutters can finish 4 uniforms per hour.

Thus, the time taken to complete Little Flower uniforms is 30/4 = 7.5
hours.

Thus, each cutter has 2.5 hours left idle.

Therefore, the total idle man-hours for the cutters is 2.5 × 2 = 5 hours.

24 of 33 9/16/2011 10:16 AM
Exam Reports http://testfunda.com/LMS/Student/NewReports.aspx

Similarly, the total idle time man-hours for the tailors and the assistants
can be found as shown in the table below.

∴ Total idle man-hours = 20 + 5 + 5 = 30 hours

Hence, option 2.

22. What is the maximum number of Little Flower uniforms that Alord can
3 Marks
complete in a day?

1) 50
2) 20
3) 40
4) 30

Solution:
Number of uniforms cut in a day = Number of cutters × (Total number of
hours worked by each cutter/Time required by one cutter to cut one
uniform)

= 2 × (10/0.5)

= 40

Similarly, number of uniforms stitched in a day = 5 × (10/1)

= 50

Finally, number of uniforms with buttons and buttonholes stitched in a day


= 2 × (10/0.5)

=
40

25 of 33 9/16/2011 10:16 AM
Exam Reports http://testfunda.com/LMS/Student/NewReports.aspx

Since one uniform can be considered complete only if all three acitivities
have been completed on it, the maximum number of Little Flower uniforms
that Alord can make in a day is equal to the smallest of the three numbers
above i.e. 40.

Thus, Alord can complete a maximum of 40 Little Flower uniforms in a day.

Hence, option 3.

23. Alord has the option to hire one more employee of any category. Which
3 Marks
category should he hire to get the maximum increase in production
capacity assuming that he needs to stitch only Mayflower uniforms on
that day?

1) Tailor
2) Cutter
3) Assistant
4) Indeterminate

Solution:
Each person has 10 man-hours available on a day.

One Mayflower uniform requires 20 minutes, 1 hour and 15 minutes to


cut, stitch and put the buttons and buttonholes respectively.

Thus, 1 cutter, 1 tailor and 1 assitant can complete 30, 10 and 40


Mayflower uniforms per day.

Alord has 2 cutters, 5 tailors and 2 assistants.

Therefore, 2 cutters, 5 tailors and 2 assitants can complete 60, 50 and


80 Mayflower uniforms per day.

Since a uniform is considered complete only if all three activities have


been completed, it can be said that Alord has 50 sets of the Mayflower
uniform ready.

Thus, if Alord wants to increase the production capacity, he should hire


more tailors.

Hence, option 1.

24. If he hires one more assistant, what is the maximum number of


3 Marks
Mayflower uniforms that he can complete in a day?

26 of 33 9/16/2011 10:16 AM
Exam Reports http://testfunda.com/LMS/Student/NewReports.aspx

1) 40
2) 50
3) 60
4) 30

Solution:
Each person has 10 man-hours available on a day.

One Mayflower uniform requires 20 minutes, 1 hour and 15 minutes to


cut, stitch and put the buttons and buttonholes respectively.

Thus, 1 cutter, 1 tailor and 1 assitant can complete 30, 10 and 40


Mayflower uniforms per day.

Alord has 2 cutters, 5 tailors and 2 assistants.

Therefore, 2 cutters, 5 tailors and 2 assitants can complete 60, 50 and


80 Mayflower uniforms per day.

Hence, if there are 3 assistants, they can complete 40 × 3 = 120


uniforms per day.

However, only 50 uniforms have been stitched.

Thus, the maximum number of Mayflower uniforms that can be


completed is 50.

Hence, option 2.

25. 139 persons have signed for an elimination tournament. All players are to be
3 Marks
paired up for the first round, but because 139 is an odd number one player
gets a bye, which promotes him to the second round, without actually playing
in the first round. The pairing continues in the next round, with a bye to any
player left over. If the schedule is planned so that a minimum number of
matches is required to determine the champion, then the number of matches
which must be played is:

1) 136
2) 137
3) 138
4) 139

27 of 33 9/16/2011 10:16 AM
Exam Reports http://testfunda.com/LMS/Student/NewReports.aspx

Solution:
The concept of a knockout competition comprising n players is that the winner
is better than every other player.

Thus, the tournament is equivalent to one where the winner beats the
remaining (n − 1) players.

To do so, the winner requires (n − 1) matches.

Hence, in a knockout tournament of n players, (n − 1) matches are required


to determine the champion.

Since this tournament has 139 players, the number of matches that must be
played is 138.

Hence, option 3.

26. There are ten 50 paise coins placed on a table. Six of these show tails and
3 Marks
four show heads. A coin is chosen at random and flipped over (not tossed).
This operation is performed seven times. One of these coins is then covered.
Of the remaining nine coins, five show tails and four show heads. The
covered coin shows:

1) A head
2) A tail
3) More likely a head
4) More likely a tail

Solution:
Assume that one coin, say one showing a tail, is flipped 7 times, instead of 7
different coins. Then, it would show a head and there would be 5 heads and
5 tails.

If by covering a coin, there are 5 tails and 4 heads to be seen, then the
covered coin is showing a head.

Hence, option 1.

Alternatively,

Assume that all four coins that now show heads are flipped coins. Hence,
they originally showed tails.

Hence, the remaining two coins that originally showed tails still show tails.

28 of 33 9/16/2011 10:16 AM
Exam Reports http://testfunda.com/LMS/Student/NewReports.aspx

Now, since there are seven flips in all, only three coins that originally showed
heads can be flipped.

Therefore, they will now show tails.

Thus, the number of coins now showing tails is 2 + 3 = 5.

Thus, the only coin that is now left shows a head.

Since five tails and four heads are visible, the remaining coin (the one that is
covered) shows a head.

Hence, option 1.

27. Along a road lie an odd number of stones placed at intervals of 10 m. These
3 Marks
stones have to be assembled around the middle stone. A person can carry
only one stone at a time. A man carried out the job starting with the stone in
the middle, carrying stones in succession, thereby covering a distance of 4.8
km. Then, the number of stones on the road is:

1) 30
2) 15
3) 29
4) 31

Solution:
If the total distance covered is 4.8 km, the distance covered on each side of
the middle stone is 2.4 km.

Assume that there are n stones on each side of the middle stone.

Each stone is kept at an interval of 10 m.

Thus, the man travels a distance of 10 + 10 = 20 m to carry the first stone and
place it around the middle stone.

Similarly, the man travels a distance of 20 + 20 = 40 m to carry the second


stone and place it around the middle stone.

Thus, the distance travelled by the man on one side forms a G.P. with a = 20
and d = 20.

Since there are n stones on one side,

∴ (n/2) × [2a + (n − 1)d] = 2400

29 of 33 9/16/2011 10:16 AM
Exam Reports http://testfunda.com/LMS/Student/NewReports.aspx

∴ (n) × [40 + (n − 1)20] = 4800

∴ n × (n + 1) = 240

∴ n2 + n − 240 = 0

On solving this, you get, n = 15

Thus, there are 15 stones on one side of the road.

Therefore, the total number of stones = 15 + 1 + 15 = 31

Hence, option 4.

Group Question
Answer the following questions based on the information given below.

Alphonso, on his death bed, keeps half his property for his wife and divides the
rest equally among his three sons Ben, Carl and Dave. Some years later Ben dies
leaving half his property to his widow and the remaining to his brothers Carl and
Dave together, shared equally. When Carl makes his will he keeps half his
property for his widow and the rest he bequeaths to his younger brother Dave.
When Dave dies some years later, he keeps half his property for his widow and
the remaining for his mother. The mother now has Rs. 15,75,000.

28. What was the total property worth?


3 Marks

1) Rs. 30 lakh
2) Rs. 8 lakh
3) Rs. 18 lakh
4) Rs. 24 lakh

Solution:
Let the value of the property of Alphonso be Rs. x

Thus, Alphonso's wife gets Rs. x/2 and the three sons together get Rs.
x/2.

Thus, each son gets Rs. x/6.

Now, when Ben dies, his wife gets [x/6]/2 = Rs. x/12.

Carl and Dave together get Rs. x/12 and individually get Rs. x/24

30 of 33 9/16/2011 10:16 AM
Exam Reports http://testfunda.com/LMS/Student/NewReports.aspx

Thus, the amount with Carl as well as Ben is Rs. [(x/6) + (x/24)]

Now, Carl leaves half of this property for his wife and the other half for
Dave.

Thus, Carl's wife gets Rs. [(x/12) + (x/48)]

Thus, the amount now available with Dave is [(x/6) + (x/24)] + (1/2) ×
[(x/6) + (x/24)] i.e. Rs. (3/2) × [(x/6) + (x/24)]

Dave leaves half of this amount for his wife and the remaining for his
mother.

Thus, Dave's wife gets Rs. (3/4) × [(x/6) + (x/24)]

Now, Dave's mother has a total amount of Rs. (x/2) + (3/4) × [(x/6) +
(x/24)]

i.e. (x/2) + (x/8) + (x/32)

Thus, the total amount that Dave's mother has is 21x/32.

The amount received by each person at every stage is as shown in the


table below.

It is given that Dave's mother has Rs. 15,75,000 at the end.

∴ (21x/32) = 1575000

∴ x = (1575000 × 32)/21

= 75000 × 32

= 2400000

31 of 33 9/16/2011 10:16 AM
Exam Reports http://testfunda.com/LMS/Student/NewReports.aspx

Thus, the total property was worth Rs. 24 lakhs.

Hence, option 4.

29. What was Carl’s share when he made his will?


3 Marks

1) Rs. 4 lakh
2) Rs. 12 lakh
3) Rs. 6 lakh
4) Rs. 5 lakh

Solution:

So, the original share of Carl = 4 + 1 = 5 lakhs

Hence, option 4.

30. What was the ratio of the property owned by the widows of the three
3 Marks
sons, in the end?

1) 7 : 9 : 13
2) 8 : 10 : 15
3) 5:7:9
4) 9 : 12 : 13

Solution:
The worth of the total property x = 24 lakhs

The required ratio is = 2 : 2.5 : 3.75

This ratio is the same as 8 : 10: 15.

32 of 33 9/16/2011 10:16 AM
Exam Reports http://testfunda.com/LMS/Student/NewReports.aspx

Hence, option 2.

33 of 33 9/16/2011 10:16 AM
Exam Reports http://testfunda.com/LMS/Student/NewReports.aspx

Section I

Group Question
Answer the following questions based on the information given below.

A, B, C, D, E, F and G are brothers. Two brothers had an argument and A said to B, “You
are as old as C was when I was twice as old as D, and will be as old as E was when he
was as old as C is now”. B said to A, “You may be older than F but G is as old as I was
when you were as old as G is, and D will be as old as F was when F will be as old as G is”.

1. Who is the eldest brother?


3 Marks

1) A
2) E
3) C
4) Indeterminate

Solution:
B is as old as C was when A was twice as old as D was.

Thus, C had reached B's age before B had.

∴ C > B; A > D … (i)

B will be as old as E was when he was as old as C is now.

∴ E > B; E > C … (ii)

A may be older than F but G is as old as B was when A was as old as G is.

∴ A > F; B > G; A = B … (iii)

D will be as old as F was when F will be as old as G is.

∴ F > D; G > F … (iv)

From (i), (ii), (iii) and (iv),

E > C > A = B > G > F > D.

Thus, the eldest brother is E.

Hence, option 2.

2. Who is the youngest brother?


3 Marks

1) B

1 of 16 9/16/2011 10:17 AM
Exam Reports http://testfunda.com/LMS/Student/NewReports.aspx

2) D
3) F
4) Indeterminate

Solution:
From the solution to the first question, the youngest brother is D.

Hence, option 2.

3. Which two brothers are probably twins?


3 Marks

1) D and G
2) E and C
3) A and B
4) Indeterminate

Solution:
From the solution to the first question, it can be seen that A and B are of the
same age.

Thus, they are the only pair that can possibly be twins.

Hence, option 3.

4. Which of the following statements is false?


3 Marks

1) G has 4 elder brothers.


2) A is older than G but younger than E.
3) B has 3 elder brothers.
4) There is a pair of twins among the brothers.

Solution:
Refer to the order obtained in the solution to the first question.

E>C>A=B>G>F>D

Statement 1 : G has four elder brothers i.e. E, C, A and B.

Thus, statement 1 is true.

Statement 2 : From the order it is clear that A is older than G but younger than E.

Thus, statement 2 is true.

Statement 3 : B has 3 elder brothers.

2 of 16 9/16/2011 10:17 AM
Exam Reports http://testfunda.com/LMS/Student/NewReports.aspx

From the order it can be seen that B has only two elder brothers i.e. E and C. B
had one brother (A) who is of the same age.

Thus, statement 3 is false.

Statement 4 : There is a pair of twins among the brothers.

Since A and B have the same age, they are twins.

Thus, there is a pair of twins among the brothers.

Thus, statement 4 is true.

Hence, option 3.

Group Question
Answer the following questions based on the information given below.

Four sisters Suvarna, Tara, Uma and Vibha were playing a game such that the loser
doubled the money of the other players from her share. They played four games and each
sister lost one game in the alphabetical order. At the end of fourth game each sister had
Rs. 32 with her.

5. How many rupees did Suvarna start with?


3 Marks

1) 60
2) 34
3) 66
4) 28

Solution:
Let the amounts held by Suvarna, Tara, Uma and Vibha at any stage be denoted
by S, T, U and V respectively.

At the end of game 4, all of them have Rs 32 with them.

Since Vibha lost game 4, she would have doubled the other three players'
amount.

Thus, at the end of the game 3 (or at the beginning of game 4), Suvarna, Tara
and Uma would each have had Rs. 32/2 = Rs. 16.

Also, Vibha would have had 32 + 16 + 16 + 16 = Rs. 80

Similarly, the amount with each person at the end of each game can be
calculated as shown in the table below.

3 of 16 9/16/2011 10:17 AM
Exam Reports http://testfunda.com/LMS/Student/NewReports.aspx

It is evident from the table that Suvarna started with 66 coins.

Hence, option 3.

6. Who started with the lowest amount?


3 Marks

1) Suvarna
2) Tara
3) Uma
4) Vibha

Solution:
From the table in the solution to the first question, Vibha started with the least
amount (Rs. 10).

Hence, option 4.

7. Who started with the highest amount?


3 Marks

1) Suvarna
2) Tara
3) Uma
4) Vibha

Solution:
From the table in the solution to the first question, it is clear that Suvarna started
with the highest amount (i.e. Rs. 66)

Hence, option 1.

8. What was the amount with Uma at the end of the second game?
3 Marks

1) 36
2) 72
3) 16
4) None of these

4 of 16 9/16/2011 10:17 AM
Exam Reports http://testfunda.com/LMS/Student/NewReports.aspx

Solution:
From the table in the solution to the first question, Uma had Rs. 72 at the end of
the second game.

Hence, option 2.

Group Question
Answer the following questions based on the information given below.

A salesman entered the quantity sold and the price into the computer. Both the numbers
were two-digit numbers. But by mistake, both numbers were entered with their digits
interchanged. The total sales value remained the same, i.e. Rs. 1,148, but the inventory
reduced by 54 units.

9. What is the actual price per article?


3 Marks

1) Rs. 82
2) Rs. 41
3) Rs. 14
4) Rs. 28

Solution:
Total sales value = 1148 = Price × Quantity

To find the price and quantity, factorise 1148 in terms of two two-digit numbers.

1148 = 2 × 2 × 7 × 41

∴ 1148 = 14 × 82 = 28 × 41

Inventory is the amount of goods remaining in the store after the sale.

Since this has mistakenly reduced by 54 units, it means that the quantity has
been mistakenly shown to be more by 54 units.

82 = 28 + 54

Thus, actual number of units sold = 28

Thus, the original price has to be Rs. 41 and then after interchanging, it becomes
Rs. 14.

Thus, the actual price = Rs. 41

Hence, option 2.

10. What is the actual quantity sold?


3 Marks

5 of 16 9/16/2011 10:17 AM
Exam Reports http://testfunda.com/LMS/Student/NewReports.aspx

1) 28
2) 14
3) 82
4) 41

Solution:
From the solution to the previous question, it is clear that the actual quantity
sold is 28 units.

Hence, option 1.

11. Out of two-thirds of the total number of basketball matches scheduled, a team has
3 Marks
won 17 matches and lost 3. What is the maximum number of matches that the team
can lose and still win more than three-fourths of the total number of matches
scheduled, if it is true that no match can end in a tie?

1) 4
2) 6
3) 5
4) 3

Solution:
Matches played so far = 17 + 3 = 20

∴ Total number of matches scheduled = 20 × (3/2) = 30

If the team wants to win more than three-fourths of all the matches scheduled, they
have to win at least (3/4) × 30 i.e. at least 22.5 matches.

Thus, the team will have to win at least 23 matches.

The team has already won 17 matches.

Thus, in the next 10 matches, (23 – 17) = 6 matches have to be won.

Hence, not more than (10 – 6) = 4 matches can be lost to maintain the required the
winning ratio.

Hence, option 1.

Group Question
Answer the following questions based on the information given below.

The production pattern for a number of units (in cubic feet) per day:

6 of 16 9/16/2011 10:17 AM
Exam Reports http://testfunda.com/LMS/Student/NewReports.aspx

For a truck that can carry 2000 cubic feet, the hiring cost per day is Rs. 1,000. The storing
cost per cubic feet is Rs. 5 per day.

12. If all the units have to be sent to the market, on which days should the truck be
3 Marks
hired to minimise the total cost?

1) 2nd, 4th, 6th, 7th


2) Only on 7th
3) 2nd, 4th, 5th and 7th
4) None of these

Solution:
Hiring cost of truck per day = Rs 1,000.

Storage cost for each day of storage = Rs 5 per cubic feet.

Thus, hiring a truck is cheaper compared to storage for more than 200 cubic feet.

Thus, on all days when the number of units present is less than or equal to 200,
hiring a truck is costlier than storing the goods. Thus, on such days, the units
should be stored and not sent out.

On the other hand, on all days when the number of units present is more than
200, a truck should be hired to minimize the total cost.

The truck has to be hired on the 7th day because that is the last available day and
all the units have to be sent to the market.

Hence, the truck is hired on the 2nd, 4th, 6th and the 7th days.

Hence, option 1.

13. If storage costs are reduced to Rs. 0.8 per cubic feet per day, then on which days
3 Marks
should the trucks be hired to minimise the total cost?

7 of 16 9/16/2011 10:17 AM
Exam Reports http://testfunda.com/LMS/Student/NewReports.aspx

1) 2nd, 4th, 6th, 7th


2) Only on 7th
3) 2nd, 4th, 5th and 7th
4) None of these

Solution:
Hiring cost of truck per day = Rs 1,000.

Storage cost for each day of storage = Rs 0.8 per cubic feet.

Thus, hiring a truck is cheaper compared to storage for more than (1000/0.8) i.e.
1250 cubic feet.

Thus, on all days when the number of units present is less than or equal to 1250,
hiring a truck is costlier than storing the goods. Thus, on such days, the units should
be stored and not sent out.

On the other hand, on all days when the number of units present is more than 1250, a
truck should be hired to minimize the total cost.

If this quantity is not reached till the last day, then on the 7th day, the truck has to be
hired to transport all the units to the market.

Hence, the truck is hired only on the 7th day.

Hence, option 2.

Group Question
Answer the following questions based on the information given below.

A, B, C and D collected one rupee coins as per the following conditions:


Together they collected 100 coins.
Each one of them collected an even number of coins.
Each one of them collected at least 10 coins.
No two of them collected the same number of coins.

14. The maximum number of coins collected by any one of them cannot exceed
3 Marks
_____.

8 of 16 9/16/2011 10:17 AM
Exam Reports http://testfunda.com/LMS/Student/NewReports.aspx

1) 64
2) 36
3) 54
4) None of these

Solution:
Each person collects an even number of coins such that the minimum number of
coins with each person is 10.

To find the maximum possible number of coins collected by a single person,


assume that the other three people collect the lowest possible number of coins.

No two people collect the same number of coins.

Therefore, assume that A, B and C collect 10, 12 and 14 coins respectively.

Thus, the maximum number of coins that the fourth person i.e. D can collect =
100 − (10 + 12 + 14)

= 100
− 36

= 64

Thus, the maximum number of coins collected by a person cannot exceed 64.

Hence, option 1.

15. If A collected 54 coins, then the difference in the number of coins between the
3 Marks
one who collected the maximum number of coins and the one who collected the
second-highest number of coins must be at least _____.

1) 12
2) 24
3) 30
4) None of these

Solution:
The difference between the person who collected the most number of coins and
the one who collected the 2nd highest number of coins would be minimum when
the 2nd highest collects the maximum number of coins possible for him.

Assume that the person who collects the 2nd highest number of coins is B.

Now, B can collect the maximum number of coins when C and D collect as few
coins as possible.

9 of 16 9/16/2011 10:17 AM
Exam Reports http://testfunda.com/LMS/Student/NewReports.aspx

This can happen if one of C and D collects 10 coins while the other collects 12
coins.

In such a case, B collects 100 − 54 − (10 + 12) i.e. 24 coins

Thus, the 2nd highest collector (assumed as B) can collect a maximum of 24


coins.

Therefore, the difference between the highest and second highest person is at
least 54 − 24 i.e. 30 coins.

Hence, option 3.

16. If A collected 54 coins and B collected two more coins than the twice of the
3 Marks
number of coins collected by C, then the number of coins collected by B could
be _______.

1) 28
2) 20
3) 26
4) 22

Solution:
Let the number of coins collected by A, B, C and D be a, b, c and d respectively.

Given that: a = 54, b = (2 × c) + 2

a + b + c + d = 100

∴ b + c + d = 100 – 54 = 46

∴ 2c + 2 + c + d = 46

∴ 3c + d = 44, where c ≥ 10 and d ≥ 10

Since the minimum value of d is 10, the maximum value of 3c is 34.

For this value, the maximum value of c can be 11.33

However, c has to be even and have a value of at least 10.

Only c = 10 satisfies all three conditions.

Consider c = 10, then d = 14 and b = 22

Thus, the number of coins collected by B would be 22.

Hence, option 4.

10 of 16 9/16/2011 10:17 AM
Exam Reports http://testfunda.com/LMS/Student/NewReports.aspx

17. A, B, C, D, … X, Y, Z are players who participated in a tournament. Everyone played


3 Marks
with every other player exactly once. A win scores 2 points, a draw scores 1 point and
a loss scores 0 points. None of the matches ended in a draw. No two players had the
same score. At the end of the tournament, the ranking list is published which is in
accordance with the alphabetical order. Then ...

1) M wins over N.
2) N wins over M.
3) M does not play with N.
4) None of these

Solution:
Since no match ends in a draw and no two players have the same score, it means that
all players have won matches according to their rankings which correspond to the
alphabetical order.

Since M takes precedence over N in the ranking order, M should have won over N.

Hence, option 1.

18. Three labeled boxes containing red and white cricket balls are all mislabeled. It is
3 Marks
known that one of the boxes contains only white balls and one only red balls. The third
contains a mixture of red and white balls. You are required to correctly label the boxes
with the labels red, white and red and white by picking a sample of one ball from only
one box. What is the label on the box you should sample?

1) White
2) Red
3) Red and White
4) Not possible to determine from a sample of one ball

Solution:
Let the red, white and red/white box be denoted by R, W and RW respectively.

Since each box is incorrectly named, the box labeled:

RW could actually be R or W

R could actually be W or RW

W could actually be R or RW.

Since the label RW is an incorrect label, the ball that comes out from that box will be
the actual colour of the balls in the box.

Hence, pick one ball from RW:

11 of 16 9/16/2011 10:17 AM
Exam Reports http://testfunda.com/LMS/Student/NewReports.aspx

Case 1:

i) If the ball is a red one, then re-label the box as R.

ii) Now, the box labelled W can be re-labelled only as RW.

iii) Finally, the box labelled R can be re-labelled as W.

Case 2:

i) If the ball is a white one, then re-label the box as W.

ii) Now, the box labeled R can be re-labelled only as RW.

iii) Finally, the box labeled W can be re-labelled only as R.

Thus, if you sample a ball from the box labelled red and white, you can correctly label
all the boxes.

Hence, option 3.

Group Question
Answer the following questions based on the information given below.

Ten coins are distributed among four people P, Q, R and S such that one of them gets one
coin, another gets two coins, the third gets three coins and the fourth gets four coins. It is
known that Q gets more coins than P, and S gets fewer coins than R.

19. If the number of coins distributed to Q is twice the number distributed to P then
3 Marks
which one of the following is necessarily true?

1) R gets an even number of coins.


2) R gets an odd number of coins.
3) S gets an even number of coins.
4) S gets an odd number of coins.

Solution:
Based on the given data, the number of coins with P, Q, R and S should be 1, 2,
3 and 4 coins (not necessarily in that order).

Also, it is given that: P < Q, S < R.

Thus,

(i) Q, R cannot have 1 coin.

(ii) P, S cannot have 4 coins.

Given: Q = 2P

12 of 16 9/16/2011 10:17 AM
Exam Reports http://testfunda.com/LMS/Student/NewReports.aspx

i) If Q = 2, then P = 1, this implies that S = 3 (since S ≠ 4) and R = 4

ii) If Q = 4, then P = 2, this implies that R = 3 (since R ≠ 1) and S = 1

From the 2 possible cases shown above, it can be seen that S always gets an
odd number of coins.

Hence, option 4.

20. If R gets at least two more coins than S, then which one of the following is
3 Marks
necessarily true?

1) Q gets at least two more coins than S.


2) Q gets more coins than P.
3) P gets more coins than S.
4) P and Q together get at least five coins.

Solution:
Given: R ≥ (S + 2)

i) If R = 4, and if S = 2, this implies that Q = 3 (since Q ≠ 1) and P = 1

ii) If R = 4, and if S = 1, this implies that Q = 3 (since Q > P) and P = 2

iii) If R = 3, and if S = 1, this implies that Q = 4 (since P ≠ 4) and P = 2

From the 3 possible cases shown above, it can be seen that the statement
‘Q gets more coins than P’ is always true.

Hence, option 2.

21. If Q gets fewer coins than R, then which one of the following is not necessarily
3 Marks
true?

1) P and Q together get at least four coins.


2) Q and S together get at least four coins.
3) R and S together get at least five coins.
4) P and R together get at least five coins.

Solution:
Given: Q < R → P < Q < R and S < R, thus R = 4

Possible cases (R = 4):

(i) P = 1, Q = 2, S = 3

13 of 16 9/16/2011 10:17 AM
Exam Reports http://testfunda.com/LMS/Student/NewReports.aspx

(ii) P = 1, Q = 3, S = 2

(iii) P = 2, Q = 3, S = 1

In all of the above cases, it can be seen that:

(Q + S) ≥ 4, (R + S) ≥ 5 and (P + R) ≥ 5

Thus, the only option that is not always true is (P + Q) ≥ 4 [Case (i)].

Hence, option 1.

Group Question
Answer the following questions based on the information given below.

A young girl Roopa leaves home with x flowers and goes to the bank of a nearby river. On
the bank of the river, there are four places of worship, standing in a row. She dips all the
'x' flowers in the river. The number of flowers doubles. Then she enters the first place of
worship, offers 'y' flowers to the deity. She dips the remaining flowers into the river, and
again the number of flowers doubles. She goes to the second place of worship, offers 'y'
flowers to the deity. She dips the remaining flowers into the river, and again the number of
flowers doubles. She goes to the third place of worship, offers 'y' flowers to the deity. She
dips the remaining flowers into the river, and again the number of flowers doubles. She
goes to the fourth place of worship, offers 'y' flowers to the deity. Now she is left with no
flowers in hand.

22. If Roopa leaves home with 30 flowers, the number of flowers she offers to each
3 Marks
deity is:

1) 30
2) 31
3) 32
4) 33

Solution:
Let Roopa leave home with x flowers and offer y flowers to each deity.

After each dip, the number of flowers she has doubles and after offering flowers
to each deity, the number of flowers decreases by y.

14 of 16 9/16/2011 10:17 AM
Exam Reports http://testfunda.com/LMS/Student/NewReports.aspx

Thus, from the table, it can be seen that 16x = 15y … (i)

Hence, option 3.

23. The minimum number of flowers that could be offered to each deity is:
3 Marks

1) 0
2) 15
3) 16
4) Indeterminate

Solution:
From the solution to the first question, y = (x/15) × 16

Thus, it can be seen that y is a multiple of 16.

Hence, the minimum value of y = 16 when x = 15.

Hence, option 3.

24. The minimum number of flowers with which Roopa leaves home is:
3 Marks

1) 16
2) 15
3) 0
4) Indeterminate

Solution:
From the solution to the first question, x = (y/16) × 15.

Thus, it can be seen that x is a multiple of 15.

Hence, the minimum value of x = 15 when y = 16.

Hence, option 2.

Group Question
Answer the following questions based on the information given below.

There are m blue vessels with known volumes v1, v2, ... vm, arranged in ascending order of
volume, where v1 > 0.5 litre, and vm < 1 litre. Each of these is full of water initially. The
water from each of these is emptied into a minimum number of empty white vessels, each

15 of 16 9/16/2011 10:17 AM
Exam Reports http://testfunda.com/LMS/Student/NewReports.aspx

having volume of 1 litre. The water from a blue vessel is not emptied into a white vessel
unless the white vessel has enough empty volume to hold all the water of the blue
vessel. The number of white vessels required to empty all the blue vessels according to the
above rules was n.

25. Among the four values given below, which is the least upper bound on e, where
3 Marks
e is the total empty volume in the n white vessels at the end of the above
process?

1) mvm
2) m(1 – vm )

3) mv1
4) m(1 – v1)

Solution:
The upper bound is the least when the volumes in all the blue vessels is nearly
equal, i.e., v1 ≈ v2 ≈ … ≈ vm

Then, the empty volume in each white vessel = e1 ≈ (1 – v1) litres

Total empty value = e = m × (1 – v1)

Hence, option 4.

26. Let the number of white vessels needed be n1 for the emptying process
3 Marks
described above, if the volume of each white vessel is 2 liters. Among the
following values, which is the least upper bound on n1?

1)

2)

3) n
4)

Solution:
Minimum number of blue vessels that can be emptied into each white vessel of
2 litres capacity = 2

Hence, option 2.

16 of 16 9/16/2011 10:17 AM
Exam Reports http://testfunda.com/LMS/Student/NewReports.aspx

Section I

Group Question
Answer the following questions based on the information given below.

There are 5 cities, A, B, C, D and E connected by 7 roads as shown in the figure


below:

Design a route such that you start from any city of your choice and walk on each
of the 7 roads once and only once, not necessarily returning to the city from which
you started.

1. For a route that satisfies the above restrictions, which of the following
3 Marks
statements is true?

1) There is no route that satisfies the above restriction.


2) A route can either start at C or end at C, but not both.
3) D can be only an intermediate city in the route.
4) The route has to necessarily end at E.

Solution:

The route has to be created in such a way that you pass through each
road exactly once and you do not need to end at the same city from

1 of 34 9/16/2011 10:15 AM
Exam Reports http://testfunda.com/LMS/Student/NewReports.aspx

where you started. Also, it is not mentioned that a particular city cannot
be visited more than once. This means that you can go through one city
more than once provided you do not go through the same road.

Keeping these conditions in mind, multiple routes such as


DEBDCABC, CABCDBED and so on can be formed.

The statement in option 1 is false since it is clear that multiple


routes satisfy the given conditions.

The statement in option 2 is true as no valid route can start and end at
C.

The statement in option 3 is false since the route DEBDCABC is a valid


route and starts at D. Thus, D need not only be an intermediate city.

The statement in option 4 is false since the route DEBDCABC is a valid


route and does not end at E.

Hence, option 2.

2. From how many different cities can one start such that the given
3 Marks
restriction is satisfied?

1) One
2) Zero
3) Three
4) Two

Solution:
To ensure that the given restriction is not violated, one can start either
from D or from C.

If one starts from either of A, B or E, the given restriction gets violated.

Thus, one can start only from 2 possible cities.

Hence, option 4.

3. Fifty college teachers are surveyed as to their possession of colour TVs,


3 Marks
VCRs and tape recorders. Of them, 22 own colour TVs, 15 own VCRs and 14
own tape recorders. Nine of these college teachers own exactly two items of
these three items; and, one college teacher owns all three. How many of the
50 teachers own none of the three, colour TV, VCR or tape recorder?

2 of 34 9/16/2011 10:15 AM
Exam Reports http://testfunda.com/LMS/Student/NewReports.aspx

1) 4
2) 9
3) 10
4) 11

Solution:
The given information can be represented as shown in the following Venn
diagram:

In the above figure, the various letters represent the folowing:

a : Teachers who own only TV

b : Teachers who own only VCR

c : Teachers who own only tape recorders (TR)

x : Teachers who own a TV and VCR but not TR

y : Teachers who own a TV and TR but not VCR

z : Teachers who own a TR and VCR but not TV

r : Teachers who own all three items

d : Teachers who do not own any of the three items

From the data given:

a + b + c + x + y + z + r + d = 50 ... (i)

a + x + y + r = 22 ... (ii)

b + x + z + r = 15 ... (iii)

3 of 34 9/16/2011 10:15 AM
Exam Reports http://testfunda.com/LMS/Student/NewReports.aspx

c + y + z + r = 14 ... (iv)

Add (ii), (iii) and (iv)

a + b + c + 2(x + y + z) + 3r = 51 ... (v)

Now,

x+y+z=9 ... (vi)

and, r = 1 ... (vii)

Substituting the values from (vi) and (vii) in (v),

a + b + c = 51 − 2(9) − 3(1)

= 51 − 18 − 3

= 30

Substituting these values in (i)

∴ 30 + 9 + 1 + d = 50

∴ d = 10

Thus, 10 teachers did not own even one of the three products.

Hence, option 3.

4. For the network given, what is the total number of ways in which one way can
3 Marks
reach B from A?

4 of 34 9/16/2011 10:15 AM
Exam Reports http://testfunda.com/LMS/Student/NewReports.aspx

1) 12
2) 16
3) 20
4) 22

Solution:
The number of ways in which one can reach B from A can be found by
counting as shown below.

1. A-c-g-l-B
2. A-c-h-l-B
3. A-c-h-k-B
4. A-c-g-k-B
5. A-d-g-l-B
6. A-d-h-l-B
7. A-d-h-k-B
8. A-d-g-k-B
9. A-e-i-k-B
10. A-e-i-l-B
11. A-e-j-l-B
12. A-e-j-k-B
13. A-f-i-k-B
14. A-f-i-l-B
15. A-f-j-l-B
16. A-f-j-k-B

Thus, one can reach B from A in 16 different ways

Hence, option 2.

Alternatively,

In the network shown above, each junction is known as a node.

5 of 34 9/16/2011 10:15 AM
Exam Reports http://testfunda.com/LMS/Student/NewReports.aspx

For such a network, the total number of ways in which one can reach from A
to B can be calculated as the product of the number of routes emerging from
any node at a particular level.

i.e. There are 4 posssible routes from the origin (A) to the next level (i.e. to c,
d, e or f).

Next, from each of c, d, e and f, there are 2 possible routes to the next level
(i.e. to g, h or i, j)

Similarly, from each of g, h, i and j, there are 2 routes to the next level ( i.e. to
k and l)

Finally, from both l and k, there is one route each available to go to the
destination (B).

∴ Total number of routes = 4 × 2 × 2 × 1 = 16

Hence, option 2.

Group Question
Answer the following questions based on the information given below.

There were a hundred schools in a town. Of these, the number of schools having
a play-ground was 30, and these schools had neither a library nor a laboratory.
The number of schools having a laboratory alone was twice the number
of schools having only a library. The number of schools having a laboratory as
well as a library was one-fourth the number of schools having a laboratory alone.
The number of schools having either a laboratory or a library or both was 35.

5. How many schools had none of the three, viz., laboratory, library or
3 Marks
play-ground?

1) 20
2) 5
3) 30
4) 35

Solution:
The town had 30 schools which had a playground. However, none of
these 30 schools had either a library or a laboratory.

Thus, there was no school that had both; a playground and a


laboratory or a playground and a library.

6 of 34 9/16/2011 10:15 AM
Exam Reports http://testfunda.com/LMS/Student/NewReports.aspx

Similarly, there was no school that had all three facilities.

The number of schools having a library and a laboratory was one-fourth


of the number of schools having only a laboratory.

Let the number of schools having only a laboratory be 4x.

Thus, the number of schools with a laboratory as well as a library is x.

The number of schools having only a laboratory was twice the number
of schools having only a library.

Therefore, the number of schools having only a library is 2x.

Thus, the given data can now be represented as shown in the following
Venn diagram:

In the above Venn diagram, PG, LI and LA corresponds to the number


of schools with a playground, library and laboratory respectively.

The number of schools having either a library or a laboratory or both


was 35.

∴ 2x + x + 4x = 35

∴ 7x = 35

∴x=5

Let the number of schools that did not have any of the three facilities i.e.
playground, library and laboratory be y.

∴ 30 + 35 + y = 100

7 of 34 9/16/2011 10:15 AM
Exam Reports http://testfunda.com/LMS/Student/NewReports.aspx

∴ y = 35

Thus, 35 schools did not have any of the three facilities.

Hence, option 4.

6. What was the ratio of schools having a laboratory to those having a


3 Marks
library?

1) 1:2
2) 5:3
3) 2:1
4) 2:3

Solution:
Number of schools having a laboratory = 4x + x = 5x

Number of schools having a library = 2x + x = 3x

Thus, the required ratio = 5x : 3x, i.e. 5 : 3

Hence, option 2.

7. There are 3 clubs, A, B and C, in a town with 40, 50 and 60 members


3 Marks
respectively. While 10 people are members of all 3 clubs, 70 people are
members in exactly one club. How many people belong to exactly two clubs?

1) 20
2) 25
3) 50
4) 70

Solution:
Let a, b, and c represent the number of people who are members of only club
A, only club B and only club C respectively.

∴ a + b + c = 70 ... (i)

Let the number of people who are members only in clubs A and B, but not in
C be x.

Similarly, let the number of people who are members in only B and C be y

8 of 34 9/16/2011 10:15 AM
Exam Reports http://testfunda.com/LMS/Student/NewReports.aspx

and the number of people who are members in only C and A be z.

Thus, the number of people who belong to exactly two clubs is x + y + z

Club A has 40 members.

∴ a + x + z + 10 = 40 ... (ii)

Using similar information for clubs B and C, you can get,

b + x + y + 10 = 50 ... (iii)

c + y + z + 10 = 60 ... (iv)

Adding (i), (ii) and (iii)

(a + b + c) + 2(x + y + z) + 30 = 150

∴ 70 + 2(x + y + z) = 120

∴ x + y + z = 25

Thus, 25 people belong to exactly two clubs.

Hence, option 2.

8. A man starting at a point walks one km east, then two kms north, then one
3 Marks
km east, then one km north, then one km east and then one km north to
arrive at the destination. What is the shortest distance from the starting point
to the destination?

1)

2) 7 km
3)

4) 5 km

Solution:
The shortest distance between the starting point and the destination can be
found by first finding the horizontal as well as the vertical distance travelled
between the starting point and the destination and then applying Pythagoras'
Theorem.

Since the man alternately walked towards the east and then towards the
north, the horizontal distance will simply be the summation of the total
distance travelled towards the east. Similarly, the vertical distance will be the

9 of 34 9/16/2011 10:15 AM
Exam Reports http://testfunda.com/LMS/Student/NewReports.aspx

summation of the total distance travelled towards the north.

Thus,

The total horizontal distance travelled by the man = 1 + 1 + 1 = 3 kms

The total vertical distance travelled by the man = 2 + 1 + 1 = 4 kms

Let A and B be the initial and final position of the man.

Hence, option 4.

9. If 8 + 12 = 2, 7 + 14 = 3 then 10 + 18 = ?
3 Marks

1) 10
2) 4
3) 6
4) 18

Solution:
The actual sum of 8 and 12 is 20

The sum of the digits in 20 = 2 + 0 = 2

Using a similar logic, 7 + 14 = 21

The sum of the digits in 21 = 2 + 1 = 3

Now, 10 + 18 = 28

The sum of digits of 28 = 2 + 8 = 10

∴ 10 + 18 = 10

Hence, option 1.

10. Lata has the same number of sisters as she has brothers, but her brother
3 Marks
Shyam has twice as many sisters as he has brothers. How many children are
there in the family?

1) 7
2) 6

10 of 34 9/16/2011 10:15 AM
Exam Reports http://testfunda.com/LMS/Student/NewReports.aspx

3) 5
4) 4

Solution:
Let the number of girls and boys in the family be G and B respectivley.

Lata has the same number of sisters as she has brothers. This implies that
the number of girls reduced by 1 is the same as the number of boys.

∴G−1=B … (i)

Lata’s brother Shyam has twice as many sisters as he has brothers. This
implies that the number of boys reduced by 1 is half the number of girls.

∴ G = 2(B – 1) … (ii)

Substituting (i) in (ii), you get

G = 2(G − 2)

∴G=4

∴B=3

G+B=4+3=7

Thus, there are 7 children in the family.

Hence, option 1.

11. A cube is made into a number of cubelets by dividing each edge into four
3 Marks
equal parts. The top face of the cube is a square ABCD. Take the diagonal
AC and push a knife downwards so that the cube is cut into two prisms of
equal size. How many cubelets are cut by the knife in this operation?

1) 16
2) 48
3) 4
4) 8

Solution:
When the cube is cut into four equal parts on each edge, the total number of
cublets formed is 4 × 4 × 4 = 64 (length × breadth × depth)

11 of 34 9/16/2011 10:15 AM
Exam Reports http://testfunda.com/LMS/Student/NewReports.aspx

The top face of the cube can be represented as given below. The numbers 1,
2, 3 and 4 represent the four cubelets whose top face forms the diagonal AC
of the square ABCD.

Consider cubelet 1. It is evident that there are 3 more cubelets exactly below
it (because the depth is 4).

Similarly, cubelets 2, 3 and 4 also have 3 cubelets each under them.

Thus, when the knife cuts downwards through cubelet 1, it actually cuts
cubelet 1 and the three cubelets exactly below it i.e. a total of 4 cubelets.

Similarly, it cuts 4 cubelets each when it cuts through cubelets 2, 3 and 4.

Thus, it cuts a total of 4 × 4 = 16 cubelets.

Hence, option 1.

12. If m # n = m + n + mn, then 2 # 3 = ?


3 Marks

1) 0
2) 6
3) 11
4) 20

Solution:
m # n = m + n + mn

∴ 2 # 3 = 2 + 3 + (2 × 3) = 11

Hence, option 3.

13. If m # n = m + n + mn and for any m, there is a number q such that m # q =


3 Marks
m, then q = ?

1) 0

12 of 34 9/16/2011 10:15 AM
Exam Reports http://testfunda.com/LMS/Student/NewReports.aspx

2) –1
3) 1
4) q does not exist

Solution:
m # n = m + n + mn

It is given that m # q = m

∴ m + q + mq = m

∴ q(1 + m) = 0

∴ q = 0 or 1 + m = 0

∴ q = 0 or m = −1

It is given that m can take any value.

∴ q= 0

Hence, option 1.

14. 100 Management students who read at least one of three available business
3 Marks
magazines are surveyed to observe the readership pattern. It is found that 80
students read Business India, 50 read Business World, and 30 read Business
Today. Five students read all the three magazines. How many read exactly
two magazines?

1) 50
2) 10
3) 95
4) 25

Solution:
Since all 100 students read at least one of the three available magazines,
there is no student who does not read any of the three magazines.

The general Venn diagram for this survey can be as shown below where BI,
BW and BT represent Business India, Business World and Business Today
respectively.

13 of 34 9/16/2011 10:15 AM
Exam Reports http://testfunda.com/LMS/Student/NewReports.aspx

From the given information, it can be noted that:

(a + b + c) + (x + y + z) + r = 100 ... (i)

a + x + y + r = 80 ... (ii)

b + x + z + r = 50 ... (iii)

c + y + z + r = 30 ... (iv)

r = 5 ... (v)

Adding (ii), (iii) and (iv), you get:

(a + b + c) + 2(x + y + z) + 3r = 160 … (vi)

∴ a + b + c + 2(x + y + z) = 145 ... (vii)

Subtracting (i) from (vii), and substiuting the value of r, you get

x + y + z = 50

Thus, the number of people who read exactly 2 magazines = x + y + z = 50

Hence, option 1.

15. A young girl counted the fingers of her left hand in the following manner. She
3 Marks
started calling the thumb 1, the index finger 2, middle finger 3, ring finger
4 and little finger 5; then reversed the direction, calling the ring finger 6,
middle finger 7, index finger 8, thumb 9 and then back to the index finger for
10, middle finger for 11, and so on. She counted up to 1994. She ended on
her..

1) Thumb
2) Index finger

14 of 34 9/16/2011 10:15 AM
Exam Reports http://testfunda.com/LMS/Student/NewReports.aspx

3) Middle finger
4) Ring finger

Solution:
Note that all five fingers were counted in the first iteration while only four
fingers were counted in every iteration after that.

The girl started with the thumb at 1 before reaching the thumb again at 9.

Similarly, she would reach the thumb again at 17, 25 and so on after every 8
counts.

Thus, the number assigned to the thumb would always be of the form 8n + 1.

Similarly, the number assigned to the index finger would be of the form 8n +
2, for the middle finger would be 8n + 3 and so on.

1994 = 8(249) + 2

Thus, 1994 is of the form 8n + 2

Hence, 1994 would represent the index finger.

Hence, she stopped at the index finger.

Hence, option 2.

16. Out of 100 families in a certain neighbourhood, 45 own radios, 75 own TVs
3 Marks
while 25 have VCRs. Only 10 families have all three appliances and each
VCR owner also has a TV. If 25 families have only a radio, how many families
own only a TV?

1) 30
2) 35
3) 40
4) 45

Solution:
Each VCR owner also has a TV. This implies that there is no family that owns
only a VCR.

Similarly, there is no family that owns only a VCR and radio, but not a TV.

Number of families that own a VCR = 25

15 of 34 9/16/2011 10:15 AM
Exam Reports http://testfunda.com/LMS/Student/NewReports.aspx

Number of families that own all three appliances = 10

Families that own a VCR = Families that own only a VCR + Families that own
only TV and VCR + Families that own only VCR and radio + Families that own
all three appliances.

∴ 25 = 0 + Number of families that own only TV and VCR + 0 + 10

∴ Number of families that own only TV and VCR = 15

Similarly, number of families that own only TV and radio = 45 − (25 + 10 + 0)


= 10

∴ Families with only TV = Families with TV − (Families with only TV and radio
+ Families with only TV and VCR + Families with all three appliances)

= 75 − (10 + 15 + 10)

= 40

Hence, option 3.

Group Question
Answer the following questions based on the information given below.

85 children went to an amusement park where they could ride on the merry-
go-round, roller coaster or Ferris wheel. It is known that 20 of them took all three
rides, and 55 of them took at least two of the three rides. Each ride costs Re. 1
and the total receipts of the amusement park were Rs. 145.

17. How many children did not try any of the rides?
3 Marks

1) 5
2) 10
3) 15
4) 20

Solution:
The number of children that took atleast 2 rides = 55

The number of children that took all three rides = 20

Therefore, the number of children that took exactly 2 rides = 55 – 20 =


35

16 of 34 9/16/2011 10:15 AM
Exam Reports http://testfunda.com/LMS/Student/NewReports.aspx

Each ride costs Re. 1 and the total collections were Rs. 145

Total collections = (No. of children who took exactly 1 ride × 1) + (No.


of children who took exactly 2 rides × 2) + (No. of children who took all
3 rides × 3)

If the number of children that took exactly 1 ride is n, then

145 = n + (35 × 2) + (20 × 3)

∴ n = 15

Thus, the number of children who took at least one ride = 15 + 35 + 20


= 70

∴ Number of children who did not take even a single ride = 85 − 70 =


15

Hence, option 3.

18. How many children took exactly one ride?


3 Marks

1) 5
2) 10
3) 15
4) 20

Solution:
From the solution to the previous question, the number of children that
took exactly 1 ride = 15

Hence, option 3.

Group Question
Answer the following questions based on the information given below.

“Kya-Kya” is an island in the South Pacific. The inhabitants of “Kya-Kya” always


answer any question with two sentences, one of which is always true and the
other is always false.

19. You are walking on a road and come to a fork. You ask the inhabitants
3 Marks
Ram, Laxman and Lila, “Which road will take me to the village?"
Ram says, “I never speak to strangers. I am new to these parts.”

17 of 34 9/16/2011 10:15 AM
Exam Reports http://testfunda.com/LMS/Student/NewReports.aspx

Laxman says, “I am married to Lila. Take the left road.”


Lila says, “I am married to Ram. He is not new to this place.”
Which of the following statements is true?

1) The left road takes you to the village


2) The right road takes you to the village
3) Lila is married to Laxman
4) None of these

Solution:
Since Ram is talking to you, he speaks to strangers. This implies that
his first statement is False.

Therefore, Ram's second statement is True i.e. he is new to these parts.

This makes Lila's second statement False and her first statement True
i.e. Lila is married to Ram.

Finally, this means that Laxman's first statement becomes False and
second becomes True i.e. “Take the left road” is a True statement.

Thus, the left road takes you to the village.

Hence, option 1.

20. You find that your boat is stolen. You question three inhabitants of the
3 Marks
island and they reply as follows:
John says, “I didn’t do it. Matthew didn’t do it.”
Matthew says, “I didn’t do it. Krishna didn’t do it.”
Krishna says, “I didn’t do it. I don’t know who did it.”
Who stole your boat?

1) John
2) Matthew
3) Krishna
4) None of them

Solution:
If the second statement of Matthew and first statement of Krishna is
true, then Matthew has stolen the boat which implies that his first
statement is false. So, the first statement of John is true and his second
statement is false.

18 of 34 9/16/2011 10:15 AM
Exam Reports http://testfunda.com/LMS/Student/NewReports.aspx

Hence, option 2.

21. You want to speak to the chief of the village. You question three
3 Marks
inhabitants, Amar, Bobby and Charles. Only Bobby is wearing a red
shirt.
Amar says, “I am not Bobby’s son. The chief wears a red shirt.”
Bobby says, “I am Amar’s father, Charles is the chief.”
Charles says, “The chief is one among us. I am the chief.”
Who is the chief?

1) Amar
2) Bobby
3) Charles
4) None of them

Solution:
Since it is mentioned in two places that “Charles is the chief”, start by
assuming that this statement is True.

Thus, if Charles is the chief, the chief is definitely one among Amar,
Bobby and Charles.

Thus, Charles' first statement also turns out to be True.

Thus, both the statements made by Charles become True, which is not
possible as per the condition of the problem.

Thus, Charles' second statement has to be definitely False.

This makes Bobby's second statement also False, and consequently,


Bobby's first statement True.

Thus, Bobby is Amar's father.

Therefore, Amar's first statement is False and second statement has to


be True.

Thus, the chief has to be a person wearing a red shirt.

Since Bobby is the only person wearing a red shirt, Bobby has to be the
chief.

Hence, option 2.

22. There is only one pilot on the island. You interview three men; Koik,

19 of 34 9/16/2011 10:15 AM
Exam Reports http://testfunda.com/LMS/Student/NewReports.aspx

3 Marks
Lony and Mirna. You also notice that Koik is wearing a cap.
Mirna says, “Lony’s father is the pilot. Lony is not the priest’s son.”
Koik says, “I am the priest. On this island, only priests can wear caps.”
Lony says, “I am the priest’s son. Koik is not the priest.”
Which of the following statements is true?

1) Lony is not Koik’s son


2) Koik is the pilot
3) Mirna is the pilot
4) Lony is the priest

Solution:
Consider Koik's statements.

If his second statement is true, then as he is wearing a cap, his first


statement also is true, which is not possible.

Therefore, Koik's second statement is false and hence he is the priest.

This makes Lony's second statement false and first statement true,
which in turn makes Mirna's second statement false and first statement
true.

Thus we can see that Lony's father, Koik is the priest as well as the
pilot.

Hence, option 2.

Group Question
Answer the following questions based on the information given below.

There are three different cable channels, namely Ahead, Luck and Bang. In a
survey it was found that 85% of the viewers respond to Bang, 20% to Luck, and
30% to Ahead. 20% of the viewers respond to exactly two channels and 5% to
none.

23. What percentage of the viewers responded to all three channels?


3 Marks

1) 10
2) 12
3) 14
4) None of these

20 of 34 9/16/2011 10:15 AM
Exam Reports http://testfunda.com/LMS/Student/NewReports.aspx

Solution:
A general Venn diagram can be drawn to represent the given data as
shown under:

Assume the total number of viewers to be 100.

Since 5% viewers do not respond to any channel, the number of viewers


who respond to at least one channel is 95.

∴ (a + b + c) + (x + y + z) + r = 95 ... (i)

The number of viewers who respond to Ahead, Bang and Luck is 30, 85
and 20 respectively.

∴ a + x + y + r = 30 ... (ii)

b + x + z + r = 85 ... (iii)

c + y + z + r = 20 ... (iv)

Adding (ii), (iii) and (iv),

(a + b + c) + 2(x + y + z) + 3r = 135 ... (v)

Now, there are 20 users who respond to exactly 2 channels.

∴ x + y + z = 20 ... (vi)

Substituting the value from (vi) in (i) and (v), you get

(a + b + c) + r = 75 ... (vii)

and, (a + b + c) + 3r = 95 ... (viii)

(viii) − (vii) gives, r = 10

21 of 34 9/16/2011 10:15 AM
Exam Reports http://testfunda.com/LMS/Student/NewReports.aspx

∴ a + b + c = 65

Thus, 10 out of 100 viewers responded to all three channels.

Thus, 10% of the viewers responded to all three channels.

Hence, option 1.

24. Assuming 20% respond to Ahead and Bang and 16% respond to Bang
3 Marks
and Luck, what is the percentage of viewers who watch only Luck?

1) 20
2) 10
3) 16
4) None of these

Solution:
Using the terminology and the values obtained in the solution to the
previous question,

x + r = 20

and

z + r = 16

Using the value of r = 10 in the two equations above, you get:

x = 10 and z = 6

x + y + z = 20

∴ y=4

The number of viewers who followed Luck = c + y + z + r = 20

∴ c + 4 + 6 + 10 = 20

∴c=0

Thus, no viewer follows only Luck.

Hence, option 4.

22 of 34 9/16/2011 10:15 AM
Exam Reports http://testfunda.com/LMS/Student/NewReports.aspx

Group Question
Answer the following questions based on the information given below.

In a locality, there are five small cities, A, B, C, D and E. The distances of these
cities from each other are as follows:

AB = 2 km AC = 2 km AD > 2 km AE > 3 km BC = 2 km

BD = 4 km BE = 3 km CD = 2 km CE = 3 km DE > 3 km

25. If a ration shop is to be set up within a maximum of 2 km of each city,


3 Marks
how many ration shops will be required?

1) 2
2) 3
3) 4
4) 5

Solution:
Given distances between the points:

AB = 2 km, AC = 2 km, AD > 2 km, AE > 3 km, BC = 2 km, BD = 4 km,


BE = 3 km, CD = 2 km, CE = 3 km, DE > 3 km.

Consider the above conditions:

(i) AB = AC = BC = 2 km. Hence it can be seen that A, B and C form an


equilateral triangle of side 2 km.

(ii) BD = 4 km, CD = 2 km. Hence it can be seen that point D lies on the
extended straight line connecting B and C and at a distance of 2 km
from C.

(iii) BE = CE = 3 km, AE > 3 km. Hence point E should lie on the


perpendicular bisector of the line BC. Point A lies on one side, hence E
should lie on the other side of the line BC.

Thus, the overall arrangement is as shown in the figure below:

23 of 34 9/16/2011 10:15 AM
Exam Reports http://testfunda.com/LMS/Student/NewReports.aspx

City C is at a distance of 2 km from A, B as well as D.

Thus, one ration shop at C can suffice for A, B, C as well as D.

However, city E is 3 km away from city C.

Thus, the ration shop at city C cannot serve city E.

Therefore, a separate ration shop is required for city E.

Thus, all the 5 cities can be properly served using just 2 ration shops.

Hence, option 1.

26. If a ration shop is to be set up within 3 km of each city, how many ration
3 Marks
shops will be required?

1) 1
2) 2
3) 3
4) 4

Solution:
Using the figure obtained in the solution to the previous question; if a
ration shop is required within 3 km of each city, then a single ration
shop at city C can serve all the cities under consideration.

Hence, option 1.

27. A cube of sides 12 cm is painted red on all the faces and then cut into
3 Marks
smaller cubes, each of sides 3 cm. What is the total number of smaller cubes

24 of 34 9/16/2011 10:15 AM
Exam Reports http://testfunda.com/LMS/Student/NewReports.aspx

having none of their faces painted?

1) 16
2) 8
3) 12
4) 24

Solution:
Number of smaller cubes on each side (n) = 12/3 = 4

Thus, the total number of smaller cubes formed = 43 = 64

If one wants the side of a cube to not be painted, one has to ignore one small
cube exactly in front of that side.

Thus, when one wants a small cube that has none of its sides painted, one
has to ignore one cube from each side. In other words, one has to ignore two
small cubes in each dimension.

Thus, the number of small cubes that have none of their sides painted = (n –
2)3 = 23 = 8

Hence, option 2.

28. In a locality, two-thirds of the people have cable-TV, one-fifth have VCRs, and
3 Marks
one-tenth have both. What is the fraction of people having either cable-TV or a
VCR?

1)

2)

3)

4)

Solution:
Fraction of people having either cable-TV or VCR = Fraction of people having
cable-TV + Fraction of people having VCR − Fraction of people having both

25 of 34 9/16/2011 10:15 AM
Exam Reports http://testfunda.com/LMS/Student/NewReports.aspx

Hence, option 4.

Group Question
Answer the following questions based on the information given below.

A survey of 200 people in a community who watched at least one of the three
channels - BBC, CNN, DD - showed that 80% of the people watched Doordarshan
(DD), 22% watched BBC, and 15% watched CNN.

29. What is the maximum percent of people who can watch all three
3 Marks
channels?

1) 12.5
2) 8.5
3) 15
4) Insufficient data

Solution:
All 200 people watched at least one channel. This implies that there
was no one who did not watch any of the three channels.

The general Venn diagram for the channels can be drawn as shown
below.

The percentage of people who can watch all three channels can be
maximum only if all the people watching one particular channel also
watch the other two channels.

Note that since only 15% people watch CNN, all of the 80% people who
watch DD cannot watch CNN.

26 of 34 9/16/2011 10:15 AM
Exam Reports http://testfunda.com/LMS/Student/NewReports.aspx

Thus, the percentage can be maximum if all the people who watch the
channel with the least viewership also watch the other two channels
that have a higher viewership.

Thus, the percentage can be maximum if the 15% people who watch
CNN also watch BBC as well as DD.

Thus, the maximum percentage of people who can wacth all three
channels is 15%.

Hence, option 3.

30. If 5% of the people watched DD and CNN, 10% watched DD and BBC,
3 Marks
then what percent of the people watched BBC and CNN only?

1) 2%
2) 5%
3) 8.5%
4) Indeterminate

Solution:
Consider the Venn diagram made in the previous solution as well as the
data given in the question.

80% people watched DD, 22% watched BBC and 15% watched CNN.

Thus, the actual number of people watching DD, BBC and CNN was
160, 44 and 30 respectively.

∴ d + x + y + r = 160 ... (i)

b + x + z + r = 44 ... (ii)

c + y + z + r = 30 ... (iii)

Now, 5% of the people watched DD and CNN while 10% watched DD


and BBC

Thus, 10 people watched DD and CNN while 20 people watched DD


and BBC.

∴ y + r = 10 ... (iv)

and

27 of 34 9/16/2011 10:15 AM
Exam Reports http://testfunda.com/LMS/Student/NewReports.aspx

x + r = 20 ... (v)

The number of people watching BBC and CNN only is z.

Now, every person watched at least one channel.

∴ (b + c + d) + (x + y + z) + r = 200 ... (vi)

Adding (i), (ii) and (iii), you get,

(b + c + d) + 2(x + y + z) + 3r = 234 ... (vii)

(vii) − (vi) gives

x + y + z + 2r = 34

∴ (x + r) + (y + r) + z = 34

Substituting the values from (iv) and (v) in the above equation, you get,

20 + 10 + z = 34

∴z=4

Thus, 4 people out of 200 watch only BBC and CNN.

Thus, 2% of the people watch only BBC and CNN.

Hence, option 1.

31. Referring to the previous question, how many percent of the people
3 Marks
watched all the three channels?

1) 3.5%
2) 0%
3) 8.5%
4) Indeterminate

Solution:
The percentage of people who watched all three channels can be found
if the value of r can be found.

With the given data, it is not possible to find r.

Hence, option 4.

28 of 34 9/16/2011 10:15 AM
Exam Reports http://testfunda.com/LMS/Student/NewReports.aspx

Group Question
Answer the following questions based on the information given below.

a # b = a + b, if a and b both are positive, else a # b = 1

a ∇ b = (ab)a + b, if ab is positive, else a ∇ b = 1

32.
3 Marks

1)

2) 1
3)

4) 3

Solution:
(i) a # b = a + b, if a, b > 0; else a # b = 1

(ii) a ∇ b = (ab)a + b, if ab > 0; else a ∇ b = 1

Therefore,

Hence, option 3.

33.
3 Marks

1)

2)

3)

4) None of these

29 of 34 9/16/2011 10:15 AM
Exam Reports http://testfunda.com/LMS/Student/NewReports.aspx

Solution:
log10 0.1 = log10 (10)−1 = −1

Thus, one of the parts of the second term of the numerator is negative
and so this second term becomes 1 (as per the alternate condition)

Thus,

Hence, option 1.

34.
3 Marks

1) X = 1, Y = 2
2) X > 0, Y < 0
3) Both X and Y are positive.
4) Both X and Y are negative.

Solution:
If X and Y lie on the same side of the number line, the numerator as
well as denominator both have one term that is negative.

Thus, the value of the numerator as well as the denominator becomes


1.

Thus, the value of the expression becomes 1 for the case where both X
and Y are positive as well as for the case where both X and Y are
negative.

Hence, options 3 and 4 do not yield the desired value, and


consequently, can be eliminated.

The values in option 1 are a specific case of option 3 and so, they also
yield a value of 1 for the expression.

Hence, option 1 can also be eliminated.

Consider option 2: X > 0, Y < 0

The numerator now has both terms positive and becomes X + Y

30 of 34 9/16/2011 10:15 AM
Exam Reports http://testfunda.com/LMS/Student/NewReports.aspx

On the other hand, the denominator has both terms negative and
becomes (XY)−(X + Y)

Thus, the value of the expression becomes (X + Y)/(XY)−(X + Y) i.e. (X +


Y) × (XY)(X + Y)

Among the options given, this is the only option that give the value of
the expression as 3/8.

Hence, option 2.

35. In a survey of political preference, 78% of those asked were in favor of at least
3 Marks
one of the proposals: I, II and III. 50% of those asked favored proposal I, 30%
favored proposal II, and 20% favored proposal III. If 5% of those asked
favored all three of the proposals, what percentage of those asked favored
more than one of the 3 proposals?

1) 10
2) 12
3) 17
4) 22

Solution:
The information given can be expressed as a general Venn Diagram as
shown below.

Let the number of respondents be 100.

Thus, 78 respondents favored at least one proposal.

∴ (a + b + c) + (x + y + z) + r = 78 ... (i)

31 of 34 9/16/2011 10:15 AM
Exam Reports http://testfunda.com/LMS/Student/NewReports.aspx

The number of people who favored proposals I, II, and III was respectively 50,
30 and 20.

∴ a + x + y + r = 50 ... (ii)

b + x + z + r = 30 ... (iii)

c + y + z + r = 20 ... (iv)

(ii) + (iii) + (iv) gives,

(a + b + c) + 2(x + y + z) + 3r = 100 ... (v)

(v) − (i) gives,

(x + y + z) + 2r = 22 ... (vi)

5 people favored all three proposals.

∴ r = 5 ... (vii)

Substitute the value of r in (vi)

∴ (x + y + z) = 12 ... (viii)

The number of people who favored more than one of the three proposals = (x
+ y + z) + r

= 12
+5

= 17

Thus, 17 out of 100 people favored more than one of the three proposals.

Hence, 17% people favored more than one of the three proposals.

Hence, option 3.

Group Question
Answer the following questions based on the information given below.

A robot moves on a graph sheet with x and y-axes. The robot is moved by feeding
it a sequence of instructions. The different instructions that can be used in moving
it are:

32 of 34 9/16/2011 10:15 AM
Exam Reports http://testfunda.com/LMS/Student/NewReports.aspx

36. The robot reaches point (6, 6) when a sequence of three instructions is
3 Marks
executed, the first of which is a GOTO (x, y) instructions, the second is
WALK X(2) and the third is WALK Y(4). What is the value of x and y
respectively?

1) 2, 4
2) 0, 0
3) 4, 2
4) 2, 2

Solution:
The final position of the robot is (6, 6) after the instruction WALK Y(4).

Therefore, the position of the robot just before this instruction and after
the instruction WALK X(2) is (6, 2).

Similarly, the position of the robot just before the instruction WALK X(2)
is (4, 2).

This is also the position of the robot after the GOTO (x, y) instruction

According to the conditions, if the robot receives a GOTO (x, y), it


directly reaches the position (x, y).

Since the robot is at (4, 2) after the GOTO instruction, it must have
received the instruction as GOTO (4, 2).

∴ x = 4 and y = 2

Hence, option 3.

37. The robot is initially at (x, y), x > 0 and y < 0. The minimum number of
3 Marks
instructions needed to be executed to bring it to the origin (0, 0) if you
are prohibited from using the GOTO instruction is:

33 of 34 9/16/2011 10:15 AM
Exam Reports http://testfunda.com/LMS/Student/NewReports.aspx

1) 2
2) 1
3) x+y
4) 0

Solution:
Since the x as well as y co-ordinate is not zero and GOTO is not
allowed, each co-ordinate has to be separately brought to zero.

This can be done using a total of two instructions (one WALK X and one
WALK Y).

The two instructions required are:

WALK X(−x)

WALK Y(+y)

Hence, option 1.

34 of 34 9/16/2011 10:15 AM
Exam Reports http://testfunda.com/LMS/Student/NewReports.aspx

Section I

Group Question
Answer the following questions based on the information given below.

There are three bottles of water, A, B, C, whose capacities are 5 litres, 3 litres, and
2 litres respectively. For transferring water from one bottle to another and to drain
out the bottles, there exists a piping system. The flow thorough these pipes is
computer controlled. The computer that controls the flow through these pipes can
be fed with three types of instructions, as explained below

Initially, A is full with water, and B and C are empty.

[CAT 2000]

1. After executing a sequence of three instructions, bottle A contains one


3 Marks
litre of water. The first and the third of these instructions are shown
below

First instruction FILL (C, A)

Third instruction FILL (C, A)

Then which of the following statements about the instructions is true?

1) The second instruction is FILL (B, A)


2) The second instruction is EMPTY (C, B)
3) The second instruction transfers water from B to C
4) The second instruction involves using the water in bottle A.

Solution:
Instruction 1: FILL (C, A)

1 of 36 9/16/2011 10:23 AM
Exam Reports http://testfunda.com/LMS/Student/NewReports.aspx

2 litres of water is transferred from A to C

∴ A has 3 litres of water left and C has 2 litres of water.

Instruction 3: FILL (C, A)

2 litres of water is transferred from A to C

∴ A has 1 litres of water left and C has 2 litres of water.

Since the third instruction again transfers water from A to C, C should


be empty before the third operation. Thus, instruction 2 should be either
“EMPTY (C, B)” or “DRAIN (C)”.

Hence, option 2.

2. Consider the same sequence of three instructions and the same initial
3 Marks
state mentioned above. Three more instructions are added at the end of
the above sequence to have A contain 4 litres of water. In this total
sequence of six instructions, the fourth one is DRAIN (A). This is the
only DRAIN instruction in the entire sequence. At the end of the
execution of the above sequence, how much water (in litres) is
contained in C?

1) One
2) Two
3) Zero
4) None of these

Solution:
After third instruction A has 1 litre, B has 2 litres and C has 2 litres of
water.

Instruction 4: DRAIN A

Instruction 5: EMPTY (B, A)

Instruction 6: EMPTY (C, A)

After 6th instruction, C has no water as all 2 litres have been transferred
to A.

Hence, option 3.

2 of 36 9/16/2011 10:23 AM
Exam Reports http://testfunda.com/LMS/Student/NewReports.aspx

3. There is a vertical stack of books marked 1, 2, and 3 on Table-A, with 1 at the


3 Marks
bottom and 3 on top. These are to be placed vertically on Table-B with 1 at
the bottom and 2 on the top, by making a series of moves from one table to
the other. During a move, the topmost book, or the topmost two books, or all
the three, can be moved from one of the tables to the other. If there are any
books on the other table, the stack being transferred should be placed on top
of the existing books, without changing the order of books in the stack that is
being moved in that move. If there are no books on the other table, the stack
is simply placed on the other table without disturbing the order of books in it.
What is the minimum number of moves in which the above task can be
accomplished?

[CAT 2000]

1) One
2) Two
3) Three
4) Four

Solution:
In four moves the task can be accomplished.
One of the ways is given below.

Initial state: Table A: 3(on top) 2(in middle) 1(bottom)

Step 1: Move book 3 from table A to table B

Step 2: Move book 2 from table A to table B

Step 3: Move books 2(on top) and 3(below book 2) from table B to table A on
book 1(bottommost)

Step 4: Move books 2, 3 and 1 from table A to table B.

Hence, option 4.

4. ABCDEFGH is a regular octagon. A and E are opposite vertices of the


3 Marks
octagon. A frog starts jumping from vertex to vertex, beginning from A. From
any vertex of the octagon except E, it may jump to either of the two adjacent
vertices. When it reaches E, the frog stops and stays there. Let an be the
number of distinct paths of exactly n jumps ending in E. Then what is the
value of a2n – 1?

[CAT 2000]

3 of 36 9/16/2011 10:23 AM
Exam Reports http://testfunda.com/LMS/Student/NewReports.aspx

1) Zero
2) Four
3) 2n – 1
4) Cannot be determined

Solution:
The frog has to jump at least four times to reach E.

i.e. a4 = 2

(i.e. (i)A-B, B-C,C-D and D-E; (ii) A-H, H-G, G-F and F-E).

If the frog keeps jumping on left (right) hand side vertices it will not take more
than four jumps.

If it jumps on right vertex and then keeps on jumping left vertices then it will
take 6 jumps to reach E.

∴ We will not find any path such that the frog takes 5 jumps and reaches E.

∴ a5 = 0

Similarly, we find that only for even values of n the frog can reach E.

2n − 1 is an odd number

∴ No route is possible with odd number of jumps.

Hence, option 1.

Group Question
Answer the following questions based on the information given below.

The following sketch shows the pipelines carrying material from one location to
another. Each location has a demand for material. The demand at Vaishali is 400,
at Jyotishmati is 400, at Panchal is 700, and at Vidisha is 200. Each arrow
indicates the direction of material flow through the pipeline. The flow from Vaishali
to Jyotishmati is 300. The quantity of material flow is such that the demands at all
these locations are exactly met. The capacity of each pipeline is 1000.

[CAT 2001]

4 of 36 9/16/2011 10:23 AM
Exam Reports http://testfunda.com/LMS/Student/NewReports.aspx

5. The quantity moved from Avanti to Vidisha is


3 Marks

1) 200
2) 800
3) 700
4) 1000

Solution:

Demand at Panchal = 700

∴ Quantity of material flowing in the pipeline from Jyotishmati to


Panchal = 700

Also, Flow from Vaishali to Jyotishmati + Flow from Vidisha to


Jyotishmati = Demand at Jyotishmati + Demand at Panchal

∴ 300 + Flow from Vidisha to Jyotishmati = 400 + 700 = 1100

∴ Flow from Vidisha to Jyotishmati = 1100 − 300 = 800

5 of 36 9/16/2011 10:23 AM
Exam Reports http://testfunda.com/LMS/Student/NewReports.aspx

Now, Flow from Avanti to Vaishali = Demand at Vaishali + Flow from


Vaishali to Jyotishmati

∴ Flow from Avanti to Vaishali = 300 + 400 = 700

Also, Flow from Avanti to Vidisha = Demand at Vidisha + Flow from


Vidisha to Jyotishmati

∴ Flow from Avanti to Vidisha = 200 + 800 = 1000

The diagram above shows the flow of the material in the pipelines and
the demand at the respective points.

∴ Quantity of material moved from Avanti to Vidisha = 1000

Hence, option 4.

6. The free capacity available at the Avanti-Vaishali pipeline is


3 Marks

1) 0
2) 100
3) 200
4) 300

Solution:
From the flow diagram, we have,

Each pipeline has a capacity of 1000 and the flow in the Avanti-Vaishali
pipeline is 700.

∴ Free capacity available = 1000 − 700 = 300

Hence, option 4.

7. What is the free capacity available in the Avanti-Vidisha pipeline?


3 Marks

1) 300
2) 200
3) 100
4) 0

6 of 36 9/16/2011 10:23 AM
Exam Reports http://testfunda.com/LMS/Student/NewReports.aspx

Solution:
From the flow diagram, the Avanti-Vidisha pipeline carries 1000.

∴ The Avanti-Vidisha pipeline has no free capacity.

Hence, option 4.

8. While Balbir had his back turned, a dog ran into his butcher shop, snatched a
3 Marks
piece of meat off the counter and ran out. Balbir was mad when he realised
what had happened. He asked three other shopkeepers, who had seen the
dog, to describe it. The shopkeepers really didn't want to help Balbir. So each
of them made a statement which contained one truth and one lie.

A. Shopkeeper Number 1 said: "The dog had black hair and a long tail."
B. Shopkeeper Number 2 said: "The dog has a short tail and wore a collar."
C. Shopkeeper Number 3 said: "The dog had white hair and no collar."

Based on the above statements, which of the following could be a correct


description?

[CAT 2001]

1) The dog had white hair, short tail and no collar.


2) The dog had white hair, long tail and a collar.
3) The dog had black hair, long tail and a collar.
4) The dog had black hair, long tail and no collar.

Solution:
According to the statement of the shopkeeper number 1, the dog had either
black hair or a long tail, but definitely not both.

∴ Options 3 and 4 can be eliminated.

According to the statement of shopkeeper number 3, the dog had either white
hair or wore a collar, but not both.

∴ Option 1 can be eliminated.

Hence, option 2.

9. Eight people carrying food baskets are going for a picnic on motorcycles.
3 Marks
Their names A, B, C, D, E, F, G, and H. They have four motorcycles, M1, M2,
M3 and M4 among them. They also have four food baskets O, P, Q and R of

7 of 36 9/16/2011 10:23 AM
Exam Reports http://testfunda.com/LMS/Student/NewReports.aspx

different sizes and shapes and each can be carried only on motorcycles M1,
M2, M3, or M4, respectively. No more than two persons can travel on a
motorcycle and no more than one basket can be carried on a motorcycle.
There are two husband-wife pairs in this group of eight people and each pair
will ride on a motorcycle together. C cannot travel with A or B. E cannot travel
with B or F. G cannot travel with F or H or D. The husband-wife pairs must
carry baskets O and P. Q is with A and P is with D. F travels on M1 and E
travels on M2 motorcycles. G is with Q, and B cannot go with R. Who is
travelling with H?

1) A
2) B
3) C
4) D

Solution:
As, O, P, Q, R are on motorcycle M1, M2, M3, M4 respectively. Hence we get,

Now, A is with Q and G is with Q hence A is with G on motorcycle M3.

And F is on M1 and E is on M2.

P is with D hence D is with E.

As B cannot go with R hence he has to go with F on motorcycle M1.

Hence C and H go together on motorcycle M4.

Hence we get,

Hence C goes with H.

8 of 36 9/16/2011 10:23 AM
Exam Reports http://testfunda.com/LMS/Student/NewReports.aspx

Hence, option 3.

10. In a family gathering there are two males who are grandfathers and four
3 Marks
males who are fathers. In the same gathering there are two females who are
grandmothers and four females who are mothers. There is at least one
grandson or a granddaughter present in this gathering. There are two
husband-wife pairs in this group. These can either be a grandfather and a
grandmother, or a father and a mother. The single grandfather (who wife is
not present) has two grandsons and a son present. The single grandmother
(whose husband is not present) has two granddaughters and a daughter
present. A grandfather or a grandmother present with their spouses does not
have any grandson or granddaughter present. What is the minimum number
of people present in this gathering?

[CAT 2001]

1) 10
2) 12
3) 14
4) 16

Solution:

Here "-" represents the Husband-Wife pair.

∵ We want the minimum number of people, let us assume that the two
grandfathers and two grandmothers have already been counted as fathers
and mothers respectively.

∴ We have 4 fathers, 4 mothers, 2 grandsons and 2 granddaughters.

∴ Minimum number of people present = 4 + 4 + 2 + 2 = 12

Hence, option 2.

Group Question

9 of 36 9/16/2011 10:23 AM
Exam Reports http://testfunda.com/LMS/Student/NewReports.aspx

Answer the following questions based on the information given below.

A and B are two sets (e.g. A = mothers, B = women). The elements that could
belong to both the sets (e.g. women who are mothers) is given by the set C = A.B.
The elements which could belong to either A or B, or both, is indicated by the set
D = A ∪ B. A set that does not contain any elements is known as a null set,
represented by ϕ (for example, if none of the women in the set B is a mother, they
C = A.B is a null set, or C = ϕ).

Let 'V' signify the set of all vertebrates; 'M' the set of all mammals; 'D' dogs; 'F'
fish; 'A' Alsatian and 'P', a dog named Pluto.

[CAT 2001]

11. Given that X = M.D is such that X = D, which of the following is true?
3 Marks

1) All dogs are mammals.


2) Some dogs are mammals.
3) X=ϕ
4) All mammals are dogs.

Solution:
X = M.D implies some dogs are mammals and X = D implies the set of
dogs.

This implies all dogs are mammals.

Hence, option 1.

12. If Y = F. (D.V), is not a null set, it implies that


3 Marks

1) All fish are vertebrates


2) All dogs are vertebrates
3) Some fish are dogs
4) None of the above

Solution:
Y = F.(D.V) implies some vertebrates are dogs and some vertebrate
dogs are fishes.

∴ Some fishes are dogs.

10 of 36 9/16/2011 10:23 AM
Exam Reports http://testfunda.com/LMS/Student/NewReports.aspx

Hence, option 3.

13. If Z = (P.D) ∪ M, then


3 Marks

1) The elements of Z consist of Pluto the dog or any other


mammal.
2) Z implies any dog or mammal.
3) Z implies Pluto or any dog that is a mammal.
4) Z is a null set.

Solution:
Z = (P.D) ∪ M

P.D means a dog called Pluto and P ∪ M means Pluto the dog and the
other mammals.

Hence, option 1.

14. If P.A = ϕ and P ∪ A = D, then which of the following is true?


3 Marks

1) Pluto and Alsatians are dogs.


2) Pluto is an Alsatian.
3) Pluto is not an Alsatian.
4) D is a null set.

Solution:
P. A = ϕ means that "Pluto is not an Alsatian".

Hence, option 3.

Group Question
Answer the following questions based on the information given below.

In a country, the following signals are applicable:

11 of 36 9/16/2011 10:23 AM
Exam Reports http://testfunda.com/LMS/Student/NewReports.aspx

A man headed towards north and follows the given signals as:

[CAT 2002]

15. What is the total distance covered by the man till the last signal?
3 Marks

1) 90 km
2) 120 km
3) 110 km
4) 84 km

Solution:
The movement can be tabulated as below:

= 10 + 10 + 20 + 40 + 10

12 of 36 9/16/2011 10:23 AM
Exam Reports http://testfunda.com/LMS/Student/NewReports.aspx

= 90 km

Hence, option 1.

16. What is his net displacement with respect to the starting point?
3 Marks

1) 40 km towards South West


2) 50 km towards North East
3) 40 km towards North East
4) 60 km towards South West

Solution:
The map of his movements is as shown below:

∴ The net displacement with respect to the starting point

Hence, option 2.

13 of 36 9/16/2011 10:23 AM
Exam Reports http://testfunda.com/LMS/Student/NewReports.aspx

17. If the first signal after the starting point, is 1 Red and 2 Green lights,
3 Marks
then what is the total distance covered by the man till the last signal?

1) 90 km
2) 50 km
3) 40 km
4) 80 km

Solution:
Since only the direction of the man has changed, the total distance
covered by him will still remain the same.

∴ Total distance = 90 km

Hence, option 1.

18. Instead of heading North, if the man was heading South, then by the
3 Marks
end of the journey, he was

1) 50 km towards South, 50 km towards West from his starting


point.
2) 50 km towards North, 50 km towards West from his starting
point.
3) 60 km towards North, 40 km towards West from his starting
point.
4) 40 km towards South, 30 km towards West from his starting
point.

Solution:
If instead of North, the man headed for South, the map given in the
solution of the second question in the set will get inverted and flipped.

∴ He will end up south-west of his starting position, instead of


north-east.

∴ He ends up at 30 km West and 40 km South of his starting point.

Hence, option 4.

19. In each of the following questions there are two blanks marked I & II. The
3 Marks words to fill in these blanks are given against I as (A, B, C, D) and II as (P, Q,
R, S). The right words to fill in these blanks are given as four alternatives. The

14 of 36 9/16/2011 10:23 AM
Exam Reports http://testfunda.com/LMS/Student/NewReports.aspx

words on either side of the sign (::) have a similar relationship. That alternative
which signifies this relationship is your answer.
For what reason Purohit did not get the offer of employment?

Statement:

I. Purohit passed the interview.


II. Purohit's friend passed the medical test who passed the interview along with
Purohit.
III. Purohit's father did not want him to take the job.
IV. Purohit has another employment offer from another company.
V. Purohit did not clear the mandatory medical test.

[CAT 2002]

1) III and IV only


2) III, IV and V only
3) I, III and IV only
4) V only

Solution:
Non-clearance of the mandatory medical test is the reason for not getting the
offer of employment.

Hence, option 4.

20. Each question given below is followed by five statements numbered I, II, III, IV
3 Marks and V. The answer choice given below each question consists of one or more
statements. You have to choose the choice which gives more relevant / useful
information in answering the question correctly. Read all the statements
together with the question and choose your answer
What were the possible reasons due to which DESCO incurred losses for the
last two years?

Statement:

I. The company's shares are not registered in the stock exchange.


II. The company does not export its products.
III. The company has an inefficient labour force.
IV. The price of its product has fallen in the last two years due to competitive market.
V. Entry of similar foreign goods at a cheaper rate.

[CAT 2002]

15 of 36 9/16/2011 10:23 AM
Exam Reports http://testfunda.com/LMS/Student/NewReports.aspx

1) Only III, IV and V


2) Only II, III and IV
3) Only IV and V
4) Only I and II

Solution:
Inefficient labour forces, fall in product price, and entry of similar foreign
goods at lower rates are the possible reasons for incurring losses by DESCO.

Hence, option 1.

21. Each question given below is followed by five statements numbered I, II, III, IV
3 Marks and V. The answer choice given below each question consists of one or more
statements. You have to choose the choice which gives more relevant / useful
information in answering the question correctly. Read all the statements
together with the question and choose your answer
On which day of the week did Sunil get his letter of promotion?

Statement:

I. Sunil purchased a new shirt on Friday


II. Sunil was given a party that Saturday.
III. Sunil was given the letter of promotion on the day before he purchased the shirt.
IV. Tuesday being his birthday, Sunil gave a party to all his friends.
V. Sunil's friend was promoted on Friday.

[CAT 2002]

1) I and II only
2) II, III and IV only
3) I and III only
4) II, III and V only

Solution:
Sunil purchased a new shirt on Friday and he got the letter of promotion one
day before.

∴ Sunil got his letter of promotion on Thursday.

Hence, option 3.

Group Question

16 of 36 9/16/2011 10:23 AM
Exam Reports http://testfunda.com/LMS/Student/NewReports.aspx

Answer the following questions based on the information given below.

New Age Consultants have three consultants Gyani, Medha and Buddhi. The sum
of the number of projects handled by Gyani and Buddhi individually is equal to
the number of projects in which Medha is involved. All three consultants are
involved together in 6 projects. Gyani works with Medha in 14 projects. Buddhi
has 2 projects with Medha but without Gyani and 3 projects with Gyani but
without Medha. The total number of projects for New Age Consultants is one less
than twice the number of projects in which more than one consultant is involved.

[CAT 2003 Leaked Test]

22. What is the number of projects in which Gyani alone is involved?


3 Marks

1) Uniquely equal to zero


2) Uniquely equal to 1
3) Uniquely equal to 4
4) Cannot be determined uniquely

Solution:

Let a be the number of projects in which only Gyani is involved, g be the


number of projects in which only Buddhi is involved and c be the
number of projects in which only Medha is involved.

From the data, d = 6

b + d = 14

∴b=8

Also, e = 3 and f = 2

It is given that

17 of 36 9/16/2011 10:23 AM
Exam Reports http://testfunda.com/LMS/Student/NewReports.aspx

a+g=b+c+d+f

∴ a − c + g = 16 … (i)

Number of projects involving more than 1 consultant = 6 + 8 + 2 + 3 =


19

∴ Total number of projects = 2 × 19 − 1 = 37

a + b + c + d + e + f + g = 2 × (b + d + e + f) − 1

∴ a + c + g = 19 − 1 = 18 … (ii)

Solving (i) and (ii), we get,

c = 1 and a + g = 17

∴ a cannot be determined uniquely.

Hence, option 4.

23. What is the number of projects in which Medha alone is involved?


3 Marks

1) Uniquely equal to zero


2) Uniquely equal to 1
3) Uniquely equal to 4
4) Cannot be determined uniquely

Solution:
From the solution to the previous question, we get, c = 1

∴ Number of projects in which Medha alone is involved = 1

Hence, option 2.

Group Question
Answer the following questions based on the information given below.

Shown below is a layout of major streets in a city.

Two days (Thursday and Friday) are left for campaigning before a major election,
and the city administration has received requests from five political parties for
taking out their processions along the following routes.

18 of 36 9/16/2011 10:23 AM
Exam Reports http://testfunda.com/LMS/Student/NewReports.aspx

Congress: A-C-D-E
BJP: A-B-D-E
SP: A-B-C-E
BSP: B-C-E
CPM: A-C-D

Street B-D cannot be used for a political procession on Thursday due to a


religious procession. The district administration has a policy of not allowing more
than one procession to pass along the same street on the same day. However, the
administration must allow all parties to take out their procession during these two
days.

[CAT 2003 Re-Test]

24. Congress procession can be allowed:


3 Marks

1) Only on Thursday.
2) Only on Friday.
3) On either day.
4) Only if the religious procession is cancelled.

Solution:

∵ Route BD cannot be used on Thursday.

∴ BJP has to take out its procession on Friday.

∵ Route DE is common to BJP and Congress.

∴ Congress should take out its procession on Thursday.

19 of 36 9/16/2011 10:23 AM
Exam Reports http://testfunda.com/LMS/Student/NewReports.aspx

∵ Route AB is common to SP and BJP.

∴ SP should take out its procession on Thursday.

∴ BSP and CPM have to take out their processions on Friday.

∴ Congress procession can be allowed only on Thursday.

Hence, option 1.

25. Which of the following is not true?


3 Marks

1) Congress and SP can take out their processions on the same


day.
2) The CPM procession cannot be allowed on Thursday.
3) The BJP procession can only take place on Friday.
4) Congress and BSP can take out their processions on the
same day.

Solution:
From the explanation given in the solution of the first question we get
that,

Congress takes out a procession on Thursday and BSP on Friday.

∴ Only option 4 is not true.

Hence, option 4.

26. 70 percent of the employees in a multinational corporation have VCD players,


3 Marks
75 percent have microwave ovens, 80 percent have ACs and 85 percent have
washing machines. At least what percentage of employees has all four
gadgets?

[CAT 2003 Re-Test]

1) 15%
2) 5%
3) 10%
4) Cannot be determined

20 of 36 9/16/2011 10:23 AM
Exam Reports http://testfunda.com/LMS/Student/NewReports.aspx

Solution:
70% have VCD Players.

∴ 30% do not have VCD Players.

75% have microwave ovens.

∴ 25% do not have microwave ovens.

80% have ACs.

∴ 20% do not have ACs.

85% have washing machines.

∴ 15% do not have washing machines.

∴ 30 + 25 + 15 + 20 = 90% of employees do not have at least 1 gadget.

∴ Minimum percentage of employees who has all the four gadgets

= 100 – 90 = 10%

Hence, option 3.

Alternatively,

Minimum percentage of employees which possess both VCDs and


Microwaves

= 70% + 75% – 100%

= 45%

Minimum percentage of employees which possess both ACs and Washing


machines

= 80% + 85% – 100%

= 65%

∴ Minimum percentage of employees which possess all the four gadgets

= 45% + 65% – 100%

= 10%

21 of 36 9/16/2011 10:23 AM
Exam Reports http://testfunda.com/LMS/Student/NewReports.aspx

Hence, option 3.

Group Question
Answer the following questions based on the information given below.

The seven basic symbols in a certain numeral system and their respective values
are as follows:

I = 1, V = 5, X = 10, L = 50, C = 100, D = 500, and M = 1000

In general, the symbols in the numeral system are read from left to right, starting
with the symbol representing the largest value; the same symbol cannot occur
continuously more than three times; the value of the numeral is the sum of the
values of the symbols.

For example, XXVII = 10 + 10 + 5 + 1 + 1 = 27.

An exception to the left-to-right reading occurs when a symbol is followed


immediately by a symbol of greater value; then, the smaller value is subtracted
from the larger.

For example, XLVI = (50 – 10) + 5 + 1 = 46.

[CAT 2003 Re-Test]

27. The value of the numeral MDCCLXXXVII is:


3 Marks

1) 1687
2) 1787
3) 1887
4) 1987

Solution:
The expression MDCCLXXXVII is expanded as,

1000 + 500 + 100 + 100 + 50 + 10 + 10 + 10 + 5 + 1 + 1

= 1787

Hence, option 2.

28. The value of the numeral MCMXCIX is


3 Marks

1) 1999

22 of 36 9/16/2011 10:23 AM
Exam Reports http://testfunda.com/LMS/Student/NewReports.aspx

2) 1899
3) 1989
4) 1889

Solution:
The expression MCMXCIX is expanded as,

1000 + (1000 – 100) + (100 – 10) + (10 – 1)

= 1000 + 900 + 90 + 9

= 1999

Hence, option 1.

29. Which of the following can represent the numeral for 1995?
3 Marks

a. MCMLXXV
b. MCMXCV
c. MVD
d. MVM

1) Only (a) and (b)


2) Only (c) and (d)
3) Only (b) and (d)
4) Only (d)

Solution:
Option (a): MCMLXXV

= 1000 + (1000 – 100) + 50 + 10 + 10 + 5

= 1000 + 900 + 50 + 25 = 1975

Option (b): MCMXCV

= 1000 + (1000 – 100) + (100 – 10) + 5

= 1000 + 900 + 90 + 5 = 1995

Option (c): MVD

23 of 36 9/16/2011 10:23 AM
Exam Reports http://testfunda.com/LMS/Student/NewReports.aspx

= 1000 + (500 – 5) = 1000 + 495 = 1495

Option (d): MVM

= 1000 + (1000 – 5) = 1000 + 995 = 1995

Options (b) and (d) give 1995.

Hence, option 3.

Group Question
Answer the following questions based on the information given below.

A study was conducted to ascertain the relative importance that employees in five
different countries assigned to five different traits in their Chief Executive Officers.
The traits were compassion (C), decisiveness (D), negotiation skills (N), public
visibility (P), and vision (V). The level of dissimilarity between two countries is the
maximum difference in the ranks allotted by the two countries to any of the five
traits. The following table indicates the rank order of the five traits for each
country.

[CAT 2004]

30. Three of the following four pairs of countries have identical levels of
3 Marks
dissimilarity. Which pair is the odd one out?

1) Malaysia and China


2) China and Thailand
3) Thailand and Japan
4) Japan and Malaysia

Solution:
Let us consider the options:

1. Malaysia and China: The maximum level of dissimilarity is 4 for ‘V’ or

24 of 36 9/16/2011 10:23 AM
Exam Reports http://testfunda.com/LMS/Student/NewReports.aspx

‘N’.

2. China and Thailand: The maximum level of dissimilarity is 4 for ‘V’.

3. Thailand and Japan: The maximum level of dissimilarity is 4 for ‘D’.

4. Japan and Malaysia: The maximum level of dissimilarity is 3 for ‘V’ or


‘N’.

Hence, option 4.

31. Which amongst the following countries is most dissimilar to India?


3 Marks

1) China
2) Japan
3) Malaysia
4) Thailand

Solution:

The table gives levels of dissimilarity between India and the other
countries for all traits.

Clearly, Japan is most dissimilar to India (since its level of dissimilarity is


4).

Hence, option 2.

32. Which of the following countries is least dissimilar to India?


3 Marks

1) China
2) Japan
3) Malaysia
4) Thailand

Solution:

25 of 36 9/16/2011 10:23 AM
Exam Reports http://testfunda.com/LMS/Student/NewReports.aspx

From the above table, China is least dissimilar to India (since its level of
dissimilarity is only 2).

Hence, option 1.

33. Which of the following pairs of countries are most dissimilar?


3 Marks

1) China and Japan


2) India and China
3) Malaysia and Japan
4) Thailand and Japan

Solution:
Consider the levels of dissimilarities for the four options.

China and Japan ≡ 3 (for 'D')

India and China ≡ 2 (for 'N')

Malaysia and Japan ≡ 3 (for 'N' or 'V')

Thailand and Japan ≡ 4 (for 'D')

Hence, option 4.

Group Question
Answer the following questions based on the information given below.

Help Distress (HD) is an NGO involved in providing assistance to people suffering


from natural disasters. Currently, it has 37 volunteers. They are involved in three
projects: Tsunami Relief (TR) in Tamil Nadu, FloodRelief (FR) in Maharashtra, and
Earthquake Relief (ER) in Gujarat. Each volunteer working with Help Distress has
to be involved in at least one relief work project.

A Maximum number of volunteers are involved in the FR project. Among them, the
number of volunteers involved in FR project alone is equal to the volunteers having
additional involvement in the ER project.
The number of volunteers involved in the ER project alone is double the number of

26 of 36 9/16/2011 10:23 AM
Exam Reports http://testfunda.com/LMS/Student/NewReports.aspx

volunteers involved in all the three projects.


17 volunteers are involved in the TR project.
The number of volunteers involved in the TR project alone is one less than the number of
volunteers involved in ER Project alone.
Ten volunteers involved in the TR project are also involved in at least one more project.

[CAT 2005]

34. Based on the information given above, the minimum number of


3 Marks
volunteers involved in both FR and TR projects, but not in the ER
project is:

1) 1
2) 3
3) 4
4) 5

Solution:

17 volunteers are involved in the TR project and 10 in TR are also


involved in other projects. Thus, 7 volunteers are involved only in TR.

∴ 8 volunteers are involved in ER alone.

∴ 4 volunteers are involved in all the three projects.

Let x people be involved in FR alone.

∴ Number of people involved in FR and ER but not TR = x – 4

Now, a + b + 4 = 10

27 of 36 9/16/2011 10:23 AM
Exam Reports http://testfunda.com/LMS/Student/NewReports.aspx

∴a+b=6

Also, 7 + a + b + 4 + x + x – 4 + 8 = 37

∴ 2x = 16 or x = 8

Number of Volunteers involved in FR > Number of Volunteers involved in


TR

And Number of Volunteers involved in FR > Number of Volunteers


involved in ER

∴ 16 + a > 17 and 16 + a > 16 + b or a > b

∴ a and b can be (6, 0), (5, 1), (4, 2)

The minimum number of volunteers involved in both FR and TR


projects, but not in the ER Project = minimum value of a = 4

Hence, option 3.

35. Which of the following additional information would enable to find the
3 Marks
exact number of volunteers involved in various projects?

1) Twenty volunteers are involved in FR.


2) Four volunteers are involved in all the three projects.
3) Twenty three volunteers are involved in exactly one project.
4) No need for any additional information.

Solution:
We can obtain the information in options 2 and 3 from the initial data.

Based on the information given in the explanation to the first question,


the information in option 1 will give us the value of a, which in turn will
give us the value of b. Thus, option 1 would enable us to find the exact
number of volunteers involved in various projects.

Hence, option 1.

36. After some time, the volunteers who were involved in all the three
3 Marks
projects were asked to withdraw from one project. As a result, one of
the volunteers opted out of the TR project, and one opted out of the ER
project, while the remaining ones involved in all the three projects opted
out of the FR project. Which of the following statements, then,

28 of 36 9/16/2011 10:23 AM
Exam Reports http://testfunda.com/LMS/Student/NewReports.aspx

necessarily follows?

1) The lowest number of volunteers is now in TR project.


2) More volunteers are now in FR project as compared to ER
project.
3) More volunteers are now in TR project as compared to ER
project.
4) None of the above

Solution:

After the volunteers withdraw as mentioned, the number of volunteers


working on different projects is as shown.

∴ Number of volunteers working on TR = 7 + 6 + 3 = 16

Number of volunteers working on FR = 14 + a

Number of volunteers working on ER = 15 + b

Considering the possible values of a and b, 14 + a > 15 + b

∴ More volunteers are now in FR than in ER

Hence, option 2.

37. After the withdrawal of volunteers, as indicated in Question 89, some


3 Marks
new volunteers joined the NGO. Each one of them was allotted only
one project in a manner such that, the number of volunteers working in

29 of 36 9/16/2011 10:23 AM
Exam Reports http://testfunda.com/LMS/Student/NewReports.aspx

one project alone for each of the three projects became identical. At that
point, it was also found that the number of volunteers involved in FR
and ER projects was the same as the number of volunteers involved in
TR and ER projects. Which of the projects now has the highest number
of volunteers?

1) ER
2) FR
3) TR
4) Cannot be determined

Solution:
Let m volunteers be added to TR project and n be added to each of FR
and ER projects.

Then, 7 + m = 8 + n

∴m=n+1

Also, b + 2 = 5

∴ b = 3 and a = 3

Number of volunteers working on TR = 7 + n + 1 + 4 + 5 = 17 + n

Number of volunteers working on FR = 17 + n

Number of volunteers working on ER = 18 + n

Thus, ER has the highest number of volunteers.

Hence, option 1.

38. Three Englishmen and three Frenchmen work for the same company. Each of
3 Marks
them knows a secret not known to others. They need to exchange these
secrets over person-to-person phone calls so that eventually each person
knows all six secrets. None of the Frenchmen knows English, and only one
Englishman knows French. What is the minimum number of phone calls
needed for the above purpose?

[CAT 2005]

1) 5

30 of 36 9/16/2011 10:23 AM
Exam Reports http://testfunda.com/LMS/Student/NewReports.aspx

2) 10
3) 9
4) 15

Solution:
Let E1, E2 and E3 be the three Englishmen and F1, F2 and F3 be the three
Frenchmen.

Let E3 be the only Englishman knowing French.

Now, Let A ↔ B denote a phone call between A and B, where they both tell
each other their secrets.The following phone calls will ensure that all six
persons know all the six secrets.

1. E1 ↔ E2

2. E2 ↔ E3 (Now E3 knows all the secrets with the Englishmen)

3. F1 ↔ F 2

4. F2 ↔ F 3 (Now F3 knows all the secrets with the Frenchmen)

5. F3 ↔ E3 (Now F3 and E3 know all the secrets)

6. E3 ↔ E2

7. E2 ↔ E1

8. F3 ↔ F 2

9. F2 ↔ F 1

Thus, a minimum of 9 calls are needed to pass all the secrets to all the six
persons.

Hence, option 3.

Group Question
Answer the following questions based on the information given below.

A significant amount of traffic flows from point S to point T in the one-way street
network shown below. Points A, B, C, and D are junctions in the network, and the
arrows mark the direction of traffic flow. The fuel cost in rupees for travelling along
a street is indicated by the number adjacent to the arrow representing the street.

31 of 36 9/16/2011 10:23 AM
Exam Reports http://testfunda.com/LMS/Student/NewReports.aspx

Motorists travelling from point S to point T would obviously take the route for
which the total cost of travelling is the minimum. If two or more routes have the
same least travel cost, then motorists are indifferent between them. Hence, the
traffic gets evenly distributed among all the least cost routes.

The government can control the flow of traffic only by levying appropriate toll at
each junction. For example, if a motorist takes the route S-A-T (using junction A
alone), then the total cost of travel would be Rs. 14 (i.e. Rs. 9 + Rs. 5) plus the
toll charged at junction A.

[CAT 2006]

39. If the government wants to ensure that all motorists travelling from S to
3 Marks
T pay the same amount (fuel costs and toll combined) regardless of the
route they choose and the street from B to C is under repairs (and
hence unusable), then a feasible set of toll charged (in rupees) at
junctions A, B, C, and D respectively to achieve this goal is:

1) 2, 5, 3, 2
2) 0, 5, 3, 1
3) 1, 5, 3, 2
4) 2, 3, 5, 1
5) 1, 3, 5, 1

Solution:
Let the toll charged at junctions A, B, C and D be a, b, c and d
respectively.

Since the cost of travel including toll on routes S-A-T, S-D-T, S-B-A-T
and S-D-C-T is the same,

∴ 14 + a = 13 + d = 9 + a + b = 10 + c + d

Thus, b = 5, d – a = 1, c = 3

32 of 36 9/16/2011 10:23 AM
Exam Reports http://testfunda.com/LMS/Student/NewReports.aspx

If a = 0, d = 1, If a = 1, d = 2 and if a = 2, d = 3

Hence, both options 2 and 3 satisfy the given criteria.

Note: The question makers took care of this inconsistency while


calculating scores.

40. If the government wants to ensure that no traffic flows on the street from
3 Marks
D to T, while equal amount of traffic flows through junctions A and C,
then a feasible set of toll charged (in rupees) at junctions A, B, C, and D
respectively to achieve this goal is:

1) 1, 5, 3, 3
2) 1, 4, 4, 3
3) 1, 5, 4, 2
4) 0, 5, 2, 3
5) 0, 5, 2, 2

Solution:
Since the cost of travel including toll on routes S-A-T, S-B-C-T, S-B-A-T
and S-D-C-T is the same,

∴ 14 + a = 7 + b + c = 9 + a + b = 10 + c + d

∴ b = 5, d = 2, c – a = 2

Only option 5 satisfies these criteria.

Hence, option 5.

41. If the government wants to ensure that all routes from S to T get the
3 Marks
same amount of traffic, then a feasible set of toll charged (in rupees) at
junctions A, B, C, and D respectively to achieve this goal is:

1) 0, 5, 2, 2
2) 0, 5, 4, 1
3) 1, 5, 3, 3
4) 1, 5, 3, 2
5) 1, 5, 4, 2

33 of 36 9/16/2011 10:23 AM
Exam Reports http://testfunda.com/LMS/Student/NewReports.aspx

Solution:
Since the cost of travel including toll on all routes is the same.

∴ 14 + a = 7 + b + c = 13 + d = 9 + a + b = 10 + c + d

∴ b = 5, d = 2, c = 3 and a = 1

Hence, option 4.

42. If the government wants to ensure that the traffic at S gets evenly
3 Marks
distributed along streets from S to A, from S to B, and from S to D, then
a feasible set of toll charged (in rupees) at junctions A, B, C, and D
respectively to achieve this goal is:

1) 0, 5, 4, 1
2) 0, 5, 2, 2
3) 1, 5, 3, 3
4) 1, 5, 3, 2
5) 0, 4, 3, 2

Solution:
If we make the cost of travelling on all the routes equal, traffic along S-B
will be twice that along S-A.

But we want traffic along S-A, S-B and S-D to be the same.

As routes lead to C from both B and D, we can increase the toll at C so


that the cost of travelling along S-B-C-T and S-D-C-T is more than that
along the other three routes.

Now, 14 + a = 9 + b = 13 + d

∴ a = 0, b = 5 and d =1

Also, 7 + b + c > 14 and 10 + d + c > 14

∴c>3

Hence, option 1.

43. The government wants to devise a toll policy such that the total cost to
3 Marks
the commuters per trip is minimized. The policy should also ensure that
not more than 70 per cent of the total traffic passes through junction B.

34 of 36 9/16/2011 10:23 AM
Exam Reports http://testfunda.com/LMS/Student/NewReports.aspx

The cost incurred by the commuter travelling from point S to point T


under this policy will be:

1) Rs. 7
2) Rs. 9
3) Rs. 10
4) Rs. 13
5) Rs. 14

Solution:
If toll charges at all junctions are made 0, 100% traffic will pass through
S-B-C-T. This is not possible.

If toll charges at A and B are made 0, then 100% traffic will pass
through S-B-A-T. This is also not possible.

If toll charges at C and D are made 0, that at B are made Rs. 3, then
the traffic will get equally divided between S-D-C-T and S-B-C-T.

Thus, the cost incurred will be Rs. 10.

Hence, option 3.

44. A survey was conducted of 100 people to find out whether they had read
3 Marks
recent issues of Golmal, a monthly magazine. The summarized information
regarding readership in 3 months is given below:

Only September: 18; September but not August: 23; September and July:
8; September: 28; July: 48; July and August: 10; None of the three months:
24.

What is the number of surveyed people who have read exactly two
consecutive issues (out of the three)?

[CAT 2006]

1) 7
2) 9
3) 12
4) 14
5) 17

35 of 36 9/16/2011 10:23 AM
Exam Reports http://testfunda.com/LMS/Student/NewReports.aspx

Solution:

100 – 24 = 76 had read at least one issue.

If x people read all the three issues, then (8 – x) people read only the
September and July issues.

23 people read the September issue but not the August issue.

∴ 18 + 8 – x = 23

∴x=3

As 28 people read the September issue, [28 – (8 – 3) – 3 – 18] = 2 people


read only the August and September issues.

As 10 people read the July and August issues, 10 – 3 = 7 people read only
the July and August issues.

∴ The number of people who have read exactly two consecutive issues = 7 +
2=9

Hence, option 2.

36 of 36 9/16/2011 10:23 AM
http://www.time4education.com/onlinecattests/forprint.asp?ar=LDI&tn=1

1 of 6 1/4/2010 12:26 AM
http://www.time4education.com/onlinecattests/forprint.asp?ar=LDI&tn=1

2 of 6 1/4/2010 12:26 AM
http://www.time4education.com/onlinecattests/forprint.asp?ar=LDI&tn=1

3 of 6 1/4/2010 12:26 AM
http://www.time4education.com/onlinecattests/forprint.asp?ar=LDI&tn=1

4 of 6 1/4/2010 12:26 AM
http://www.time4education.com/onlinecattests/forprint.asp?ar=LDI&tn=1

5 of 6 1/4/2010 12:26 AM
http://www.time4education.com/onlinecattests/forprint.asp?ar=LDI&tn=1

6 of 6 1/4/2010 12:26 AM
http://www.time4education.com/onlinecattests/forprint.asp?ar=LDI&tn=10

1 of 6 1/4/2010 12:34 AM
http://www.time4education.com/onlinecattests/forprint.asp?ar=LDI&tn=10

2 of 6 1/4/2010 12:34 AM
http://www.time4education.com/onlinecattests/forprint.asp?ar=LDI&tn=10

3 of 6 1/4/2010 12:34 AM
http://www.time4education.com/onlinecattests/forprint.asp?ar=LDI&tn=10

4 of 6 1/4/2010 12:34 AM
http://www.time4education.com/onlinecattests/forprint.asp?ar=LDI&tn=10

5 of 6 1/4/2010 12:34 AM
http://www.time4education.com/onlinecattests/forprint.asp?ar=LDI&tn=10

6 of 6 1/4/2010 12:34 AM
http://www.time4education.com/onlinecattests/forprint.asp?ar=LDI&tn=11

1 of 6 1/4/2010 12:35 AM
http://www.time4education.com/onlinecattests/forprint.asp?ar=LDI&tn=11

2 of 6 1/4/2010 12:35 AM
http://www.time4education.com/onlinecattests/forprint.asp?ar=LDI&tn=11

3 of 6 1/4/2010 12:35 AM
http://www.time4education.com/onlinecattests/forprint.asp?ar=LDI&tn=11

4 of 6 1/4/2010 12:35 AM
http://www.time4education.com/onlinecattests/forprint.asp?ar=LDI&tn=11

5 of 6 1/4/2010 12:35 AM
http://www.time4education.com/onlinecattests/forprint.asp?ar=LDI&tn=11

6 of 6 1/4/2010 12:35 AM
http://www.time4education.com/onlinecattests/forprint.asp?ar=LDI&tn=12

1 of 5 1/4/2010 12:36 AM
http://www.time4education.com/onlinecattests/forprint.asp?ar=LDI&tn=12

2 of 5 1/4/2010 12:36 AM
http://www.time4education.com/onlinecattests/forprint.asp?ar=LDI&tn=12

3 of 5 1/4/2010 12:36 AM
http://www.time4education.com/onlinecattests/forprint.asp?ar=LDI&tn=12

4 of 5 1/4/2010 12:36 AM
http://www.time4education.com/onlinecattests/forprint.asp?ar=LDI&tn=12

5 of 5 1/4/2010 12:36 AM
http://www.time4education.com/onlinecattests/forprint.asp?ar=LDI&tn=13

1 of 5 1/4/2010 12:36 AM
http://www.time4education.com/onlinecattests/forprint.asp?ar=LDI&tn=13

2 of 5 1/4/2010 12:36 AM
http://www.time4education.com/onlinecattests/forprint.asp?ar=LDI&tn=13

3 of 5 1/4/2010 12:36 AM
http://www.time4education.com/onlinecattests/forprint.asp?ar=LDI&tn=13

4 of 5 1/4/2010 12:36 AM
http://www.time4education.com/onlinecattests/forprint.asp?ar=LDI&tn=13

5 of 5 1/4/2010 12:36 AM
http://www.time4education.com/onlinecattests/forprint.asp?ar=LDI&tn=14

1 of 6 1/4/2010 12:37 AM
http://www.time4education.com/onlinecattests/forprint.asp?ar=LDI&tn=14

2 of 6 1/4/2010 12:37 AM
http://www.time4education.com/onlinecattests/forprint.asp?ar=LDI&tn=14

3 of 6 1/4/2010 12:37 AM
http://www.time4education.com/onlinecattests/forprint.asp?ar=LDI&tn=14

4 of 6 1/4/2010 12:37 AM
http://www.time4education.com/onlinecattests/forprint.asp?ar=LDI&tn=14

5 of 6 1/4/2010 12:37 AM
http://www.time4education.com/onlinecattests/forprint.asp?ar=LDI&tn=14

6 of 6 1/4/2010 12:37 AM
http://www.time4education.com/onlinecattests/forprint.asp?ar=LDI&tn=15

1 of 5 1/4/2010 12:39 AM
http://www.time4education.com/onlinecattests/forprint.asp?ar=LDI&tn=15

2 of 5 1/4/2010 12:39 AM
http://www.time4education.com/onlinecattests/forprint.asp?ar=LDI&tn=15

3 of 5 1/4/2010 12:39 AM
http://www.time4education.com/onlinecattests/forprint.asp?ar=LDI&tn=15

4 of 5 1/4/2010 12:39 AM
http://www.time4education.com/onlinecattests/forprint.asp?ar=LDI&tn=15

5 of 5 1/4/2010 12:39 AM
http://www.time4education.com/onlinecattests/forprint.asp?ar=LDI&tn=16

1 of 5 1/4/2010 12:40 AM
http://www.time4education.com/onlinecattests/forprint.asp?ar=LDI&tn=16

2 of 5 1/4/2010 12:40 AM
http://www.time4education.com/onlinecattests/forprint.asp?ar=LDI&tn=16

3 of 5 1/4/2010 12:40 AM
http://www.time4education.com/onlinecattests/forprint.asp?ar=LDI&tn=16

4 of 5 1/4/2010 12:40 AM
http://www.time4education.com/onlinecattests/forprint.asp?ar=LDI&tn=16

5 of 5 1/4/2010 12:40 AM
http://www.time4education.com/onlinecattests/forprint.asp?ar=LDI&tn=17

1 of 6 1/4/2010 12:42 AM
http://www.time4education.com/onlinecattests/forprint.asp?ar=LDI&tn=17

2 of 6 1/4/2010 12:42 AM
http://www.time4education.com/onlinecattests/forprint.asp?ar=LDI&tn=17

3 of 6 1/4/2010 12:42 AM
http://www.time4education.com/onlinecattests/forprint.asp?ar=LDI&tn=17

4 of 6 1/4/2010 12:42 AM
http://www.time4education.com/onlinecattests/forprint.asp?ar=LDI&tn=17

5 of 6 1/4/2010 12:42 AM
http://www.time4education.com/onlinecattests/forprint.asp?ar=LDI&tn=17

6 of 6 1/4/2010 12:42 AM
http://www.time4education.com/onlinecattests/forprint.asp?ar=LDI&tn=18

1 of 6 1/4/2010 12:44 AM
http://www.time4education.com/onlinecattests/forprint.asp?ar=LDI&tn=18

2 of 6 1/4/2010 12:44 AM
http://www.time4education.com/onlinecattests/forprint.asp?ar=LDI&tn=18

3 of 6 1/4/2010 12:44 AM
http://www.time4education.com/onlinecattests/forprint.asp?ar=LDI&tn=18

4 of 6 1/4/2010 12:44 AM
http://www.time4education.com/onlinecattests/forprint.asp?ar=LDI&tn=18

5 of 6 1/4/2010 12:44 AM
http://www.time4education.com/onlinecattests/forprint.asp?ar=LDI&tn=18

6 of 6 1/4/2010 12:44 AM
http://www.time4education.com/onlinecattests/forprint.asp?ar=LDI&tn=19

1 of 6 1/4/2010 12:45 AM
http://www.time4education.com/onlinecattests/forprint.asp?ar=LDI&tn=19

2 of 6 1/4/2010 12:45 AM
http://www.time4education.com/onlinecattests/forprint.asp?ar=LDI&tn=19

3 of 6 1/4/2010 12:45 AM
http://www.time4education.com/onlinecattests/forprint.asp?ar=LDI&tn=19

4 of 6 1/4/2010 12:45 AM
http://www.time4education.com/onlinecattests/forprint.asp?ar=LDI&tn=19

5 of 6 1/4/2010 12:45 AM
http://www.time4education.com/onlinecattests/forprint.asp?ar=LDI&tn=19

6 of 6 1/4/2010 12:45 AM
http://www.time4education.com/onlinecattests/forprint.asp?ar=LDI&tn=2

1 of 6 1/4/2010 12:27 AM
http://www.time4education.com/onlinecattests/forprint.asp?ar=LDI&tn=2

2 of 6 1/4/2010 12:27 AM
http://www.time4education.com/onlinecattests/forprint.asp?ar=LDI&tn=2

3 of 6 1/4/2010 12:27 AM
http://www.time4education.com/onlinecattests/forprint.asp?ar=LDI&tn=2

4 of 6 1/4/2010 12:27 AM
http://www.time4education.com/onlinecattests/forprint.asp?ar=LDI&tn=2

5 of 6 1/4/2010 12:27 AM
http://www.time4education.com/onlinecattests/forprint.asp?ar=LDI&tn=2

6 of 6 1/4/2010 12:27 AM
http://www.time4education.com/onlinecattests/forprint.asp?ar=LDI&tn=20

1 of 5 1/4/2010 12:46 AM
http://www.time4education.com/onlinecattests/forprint.asp?ar=LDI&tn=20

2 of 5 1/4/2010 12:46 AM
http://www.time4education.com/onlinecattests/forprint.asp?ar=LDI&tn=20

3 of 5 1/4/2010 12:46 AM
http://www.time4education.com/onlinecattests/forprint.asp?ar=LDI&tn=20

4 of 5 1/4/2010 12:46 AM
http://www.time4education.com/onlinecattests/forprint.asp?ar=LDI&tn=20

5 of 5 1/4/2010 12:46 AM
http://www.time4education.com/onlinecattests/forprint.asp?ar=LDI&tn=21

1 of 6 1/4/2010 12:47 AM
http://www.time4education.com/onlinecattests/forprint.asp?ar=LDI&tn=21

2 of 6 1/4/2010 12:47 AM
http://www.time4education.com/onlinecattests/forprint.asp?ar=LDI&tn=21

3 of 6 1/4/2010 12:47 AM
http://www.time4education.com/onlinecattests/forprint.asp?ar=LDI&tn=21

4 of 6 1/4/2010 12:47 AM
http://www.time4education.com/onlinecattests/forprint.asp?ar=LDI&tn=21

5 of 6 1/4/2010 12:47 AM
http://www.time4education.com/onlinecattests/forprint.asp?ar=LDI&tn=21

6 of 6 1/4/2010 12:47 AM
http://www.time4education.com/onlinecattests/forprint.asp?ar=LDI&tn=22

1 of 6 1/4/2010 12:49 AM
http://www.time4education.com/onlinecattests/forprint.asp?ar=LDI&tn=22

2 of 6 1/4/2010 12:49 AM
http://www.time4education.com/onlinecattests/forprint.asp?ar=LDI&tn=22

3 of 6 1/4/2010 12:49 AM
http://www.time4education.com/onlinecattests/forprint.asp?ar=LDI&tn=22

4 of 6 1/4/2010 12:49 AM
http://www.time4education.com/onlinecattests/forprint.asp?ar=LDI&tn=22

5 of 6 1/4/2010 12:49 AM
http://www.time4education.com/onlinecattests/forprint.asp?ar=LDI&tn=22

6 of 6 1/4/2010 12:49 AM
http://www.time4education.com/onlinecattests/forprint.asp?ar=LDI&tn=23

1 of 6 1/4/2010 12:51 AM
http://www.time4education.com/onlinecattests/forprint.asp?ar=LDI&tn=23

2 of 6 1/4/2010 12:51 AM
http://www.time4education.com/onlinecattests/forprint.asp?ar=LDI&tn=23

3 of 6 1/4/2010 12:51 AM
http://www.time4education.com/onlinecattests/forprint.asp?ar=LDI&tn=23

4 of 6 1/4/2010 12:51 AM
http://www.time4education.com/onlinecattests/forprint.asp?ar=LDI&tn=23

5 of 6 1/4/2010 12:51 AM
http://www.time4education.com/onlinecattests/forprint.asp?ar=LDI&tn=23

6 of 6 1/4/2010 12:51 AM
http://www.time4education.com/onlinecattests/forprint.asp?ar=LDI&tn=24

1 of 5 1/4/2010 12:52 AM
http://www.time4education.com/onlinecattests/forprint.asp?ar=LDI&tn=24

2 of 5 1/4/2010 12:52 AM
http://www.time4education.com/onlinecattests/forprint.asp?ar=LDI&tn=24

3 of 5 1/4/2010 12:52 AM
http://www.time4education.com/onlinecattests/forprint.asp?ar=LDI&tn=24

4 of 5 1/4/2010 12:52 AM
http://www.time4education.com/onlinecattests/forprint.asp?ar=LDI&tn=24

5 of 5 1/4/2010 12:52 AM
http://www.time4education.com/onlinecattests/forprint.asp?ar=LDI&tn=25

1 of 6 1/4/2010 12:53 AM
http://www.time4education.com/onlinecattests/forprint.asp?ar=LDI&tn=25

2 of 6 1/4/2010 12:53 AM
http://www.time4education.com/onlinecattests/forprint.asp?ar=LDI&tn=25

3 of 6 1/4/2010 12:53 AM
http://www.time4education.com/onlinecattests/forprint.asp?ar=LDI&tn=25

4 of 6 1/4/2010 12:53 AM
http://www.time4education.com/onlinecattests/forprint.asp?ar=LDI&tn=25

5 of 6 1/4/2010 12:53 AM
http://www.time4education.com/onlinecattests/forprint.asp?ar=LDI&tn=25

6 of 6 1/4/2010 12:53 AM
http://www.time4education.com/onlinecattests/forprint.asp?ar=LDI&tn=26

1 of 6 1/4/2010 12:54 AM
http://www.time4education.com/onlinecattests/forprint.asp?ar=LDI&tn=26

2 of 6 1/4/2010 12:54 AM
http://www.time4education.com/onlinecattests/forprint.asp?ar=LDI&tn=26

3 of 6 1/4/2010 12:54 AM
http://www.time4education.com/onlinecattests/forprint.asp?ar=LDI&tn=26

4 of 6 1/4/2010 12:54 AM
http://www.time4education.com/onlinecattests/forprint.asp?ar=LDI&tn=26

5 of 6 1/4/2010 12:54 AM
http://www.time4education.com/onlinecattests/forprint.asp?ar=LDI&tn=26

6 of 6 1/4/2010 12:54 AM
http://www.time4education.com/onlinecattests/forprint.asp?ar=LDI&tn=27

1 of 6 1/4/2010 12:56 AM
http://www.time4education.com/onlinecattests/forprint.asp?ar=LDI&tn=27

2 of 6 1/4/2010 12:56 AM
http://www.time4education.com/onlinecattests/forprint.asp?ar=LDI&tn=27

3 of 6 1/4/2010 12:56 AM
http://www.time4education.com/onlinecattests/forprint.asp?ar=LDI&tn=27

4 of 6 1/4/2010 12:56 AM
http://www.time4education.com/onlinecattests/forprint.asp?ar=LDI&tn=27

5 of 6 1/4/2010 12:56 AM
http://www.time4education.com/onlinecattests/forprint.asp?ar=LDI&tn=27

6 of 6 1/4/2010 12:56 AM
http://www.time4education.com/onlinecattests/forprint.asp?ar=LDI&tn=28

1 of 6 1/4/2010 12:57 AM
http://www.time4education.com/onlinecattests/forprint.asp?ar=LDI&tn=28

2 of 6 1/4/2010 12:57 AM
http://www.time4education.com/onlinecattests/forprint.asp?ar=LDI&tn=28

3 of 6 1/4/2010 12:57 AM
http://www.time4education.com/onlinecattests/forprint.asp?ar=LDI&tn=28

4 of 6 1/4/2010 12:57 AM
http://www.time4education.com/onlinecattests/forprint.asp?ar=LDI&tn=28

5 of 6 1/4/2010 12:57 AM
http://www.time4education.com/onlinecattests/forprint.asp?ar=LDI&tn=28

6 of 6 1/4/2010 12:57 AM
http://www.time4education.com/onlinecattests/forprint.asp?ar=LDI&tn=29

1 of 7 1/4/2010 12:59 AM
http://www.time4education.com/onlinecattests/forprint.asp?ar=LDI&tn=29

2 of 7 1/4/2010 12:59 AM
http://www.time4education.com/onlinecattests/forprint.asp?ar=LDI&tn=29

3 of 7 1/4/2010 12:59 AM
http://www.time4education.com/onlinecattests/forprint.asp?ar=LDI&tn=29

4 of 7 1/4/2010 12:59 AM
http://www.time4education.com/onlinecattests/forprint.asp?ar=LDI&tn=29

5 of 7 1/4/2010 12:59 AM
http://www.time4education.com/onlinecattests/forprint.asp?ar=LDI&tn=29

6 of 7 1/4/2010 12:59 AM
http://www.time4education.com/onlinecattests/forprint.asp?ar=LDI&tn=29

7 of 7 1/4/2010 12:59 AM
http://www.time4education.com/onlinecattests/forprint.asp?ar=LDI&tn=3

1 of 6 1/4/2010 12:28 AM
http://www.time4education.com/onlinecattests/forprint.asp?ar=LDI&tn=3

2 of 6 1/4/2010 12:28 AM
http://www.time4education.com/onlinecattests/forprint.asp?ar=LDI&tn=3

3 of 6 1/4/2010 12:28 AM
http://www.time4education.com/onlinecattests/forprint.asp?ar=LDI&tn=3

4 of 6 1/4/2010 12:28 AM
http://www.time4education.com/onlinecattests/forprint.asp?ar=LDI&tn=3

5 of 6 1/4/2010 12:28 AM
http://www.time4education.com/onlinecattests/forprint.asp?ar=LDI&tn=3

6 of 6 1/4/2010 12:28 AM
http://www.time4education.com/onlinecattests/forprint.asp?ar=LDI&tn=30

1 of 6 1/4/2010 1:00 AM
http://www.time4education.com/onlinecattests/forprint.asp?ar=LDI&tn=30

2 of 6 1/4/2010 1:00 AM
http://www.time4education.com/onlinecattests/forprint.asp?ar=LDI&tn=30

3 of 6 1/4/2010 1:00 AM
http://www.time4education.com/onlinecattests/forprint.asp?ar=LDI&tn=30

4 of 6 1/4/2010 1:00 AM
http://www.time4education.com/onlinecattests/forprint.asp?ar=LDI&tn=30

5 of 6 1/4/2010 1:00 AM
http://www.time4education.com/onlinecattests/forprint.asp?ar=LDI&tn=30

6 of 6 1/4/2010 1:00 AM
http://www.time4education.com/onlinecattests/forprint.asp?ar=LDI&tn=4

1 of 6 1/4/2010 12:29 AM
http://www.time4education.com/onlinecattests/forprint.asp?ar=LDI&tn=4

2 of 6 1/4/2010 12:29 AM
http://www.time4education.com/onlinecattests/forprint.asp?ar=LDI&tn=4

3 of 6 1/4/2010 12:29 AM
http://www.time4education.com/onlinecattests/forprint.asp?ar=LDI&tn=4

4 of 6 1/4/2010 12:29 AM
http://www.time4education.com/onlinecattests/forprint.asp?ar=LDI&tn=4

5 of 6 1/4/2010 12:29 AM
http://www.time4education.com/onlinecattests/forprint.asp?ar=LDI&tn=4

6 of 6 1/4/2010 12:29 AM
http://www.time4education.com/onlinecattests/forprint.asp?ar=LDI&tn=5

1 of 4 1/4/2010 12:30 AM
http://www.time4education.com/onlinecattests/forprint.asp?ar=LDI&tn=5

2 of 4 1/4/2010 12:30 AM
http://www.time4education.com/onlinecattests/forprint.asp?ar=LDI&tn=5

3 of 4 1/4/2010 12:30 AM
http://www.time4education.com/onlinecattests/forprint.asp?ar=LDI&tn=5

4 of 4 1/4/2010 12:30 AM
http://www.time4education.com/onlinecattests/forprint.asp?ar=LDI&tn=6

1 of 5 1/4/2010 12:30 AM
http://www.time4education.com/onlinecattests/forprint.asp?ar=LDI&tn=6

2 of 5 1/4/2010 12:30 AM
http://www.time4education.com/onlinecattests/forprint.asp?ar=LDI&tn=6

3 of 5 1/4/2010 12:30 AM
http://www.time4education.com/onlinecattests/forprint.asp?ar=LDI&tn=6

4 of 5 1/4/2010 12:30 AM
http://www.time4education.com/onlinecattests/forprint.asp?ar=LDI&tn=6

5 of 5 1/4/2010 12:30 AM
http://www.time4education.com/onlinecattests/forprint.asp?ar=LDI&tn=7

1 of 6 1/4/2010 12:31 AM
http://www.time4education.com/onlinecattests/forprint.asp?ar=LDI&tn=7

2 of 6 1/4/2010 12:31 AM
http://www.time4education.com/onlinecattests/forprint.asp?ar=LDI&tn=7

3 of 6 1/4/2010 12:31 AM
http://www.time4education.com/onlinecattests/forprint.asp?ar=LDI&tn=7

4 of 6 1/4/2010 12:31 AM
http://www.time4education.com/onlinecattests/forprint.asp?ar=LDI&tn=7

5 of 6 1/4/2010 12:31 AM
http://www.time4education.com/onlinecattests/forprint.asp?ar=LDI&tn=7

6 of 6 1/4/2010 12:31 AM
http://www.time4education.com/onlinecattests/forprint.asp?ar=LDI&tn=8

1 of 5 1/4/2010 12:32 AM
http://www.time4education.com/onlinecattests/forprint.asp?ar=LDI&tn=8

2 of 5 1/4/2010 12:32 AM
http://www.time4education.com/onlinecattests/forprint.asp?ar=LDI&tn=8

3 of 5 1/4/2010 12:32 AM
http://www.time4education.com/onlinecattests/forprint.asp?ar=LDI&tn=8

4 of 5 1/4/2010 12:32 AM
http://www.time4education.com/onlinecattests/forprint.asp?ar=LDI&tn=8

5 of 5 1/4/2010 12:32 AM
http://www.time4education.com/onlinecattests/forprint.asp?ar=LDI&tn=9

1 of 6 1/4/2010 12:33 AM
http://www.time4education.com/onlinecattests/forprint.asp?ar=LDI&tn=9

2 of 6 1/4/2010 12:33 AM
http://www.time4education.com/onlinecattests/forprint.asp?ar=LDI&tn=9

3 of 6 1/4/2010 12:33 AM
http://www.time4education.com/onlinecattests/forprint.asp?ar=LDI&tn=9

4 of 6 1/4/2010 12:33 AM
http://www.time4education.com/onlinecattests/forprint.asp?ar=LDI&tn=9

5 of 6 1/4/2010 12:33 AM
http://www.time4education.com/onlinecattests/forprint.asp?ar=LDI&tn=9

6 of 6 1/4/2010 12:33 AM
PAST CAT PAPER – 2002
Instructions:
1. The Test Paper contains 150 questions. The duration of the test is 120 minutes.
2. The paper is divided into three sections. Section-I: 50 Q:, Section-II: 50 Q:, Section-III: 50 Q.
3. Wrong answers carry negative marks. There is only one correct answer for each question.

Section – 1
Directions for questions 1 to 6: Answer the questions independently.
Four students — Ashish, Dhanraj, Felix and Sameer sat for the Common Entrance Exam for Management
(CEEM). One student got admission offers from three NIMs (National Institutes of Management), another
from two NIMs, the third from one NIM, while the fourth got none. Below are some of the facts about who
got admission offers from how many NIMs and what is their educational background.
I. The one who is an engineer didn’t get as many admissions as Ashish.
II. The one who got offer for admissions in two NIMs isn’t Dhanraj nor is he a chartered accountant.
III. Sameer is an economist.
IV. Dhanraj isn’t an engineer and received more admission offers than Ashish.
V. The doctor got the most number of admission offers.

1. Which one of the following statements is necessarily true?


1. Ashish is a chartered accountant and got offer for admission in three NIMs.
2. Dhanraj is a doctor and got admission offer in one NIM.
3. Sameer is an economist who got admission offers in two NIMs.
4. Felix who is not an engineer did not get any offer for admission.

2. Five boys went to a store to buy sweets. One boy had Rs. 40. Another boy had Rs. 30. Two other
boys had Rs. 20 each. The remaining boy had Rs. 10. Below are some more facts about the initial
and final cash positions.
I. Alam started with more than Jugraj.
II. Sandeep spent Rs. 1.50 more than Daljeet.
III. Ganesh started with more money than just only one other person.
2
IV. Daljeet started with of what Sandeep started with.
3
V. Alam spent the most, but did not end with the least.
VI. Jugraj spent the least and ended with more than Alam or Daljeet.
VII. Ganesh spent Rs.3.50.
VIII. Alam spent 10 times more than what Ganesh did.

In the choices given below, all statements except one are false. Which one of the following statements
can be true?
1. Alam started with Rs. 40 and ended with Rs. 9.50
2. Sandeep started with Rs. 30 and ended with Re. 1
3. Ganesh started with Rs. 20 and ended with Rs. 4
4. Jugraj started with Rs. 10 and ended with Rs. 7

PAST CAT PAPER – 2002 Page 1


3. In a hospital there were 200 diabetes, 150 hyperglycaemia and 150 gastro-enteritis patients.
Of these, 80 patients were treated for both diabetices and hyperglycaemia. Sixty patients were
treated for gastro-enteritis and hyperglycaemia, while 70 were treated for diabetes and gastro-
enteritis. Some of these patients have all the three diseases. Dr. Dennis treats patients with only
gastro-enteritis. Dr. Paul is a generalist. Therefore, he can treat patients with multiple diseases.
Patients always prefer a specialist for their disease. If Dr. Dennis had 80 patients, then the other
three doctors can be arranged in terms of the number of patients treated as:
1. Paul > Gerard > Hormis 2. Paul > Hormis > Gerard
3. Gerard > Paul > Hormis 4. None of these

4. Three children won the prizes in the Bournvita Quiz contest. They are from the schools: Loyola,
Convent and Little Flowers, which are located at different cities. Below are some of the facts about
the schools, the children and the city they are from.
I. One of the children is Bipin.
II. Loyola School’s contestant did not come first.
III. Little Flower’s contestant was named Riaz.
IV. Convent School is not in Hyderabad.
V. The contestant from Pune is not from Loyola School.
VI. The contestant from Bangalore did not come first.
VII. Convent School’s contestant’s name is not Balbir.

Which of the following statements is true?


1. 1st prize: Riaz (Little Flowers), 2nd prize: Bipin (Convent), 3rd prize: Balbir (Loyola)
2. 1st prize: Bipin (Convent), 2nd prize: Riaz (Little Flowers), 3rd prize: Balbir (Loyola)
3. 1st prize: Riaz (Little Flowers), 2nd prize: Balbir (Loyola), 3rd prize: Bipin (Convent)
4. 1st prize: Bipin (Convent), 2nd prize: Balbir (Loyola), 3rd prize: Riaz (Litttle Flowers)

5. Two boys are playing on a ground. Both the boys are less than 10 years old. Age of the younger boy
is equal to the cube root of the product of the age of the two boys. If we place the digit representing
the age of the younger boy to the left of the digit representing the age of the elder boy, we get the age
of father of the younger boy. Similarly, if we place the digit representing the age of the elder boy to
the left of the digit representing the age of the younger boy and divide the figure by 2, we get the age
of mother of the younger boy. The mother of the younger boy is younger to his father by 3 years.
Then, what is the age of the younger boy?
1. 3 2. 4 3. 2 4. None of these

6. Flights A and B are scheduled from an airport within the next one hour. All the booked passengers
of the two flights are waiting in the boarding hall after check-in. The hall has a seating capacity of
200, out of which 10% remained vacant. 40% of the waiting passengers are ladies. When boarding
announcement came, passengers of flight A left the hall and boarded the flight. Seating capacity of
each flight is two-third of the passengers who waited in the waiting hall for both the flights put
together. Half the passengers who boarded flight A are women. After boarding for flight A, 60% of the
waiting hall seats became empty. For every twenty of those who are still waiting in the hall for flight
B, there is one air hostess in flight A. What is the ratio of empty seats in flight B to the number of
air hostesses in flight A?
1. 10 : 1 2. 5 : 1 3. 20 : 1 4. 1 : 1

Page 2 PAST CAT PAPER – 2002


Directions for questions 7 to 10: Answer the questions based on the information given below.
A country has the following types of traffic signals.
3 red lights = stop
2 red lights = turn left
1 red light = turn right
3 green lights = go at 100 km/hr speed
2 green lights = go at 40 km/hr speed
1 green light = go at 20 km/hr speed
A motorist starts at a point on a road and follows all traffic signals. His car is heading towards the north.
He encounters the following signals (the time mentioned in each case below is applicable after crossing
the previous signal).
Starting point - 1 green light
After half an hour, 1st signal - 2 red and 2 green lights
After 15 min, 2nd signal - 1 red light
After half an hour, 3rd signal - 1 red and 3 green lights
After 24 min, 4th signal - 2 red and 2 green lights
After 15 min, 5th signal - 3 red lights

7. The total distance travelled by the motorist from the starting point till the last signal is
1. 90 km 2. 100 km 3. 120 km 4. None of these

8. What is the position (radial distance) of the most motorist when he reaches the last signal?
1. 45 km directly north of the starting point
2. 30 km directly to the east of the starting point
3. 50 km away to the north-east of the starting point
4. 45 km away to the north-west of the starting point

9. After the starting point if the 1st signal were 1 red and 2 green lights, what would be the final position
of the motorist?
1. 30 km to the west and 20 km to the south 2. 30 km to the west and 40 km to the north
3. 50 km to the east and 40 km to the north 4. Directly 30 km to the east

10. If at the starting point, the car was heading towards south, what would be the final position of the
motorist?
1. 30 km to the east and 40 km to the south 2. 50 km to the east and 40 km to the south
3. 30 km to the west and 40 km to the south 4. 50 km to the west and 20 km to the north

PAST CAT PAPER – 2002 Page 3


Directions for questions 11 to 13: Answer these questions based on the table given below.
The following table provides data on the different countries and location of their capitals. (the data may not
match the actual Latitude, Longitudes) Answer the following questions on the basis of this table.

S.No. Country Capital Latitude Longitude


1 Argentina Buenos Aires 34.30 S 58.20 E
2 Australia Canberra 35.15 S 149.08 E
3 Austria Vienna 48.12 N 16.22 E
4 Bulgaria Sofia 42.45 N 23.20 E
5 Brazil Brasilia 15.47 S 47.55 E
6 Canada Ottawa 45.27 N 75.42 E
7 Cambodia Phnom Penh 11.33 N 104.55 E
8 Equador Quito 0.15 S 78.35 E
9 Ghana Accra 5.35 N 0.6 E
10 Iran Teheran 35.44 N 51.30 E
11 Ireland Dublin 53.20 N 6.18 E
12 Libya Tripoli 32.49 N 13.07 E
13 Malaysia Kuala Lumpur 3.9 N 101.41 E
14 Peru Lima 12.05 S 77.0 E
15 Poland W arsaw 52.13 N 21.0 E
16 New Zealand W ellington 41.17 S 174.47 E
17 Saudi Arabia Riyadh 24.41 N 46.42 E
18 Spain Madrid 40.25 N 3.45 W
19 Sri Lanka Colombo 6.56 N 79.58 E
20 Zambia Lusaka 15.28 S 28.16 E

11. What percentage of cities located within 10°E and 40°E (20° East and 40° East) lie in the Southern
Hemisphere?
1. 15% 2. 20% 3. 25% 4. 30%

12. The number of cities whose names begin with a consonant and are in the Northern Hemisphere in
the table
1. exceeds the number of cities whose names begin with a consonant and are in the southern
hemisphere by 1
2. exceeds the number of cities whose names begin with a consonant and are in the southern
hemisphere by 2
3. is less than the number of cities whose names begin with a consonant and are in the east of the
meridian by 1
4. is less than the number of countries whose name begins with a consonant and are in the east
of the meridian by 3

13. The ratio of the number of countries whose name starts with vowels and located in the southern
hemisphere, to the number of countries, the name of whose capital cities starts with a vowel in the
table above is
1. 3 : 2 2. 3 : 3 3. 3 : 1 4. 4 : 3

Page 4 PAST CAT PAPER – 2002


Directions for questions 14 to 21: Each item is followed by two statements, A and B. Answer each
question using the following instructions.
Choose 1 if the question can be answered by one of the statement alone but not by the other.
Choose 2 if the question can be answered by using either statement alone.
Choose 3 if the question can be answered by using both the statements together, but cannot be
answered by using either statement alone.
Choose 4 if the question cannot be answered by either of the statements.

14. In a hockey match, the Indian team was behind by 2 goals with 5 min remaining. Did they win the
match?
A. Deepak Thakur, the Indian striker, scored 3 goals in the last 5 min of the match.
B. Korea scored a total of 3 goals in the match.
1. 1 2. 2 3. 3 4. 4
15. Four students were added to a dance class. Would the teacher be able to divide her students evenly
into a dance team (or teams) of 8?
A. If 12 students were added, the teacher could put everyone in teams of 8 without any leftovers.
B. The number of students in the class is currently not divisible by 8.
1. 1 2. 2 3. 3 4. 4
16. Is x = y?
 1 1
A. (x + y)  +  = 4
x y

B. (x − 50)2 = (y − 50)2
1. 1 2. 4 3. 3 4. 4
17. A dress was initially listed at a price that would have given the store a profit of 20% of the wholesale
cost. What was the wholesale cost of the dress?
A. After reducing the listed price by 10%, the dress sold for a net profit of $10.
B. The dress is sold for $50.
1. 1 2. 2 3. 3 4. 4

18. Is 500 the average (arithmetic mean) score in the GMAT?


A. Half of the people who take the GMAT score above 500 and half of the people score below 500.
B. The highest GMAT score is 800 and the lowest score is 200.
1. 1 2. 2 3. 3 4. 4

19. Is |x – 2| < 1?
A. |x| < 1
B. |x – 1| < 2
1. 2 2. 1 3. 3 4. 4

20. People in a club either speak French or Russian or both. Find the number of people in a club who
speak only French.
A. There are 300 people in the club and the number of people who speak both French and Russian
is 196.
B. The number of people who speak only Russian is 58.
1. 1 2. 2 3. 3 4. 4

PAST CAT PAPER – 2002 Page 5


21. A sum of Rs. 38,500 was divided among Jagdish, Punit and Girish. Who received the minimum
amount?
2
A. Jadgish received of what Punit and Girish received together.
9
3
B. Punit received of what Jadgish and Girish received together.
11
1. 1 2. 2 3. 3 4. 4

Directions for questions 22 to 25: Answer the questions based on the following information.
The following table gives details regarding the total earnings of 15 employees and the number of days they
have worked on complex, medium and simple operation in the month of June 2002. Even though the
employees might have worked on an operation, they would be eligible for earnings only if they have minimum
level of efficiency.

Total Earnings Total Days


Emp. No Complex Medium Simple Total Complex Medium Simple Total
2001147 82.98 636.53 719.51 3.00 0.00 23.00 26.00
2001148 51.53 461.73 513.26 3.33 1.67 16.00 21.00
2001149 171.1 79.10 250.81 5.50 4.00 8.50 18.00
2001150 100.47 497.47 597.95 6.00 4.67 7.33 18.00
2001151 594.43 159.64 754.06 9.67 13.33 0.00 23.00
2001156 89.70 89.70 8.00 0.00 1.00 9.00
2001158 472.31 109.73 582.04 1.39 9.61 0.00 11.00
2001164 402.25 735.22 213.67 1351.14 5.27 12.07 0.67 18.00
2001170 576.57 576.57 21.00 0.00 0.00 21.00
2001171 286.48 6.10 292.57 8.38 4.25 0.38 13.00
2001172 512.10 117.46 629.56 10.00 8.50 3.50 22.00
2001173 1303.88 1303.88 25.50 0.00 0.50 26.00
2001174 1017.94 1017.9 26.00 0.00 0.00 26.00
2001179 46.56 776.19 822.75 2.00 19.00 0.00 21.00
2001180 116.40 1262.79 1379.19 5.00 19.00 0.00 24.00
22. The number of employees who have earned more than Rs. 50 per day in complex operations is
1. 4 2. 3 3. 5 4. 6

23. The number of employees who have earned more than Rs. 600 and having more than 80% attendance
(there are 25 regular working days in June 2002; some might be coming on overtime too) is
1. 4 2. 5 3. 6 4. 7

24. The employee number of the person who has earned the maximum earnings per day in medium
operation is
1. 2001180 2. 2001164 3. 2001172 4. 2001179

25. Among the employees who were engaged in complex and medium operations, the number of
employees whose average earning per day in complex operations is more than average earning per
day in medium operations is
1. 2 2. 3 3. 5 4. 7

Page 6 PAST CAT PAPER – 2002


Directions for questions 26 to 33: Answer the questions based on the table given below:

The following table shows the revenue and expenses in millions of Euros (European currency) associated
with REPSOL YPF company’s oil and gas producing activities in operations in different parts of the world
for 1998-2000.
REPSOL YPF’S Operations of Oil and Gas Producing Activities
S. No. Item Year Total Spain North Argentina Rest of Far North Rest of
World Africa & Latin East Sea the
Middle America World
East
1998 916 70 366 281 34 82 78 5
1 Revenue 1999 3374 55 666 2006 115 301 140 91
2000 8328 394 1290 5539 482 603 0 20
1998 668 39 255 187 57 63 52 15
2 Expenses 1999 1999 48 325 1168 131 204 65 58
2000 3709 43 530 2540 252 311 0 33
Income
1998 248 31 111 94 –23 19 26 –10
before
Taxes &
1999 1375 7 341 838 –16 97 75 33
3 Charges
(Revenue-
Expenses) 2000 4619 351 760 2999 230 292 0 –13
=[(1)-(2)]
1998 152 6 104 33 –3 9 6 –3
Taxes &
4 1999 561 3 169 338 –6 39 21 –3
Charges
2000 1845 126 404 1150 61 103 0 1
Net
1998 96 25 7 61 –20 10 20 –7
Income
5 Taxes
1999 814 4 172 500 –10 58 54 36
Charges
[=(3)-(4)] 2000 2774 225 356 1849 169 189 0 –14

26. How many operations (Spain, North Africa and Middle East,..) of the company accounted for less
than 5% of the total revenue earned in 1999?
1. 2 2. 3 3. 4 4. None of these

27. How many operations (Spain, North Africa and Middle East…) of the company witnessed more than
200% revenue from 1999 to 2000?
1. 1 2. 2 3. 3 4. None of these

28. How many operations registered a sustained yearly increase in income before taxes and charges
from 1998 to 2000?
1. 3 2. 4 3. 5 4. None of these

PAST CAT PAPER – 2002 Page 7


29. Ignoring the loss making operations of the company in 1998, for how many operations was the
percentage increase in net income before taxes and charges higher than the average from 1998 to
1999?
1. 0 2. 1 3. 2 4. None of these

30. If profitability is defined as the ratio of net income after taxes and charges to expense, which of the
following statements is true?
1. The Far East operations witnessed its highest profitability in 1998
2. The North Sea operations increased its profitability from 1998 to 1999
3. The operations in Argentina witnessed a decrease in profitability from 1998 to 1999
4. Both 2 and 3 are true

31. In 2000, which among the following countries had the best profitability?
1. North Africa and Middle East 2. Spain
3. Rest of Latin America 4. Far East

32. If efficiency is defined as the ratio of revenue to expenses, which operation was the least efficient in
2000?
1. Spain 2. Argentina 3. Far East 4. None of these

33. Of the following statements, which one is not true?


1. The operations in Spain had the best efficiency in 2000
2. The Far East operations witnessed an efficiency improvement from 1999 to 2000
3. The North Sea operations witnessed an efficiency improvement from 1998 to 1999
4. In 1998, the operations in Rest of Latin America were the least efficient

Directions for questions 34 and 35: Answer the questions based on the pie charts given below.

Sw itzerland Pakistan
Sw itzerland Pakistan 11% 16%
USA
20% 12%
15%
India
20%

USA
17% Turkey
India
Turkey China 15%
China 26%
16% 15%
17%
Chart 1
Chart 2

Chart 1 shows the distribution by value of top 6 suppliers of MFA Textiles in 1995. Chart 2 shows the
distribution by quantity of top 6 suppliers of MFA Textiles in 1995. The total value is 5760 million
Euro (European currency). The total quantity is 1.055 million tonnes.

34. The country which has the highest average price is


1. USA 2. Switzerland 3. Turkey 4. India

Page 8 PAST CAT PAPER – 2002


35. The average price in Euro per kilogram for Turkey is roughly
1. 6.20 2. 5.60 3. 4.20 4. 4.80

Directions for questions 36 to 41: Answer these questions based on the tables given below:

There are 6 refineries, 7 depots and 9 districts. The refineries are BB, BC, BD, BE, BF and BG. The depots
are AA, AB, AC, AD, AE, AF and AG. The districts are AAA, AAB, AAC, AAD, AAE, AAF, AAG, AAH, and
AAI. Table A gives the cost of transporting one unit from refinery to depot. Table B gives the cost of
transporting one unit from depot to a district.

Table A

BB BC BD BE BF BG
AA 928.2 537.2 567.8 589.9 589.9 800.1
AB 311.1 596.7 885.7 759.9 759.9 793.9
AC 451.1 0 320.1 780.1 720.7 1000.1
AD 371.1 150.1 350.1 750.1 650.4 980.1
AE 1137.3 314.5 0 1157.7 1157.7 1023.4
AF 617.1 516.8 756.5 1065.9 1065.9 406.3
AG 644.3 299.2 537.2 1093.1 1093.1 623.9

Table B

AA AB AC AD AE AF AG
AAA 562.7 843.2 314.5 889.1 0 754.8 537.2
AAB 532.7 803.2 284.5 790.5 95.2 659.6 442
AAC 500.7 780.2 0 457.3 205.7 549.1 331.5
AAD 232.9 362.1 286.2 275.4 523.6 525.3 673.2
AAE 345.1 268.6 316.2 163.2 555.9 413.1 227.8
AAF 450.1 644.3 346.2 372.3 933.3 402.9 379.1
AAG 654.5 0 596.7 222.7 885.7 387.6 348.5
AAH 804.1 149.6 627.2 360.4 1035.3 537.2 498.1
AAI 646 255 433.5 137.7 698.7 112.2 161.5

36. What is the least cost of sending one unit from any refinery to any district?
1. 95.2 2. 0 3. 205.7 4. 284.5

37. What is the least cost of sending one unit from any refinery to the district AAB?
1. 0 2. 284.5 3. 95.2 4. None of these

38. What is the least cost of sending one unit from refinery BB to any district?
1. 284.5 2. 311.1 3. 451.1 4. None of these

39. What is the least cost of sending petrol from refinery BB to district AAA?
1. 765.6 2. 1137.3 3. 1154.3 4. None of these

PAST CAT PAPER – 2002 Page 9


40. How many possible ways are there for sending petrol from any refinery to any district?
1. 63 2. 42 3. 54 4. 378

41. The largest cost of sending petrol from any refinery to any district is
1. 2172.6 2. 2193.0 3. 2091.0 4. None of these

Directions for questions 42 to 47: Answer the questions based on the chart given below.
The chart given below indicates the annual sales tax revenue collections (in rupees in crores) of seven
states from 1997 to 2001. The values given at the top of each bar represents the total collections in that
year.
49.638
50000
4300
45000 W B W es t Bengal
42.348
6375
40000 3500 UP Uttar Prdes h
36.068 5270
35000 33.168 3118 8015 TN Tamil Nadu
29.870 2844
30000 4458 7000
W B 2704 3935
25000 UP 3473 6113 12034 MA Maharas htra
5604
TN 5341 10284
20000
8067
7826 5413 KA Karnataka
15000 MA 7290 4839
3829 4265
10000 KA 3510 6300 GU Gujarat
5400
4402 4796
5000 GU 4026
7202 A P A ndhra Prades h
A P 3526 4728 5251 6055
0
1996-97 1997-98 1998-99 1999-2000 2000-01

42. If for each year, the states are ranked in terms of the descending order of sales tax collections,
how many states do not change the ranking more than once over the five years?
1. 1 2. 5 3. 3 4. 4

43. Which of the following states has changed its relative ranking most number of times when you
rank the states in terms of the descending volume of sales tax collections each year?
1. Andhra Pradesh 2. Uttar Pradesh 3. Karnataka 4. Tamil Nadu

44. The percentage share of sales tax revenue of which state has increased from 1997 to 2001?
1. Tamil Nadu 2. Karnataka 3. Gujarat 4. Andhra Pradesh

45. Which pair of successive years shows the maximum growth rate of tax revenue in Maharashtra?
1. 1997 to 1998 2. 1998 to 1999 3. 1999 to 2000 4. 2000 to 2001

Page 10 PAST CAT PAPER – 2002


46. Identify the state whose tax revenue increased exactly by the same amount in two successive pair
of years?
1. Karnataka 2. West Bengal 3. Uttar Pradesh 4. Tamil Nadu

47. Which state below has been maintaining a constant rank over the years in terms of its contribution
to total tax collections?
1. Andhra Pradesh 2. Karnataka 3. Tamil Nadu 4. Uttar Pradesh

Directions for questions 48 to 50: Answer the questions based on the table given below.
The table below gives information about four different crops, their different quality, categories and the
regions where they are cultivated. Based on the information given in the table answer the questions below.

Type of Crop Quality Region


High R1, R2, R3, R4, R5
Crop - 1 Medium R6, R7, R8
Low R9, R10, R11
High R5, R8, R12
Crop - 2 Medium R9, R13
Low R6, R7, R8
High R2, R6, R7. R13
Crop - 3 Medium R3, R9, R11
Low R1, R4
High R3, R10, R11
Crop - 4 Medium R1, R2, R4
Low R5, R9

48. How many regions produce medium qualities of Crop-1 or Crop-2 and also produce low quality of
Crop-3 or Crop-4?
1. Zero 2. One 3. Two 4. Three

49. Which of the following statements is true?


1. All medium quality Crop-2 producing regions are also high quality Crop-3 producing regions.
2. All high quality Crop-1 producing regions are also medium and low Crop-4 producing regions.
3. There are exactly four Crop-3 producing regions, which also produce Crop-4 but not Crop-2.
4. Some Crop-3 producing regions produce Crop-1, but not high quality Crop-2.

50. How many low quality Crop-1 producing regions are either high quality Crop-4 producing regions or
medium quality Crop-3 producing regions?
1. One 2. Two 3. Three 4. Zero

PAST CAT PAPER – 2002 Page 11


Section – 1I

DIRECTIONS for Questions 51 to 53: In each question, there are two statements: A and B, either of
which can be true or false on the basis of the information given below.

A research agency collected the following data regarding the admission process of a reputed manage-
ment school in India.

Number bought Number Number Number


Year Gender application appeared for called for selected for
forms written test interviews the course
Male 61205 59981 684 171
2002
Female 19236 15389 138 48
Male 63298 60133 637 115
2003
Female 45292 40763 399 84

Choose (1) if only A is true


Choose (2) if only B is true
Choose (3) if both A and B are true
Choose (4) if neither A nor B is true

51. Statement A: The success rate of moving from written test to interview stage for males was worse
than for females in 2003.

Statement B: The success rate of moving from written test to interview stage for females was better
in 2002 than in 2003.

52. Statement A: In 2002, the number of females selected for the course as a proportion of the number
of females who bought application forms, was higher than the corresponding proportion for males.

Statement B: In 2002, among those called for interview, males had a greater success rate than
females.

53. Statement A: The percentage of absentees in the written test among females decreased from 2002
to 2003.

Statement B: The percentage of absentees in the written test among males was larger than among
females in 2003.

Page 18 CAT PAPER – 2003 (Leaked)


DIRECTIONS for Questions 54 to 57: Answer the questions on the basis of the information given below.

The length of an infant is one of the measures of his/her development in the early stages of his/her life.
The figure below shows the growth chart of four infants in the first five months of life.

65

60
Seeta
Length (cm)

Geeta
55
Ram
Shyam
50

45
0 1 2 3 4 5
Month

54. After which month did Seeta's rate of growth start to decline?
1. Second month 2. Third month 3. Fourth month 4. Never

55. Who grew at the fastest rate in the first two months of life?
1. Geeta 2. Seeta 3. Ram 4. Shyam

56. The rate of growth during the third month was the lowest for
1. Geeta 2. Seeta 3. Ram 4. Shyam

57. Among the four infants, who grew the least in the first five months of life?
1. Geeta 2. Seeta 3. Ram 4. Shyam

CAT PAPER – 2003 (Leaked) Page 19


DIRECTIONS for Questions 58 to 60: Answer the questions on the basis of the information given below.
The table below provides certain demographic details of 30 respondents who were part of a survey. The
demographic characteristics are: gender, number of children, and age of respondents. The first number in
each cell is the number of respondents in that group. The minimum and maximum age of respondents in
each group is given in brackets. For example, there are five female respondents with no children and
among these five, the youngest is 34 years old, while the oldest is 49.

No. of children Male Female Total


0 1 (38, 38) 5 (34, 49) 6
1 1 (32, 32) 8 (35, 57) 9
2 8 (21, 65) 3 (37, 63) 11
3 2 (32, 33) 2 (27, 40) 4
Total 12 18 30

58. The percentage of respondents aged less than 40 years is at least


1. 10% 2. 16.67% 3. 20.0% 4. 30%

59. Given the information above, the percentage of respondents older than 35 can be at most
1. 30% 2. 73.33% 3. 76.67% 4. 90%

60. The percentage of respondents that fall into the 35 to 40 years age group (both inclusive) is at least
1. 6.67% 2. 10% 3. 13.33% 4. 26.67%

DIRECTIONS for Questions 61 to 63: Answer the questions on the basis of the information given below.

Spam that enters our electronic mailboxes can be classified under several spam heads. The following table
shows the distribution of such spam worldwide over time. The total number of spam emails received during
December 2002 was larger than the number received in June 2003. The total number of spam emails
received during September 2002 was larger than the number received in March 2003. The figures in the
table represent the percentage of all spam emails received during that period, falling into those respective
categories.

Category Sep-02 Dec-02 Mar-03 Jun-03


Adult 38 33 19 17
Financial 25 30 37 45
Health 11 19 5 18
Internet 5 3 10 6
Products 3 7 10 11
Scams 5 6 11 2
Others 13 2 8 1

61. In which category was the percentage of spam emails increasing but at a decreasing rate?
1. Financial 2. Scams 3. Products 4. None of the above

Page 20 CAT PAPER – 2003 (Leaked)


62. In the health category, the number of spam emails received in December 2002 as compared to
June 2003.
1. was larger 2. was smaller 3. was equal 4. cannot be determined

63. In the financial category, the number of spam emails received in September 2002 as compared to
March 2003.
1. was larger 2. was smaller 3. was equal 4. cannot be determined

DIRECTIONS for Questions 64 to 66: Answer the questions on the basis of the information given below.
One of the functions of the Reserve Bank of India is to mobilize funds for the Government of India by issuing
securities. The following table shows details of funds mobilized during the period July 2002 - July 2003.
Notice that on each date there were two rounds of issues, each with a different maturity.

Non-
Competitive Non- Total
Date of Notified competitive Competitive bids Coupon Implicit
Maturity bids competitive amount
issue amount bids accepted rate % yield %
received bids accepted mobilized
received
Rs. Crore Years No. No. No. Value No. Value Rs. Crore
17-Jul-02 40 15 229 23 66 15.21 23 0.37 16 8.07 7.80
17-Jul-02 30 10 145 12 90 29.88 12 0.12 30 6.72 6.72
5-Aug-02 50 9 324 13 105 49.68 13 0.33 50 9.39 7.24
5-Aug-02 20 24 163 9 34 19.81 9 0.19 20 10.18 7.93
28-Aug-02 50 15 260 26 157 48.92 26 1.08 50 7.46 7.46
28-Aug-02 20 30 119 15 67 19.61 15 0.39 20 7.95 7.95
11-Sep-02 40 15 261 22 152 38.93 22 1.07 40 7.46 7.44
11-Sep-02 30 20 131 20 98 29.44 20 0.56 30 8.35 7.70
9-Oct-02 40 11 361 26 119 39.22 26 0.78 40 7.27 7.14
9-Oct-02 30 30 91 15 39 29.52 15 0.48 30 7.95 7.89
7-Nov-02 40 17 245 14 20 39.71 14 0.29 40 10.03 7.26
7-Nov-02 30 24 166 11 49 29.70 11 0.31 30 10.18 7.48
9-Apr-03 40 20 245 25 65 39.53 25 1.47 40 6.30 6.30
9-Apr-03 50 11 236 24 201 49.40 24 0.60 50 7.37 5.98
23-Apr-03 50 15 319 26 134 48.98 26 1.02 50 6.25 6.10
23-Apr-03 20 29 131 19 9 19.39 19 0.61 20 7.95 6.33
5-May-03 60 10 314 14 98 59.69 14 0.31 60 7.27 5.97
5-May-03 30 20 143 14 118 29.58 14 0.42 30 6.30 6.35
4-Jun-03 30 25 187 19 15 28.50 19 1.50 30 6.13 6.13
4-Jun-03 60 9 378 21 151 59.09 21 0.91 60 6.85 5.76
2-Jul-03 50 11 298 20 116 49.05 20 0.95 50 7.37 5.76
2-Jul-03 30 25 114 20 45 28.64 20 1.36 30 6.31 6.10
16-Jul-03 60 17 371 29 115 57.00 29 3.10 60 6.35 5.97
16-Jul-03 30 29 134 22 12 29.32 22 0.68 30 7.95 6.20
Total 930 906

64. How many times was the issue of securities under-subscribed, i.e., how often did the total amount
mobilized fall short of the amount notified?
1. 0 2. 1 3. 2 4. 3

CAT PAPER – 2003 (Leaked) Page 21


65. Which of the following is true?
1. The second round issues have a higher maturity than the first round for all dates.
2. The second round issue of any date has a lower maturity only when the first round notified
amount exceeds that of the second round.
3. On at least one occasion, the second round issue having lower maturity received a higher
number of competitive bids.
4. None of the above three statements is true.

66. Which of the following statements is NOT true?


1. Competitive bids received always exceed non-competitive bids received.
2. The number of competitive bids accepted does not always exceed the number of non-competitive
bids accepted.
3. The value of competitive bids accepted on any particular date is never higher for higher maturity.
4. The value of non-competitive bids accepted in the first round is always greater than that in the
second round.

DIRECTIONS for Questions 67 to 69: Answer the questions on the basis of the information given below.
Each point in the graph below shows the profit and turnover data for a company. Each company belongs
to one of the three industries: textile, cement and steel.

P rofit
400

300 Textile
C em ent
200 Ste el

100

100 0 200 0 300 0 400 0


Turnover

67. For how many companies does the profit exceed 10% of turnover?
1. 8 2. 7 3. 6 4. 5

68. For how many steel companies with a turnover of more than 2000 is the profit than 300?
1. 0 2. 1 3. 2 4. 7

69. An investor wants to buy stock of only steel or cement companies with a turnover more than
1000 and profit exceeding 10% of turnover. How many choices are available to the investor?
1. 4 2. 5 3. 6 4. 7

Page 22 CAT PAPER – 2003 (Leaked)


DIRECTIONS for Questions 70 to 72: Answer the questions on the basis of the information given below.

Details of the top 20 MBA schools in the US as ranked by US News and World Report, 1997 are given
below.

School Overall Ranking by Ranking Ranking by Median % Annual


ranking Academics by placement starting employed tuition fee
recruiters salary

Stanford University 1 1 3 1 $82,000 98.9 $23,100


Harvard University 2 1 2 4 $80,000 96.4 $23,840
University of
3 1 4 2 $79,000 100.0 $24,956
Pennsylvania
Massachusetts Institute
4 1 4 3 $78,000 98.8 $23,900
of Technology
University of Chicago 5 1 8 10 $65,000 98.4 $23,930
Northwestern University 6 1 1 11 $70,000 93.6 $23,025
Columbia University 7 9 10 5 $83,000 96.2 $23,830
Dartmouth College 8 12 11 6 $70,000 98.3 $23,700
Duke Univrsity 9 9 7 8 $67,500 98.5 $24,380
University of
10 7 12 12 $70,000 93.7 $18,788
California—Berkeley
University of Virginia 11 12 9 9 $66,000 98.1 $19,627
University of
12 7 6 14 $65,000 99.1 $23,178
Michigan—Ann Arbor
New York University 13 16 19 7 $70,583 97 $23,554
Carnegie Mellon
14 12 18 13 $67,200 96.6 $22,200
University
Yale University 15 18 17 22 $65,000 91.5 $23,220
Univ. of North
16 16 16 16 $60,000 96.8 $14,333
Carolina—Chapel Hill
University of
17 9 13 38 $65,000 82.2 $19,431
California—Los Angeles
University of
18 18 13 24 $60,000 97.3 $11,614
Texas—Austin
Indiana
19 18 20 17 $61,500 95.2 $15,613
University—Bloomington
Cornell University 20 12 15 36 $64,000 85.1 $23,151

70. Madhu has received admission in all schools listed above. She wishes to select the highest overall
ranked school whose a) annual tuition fee does not exceed $23,000 and b) median starting salary is
at least $70,000. Which school will she select?
1. University of Virginia. 2. University of Pennsylvania
3. Northwestern University 4. University of California - Berkeley

CAT PAPER – 2003 (Leaked) Page 23


71. In terms of staring salary and tuition fee, how many schools are uniformly better (higher median
starting salary AND lower tuition fee) than Dartmouth College?
1. 1 2. 2 3. 3 4. 4

72. How many schools in the list above have single digit rankings on at least 3 of the 4 parameters
(overall ranking, ranking by academics, ranking by recruiters and ranking by placement)?
1. 10 2. 5 3. 7 4. 8

DIRECTIONS for Questions 73 to 75: Answer the questions on the basis of the information given below.

Table A below provides data about ages of children in a school. For the age given in the first column, the
second column gives the number of children not exceeding the age. For example, first entry indicates that
there are 9 children aged 4 years or less. Tables B and C provide data on the heights and weights respec-
tively of the same group of children in a similar format. Assuming that an older child is always taller and
weighs more than a younger child, answer the following questions.

Table A Table B Table C


Age (years) Number Height (cm.) Number Weight (kg.) Number
4 9 115 6 30 8
5 12 120 11 32 13
6 22 125 24 34 17
7 35 130 36 36 28
8 42 135 45 38 33
9 48 140 53 40 46
10 60 145 62 42 54
11 69 150 75 44 67
12 77 155 81 46 79
13 86 160 93 48 91
14 100 165 100 50 100

73. What is the number of children of age 9 years of less whose height does not exceed 135 cm?
1. 48 2. 45 3. 3 4. Cannot be determined

74. How many children of age more than 10 years are taller than 150 cm and do not weigh more than
48 kg?
1. 16 2. 40 3. 9 4. Cannot be determined

75. Among the children older than 6 years but not exceeding 12 years, how many weigh more than
38 kg.?
1. 34 2. 52 3. 44 4. Cannot be determined

Page 24 CAT PAPER – 2003 (Leaked)


DIRECTIONS for Questions 76 to 77: Answer the questions on the basis of the information given below.

An industry comprises four firms (A, B, C, and D). Financial details of these firms and of the industry as a
whole for a particular year are given below. Profitability of a firm is defined as profit as a percentage of sales.

Figures in Rs. A B C D Total


Sales 24568 25468 23752 15782 89570
Operating costs 17198 19101 16151 10258 62708
Interest costs 2457 2292 2850 1578 9177
Profit 4914 4075 4750 3946 17684

76. Which firm has the highest profitability?


1. A 2. B 3. C 4. D

77. If firm A acquires firm B, approximately what percentage of the total market (total sales) will they
corner together?
1. 55% 2. 45% 3. 35% 4. 50%

DIRECTIONS for Questions 78 to 80: Answer the questions on the basis of the information given below.

A, B, C, D, E, and F are a group of friends. There are two housewives, one professor, one engineer, one
accountant and one lawyer in the group. There are only two married couples in the group. The lawyer is
married to D, who is a housewife. No woman in the group is either an engineer or an accountant. C, the
accountant, is married to F, who is a professor. A is married to a housewife. E is not a housewife.

78. Which of the following is one of the married couples?


1. A & B 2. B & E 3. D & E 4. A & D

79. What is E's profession?


1. Engineer 2. Lawyer 3. Professor 4. Accountant

80. How many members of the group are males?


1. 2 2. 3 3. 4 4. Cannot be determined

CAT PAPER – 2003 (Leaked) Page 25


DIRECTIONS for Questions 81 and 82: Answer the questions on the basis of the information given below.

The Head of a newly formed government desires to appoint five of the six elected members A, B, C, D, E
and F to portfolios of Home, Power, Defence, Telecom and Finance. F does not want any portfolio if D gets
one of the five. C wants either Home or Finance or no portfolio. B says that if D gets either Power or
Telecom then she must get the other one. E insists on a portfolio if A gets one.

81. Which is a valid assignment?


1. A-Home, B-Power, C-Defence, D-Telecom, E-Finance.
2. C-Home, D-Power, A-Defence, B-Telecom, E-Finance.
3. A-Home, B-Power, E-Defence, D-Telecom, F-Finance.
4. B-Home, F-Power, E-Defence, C-Telecom, A-Finance.

82. If A gets Home and C gets Finance, then which is NOT a valid assignment of Defense and Telecom?
1. D-Defence, B-Telecom. 2. F-Defence, B-Telecom.
3. B-Defence, E-Telecom. 4. B-Defence, D-Telecom.

DIRECTIONS for Questions 83 to 85: Answer the questions on the basis of the information given below.

Rang Barsey Paint Company (RBPC) is in the business of manufacturing paints. RBPC buys RED, YELLOW,
WHITE, ORANGE, and PINK paints. ORANGE paint can be also produced by mixing RED and YELLOW
paints in equal proportions. Similarly, PINK paint can also be produced by mixing equal amounts of RED
and WHITE paints. Among other paints, RBPC sells CREAM paint, (formed by mixing WHITE and YELLOW
in the ratio 70:30) AVOCADO paint (formed by mixing equal amounts of ORANGE and PINK paint) and
WASHEDORANGE paint (formed by mixing equal amounts of ORANGE and WHITE paint). The following
table provides the price at which RBPC buys paints.

Color Rs./litre
RED 20
YELLOW 25
WHITE 15
ORANGE 22
PINK 18

83. The cheapest way to manufacture AVOCADO paint would cost


1. Rs. 19.50 per litre. 2. Rs. 19.75 per litre
3. Rs. 20.00 per litre. 4. Rs. 20.25 per litre.

84. WASHEDORANGE can be manufactured by mixing


1. CREAM and RED in the ratio 14:10.
2. CREAM and RED in the ratio 3:1.
3. YELLOW and PINK in the ratio 1:1.
4. RED, YELLOW, and WHITE in the ratio 1:1:2.

Page 26 CAT PAPER – 2003 (Leaked)


85. Assume that AVOCADO, CREAM and WASHEDORANGE each sells for the same price. Which
of the three is the most profitable to manufacture?
1. AVOCADO 2. CREAM
3. WASHEDORANGE 4. Sufficient data is not available.

DIRECTIONS for Questions 86 to 88: Answer the questions on the basis of the information given below.

Seven varsity basketball players (A, B, C, D, E, F, and G) are to be honoured at a special luncheon. The
players will be seated on the dais in a row. A and G have to leave the luncheon early and so must be seated
at the extreme right. B will receive the most valuable player's trophy and so must be in the centre to
facilitate presentation. C and D are bitter rivals and therefore must be seated as far apart as possible.

86. Which of the following cannot be seated at either end?


1. C 2. D 3. F 4. G

87. Which of the following pairs cannot be seated together?


1. B & D 2. C & F 3. D & G 4. E & A

88. Which of the following pairs cannot occupy the seats on either side of B?
1. F & D 2. D & E 3. E & G 4. C & F

DIRECTIONS for Questions 89 to 92: In each question there are two statements: A and B.

Choose (1) if the question can be answered by one of the statements alone but not by the other.
Choose (2) if the question can be answered by using either statement alone.
Choose (3) if the question can be answered by using both the statements together but cannot be answered
using either statement alone.
Choose (4) if the question cannot be answered even b using both the statements A and B.

89. F and M are father and mother of S, respectively. S has four uncles and three aunts. F has two
siblings. The siblings of F and M are unmarried. How many brothers does M have?

A. F has two brothers.


B. M has five siblings.

90. A game consists of tossing a coin successively. There is an entry fee of Rs. 10 and an additional fee
of Re. 1 for each toss of coin. The game is considered to have ended normally when the coin turns
heads on two consecutive throws. In this case the player is paid Rs. 100. Alternatively, the player
can choose to terminate the game prematurely after any of the tosses. Ram has incurred a loss of
Rs. 50 by playing this game. How many times did he toss the coin?

A. The game ended normally.


B. The total number of tails obtained in the game was 138.

CAT PAPER – 2003 (Leaked) Page 27


91. Each packet of SOAP costs Rs. 10. Inside each packet is a gift coupon labelled with one of the
letters S, O, A and P. If a customer submits four such coupons that make up the word SOAP, the
customer gets a free SOAP packets. Ms. X kept buying packet after packet of SOAP till she could
get one set of coupons that formed the word SOAP. How many coupons with label P did she get in
the above process?

A. The last label obtained by her was S and the total amount spent was Rs. 210.
B. The total number of vowels obtained was 18.

92. If A and B run a race, then A wins by 60 seconds. If B and C run the same race, then B wins by 30
seconds. Assuming that C maintains a uniform speed what is the time taken by C to finish the
race?

A. A and C run the same race and A wins by 375 metres.


B. The length of the race is 1 km.

DIRECTIONS for Questions 93 to 94: Answer the questions on the basis of the information given below.

Some children were taking free throws at the basketball court in school during lunch break. Below are
some facts about how many baskets these children shot.

i. Ganesh shot 8 baskets less than Ashish.


ii. Dhanraj and Ramesh together shot 37 baskets.
iii. Jugraj shot 8 baskets more than Dhanraj.
iv. Ashish shot 5 baskets more than Dhanraj.
v. Ashish and Ganesh together shot 40 baskets.

93. Which of the following statements is true?


1. Ramesh shot 18 baskets and Dhanraj shot 19 baskets.
2. Ganesh shot 24 baskets and Ashish shot 16 baskets.
3. Jugraj shot 19 baskets and Dhanraj shot 27 baskets.
4. Dhanraj shot 11 baskets and Ashish shot 16 baskets.

94. Which of the following statements is true?


1. Dhanraj and Jugraj together shot 46 baskets.
2. Ganesh shot 18 baskets and Ramesh shot 21 baskets.
3. Dhanraj shot 3 more baskets than Ramesh.
4. Ramesh and Jugraj together shot 29 baskets.

Page 28 CAT PAPER – 2003 (Leaked)


DIRECTIONS for Questions 95 to 97: Answer the questions on the basis of the information given below.
Five women decided to go shopping to M.G. Road, Bangalore. They arrived at the designated meeting
place in the following order: 1. Archana, 2. Chellamma, 3. Dhenuka, 4. Helen, and 5. Shahnaz. Each
woman spent at least Rs. 1000. Below are some additional facts about how much they spent during their
shopping spree.
i. The woman who spent Rs. 2234 arrived before the lady who spent Rs. 1193.
ii. One woman spent Rs. 1340 and she was not Dhenuka.
iii. One woman spent Rs. 1378 more than Chellamma.
iv. One woman spent Rs. 2517 and she was not Archana.
v. Helen spent more than Dhenuka.
vi. Shahnaz spent the largest amount and Chellamma the smallest.
95. What was the amount spent by Helen?
1. Rs. 1193 2. Rs. 1340 3. Rs. 2234 4. Rs. 2517

96. Which of the following amounts was spent by one of them?


1. Rs. 1139 2. Rs. 1378 3. Rs. 2571 4. Rs. 2718

97. The woman who spent Rs. 1193 is


1. Archana 2. Chellamma 3. Dhenuka 4. Helen

DIRECTIONS for Questions 98 to 100: Answer the questions on the basis of the information given below.
Five friends meet every morning at Sree Sagar restaurant for an idli-vada breakfast. Each consumes a
different number of idlis and vadas. The number of idlis consumed are 1, 4, 5, 6, and 8, while the number of
vadas consumed are 0, 1, 2, 4, and 6. Below are some more facts about who eats what and how much.

i. The number of vadas eaten by Ignesh is three times the number of vadas consumed by the person
who eats four idlis.
ii. Three persons, including the one who eats four vadas eat without chutney.
iii. Sandeep does not take any chutney.
iv. The one who eats one idli a day does not eat any vadas or chutney. Further, he is not Mukesh.
v. Daljit eats idli with chutney and also eats vada.
vi. Mukesh, who does not take chutney, eats half as many vadas as the person who eats twice as
many idlis as he does.
vii. Bimal eats two more idlis than Ignesh, but Ignesh eats two more vadas than Bimal.

98. Which one of the following statements is true?


1. Daljit eats 5 idlis 2. Ignesh eats 8 idlis
3. Bimal eats 1 idli. 4. Bimal eats 6 idlis.
99. Which of the following statements is true?
1. Sandeep eats 2 vadas. 2. Mukesh eats 4 vadas.
3. Ignesh eats 6 vadas. 4. Bimal eats 2 vadas.
100. Which of the following statements is true?
1. Mukesh eats 8 idlis and 4 vadas but no chutney.
2. The person who eats 5 idlis and 1 vada does not take chutney.
3. The person who eats equal number of vadas and idlis also takes chutney.
4. The person who eats 4 idlis and 2 vadas also takes chutney.

CAT PAPER – 2003 (Leaked) Page 29


a teacher's situation, so (2) is out. are in (4) also indicate 64. 2 It happened only once i.e; on 17-Jul-02
a compulsive situation which is not evident in the
sentence, so (4) is out. 65. 3 From the table we can see that for issue dated,
04 June-03, the 2nd round issue has a lower maturity
50. 4 Negative reinforcements foster negative behavior. (1), and the competitive bids received are higher.
(2) and (3) are easily ruled out as giving, bestowing or
conferring rewards cannot possibly encourage 66. 4 For any issue the value of non-competitive bids in 2nd
negative behaviour. Withholding and fostering thus round is greater than the 1st round.
presents the right situation here.
67. 2 Just draw a diagonal line from bottom left point to top
51. 4 From the data both statements are false. right point. All companies lying above this line have
profit in excess of 10% of turnover.
52. 4 From the data both statements are false. From the graph there are 7 companies, has the profit
10% of turnover.
53. 1 From the data statement "A" is true.
68. 3 From the graph there are 2 steel companies with a
54. 2 It is evident from graph Seeta's growth rate decreased turnover of more than 2000 and profit less than 300.
from third month as this is the first time the slope has
decreased. 69. 2 From the graph there are 5 companies.
55. 1 Geeta grew at fastest rate in first two months (the
slope of the line in this period is steepest for Geeta). 70. 4 By looking up the table, in University of California -
Berkeley median starting salary is $70,000 and annual
56. 1 Geeta grew lowest in third month (during this period, tuition fee is $18,788.
the slope was least for Geeta).
71. 2 By looking up the table, the number of schools, uniformly
57. 4 Seeta increased 7cm on 50 and shyam 7cm on 53cm, better than Dartmouth College is 2.
Hence Shyam grew least.
72. 4 By counting from the table, eight rows of first nine
row schools satisfy the given condition.
9
58. 4 × 100 = 30%
30 73. 2 There are 45 children of height not exceeding 135 and
48 children of age not exceeding 9 yrs. Consider the
tallest child of the 45 children with height not exceeding
23
59. 3 × 100 = 76.67% 135. We can be very sure that his age is less than
30 9 yrs as taller children have higher weights. Thus all
45 children of heights not exceeding 135 will have
4 age not exceeding 9 yrs.
60. 3 × 100 = 13.33%
30
74. 1 Using the same logic as above, there are 25 children
taller than 150 cms and more than 10 years of age.
61. 3 Incase of Products, percentage of spam emails is There are 9 children of weights more than 48. These
increasing but at decreasing rate, from Sep 2002 to 9 children are surely included in the 25 children taller
7−3 than 150 cms and more than 10 years of age because
Dec 2002 products increased by ≈ 133% and in of the assumption given. Thus 25 – 9 = 16 children
3
satisfy the condition.
7−4
Mar 2003 about ≈ 43% and in Jun 2003 75. 3 There are 55 children not exceeding 12 years but
7
older than 6 years. Again 33 children weigh less than
 11 − 10  or equal to 38. Of these, 22 are those who are less
 ≈ 10%  than 6 years of age. Thus 11 of the 55 students weigh
 10 
less than or equal to 38 kg.
So the answer is 55 – 11 = 44.
62. 1 Since percentage of spam is Dec 2002 is higher than
June 2003, and the number of total e-mails received is 76. 4 Profitability is defined as percentage of sales.
higher, hence number received in Dec 2002 is higher. Approximately Firm A has 25% profit, B has 16.66%, C
has 20% and D has approximately 30% profit.
63. 4 Cannot be determined as in Sept 2002 percentage is
lower as compared to March 2003, however the total
number of emails received in Sept 2003 is higher than
that in March 2002. Thus we cannot say anything.

Page 4 CAT PAPER – 2003(Leaked) Explanations


84. 4 Mixing equal amounts of ORANGE and WHITE can
24568 + 25468
77. 1 × 100 = 55% make WASHEDORANGE, ORANGE can be made by
89570 mixing equal amounts of RED and YELLOW. So the
ratio of RED, YELLOW and WHITE is 1 : 1 : 2
For questions 78 to 80:
85. 2 If cost of AVOCADO paint is Rs.19.75
(+) - Male
7 × 15 + 3 × 75
(-) - Female The cost of the CREAM is = Rs. 18
10
A(Lawyer)(+)-----Couple ------ D (Housewife)(-) And cost of WASHEDORANGE is Rs.18.50
C (Accountant)(+)-----Couple-----F(Professor)(-) So CREAM is the most profitable.
(Or) F (Professor)(+)-----Couple-----C(Accountant)(-)
For questions 86 to 88:
(B)(Housewife)(-)
(E)(Engineer)(+)
1 2 3 4 5 6 7
78. 4 C B D A G
79. 1 D B C A G
D B C G A
80. 3 D B C G A
For questions 81 and 82:
86. 3 From given options F is the only possibility.
If D gets portfolio F does not or vice-versa.
C wants only Home or Finance or none 87. 4 If we look at the options D & G can sit together, C & F
If D gets Power B must get Telecom or D - Telecom then B must can sit together, B & D can sit together and E & A is the
get Power only option which is not possible.
If A gets a portfolion E should get.
88. 3 E & G is the only possibility.

81. 2 (1) gets eliminated because C can have either home 89. 1 S has 4 uncles and from statement A. F has two
or finance. brothers. Hence, the other 2 uncles of S must be the
(3) gets eliminated because F and D cannot be in the brothers of M. Statement B does not give any additional
same team. information.
(4) gets eliminated because C cannot have telecom
portfolio. 90. 2 From both statements individually. If x is the number of
Hence (2) is correct. tosses he took, from statement I we get the equation
10 + x – 100 = 50. Thus x = 140.
82. 4 B-Defence, D - Telecom because if D gets Telcom then From statement II individually, we have x > 138.
B must get Power. Thus we are sure he has paid up more than 148. If he
incurs a loss of only Rs. 50, the game has to end
83. 2 AVOCADO paint is mixture of ORANGE and PINK in normally. Thus the above state of his taking 150 shots
equal quantities. with first 138 as tails and 139 and 140 throw as heads
If ORANGE is made using RED and YELLOW, then the is the scenario. With no other scenario will a loss of
just 50 and 138 tails show up.
20 + 25
cost of ORANGE would be = 22.5 which is
2 91. 3 Since Ms. X bought 21 packets out of which there are
greater than the cost of the ORANGE. 18 O’s and A’s in total. Since she got one S, there has
If we make PINK by RED and WHITE, the cost of PINK to be 2 P’s which she bought. Hence, both the
20 + 15 statements are required.
would be = 17.5 which is less than the cost
2
of the PINK paint.
22 + 17.5
Hence, the cost of the AVOCADO is = 19.75
2

CAT PAPER – 2003(Leaked) Explanations Page 5


92. 3 If A takes X seconds then B takes (x + 60) seconds to 100. 3
run 1000 m.
Ratio of speeds of A and C = 1000 : 625 = 8 : 5 For questions 101 to 102:
Ratio of times taken by A and C = 5 : 8
If B takes y second then C takes y + 30 seconds to run S, M and R in all spend 1248 bahts.
1000 m. Initially M pays 211 bahts and R pays 92 bahts.
Hence 5 (y + 30) = 8x …(i) Remaining is paid by S i.e; 945 bahts
1000 1000 If 1248 is divided equally among S, M & R and each has to
and x + 60 = y …(ii) spend 415 bahts
Hence M has to pay S 205 bahts which is 5 Dollars.
Solving we get the values of x and y.
And R has to pay 324 bahts to S.
Hence both statements are required.
101. 4
For questions 93 and 94:
102. 3
G+8=A
D + R = 37
103. 4 Putting the value of M in either equation, we get
J=D+8
G + B = 17.
A=D+5
Hence neither of two can be uniquely determined.
A + G = 40
Solving we get
104. 2 As per the given data we get the following:
2G = 32, G = 16, A = 24
D = 19, j = 27, R = 18 M ed ha
G ia ni
93. 1 G 8 M
6
3 2
94. 1 D + J = 46
B
For questions 95 to 97:
B u dd hi
Five shopping women spending various amounts with
conditions G + B = M + 16
Also, M + B + G + 19 = (2 × 19) – 1
One of the women spent 2517 – 1378 = 1139 who is i.e. (G + B) = 18 – M
Chellamma. This is the only possibility as if we add 1378 even Thus, M + 16 = 18 – M
to the least amount of 1193, we will not be able to satisfy all the i.e. M = 1
conditions given simultaneously.
105. 3 2x – x – 1 = 0
⇒ 2x – 1 = x
A C D H S If we put x = 0, then this is satisfied and if we put
2234 1139 1193 1340 2517 x = 1, then also this is satisfied.
Now we put x = 2, then this is not valid.
95. 2 96. 1 97. 3
106. 2 For the curves to intersect, log10 x = x–1
For questions 98 to 100: 1
Thus, log10 x = or x x = 10
x
Shree Sagar restaurant and idli-vada breakfast
This is possible for only one value of x (2 < x < 3).

Idli Vada 107. 4 The surface area of a sphere is proportional to the


Ignesh 6 6 square of the radius.
Sandeep 1 0 SB 4
Thus, = (S. A. of B is 300% higher than A)
Mukesh 4 2 SA 1
Daljit 5 1
r 2
Bimal 8 4 ∴ B =
rA 1
98. 1

99. 3

Page 6 CAT PAPER – 2003(Leaked) Explanations


Section – III

Directions for questions 111 to 120: Each question consists of five statements followed by options
consisting of three statements put together in a specific order. Choose the option which indicates a valid
argument, that is, where the third statement is a conclusion drawn from the preceding two statements.

Example:
A. All cigarettes are hazardous to health.
B. Brand X is a cigarette.
C. Brand X is hazardous to health.
ABC is a valid option, where statement C can be concluded from statements A and B.

111. A. All software companies employ knowledge workers.


B. Tara Tech employs knowledge workers.
C. Tara Tech is a software company.
D. Some software companies employ knowledge workers.
E. Tara Tech employs only knowledge workers.
a. ABC b. ACB c. CDB d. ACE

112. A. Traffic congestion increases carbon monoxide in the environment.


B. Increase in carbon monoxide is hazardous to health.
C. Traffic congestion is hazardous to health.
D. Some traffic congestion does not cause increased carbon monoxide.
E. Some traffic congestion is not hazardous to health.
a. CBA b. BDE c. CDE d. BAC

113. A. Apples are not sweets.


B. Some apples are sweet.
C. All sweets are tasty.
D. Some apples are not tasty.
E. No apple is tasty.
a. CEA b. BDC c. CBD d. EAC

114. A. Some towns in India are polluted.


B. All polluted towns should be destroyed.
C. Town Meghana should be destroyed.
D. Town Meghana is polluted.
E. Some towns in India should be destroyed.
a. BDE b. BAE c. ADE d. CDB

115. A. No patriot is a criminal.


B. Bundledas is not a criminal.
C. Bundledas is a patriot.
D. Bogusdas is not a patriot.
E. Bogusdas is a criminal.
a. ACB b. ABC c. ADE d. ABE

CAT PAPER - 1999 Page 31


116. A. Anteaters like ants.
B. Boys are anteaters.
C. Balaram is an anteater.
D. Balaram likes ants.
E. Balaram may eat ants.
a. DCA b. ADC c. ABE d. ACD

117. A. All actors are handsome.


B. Some actors are popular.
C. Ram is handsome.
D. Ram is a popular actor.
E. Some popular people are handsome.
a. ACD b. ABE c. DCA d. EDC

118. A. Modern industry is technology-driven.


B. BTI is a modern industry.
C. BTI is technology-driven.
D. BTI may be technology-driven
E. Technology driven industry is modern.
a. ABC b. ABD c. BCA d. EBC

119. A. All Golmal islanders are blue-coloured people.


B. Some smart people are not blue-coloured people.
C. Some babies are blue-coloured.
D. Some babies are smart.
E. Some smart people are not Golmal islanders.
a. BCD b. ABE c. CBD d. None of these

120. A. MBAs are in great demand.


B. Ram and Sita are in great demand.
C. Ram is in great demand.
D. Sita is in great demand.
E. Ram and Sita are MBAs.
a. ABE b. ECD c. AEB d. EBA

Directions for questions 121 to 124: Each question has a main statement followed by four statements
labelled A, B, C and D. Choose the ordered pair of statements where the first statement implies the
second, and the two statements are logically consistent with the main statement.

121. Either the orangutan is not angry, or he frowns upon the world.
A. The orangutan frowns upon the world.
B. The orangutan is not angry.
C. The orangutan does not frown upon the world.
D. The orangutan is angry.
a. CB only b. DA only c. AB only d. CB and DA

Page 32 CAT PAPER - 1999


122. Either Ravana is a demon, or he is a hero.
A. Ravana is a hero.
B. Ravana is a demon.
C. Ravana is not a demon.
D. Ravana is not a hero.
a. CD only b. BA only c. CD and BA d. DB and CA

123. Whenever Rajeev uses the Internet, he dreams about spiders.


A. Rajeev did not dream about spiders.
B. Rajeev used the Internet.
C. Rajeev dreamt about spiders.
D. Rajeev did not use the Internet.
a. AD b. DC c. CB d. DA

124. If I talk to my professors, then I do not need to take a pill for headache.
A. I talked to my professors.
B. I did not need to take a pill for headache.
C. I needed to take a pill for headache.
D. I did not talk to my professors.
a. AB only b. DC only c. CD only d. AB and CD

Directions for questions 125 to 134: Each question has a set of four statements. Each statement has
three segments. Choose the alternative where the third segment in the statement can be logically deduced
using both the preceding two, but not just from one of them.

125. A. No cowboys laugh. Some who laugh are sphinxes. Some sphinxes are not cowboys.
B. All ghosts are florescent. Some ghost do not sing. Some singers are not florescent.
C. Cricketers indulge in swearing. Those who swear are hanged. Some who are hanged are not
cricketers.
D. Some crazy people are pianists. All crazy people are whistlers. Some whistlers are pianists.
a. A and B b. C only c. A and D d. D only

126. A. All good people are knights. All warriors are good people. All knights are warriors.
B. No footballers are ministers. All footballers are tough. Some ministers are players.
C. All pizzas are snacks. Some meals are pizzas. Some meals are snacks.
D. Some barkers are musk deer. All barkers are sloth bears. Some sloth bears are musk deer.
a. C and D b. B and C c. A only d. C only

127. A. Dinosaurs are prehistoric creatures. Water-buffaloes are not dinosaurs. Water-buffaloes are not
prehistoric creatures.
B. All politicians are frank. No frank people are crocodiles. No crocodiles are politicians.
C. No diamond is quartz. No opal is quartz. Diamonds are opals.
D. All monkeys like bananas. Some GI Joes like bananas. Some GI Joes are monkeys.
a. C only b. B only c. A and D d. B and C

CAT PAPER - 1999 Page 33


128. A. All earthquakes cause havoc. Some landslides cause havoc. Some earthquakes cause landslides.
B. All glass things are transparent. Some curios are glass things. Some curios are transparent.
C. All clay objects are brittle. All XY are clay objects. Some XY are brittle.
D. No criminal is a patriot. Ram is not a patriot. Ram is a criminal.
a. D only b. B only c. C and B d. A only

129. A. MD is an actor. Some actors are pretty. MD is pretty.


B. Some men are cops. All cops are brave. Some brave people are cops.
C. All cops are brave. Some men are cops. Some men are brave.
D. All actors are pretty ; MD is not an actor ; MD is not pretty.
a. D only b. C only c. A only d. B and C

130. A. All IIMs are in India. No BIMs are in India. No IIMs are BIMs.
B. All IIMs are in India. No BIMs are in India. No BIMs are IIMs.
C. Some IIMs are not in India. Some BIMs are not in India. Some IIMs are BIMs.
D. Some IIMs are not in India. Some BIMs are not in India. Some BIMs are IIMs.
a. A and B b. C and D c. A only d. B only

131. A. Citizens of Yes Islands speak only the truth. Citizens of Yes Islands are young people. Young
people speak only the truth.
B. Citizens of Yes Islands speak only the truth. Some Yes Islands are in Atlantic. Some citizens of
Yes Islands are in the Atlantic.
C. Citizens of Yes Islands speak only the truth. Some young people are citizens of Yes Islands.
Some young people speak only the truth.
D. Some people speak only the truth. Some citizens of Yes Islands speak only the truth. Some
people who speak only the truth are citizens of Yes Islands.
a. A only b. B only c. C only d. D only

132. A. All mammals are viviparous. Some fish are viviparous. Some fish are mammals.
B. All birds are oviparous. Some fish are not oviparous. Some fish are birds.
C. No mammal is oviparous. Some creatures are oviparous and some are not. Some creatures are
not mammals.
D. Some creatures are mammals. Some creatures are viviparous. Some mammals are viviparous.
a. A only b. B only c. C only d. D only

133. A. Many singers are not writers. All poets are singers. Some poets are not writers.
B. Giants climb beanstalks. Some chicken do not climb beanstalks. Some chicken are not giants.
C. All explorers live in snowdrifts. Some penguins live in snowdrifts. Some penguins are explorers.
D. Amar is taller than Akbar. Anthony is shorter than Amar. Akbar is shorter than Anthony.
a. A only b. B only c. B and C d. D only

134. A. A few farmers are rocket scientists. Some rocket scientists catch snakes. A few farmers catch
snakes.
B. Poonam is a kangaroo. Some kangaroos are made of teak. Poonam is made of teak.

Page 34 CAT PAPER - 1999


C. No bulls eat grass. All matadors eat grass. No matadors are bulls.
D. Some skunks drive Cadillacs. All skunks are polar bears. Some polar bears drive Cadillacs.
a. B only b. A and C c. C only d. C and D

Directions for questions 135 to 138: Answer the questions based on the following information.
The figure below represents sales and net profit in Rs. crore of IVP Ltd. for five years from 1994-95 to
1998-99. During this period the sales increased from Rs. 100 crore to Rs. 680 crore. Correspondingly, the
net profit increased from Rs. 2.5 crore to Rs. 12 crore. Net profit is defined as the excess of sales over total
costs.

700 680 14
600 12
500 8.5 12 10
400 6 8
300 250 6
200 2.5 300 290 4
4.5
100 100 2
0 0
1994-95 1995-96 1996-97 1997-98 1998-99

Sales Net profit

135. The highest percentage of growth in sales, relative to the previous year, occurred in
a. 1995-96 b. 1996-97 c. 1997-98 d. 1998-99

136. The highest percentage growth in net profit, relative to the previous year, was achieved in
a. 1998-99 b. 1997-98 c. 1996-97 d. 1995-96

137. Defining profitability as the ratio of net profit to sales, IVP Ltd., recorded the highest profitability in
a. 1998-99 b. 1997-98 c. 1994-95 d. 1996-97

138. With profitability as defined in question 137, it can be concluded that


a. profitability is non-decreasing during the five years from 1994-95 to 1998-99.
b. profitability is non-increasing during the five years from 1994-95 to 1998-99.
c. profitability remained constant during the five years from 1994-95 to 1998-99.
d. None of the above

Directions for questions 139 to 142: Answer the questions based on the following information
Consider the information provided in the figure below relating to India’s foreign trade in 1997-98 and the first
eight months of 1998-99. Total trade with a region is defined as the sum of exports and imports from that
region. Trade deficit is defined as the excess of imports over exports. Trade deficit may be negative.
A. USA G. Other East European countries
B. Germany H. OPEC
C. Other EU I. Asia
D. UK J. Other LDCs
E. Japan K. Others
F. Russia

CAT PAPER - 1999 Page 35


Source of imports
1997-98 1998-99
Imports into India $40,779 million Imports into India (April-November)
$28,126 million
J K J K A
A
4% 1% 5% 1% 9% B
I 9% B
I 5%
14% 5%
16%
C C
12% 14%

D D
H
H 6% 6%
17%
23%
E E
5% 5%
G
G
F 21% F
19%
2% 1%

Destination of exports

1997-98 1998-99
Exports from India: $33,979 million Exports from India (April-November)
$21,436 million
J K J K
A
5% 1% 5% 1% A
19% 23%
I
I
17%
20%
B
6%
B
H 6%
H C 10%
10% 14% C
G G 14%
D D
F E 12% F E
10% 6%
3% 6% 2% 5% 5%

139. What is the region with which India had the highest total trade in 1997-98?
a. USA b. Other EU countries c. OPEC d. Others

140. In 1997-98 the amount of Indian exports, million US dollars, to the region with which India had the
lowest total trade, is approximately
a. 750 b. 340 c. 220 d. 440

141. In 1997-98, the trade deficit with respect to India, billion US dollars, for the region with the highest
trade deficit with respect to India, is approximately equal to
a. 6.0 b. 3.0 c. 4.5 d. 7.5

Page 36 CAT PAPER - 1999


142. What is the region with the lowest trade deficit with India in 1997-98?
a. USA b. Asia c. Others d. Other EU countries

Directions for questions 143 and 144: Answer the questions based on the following information.
Assume that the average monthly exports from India and imports to India during the remaining four months
of 1998-99 would be the same as that for the first eight months of the year.

143. What is the region to which India’s exports registered the highest percentage growth between
1997-98 and 1998-99?
a. Other East European countries b. USA
c. Asia d. Exports have declined, no growth

144. What is the percentage growth rate in India’s total trade deficit between 1997-98 and 1998-99?
a. 43 b. 47 c. 50 d. 40

Directions for questions 145 to 148: Answer the questions based on the following information.
These questions are based on the price fluctuations of four commodities — arhar, pepper, sugar and gold
during February-July 1999 as described in the figures below.

Arhar (F riday quo tation s) Pepp er (F riday qu otation s)


2 50 0 2 00 00
Price (Rs. per quintal)

Price (Rs. per quintal)

2 30 0 1 95 00
2 10 0 1 90 00
1 90 0 1 85 00

1 70 0 1 80 00
1 50 0 1 75 00
1 30 0 1 70 00
F1 F4 M 3 A2 A5 M 3 JN 2 JL1 F1 F4 M3 A2 A5 M 3 JN 2 JL1
W eeks (1999) W eeks (1999)

Su gar (Frid ay q uotations) G old (F riday quo tations)


1 52 0 4 30 0
P rice (Rs. p er quin tal)

Price (Rs. per 10g)

1 50 0 4 20 0
1 48 0 4 10 0
1 46 0 4 00 0

1 44 0 3 90 0
1 42 0 3 80 0
1 40 0 3 70 0
F1 F4 M 3 A2 A 5 M 3 JN 2 JL1 F1 F4 M 3 A2 A 5 M 3 JN 2 JL1
W eeks (1999) W eeks (1999)

145. Price change of a commodity is defined as the absolute difference in ending and beginning prices
expressed as a percentage of the beginning. What is the commodity with the highest price change?
a. Arhar b. Pepper c. Sugar d. Gold

CAT PAPER - 1999 Page 37


146. Price volatility (PV) of a commodity is defined as follows:
PV = (Highest price during the period – Lowest price during the period)/Average price during the
period. What is the commodity with the lowest price volatility?
a. Arhar b. Pepper c. Sugar d. Gold

147. Mr X, a fund manager with an investment company invested 25% of his funds in each of the four
commodities at the beginning of the period. He sold the commodities at the end of the period. His
investments in the commodities resulted in
a. 17% profit b. 5.5% loss c. No profit, no loss d. 5.4% profit

148. The price volatility(PV) of the commodity with the highest PV during the February-July period is
approximately equal to
a. 3% b. 40% c. 20% d. 12%

Directions for questions 149 to 153: Answer the questions based on the following information.
The table below presents data on percentage population covered by drinking water and sanitation facilities
in selected Asian countries.

Population covered by drinking water and sanitation facilities


Percentage coverage

Drinking water Sanitation facilities

Urban Rural Total Urban Rural Total

India 85 79 81 70 14 29

Bangladesh 99 96 97 79 44 48

China 97 56 67 74 7 24

Pakistan 82 69 74 77 22 47

Philippines 92 80 86 88 66 77

Indonesia 79 54 62 73 40 51

Sri Lanka 88 52 57 68 62 63

Nepal 88 60 63 58 12 1

(Source: World Resources 1998-99, p. 251, UNDP, UNEP and World Bank.)

Country A is said to dominate B or A > B if A has higher percentage in total coverage for both drinking water
and sanitation facilities, and, B is said to be dominated by A, or B < A.
A country is said to be on the coverage frontier if no other country dominates it. Similarly, a country is not
on the coverage frontier if it is dominated by at least one other country.

149. Which countries are the countries on the coverage frontier?


a. India and China b. Sri Lanka and Indonesia
c. Philippines and Bangladesh d. Nepal and Pakistan

Page 38 CAT PAPER - 1999


150. Which of the following statements are true?
A. India > Pakistan and India > Indonesia B. India > China and India > Nepal
C. Sri Lanka > China D. China > Nepal
a. A and C b. B and D c. A, B and C d. B, C and D

151. Using only the data presented under ‘sanitation facilities’ columns, it can be concluded that rural
population in India, as a percentage of its total population is approximately
a. 76 b. 70 c. 73 d. Cannot be determined

152. Again, using only the data presented under ‘sanitation facilities’ columns, sequence China, Indonesia
and Philippines in ascending order of rural population as a percentage of their respective total
population. The correct order is
a. Philippines, Indonesia, China b. Indonesia, China, Philippines
c. Indonesia, Philippines, China d. China, Indonesia, Philippines

153. India is not on the coverage frontier because


A. it is lower than Bangladesh in terms of coverage of drinking water facilities.
B. it is lower than Sri Lanka in terms of coverage of sanitation facilities.
C. it is lower than Pakistan in terms of coverage of sanitation facilities.
D. it is dominated by Indonesia.
a. A and B b. A and C c. D d. None of these

Directions for questions 154 and 155: Answer the questions based on the following information.
These relate to the above table with the additional provison that the gap between the population coverages
of ‘sanitation facilities’ and ‘drinking water facilities’ is a measure of disparity in coverage.

154. The country with the most disparity in coverage of rural sector is
a. India b. Bangladesh c. Nepal d. None of these

155. The country with the least disparity in coverage of urban sector is
a. India b. Pakistan c. Philippines d. None of these

Directions for questions 156 to 165: Each question is followed by two statements I and II.
Mark:
a. if the question can be answered by any one of the statements alone, but cannot be answered by
using the other statement alone.
b. if the question can be answered by using either statement alone.
c. if the question can be answered by using both the statements together, but cannot be answered
by using either statement alone.
d. if the question cannot be answered even by using both the statements together.

156. The average weight of students in a class is 50 kg. What is the number of students in the class?
I. The heaviest and the lightest members of the class weigh 60 kg and 40 kg respectively.
II. Exclusion of the heaviest and the lightest members from the class does not change the average
weight of the students.

157. A small storage tank is spherical in shape. What is the storage volume of the tank?
I. The wall thickness of the tank is 1 cm.
II. When an empty spherical tank is immersed in a large tank filled with water, 20 l of water overflow
from the large tank.

CAT PAPER - 1999 Page 39


158. Mr X starts walking northwards along the boundary of a field from point A on the boundary, and after
walking for 150 m reaches B, and then walks westwards, again along the boundary, for another
100 m when he reaches C. What is the maximum distance between any pair of points on the
boundary of the field?
I. The field is rectangular in shape.
II. The field is a polygon, with C as one of its vertices and A as the mid-point of a side.

159. A line graph on a graph sheet shows the revenue for each year from 1990 through 1998 by points
and joins the successive points by straight-line segments. The point for revenue of 1990 is labelled
A, that for 1991 as B, and that for 1992 as C. What is the ratio of growth in revenue between
1991-92 and 1990-91?
I. The angle between AB and X-axis when measured with a protractor is 40°, and the angle between
CB and X-axis is 80°.
II. The scale of Y-axis is 1 cm = Rs. 100

160. There is a circle with centre C at the origin and radius r cm. Two tangents are drawn from an external
point D at a distance d cm from the centre. What are the angles between each tangent and the
X-axis.
I. The coordinates of D are given.
II. The X-axis bisects one of the tangents.

161. Find a pair of real numbers x and y that satisfy the following two equations simultaneously. It is
known that the values of a, b, c, d, e and f are non-zero.
ax + by = c
dx + ey = f
I. a = kd and b = ke, c = kf, k ≠ 0
II. a = b = 1, d = e = 2, f ≠ 2c

162. Three professors A, B and C are separately given three sets of numbers to add. They were expected
to find the answers to 1 + 1, 1 + 1 + 2, and 1 + 1 respectively. Their respective answers were 3, 3 and
2. How many of the professors are mathematicians?
I. A mathematician can never add two numbers correctly, but can always add three numbers
correctly.
II. When a mathematician makes a mistake in a sum, the error is +1 or –1.

163. How many students among A, B, C and D have passed the examination?
I. The following is a true statement: A and B passed the examination.
II. The following is a false statement: At least one among C and D has passed the examination.

164. What is the distance x between two cities A and B in integral number of kilometres?
I. x satisfies the equation log2 x = x
II. x ≤ 10 km

165. Mr Mendel grew 100 flowering plants from black seeds and white seeds, each seed giving rise to
one plant. A plant gives flowers of only one colour. From a black seed comes a plant giving red or
blue flowers. From a white seed comes a plant giving red or white flowers. How many black seeds
were used by Mr Mendel?
I. The number of plants with white flowers was 10.
II. The number of plants with red flowers was 70.

Page 40 CAT PAPER - 1999


107. For minimum petrol consumption, Zoheb should drive
160 HH
at 40 kmph, petrol consumption = = 6.67 l.
24 IC M
TC
For questions 108 to 110:

Opening Closing
Game Player's pick Dealer's pick TC = Instances of traffic congestion
balance balance
ICM = Instances of increase in carbon monoxide in the
Debit Credit Debit Credit environment.
( –) (+) (–) (+) HH = Instances that are hazardous to health.
The premises state that increase in carbon monoxide
1 0 0 8 16 0 -8 is hazardous to health and traffic congestion increases
carbon monoxide in the environment. Therefore, traffic
2 -8 0 10 0 10 12 congestion is hazardous to health.
(a) CBA is invalid, because if TC and ICM are both
3 12 0 6 6 0 12 separately hazardous to health (HH), no definite
relationship between TC and ICM necessarily follows.
4 12 0 8 16 0 4 (b) BDE is invalid, because the some TC that are not
ICM need not necessarily be free of any hazard to
health (HH): they could possibly pollute the environment
108. Hence, we see that the maximum gain is Rs. 12 with other noxious gases.
109. Since the maximum negative that Ghosh Babu goes HH
into is –8, he should begin with at least Rs. 8, so that TC
he does not have to borrow any money at any point. IC M

110. From the above table it is evident that in four games,


Ghosh Babu makes a profit of Rs. 4. Hence, if the final
amount left with Ghosh Babu is Rs. 100, the initial The shaded portion shows those some TC that are
amount that he had would be Rs. 96. not ICM, but could possibly be hardardous to health
(HH).
111. (b) ACB is the correct answer choice. (c) CDE is invalid, because the conclusion E is a
as it forms a valid syllogism. negation of one of the premises C. Another check
reveals the TC is the middle term, which appears in
both the premises C and D, and E, therefore, should
CEKW not appear in the conclusion E, as per the basic
SC definition of a syllogism.
TT
113. (a) CEA is the correct answer choice, as it forms a
valid syllogism.

TT
CEKW = Companies that employ knowledge workers
SC = Software companies S A
TT = Tara Tech
The premises state that all software companies
employ knowledge workers and Tara Tech is a
Figures S = Sweets
software company. Therefore, Tara Tech employs
TT = Tasty things
knowledge workers.
A = Apples
(a) ABC is invalid, because if Tara Tech employs
Sweets are a subset of tasty things. Apples are not
knowledge workers, it cannot definitely be concluded
tasty things. As apples are not a part of the main set
that it is a software company.
of tasty things, they can also not be a part of the
(c) CDB is invalid, because if only some software
subset of sweets.
companies employ knowledge workers, Tara Tech
(b) BDC is invalid, because the set of some A that are
may not be among them.
S and the set of some A that are not TT may have no
(d) ACE introduces a fourth term. ‘Companies that
relationship with each other (disjoint sets).
employ only knowledge workers’ and, therefore,
(c) CBD is invalid, because some A that are S are
violates the very definition of a syllogism, which must
definitely TT, but we have no knowledge of the
have 3 and only 3 terms.
remaining A, and hence can reach no conclusion about
them.
112. (d) BAC is the correct answer choice, as it forms a
(d) EAC is invalid, because if all A are neither S nor TT,
valid syllogism.
we cannot establish a definite relationship between S
and TT.

CAT PAPER - 1999 Page 13


114. (b) BAE is the correct answer choice, as it forms a
valid syllogism. LA
AE
SB D IT
Bl
PT

PT = Polluted towns Bl = Balaram


SBD = Things that should be destroyed AE = Anteaters
IT = Indian towns or towns in India. LA = Creatures who like ants
The shaded portion are those IT which are polluted Anteaters like ants and Balram is an anteater.
(PT) and hence should be destroyed (SBD). Therefore, Balram likes ants.
The premises state that polluted towns should be (a) DCA is invalid. Just because Balaram likes ants
destroyed and that some towns in India are polluted. and he is also an anteater, it does not logically follow
The ‘some’ in statement E refers to the polluted Indian that all anteaters like ants. It would, however, be valid
towns that should be destroyed. to conclude that ‘some anteaters like ants.’
(a) BDE is invalid, because with B and D as premises, (b) ADC is invalid, because if Balaram like ants, we
the only valid conclusion that follows is C. Town cannot definitely conclude that Balaram is an anteater,
Meghana should be destroyed. ‘Town Meghana’ cannot as evident from the Venn diagram.
be simply replaced by ‘Some town in India,’ as in E.
(c) ADE is invalid, because the syllogism has more
than 3 terms. The term SBD does not appear in the LA
premises A and D, but appears in the conclusion E as AE Bl
a new and fourth term.
(d) CDB is invalid, because we can’t talk of all PT in the Bl
conclusion B, when we only have information about
Town Meghana in both the premises C and D. In
deductive reasoning, we cannot proceed from specific
cases to general cases. It would, however, be valid to (c) ABC is invalid, because it has four terms:
conclude that ‘some polluted towns should be Anteaters, creatures who like ants, boys and Balaram.
destroyed.’ Furthermore, E: ‘Balaram may eat ants’ is a tantologous
statement and can never feature in a valid syllogism.
115. (a) ACB is the correct answer choice, as it forms a A ‘may’ statement implies ‘may not’ and is always true
valid syllogism. (tantalogous), and can always stand on its own,
without any supporting premises. Hence, such a
P statement is of no use to a student of logic, who is
C concerned with the process of reasoning, arriving at
B
a definite conclusion from definite information given in
the premises.
P = Patriots
117. (b) ABE is the correct answer choice, as it forms a
C = Criminals
valid syllogism.
B = Bundledas
Since patriot and criminal are two distinct sets, what
is patriot, cannot be criminal. Therefore, if Bundeldas
H
is a patriot, Bundeldas cannot be a criminal.
(b) ABC is invalid, because if no P is C and B is not C,
we cannot be definite that B is not P.
A
B P
P
C
B H = Handsome people
A = Actors
P = Popular people
Bundledas (B) may be a patriot (P) or not. Nothing
The shaded portion are some popular people who are
definite can be concluded.
actors and hence are handsome people.
(c) ADE is invalid, on similar lines as explained in the
All actors are handsome and some of those actors
case of (b).
are also popular. Therefore, some who are popular
(d) ABE is invalid, because the syllogism has four
are also handsome.
terms: patriots, criminals, Bundledas and Bogusdas.
(a) ACD is invalid, as there are four terms: actors,
handsome people, Ram and popular actors.
116. (d) ACD is the correct answer choice, as it forms a
Furthermore, just because Ram is handsome does not
valid syllogism.
definitely mean that he is an actor, let alone ‘a popular

Page 14 CAT PAPER - 1999


actor’. SP = Smart people
(c) and (d) are similarly invalid as option (a), because NBCP = Not blue-coloured people
they each have four terms. Those smart people who are not blue-coloured people
(shaded portion) are definitely not Golmal islanders.
118. (a) ABC is the correct answer choice, as it forms a (a) BCD is invalid, because there is no connection
valid syllogism. between some babies that are BCP and the some SP
who are not BCP.
TD
MI
SP
B TI BCP

(c) CBD is invalid, because there need not necessarily


be any relationship between the some B that are BCP
BTI = BTI and the some B that are SP.
MI = Modern Industry
TD = Thing that is technology-driven BCP SP
All modern industries are technology-driven.
BTI is a modern industry B
Therefore, BTI is technology-driven.
(b) ABD is invalid, although it is similar to the correct
option ABC.
120. (c) AEB is the correct answer choice, as it forms a
TD valid syllogism.
MI
PGD
B TI
M BA
R +S

It is invalid simply because the statement D: ‘BTI may


be technology-driven, is a tantalogous statement (that
is, it is always true, regardless of what the premises R + S = Ram and Sita
say). PGD = People in great demand
(c) BCA is invalid. Just because BTI is a modern MBA = MBA
industry and it is technology-driven, it does not mean As all MBAs are in great demand and Ram and Sita
that all MI are TD. It would, however, be valid to conclude belong to the set of MBAs, Ram and Sita are also in
that ‘Some MI are TD’. great demand.
(d) EBC is invalid, because if BTI is a modern industry, (a) ABE is invalid because R + S need not be MBAs,
we cannot definitely conclude that it is technology- just because both are in great demand.
driven, as evident from the Venn diagram. (b) ECD makes no sense.
(d) EBA is invalid. Just because R + S are MBAs and
MI in great demand does not mean that ‘all MBAs are in
great demand’. It would, however, be valid to conclude
TD the ‘some MBAs are in great demand’.
BTI
BTI 121. The main statement clearly states that if the orangutan
(O) belongs to the set of not angry (NA), he cannot
belong to the set of frowns (F). Also, O must belong to
either of the two sets.
CB states that O is not F and is NA. And DA states that
119. (b) ABE is the correct answer choice, as it forms a O is not NA and is F. Therefore, both CB and DA are
valid syllogism. logically valid.

122. The main statement clearly states that Ravana (R) is


either a demon (D) or a hero (H). He has to be at least
BCP one of the two, and if he is one, he cannot be the
other.
GI S .P N .B .C .P
DB states that R is not H but is D.
And CA states that R is not D but is H.
Therefore, both DB and CA are logically valid.
GI = Golmal islanders
BCP = Blue-coloured people

CAT PAPER - 1999 Page 15


123. The question statement clearly states that whenever ∴ Some H are not C.
Rajeev uses the Internet, he dreams about spiders.
However, this does not mean that he dreams about H
spiders only when he uses the Internet. Therefore, S
using the Internet is only one possible condition for C
Rajeev to dream about spiders. Thus, the Venn diagram
for the question will be:

D .S
C = Cricketers
U .I S = Those who swear
H = Those who are hanged
The valid conclusion is:
All C are H (ü) and not its negation: Some H are not (C)
(x)
Where,
Statement D is valid.
UI → Uses Internet
Some CP are P.
DS → Dreams of spiders All CP are W.
Answer choice (a) AD states that Rajeev did not dream ∴ Some W are P.
about spiders, and hence he did not use the Internet.
This logically follows the above Venn diagram. If the W P
main set does not occur, the subset will also not occur.
CP
Thus, AD is the right option.

124. P = I talk to my professors


Q = I do not need to take a pill for headache.
CP = Crazy people
Therefore, both AB(P ⇒ Q) and CD(neg Q ⇒ neg P)
W = Wrestlers
are logically consistent, not BA and DC.
P = Pianists
(d) ‘AB and CD’ is the correct answer choice.
The shaded portion represents those some pianists
who are crazy people and are, therefore, wrestlers.
125. (c) ‘A and D’ is the correct answer choice
And conversely, they are those some wrestlers who
C S L are pianists.

126. (a) ‘C and D’ is the correct answer choice.


Statement A is invalid: The valid conclusion would be
Statement A is valid: ‘All W are K’, rather than its converse, ‘All K are W’.
The shaded portion represents those some sphinxes
that laugh and, therefore, are not cowboys as no K
cowboys laugh. GP
Statement B is invalid. W
All G are F.
Some G are not S.
∴ Some S are not F.

F K = Knights
G S GP = Good People
W = Warriors
Statement B is invalid, because it has four terms:
footballers, ministers, tough people and players.
Statement C is valid: The ‘some M’ that are P are also
F = Florescent things
S. (shaded position).
G = Ghosts
S = Singers
S M
The valid conclusion is: Some F(shared position) are
not S. (V) and not the converse: Some S are not F. (X) P
(The some ghosts who are not singers (in shaded
portion) are also the ‘some florescent things that are
not singers’)
Statement C is invalid: S = Snacks
All C are S. P = Pizzas
All S are H. M = Meals
Statement D is valid: The ‘some MD’ that are B are also
SB. (shaded portion).

Page 16 CAT PAPER - 1999


Statement C is valid:
SB MD
B B
CO
XY

SB = Sloth bears
B = Barkers
MD = Musk deer
B = Brittle things
CO = Clay objects
127. (b) B only is the correct answer choice.
CO is a subset of B.
Statement A is invalid, as ‘No WB are PC’ cannot be
XY is a subset of CO.
validly concluded. What can, however, be validly
Therefore, XY is also a subset of B.
concluded is that ‘Some PC (which are all D in shaded
Thus, all XY are also brittle.
position) are not WB’.
Statement D is invalid.
No valid conclusion can follow from two negative
PC
premises, as negatives have the effect of separating
D WB one term from the other.

129. (b) C is the only correct answer choice.


Statement A is invalid.
D = Dinosaurs MD need not necessarily be among the some actors
PC = Prehistoric creatures who are pretty.
WB = Water buffaloes Statement B is invalid, as the third segment is not a
Statement B is valid as evident from the Venn diagram conclusion logically deduced using both the first and
second segments. In fact, the conclusion can directly
F be deduced from the second segment alone. Another
way to check for this error is to see whether the
P C
middle term ‘Cops’, that appears in the premises also
appears in the conclusion. Here, ‘Cops’ appears in all
three segments and option (b) is therefore, not a valid
F = Frank creatures syllogism.
P = Politicians Statement C is valid. Those ‘Some men’, who are cops
C = Crocodiles are also brave people (shaded portion).
P is a subset of F.
F and C are disjoint sets. BP M
As the main set F can never intersect with C, the
subset D will also never intersect or coincide with C. C
Statement C is invalid: No valid conclusion can follow
from two negative premises, as negatives have the
effect of separating one term from the others. BP = Brave people
Statement D is invalid, as the ‘Some GI Joes’ who like C = Cops
bananas need not be monkeys. M = Men
C is a subset of BP
128. (c) ‘C and B’ is the correct answer choice. Some M is C.
Statement A is invalid because it has four terms: Therefore, those M that are C, are also BP.
a. Earthquakes Statement D is invalid, MS may still be pretty without
b. Things that cause havoc being an actor.
c. Landslides
d. Things that cause landslides 130. (a) ‘A and B’ is the correct answer choice.
Statement B is valid as the ‘Some C’ which are GT are A and B are both valid: ‘No IIMs are BIMs’ or its
also T (shaded portion). converse,
‘No BIMs are IIMs’ are both equally valid.
T C
GT II

IIM B IM

T = Transparent
GT = Glass things II = Institutes in India.
C = Curios IIMs is a subset of II.
As all glass things (GT) are transparent (T), those No II is a BIM
curios (C) that are GT are definitely T. As the mainset II can never intersect with the disjoint

CAT PAPER - 1999 Page 17


set BIM, the subset IIM will also never intersect or 133. (b) B only is the correct answer choice.
coincide with BIM.
C and D are both invalid: The ‘Some IIMs’ and the ‘Some S
BIMs’ need not have any relationship between them,
just because they are not in India. W P

131. (c) Only C is the correct answer choice.


Statement A is invalid. We cannot conclude about ‘all
YP’, but only about ‘Some YP’ (shaded portion) who Statement A is invalid, as no definite relationship
are, incidentally, ‘all CY’. between P and W can be established.
Note: ‘Many’ is translated as ‘Some’ to convert the
ST statement in standard form.
YP The ‘Some S (shared portion)’ are not W, but some
other S could be W, as shown in the Venn diagram
above.

CB
CYI
G C
CYI = Citizens of Yes Islands
ST = People who speak only the truth
YP = Young people
Statement B is invalid, because it has four terms: Statement B is valid, as the ‘Some C (shaded portion)’
a. Citizens of Yes Islands that do not climb beans stalks cannot be giants.
b. People who speak only the truth G = Grants
c. Things that are in the Atlantic C = Children
d. Yes Islands CB = Creatures which climb beanstalks
Statement C is valid. The ‘Some YP’ who are CYI also C is invalid, the ‘Some penguins’ that live in snowdrifts
are people who speak only the truth. (shaded portion) need not be explorers.
D is invalid, as Amar is the tallest among the three, but
ST YP it is not clear how the heights of Akbar and Anthony
are compared.
CYI Note: This is not a ‘Categorical’ syllogism comprising
statements, as such. All S is P, No S is P, Some S is P
and some S is not P. It is a ‘relational’ syllogism
comprising relational statements that normally feature
Statement D is invalid, because the ‘some people’ and in analytical reasoning. Be alert: CAT examiner is in
the ‘some CYI’ need not have any relationship between the habit of jumbling up questions to throw you off
them, just because both speak only the truth. gear.

132. (c) C only is the correct answer choice. 134. (d) ‘C and D’ is the correct answer choice.
Statement A is invalid, as the some fish that are
viviparous need not be mammals. F CS
RS
O
B F
Statement A is invalid: The ‘Some F’ who are RS and
the ‘Some RS’ who can catch snakes need not have
any relationship between them.
Statement B is invalid. We can validly conclude that CS = People who catch snakes
the ‘Some fish (shaded portion)’ that are not oviparous RS = Rocket scientists
are definitely not birds as well. But, based on this, we F = Farmers
cannot conclude that ‘Some fish are birds’, as we Note: ‘Many’ or ‘a few’ are both translated as ‘some’
have no knowledge about the ‘remaining fish’. to convert the statements in the standard form.
Statement C is valid, as we can validly conclude that Statement B is invalid. Poonam need not be among the
the ‘Some C’ which are inside the circle O cannot be some kangaroos that are made of teak.
mammals. But nothing definite can be said about the
‘Some C’ which are outside the circle O. EG
B
O N .O . M
C
Statement C is valid. No matadors can be bulls because
Statement D is invalid, because the ‘Some C’, which the former eat grass, while the latter do not.
are V and the ‘Some C’, which are M, need not have EG: Creatures that eat grass
any relationship between them, as evident from the M: Matadors
Venn diagram.

Page 18 CAT PAPER - 1999


B: Bulls 138. It may be seen that profitability does not follow a fixed
Statement D is valid. The some PB (shaded portion) pattern as the first three statements try to generalize
are in effect the ‘Some skunks’ that drive Cadillacs. the profitability. They are not applicable.

PB DC 139. Total trade with a region is defined as: The sum of


S exports and imports from that region, from the pie
charts for 1997-98, we have the following sectors
occupying maximum area.

DC: Creature that drive Cadillacs


Country Exports Imports Trade
PB: Polar bears
S: Skunks
H OPEC 3397.9 9379.2 12795.1
135. From the graph, we know the percentage growth in
I A si a 6796 5709 12505
sales are:
E a st
Previous Current G 3397 7748 11145.9
Difference Percentage Europe
sales sales
C EU 4757 4893.5 9650.5
1995-96 100 250 150 150%
A USA 6456 3670 10126
1996-97 250 300 50 20%

1997-98 300 290 -10 –3.33% H – OPEC has the maximum trade.

1998-99 290 680 390 134.5% 140. From the pie chart, the region having lowest trade is
K.
It is but obvious from the above table that the maximum 1% of imports + 1% of exports
percentage increase relative to previous year occurred ∴ Indian exports are 1% of 3397.9 which is roughly
in 1995-96. 340 US $ million.

136. From the graph, we can again calculate the growth in 141.
profits. Imports to Exports Trade deficit
India from India
Previous Current
Difference Percentage A 3670.11 6456.01 –2785.9
profit profit
B 2038.95 2038.74 0.21
1995-96 2.5 4.5 2 80%
C 4893.48 4757.06 136.42
1996-97 4.5 6 1.5 33.33%
D 2446.74 2038.74 408
1997-98 6.0 8.5 2.5 41.66%
E 2038.95 2038.74 0.21
1998-99 8.5 12 3.5 41.2%

The highest percentage growth in net profit relative to F 815.58 1019.37 –203.79
the previous year was achieved in 1995-96.
G 7748.01 3397.9 4350.11
137.
H 9379.17 3397.9 5981.27

I 5709.06 6795.8 –1086.74


Net profit Net sales Ratio
J 1631.16 1698.95 –67.79
1994-95 2.5 100 0.025

1995-96 4.5 250 0.02 So, we see that region H has highest trade deficit of
approximately $6,000 million or $6 billion.
1996-97 6 300 0.02
142. From the pie chart for 1997-98, we get that USA which
1997-98 8.5 290 0.03 is a region A has the lowest trade deficit.
(9% of imports – 19% of total exports)
1998-99 12 680 0.018
 9 19 
 × 40779 − × 33979 
The profitability is maximum for 1997-98.  100 100 
= $– 2785.9 million.

CAT PAPER - 1999 Page 19


143. From the pie chart, we know that the exports have 146. The price volatility for each individual.
increased from three regions A, G and H as follows.

Highest Lowest Average


Co u n tr y 1998-99 1997-98 Difference PV
price price price

A USA 7395.4 6456 Arhar 2300 1500 800 1900 0.42

E a st P epper 19500 17350 2250 18425 0.12


G European 3858.5 3397.9
countries Sugar 1500 1410 90 1455 0.062

H OPEC 3215.4 3397.9 Gold 4300 3800 480 4050 0.12

Also the exports for 8 months have been given. The price volatility for sugar is least, hence answer
According to new directions, before question 143, choice is (c).
we need to calculate exports for 12 months Note: Average price can be calculated by highest
21436 × 12 price, lowest price, ending and beginning price.
= $32,154 million.
8
147. Let us assume Mr X invested Rs. 100 in each
The maximum percentage increase is therefore from
commodity.
region A.

144. India’s total trade deficits are as follows.


Price increase
Income on Rs. 100
percentage
Imports Exports Deficit
Arhar 25 25
1997-98 40779 33979 6800

1998-99 42189 32154 10035 P epper 4 4.0

Sugar –0.3 –0.3


28126 ×12
Total imports for 1998-99 =
8 Gold –7.8 –7.8
= $42,189 millions
28126 ×12 21.5
Similarly, exports for 1998-99 =
8
∴ His income is Rs. 21.5 on Rs. 400.
= $32,154 millions
21.5
∴ × 100 = 5.4% profit
400
10035 − 6800
Percentage growth rate = × 100
6800
148. As per the table in question 146, the maximum PV is
= 47.6% around 40%.
145. The price changes for each commodity are as follows. 149. Also Bangladesh > Philippines (97 > 86) for drinking
water.
Ending Beginning Difference Percentage And Philippines > any other countries for sanitation
facilities.Thus, these three countries are on the
Arhar 2125 1700 425 25 coverage frontier for two facilities.

P epper 19275 18525 750 4 150. Statement A > Statement B only if statement A has
higher percentage in total coverage for both drinking
Sugar 1435 1440 5 –0.3 water and sanitation facilities taken independently and
not as a total of the two facilities.
Gold 3820 4250 330 7.8 Thus, only statement B and statement D are
India > China
(81 > 67 and 29 > 24)
The highest price change from the graph and the above
India > Nepal
is definitely for Arhar.
(81 > 63 and 29 > 18)
Also China > Nepal (67 > 63 and 24 > 18)

Page 20 CAT PAPER - 1999


151. Let the urban population be x and rural population be coverage – Percentage denoting sanitation coverage),
y. For example, rural sector of India = 79 – 14= 65%
From the sanitation column, we have Thus, as it can be seen from the table, in rural sector
0.7x + 0.14y = 0.29(x + y) the country with most disparity is India (79 – 14)
0.41x = 0.15y = 65%.
And the country with least disparity in urban sector is
15
∴x= y Philippines (92 – 88) = 4%
41
∴ Percentage of rural population 156. Statement I gives the weight of the heaviest and lightest
members of the class but no indication as to the number
y
= × 100 of students in the class or the total weight of the
x+y students is there. The second statement is also
inconclusive, making our answer choice as (d).
y
= × 100
15 157. Statement I gives the thickness of the wall which is of
y+y
41 no use to find the volume of the tank since we do not
know the radius of the sphere.
41 Statement II gives us the answer as the volume of
= × 100
56 water displaced is equal to the volume of the immersed
= 73.2% tank (from Archimedes’ principle)
So to find the exact storage volume of the tank both
152. In the same way as the previous questions, we can the statements are needed.
find percentage of rural population for Philippines,
Indonesia and China. 158. Statement I by itself does not solve the problem but it
does tell us about the shape of the field. However, it
fails to give information about the points A, B and C as
P 50%
to whether they be at the end of the field, etc. This
data is given by the second statement, from which it is
I 66.66% known that

C 79.8% 1 00
C B

Thus, P < I < C


1 50
153. India is not on coverage frontier because
(i) it is below Bangladesh and Philippines for drinking A
water.
(ii) for sanitation facilities it is below Philippines,
Sri Lanka, Indonesia and Pakistan. 1 50

For questions 154 and 155:


The disparity for the coverage of rural sector is as follows.
The polygon has the length = 150 × 2 = 300 m and the
Rural sector Urban sector breadth = 100 m and also that it is a rectangle
(from A). Thus, the maximum distance is the diagonal
I 65 15 length of the rectangle.

B 52 20
159. R C
C 49 23

P 47 5

P 20 4
B 8 0°
I 22 6 Q

S –5 20 P A 4 0°

N 51 30

Note: Disparity = (Percentage denoting drinking facilities


90 91 92

CAT PAPER - 1999 Page 21


RQ
Ratio of revenues =
QP
Since in a line graph, the years are uniformly spaced
Q
RQ tan80°
⇒ =
QP tan 40° L C
(iii) DL = LQ
So the ratio can be determined from statement I alone.

Statement II is immaterial because we intend to find the D


ratio and not absolute figures.
P

160. P

D
P
(iv) DL = LQ
D L C
r d
Q

C
r

Q 4 more cases are possible when x-axis bisects DP.


However, for all the 8 cases, we have 2 sets of slopes
for DP and DQ. Hence angles cannnot be determined
uniquely.

161. Statement I when used to solve the sum gives us the


same equation as the second substituted in to the first
equation.
r and d are given kdx + key = kf
∴ k(dx + ey) = kf
From statement I, when co-ordinates of D are given, as k ≠ 0.
only one pair of tangents can be drawn onto the given So, it is of no use as we get infinite solutions and not
circle from D. So angle made by x-axis for each can a unique one.
be found out. Hence statement I is sufficient. Statement II gives us the following equations.
x+y=c
From statement II, all we can interpret is that x-axis 2x + 2y = f.
bisects one of the tangents. Keeping this in mind, many These are two linear equations in x and y, such that
figures are possible.
1 1 c
= ≠
2 2 f
P
c 1
D As ≠ (Given)
f 2
(i) QL = LD Which is a system having a unique solution.
C L
162. Statement II tells us that mathematicians can make
Q
mistakes which are always errors of +1 and –1.
Also statement I tells us that mathematicians can never
add 2 numbers correctly but we know he can make
mistakes also.
Again he can always add 3 numbers correctly.
Therefore, as mistakes can be made here too, we
Q cannot decide as to who is a mathematician.

163. From I, we know A and B passed the examination.


L From II, we know the condition that among C and D at
(ii) QL = LD
C D least one passed (or both passed) is false.
Therefore, it is obvious that both C and D have failed.
Thus, both statements are necessary to find the
P answer.

Page 22 CAT PAPER - 1999


164. Statement I tells us that A satisfies the equation 165. Statement I gives us the number of white flowers. But
we know that a white seed gives both red or white
log2 x = x flowers. Thus, proving statement II, gives the number
of red flowers. But both black and white seeds give
∴ 2 x =x red flowers, again providing no solutions.
i.e. x = 4, but if x = 16 then also the above relation is
true. Here we need to consider statement II in order to
get the unique value of x, i.e. x = 4.

CAT PAPER - 1999 Page 23


Section – III

Directions for questions 111 to 120: There are ten short arguments given below. Read each of the
passages and answer the question that follows it.

111. In a recent report, the gross enrolment ratios at the primary level, that is, the number of children
enrolled in classes one to five as a proportion of all children aged six to ten, were shown to be very
high for most states; in many cases they were way above 100 per cent! These figures are not worth
anything, since they are based on the official enrolment data compiled from school records. They
might as well stand for ‘gross exaggeration ratios’.

Which one of the following options best supports the claim that the ratios are exaggerated?
a. The definition of gross enrolment ratio does not exclude, in its numerator, children below six
years or above ten years enroled in classes one to five.
b. A school attendance study found that many children enrolled in the school records were not
meeting a minimum attendance requirement of 80 per cent.
c. A study estimated that close to 22 per cent of children enrolled in the class one records were
below six years of age and still to start going to school.
d. Demographic surveys show shifts in the population profile which indicate that the number of
children in the age group six to ten years is declining.

112. Szymanski suggests that the problem of racism in football may be present even today. He begins
by verifying an earlier hypothesis that clubs’ wage bills explain 90 per cent of their performance.
Thus, if players’ salaries were to be only based on their abilities, clubs that spend more should
finish higher. If there is pay discrimination against some group of players — fewer teams bidding for
black players thus lowering the salaries for blacks with the same ability as whites — that neat
relation may no longer hold. He concludes that certain clubs seem to have achieved much less than
what they could have, by not recruiting black players.

Which one of the following findings would best support Szymanski’s conclusion?
a. Certain clubs took advantage of the situation by hiring above-average shares of black players.
b. Clubs hired white players at relatively high wages and did not show proportionately good
performance.
c. During the study period, clubs in towns with a history of discrimination against blacks, under-
performed relative to their wage bills.
d. Clubs in one region, which had higher proportions of black players, had significantly lower wage
bills than their counterparts in another region which had predominantly white players.

FLT – 06 Page 31
113. The pressure on Italy’s 257 jails has been increasing rapidly. These jails are old and overcrowded.
They are supposed to hold up to 43,000 people — 9,000 fewer than now. San Vittore in Milan, which
has 1,800 inmates, is designed for 800. The number of foreigners inside jails has also been increasing.
The minister-in-charge of prisons fears that tensions may snap, and so has recommended to the
government an amnesty policy.

Which one of the following, if true, would have most influenced the recommendation of the minister?
a. Opinion polls have indicated that many Italians favour a general pardon.
b. The opposition may be persuaded to help since amnesties must be approved by a two-thirds
majority in parliament.
c. During a recent visit to a large prison, the Pope, whose pronouncements are taken seriously,
appealed for ‘a gesture of clemency’.
d. Shortly before the recommendation was made, 58 prisons reported disturbances in a period of
two weeks.

114. The offer of the government to make iodised salt available at a low price of one rupee per kilogram is
welcome, especially since the government seems to be so concerned about the ill effects of non-
iodised salt. But it is doubtful whether the offer will actually be implemented. Way back in 1994, the
government, in an earlier effort, had prepared reports outlining three new and simple but experimental
methods for reducing the costs of iodisation to about five paise per kilogram. But these reports have
remained just those — reports on paper.

Which one of the following, if true, most weakens the author’s contention that it is doubtful whether
the offer will be actually implemented?
a. The government proposes to save on costs by using the three methods it has already devised for
iodisation.
b. The chain of fair-price distribution outlets now covers all the districts of the state.
c. Many small-scale and joint-sector units have completed trials to use the three iodisation methods
for regular production.
d. The government which initiated the earlier effort is in place even today and has more information
on the effects of non-iodised salt.

115. About 96 per cent of Scandinavian moths have ears tuned to the ultrasonic pulses that bats, their
predators, emit. But the remaining 4 per cent do not have ears and are deaf. However, they have a
larger wingspan than the hearing moths, and also have higher wing-loadings — the ratio between a
wing’s area and its weight — meaning higher manoeuvrability.

Which one of the following can be best inferred from the above passage?
a. A higher proportion of deaf moths than hearing moths fall prey to bats.
b. Deaf moths may try to avoid bats by frequent changes in their flight direction.
c. Deaf moths are faster than hearing moths, and so are less prone to becoming a bat’s dinner than
hearing moths.
d. The large wingspan enables deaf moths to better receive and sense the pulses of their bat
predators.

Page 32 FLT – 06
116. Argentina’s beef cattle herd has dropped to under 50 million from 57 million ten years ago in 1990.
The animals are worth less, too: prices fell by over a third last year, before recovering slightly. Most
local meat packers and processors are in financial trouble, and recent years have seen a string of
plant closures. The Beef Producers Association has now come up with a massive advertisement
campaign calling upon Argentines to eat more beef — their ‘juicy, healthy, rotund, plate-filling’
steaks.

Which one of the following, if true, would contribute most to a failure of the campaign?
a. There has been a change in consumer preference towards eating leaner meats like chicken and
fish.
b. Prices of imported beef have been increasing, thus making locally grown beef more competitive in
terms of pricing.
c. The inability to cross-breed native cattle with improved varieties has not increased production to
adequate levels.
d. Animal rights pressure groups have come up rapidly, demanding better and humane treatment of
farmyard animals like beef cattle.

117. The problem of traffic congestion in Athens has been testing the ingenuity of politicians and town
planners for years. But the measures adopted to date have not succeeded in decreasing the number
of cars on the road in the city centre. In 1980, an odds and evens number-plate legislation was
introduced, under which odd and even plates were banned in the city centre on alternate days,
thereby expecting to halve the number of cars in the city centre. Then in 1993 it was decreed that all
cars in use in the city centre must be fitted with catalytic converters; a regulation had just then been
introduced, substantially reducing import taxes on cars with catalytic converters, the only condition
being that the buyer of such a ‘clean’ car offered for destruction a car at least 15-year-old.

Which one of the following options, if true, would best support the claim that the measures adopted
to date have not succeeded?
a. In the 1980s, many families purchased second cars with the requisite odd or even number plate.
b. In the mid-1990s, many families found it feasible to become first-time car owners by buying a car
more than 15-year-old and turning it in for a new car with catalytic converters.
c. Post-1993, many families seized the opportunity to sell their more than 15-year-old cars and buy
‘clean’ cars from the open market, even if it meant forgoing the import tax subsidy.
d. All of the above

FLT – 06 Page 33
118. Although in the limited sense of freedom regarding appointments and internal working, the
independence of the Central Bank is unequivocally ensured, the same cannot be said of its right to
pursue monetary policy without coordination with the Central Government. The role of the Central
Bank has turned out to be subordinate and advisory in nature.

Which one of the following best supports the conclusion drawn in the passage?
a. A decision of the chairman of the Central Bank to increase the bank rate by two percentage
points sent shock-waves in industry, academic and government circles alike.
b. Government has repeatedly resorted to monetisation of the debt despite the reservations of the
Central Bank.
c. The Central Bank does not need the Central Government’s nod for replacing soiled currency
notes.
d. The inability to remove coin shortage was a major shortcoming of this government.

119. The Shveta-chattra or the ‘White Umbrella’ was a symbol of sovereign political authority placed over
the monarch’s head at the time of the coronation. The ruler so inaugurated was regarded not as a
temporal autocrat but as the instrument of protective and sheltering firmament of supreme law. The
white umbrella symbol is of great antiquity and its varied use illustrates the ultimate common basis
of non-theocratic nature of states in the Indian tradition. As such, the umbrella is found, although not
necessarily a white one, over the head of Lord Ram, the Mohammedan sultans and Chatrapati
Shivaji.

Which one of the following best summarises the above passage?


a. The placing of an umbrella over the ruler’s head was a common practice in the Indian subcontinent.
b. The white umbrella represented the instrument of firmament of the supreme law and the non-
theocratic nature of Indian states.
c. The umbrella, not necessarily a white one, was a symbol of sovereign political authority.
d. The varied use of the umbrella symbolised the common basis of the non-theocratic nature of
states in the Indian tradition.

120. The theory of games is suggested to some extent by parlour games such as chess and bridge.
Friedman illustrates two distinct features of these games. First, in a parlour game played for money,
if one wins the other (others) loses (lose). Second, these games are games involving a strategy. In
a game of chess, while choosing what action is to be taken, a player tries to guess how his/her
opponent will react to the various actions he or she might take. In contrast, the card-pastime,
‘patience’ or ‘solitaire’ is played only against chance.

Which one of the following can best be described as a ‘game’?


a. The team of Tenzing Norgay and Edmund Hillary climbing Mt. Everest for the first time in human
history.
b. A national level essay writing competition.
c. A decisive war between the armed forces of India and Pakistan over Kashmir.
d. Oil Exporters’ Union deciding on world oil prices, completely disregarding the countries which
have at most minimal oil production.

Page 34 FLT – 06
Directions for questions 121 to 125: Read each of the five problems given below and choose the best
answer from among the four given choices.

121. Persons X, Y, Z and Q live in red, green, yellow or blue-coloured houses placed in a sequence on a
street. Z lives in a yellow house. The green house is adjacent to the blue house. X does not live
adjacent to Z. The yellow house is in between the green and red houses. The colour of the house, X
lives in is
a. blue b. green
c. red d. Not possible to determine

122. My bag can carry not more than ten books. I must carry at least one book each of management,
mathematics, physics and fiction. Also, for every management book I carry I must carry two or more
fiction books, and for every mathematics book I carry I must carry two or more physics books. I earn
4, 3, 2 and 1 points for each management, mathematics, physics and fiction book, respectively, I
carry in my bag. I want to maximise the points I can earn by carrying the most appropriate combination
of books in my bag. The maximum points that I can earn is
a. 20 b. 21 c. 22 d. 23

123. Five persons with names P, M, U, T and X live separately in any one of the following: a palace, a hut,
a fort, a house or a hotel. Each one likes two different colours from among the following: blue, black,
red, yellow and green. U likes red and blue. T likes black. The person living in a palace does not like
black or blue. P likes blue and red. M likes yellow. X lives in a hotel. M lives in a
a. hut b. palace c. fort d. house

124. There are ten animals — two each of lions, panthers, bison, bears, and deer — in a zoo. The
enclosures in the zoo are named X, Y, Z, P and Q and each enclosure is allotted to one of the
following attendants: Jack, Mohan, Shalini, Suman and Rita. Two animals of different species are
housed in each enclosure. A lion and a deer cannot be together. A panther cannot be with either a
deer or a bison. Suman attends to animals from among bison, deer, bear and panther only. Mohan
attends to a lion and a panther. Jack does not attend to deer, lion or bison. X, Y and Z are allotted to
Mohan, Jack and Rita respectively. X and Q enclosures have one animal of the same species. Z and
P have the same pair of animals. The animals attended by Shalini are
a. bear and bison b. bison and deer c. bear and lion d. bear and panther

125. Eighty kilogram of store material is to be transported to a location 10 km away. Any number of
couriers can be used to transport the material. The material can be packed in any number of units
of 10, 20, or 40 kg. Courier charges are Rs. 10 per hour. Couriers travel at the speed of 10 km/hr if
they are not carrying any load, at 5 km/hr if carrying 10 kg, at 2 km/hr if carrying 20 kg and at
1 km/hr if carrying 40 kg. A courier cannot carry more than 40 kg of load. The minimum cost at which
80 kg of store material can be transported to its destination will be
a. Rs. 180 b. Rs. 160 c. Rs. 140 d. Rs. 120

FLT – 06 Page 35
Directions for questions 126 to 130: Answer these questions with reference to the table given below.

Information Technology Industry in India


(Figures are in million US dollars)

1994-95 1995-96 1996-97 1997-98 1998-99

Software

Domestic 350 490 670 950 1250

Exports 485 734 1083 1750 2650

Hardware

Domestic 590 1037 1050 1205 1026

Exports 177 35 286 201 4

Peripherals

Domestic 148 196 181 229 329

Exports 6 6 14 19 18

Training 107 143 185 263 302

Maintenance 142 172 182 221 236

Networking and
36 73 156 193 237
others

Total 2041 2886 3807 5031 6052

126. The total annual exports lies between 35 and 40% to the total annual business of the IT industry, in
a. 1997-98 and 1994-95 b. 1996-97 and 1997-98
c. 1996-97 and 1998-99 d. 1996-97 and 1994-95

127. The highest percentage growth in the total IT business, relative to the previous year was achieved in
a. 1995-96 b. 1996-97
c. 1997-98 d. 1998-99

128. Which one of the following statements is correct?


a. The annual software exports steadily increased but annual hardware exports steadily declined
during 1994-99.
b. The annual peripheral exports steadily increased during 1994-99.
c. The IT business in training during 1994-99 was higher than the total IT business in maintenance
during the same period.
d. None of the above

Page 36 FLT – 06
Additional directions for questions 129 and 130:
For any activity, A, year X dominates year Y if the IT business in activity A, in the year X is greater than the
IT business in activity A in the year Y. For any two IT business activities, A and B, year X dominates year
Y if
I. the IT business in activity A, in the year X, is greater than or equal to the IT business in activity A in
the year Y,
II. the IT business in activity B, in the year X, is greater than or equal to the IT business in activity B in
the year Y and
III. there should be strict inequality in the case of at least one activity.

129. For the IT hardware business activity, which one of the following is not true?
a. 1997-98 dominates 1996-97 b. 1997-98 dominates 1995-96
c. 1995-96 dominates 1998-99 d. 1998-99 dominates 1996-97

130. For the two IT business activities, hardware and peripherals, which one of the following is true?
a. 1996-97 dominates 1995-96 b. 1998-99 dominates 1995-96
c. 1997-98 dominates 1998-99 d. None of these

Directions for questions 131 to 140: Each question is followed by two statements, I and II. Answer each
question using the following instructions.

Mark the answer as


a. if the question can be answered by one of the statements alone, but cannot be answered by
using the other statement alone.
b. if the question can be answered by using either statement alone.
c. if the question can be answered by using both the statements together, but cannot be answered
by using either statement alone.
d. if the question cannot be answered even by using both statements together.

131. Consider three real numbers, X, Y and Z. Is Z the smallest of these numbers?
I. X is greater than at least one of Y and Z.
II. Y is greater than at least one of X and Z.

132. Let X be a real number. Is the modulus of X necessarily less than 3?


I. X(X + 3) < 0
II. X(X – 3) > 0

133. How many people are watching TV programme P?


I. Number of people watching TV programme Q is 1,000 and number of people watching both the
programmes P and Q, is 100.
II. Number of people watching either P or Q or both is 1,500.

134. ∆PQR has ∠PRQ = 90°. What is the value of PR + RQ?


I. Diameter of the inscribed circle of the ∆PQR is equal to 10 cm.
II. Diameter of the circumscribed circle of the ∆PQR is equal to 18 cm.

FLT – 06 Page 37
135. Harshad bought shares of a company on a certain day, and sold them the next day. While buying
and selling he had to pay to the broker 1% of the transaction value of the shares as brokerage. What
was the profit earned by him per rupee spent on buying the shares?
I. The sales price per share was 1.05 times that of its purchase price.
II. The number of shares purchased was 100.

136. For any two real numbers:


a ⊕ b = 1 if both a and b are positive or both a and b are negative.
= –1 if one of the two numbers a and b is positive and the other negative.
What is (2 ⊕ 0) ⊕ (–5 ⊕ –6)?
I. a ⊕ b is zero if a is zero
II. a ⊕ b = b ⊕ a

137. There are two straight lines in the x-y plane with equations:
ax + by = c
dx + ey = f
Do the two straight lines intersect?
I. a, b, c, d, e and f are distinct real numbers.
II. c and f are non-zero.

138. ‘O’ is the centre of two concentric circles, AE is a chord of the outer circle and it intersects the inner
circle at points ‘B’ and ‘D’. ‘C’ is a point on the chord in between ‘B’ and ‘D’.
What is the value of AC/CE?
I. BC/CD = 1
II. A third circle intersects the inner circle at ‘B’ and ‘D’ and the point ‘C’ is on the line joining the
centres of the third circle and the inner circle.

139. Ghosh Babu has decided to take a non-stop flight from Mumbai to No-man’s-land in South America,
He is scheduled to leave Mumbai at 5 a.m., IST on December 10, 2000. What is the local time at
No-man’s-land when he reaches there?
I. The average speed of the plane is 700 km/hr.
II. The flight distance is 10,500 km.

140. What are the ages of two individuals, X and Y?


I. The age difference between them is 6 years.
II. The product of their ages is divisible by 6.

Page 38 FLT – 06
Directions for questions 141 to 145: Answer these questions based on the data provided in the table
below.
Factory Sector by Type of Ownership
All figures in the table are in percentage of the total for the corresponding column

Fi xed Gross Value


Sector Factori es Employment
capi tal output added

Publi c: 7 27.7 43.2 25.8 30.8

C entral Government 1 10.5 17.5 12.7 14.1

States or local
5.2 16.2 24.3 11.6 14.9
government

C entral and state or local


0.8 1.0 1.4 1.5 1.8
government

Joi nt: 1.8 5.1 6.8 8.4 8.1

Wholly pri vate 90.3 64.6 46.8 63.8 58.7

Others 0.9 2.6 3.2 2.0 2.4

Total 100 100 100 100 100

141. Suppose the average employment level is 60 per factory. The average employment in ‘wholly private’
factories is approximately
a. 43 b. 47 c. 50 d. 54

142. Among the firms in different sectors, value added per employee is highest in
a. Central Government b. Central and State/Local Governments
c. Joint sector d. Wholly private

143. Capital productivity is defined as the gross output value per rupee of fixed capital. The three sectors
with the higher capital productivity, arranged in descending order are
a. Joint, Wholly private, Central and States/Local
b. Wholly private, Joint, Central and States/Local
c. Wholly private, Central and State/Local, Joint
d. Joint, Wholly private, Central

144. A sector is considered ‘pareto efficient’ if its value added per employee and its value added per rupee
of fixed capital is higher than those of all other sectors. Based on the table data, the pareto efficient
sector is
a. Wholly private b. Joint
c. Central and State/Local d. others

145. The total value added in all sectors is estimated at Rs. 1,40,000 crore. Suppose the number of firms
in the joint sector is 2,700. The average value added per factory, in the Central Government is
a. Rs. 141 crore b. Rs. 14.1 crore c. Rs. 131 crore d. Rs. 13.1 crore

FLT – 06 Page 39
Directions for questions 146 to 149: Answer these questions based on the data presented in the figure
below.
FEI for a country in a year, is the ratio (expressed as a percentage) of its foreign equity inflows to its GDP.
The following figure displays the FEIs for select Asian countries for 1997 and 1998.

12 10.67
9.92
10

8
5.96 5.82
6 4.80 5.09

4
2.16 2.50
1.71
2
0.72
0
India China Malaysia South Korea Thailand

1997 1998

146. The country with the highest percentage change in FEI in 1998 relative to its FEI in 1997, is
a. India b. China c. Malaysia d. Thailand

147. Based on the data provided, it can be concluded that


a. absolute value of foreign equity inflows in 1998 was higher than that in 1997 for both Thailand and
South Korea.
b. absolute value of foreign equity inflows was higher in 1998 for Thailand and lower for China than
the corresponding values in 1997.
c. absolute value of foreign equity inflows was lower in 1998 for both India and China than the
corresponding values in 1997.
d. None of the above can be inferred

148. It is known that China’s GDP in 1998 was 7% higher than its value in 1997, while India’s GDP grew
by 2% during the same period. The GDP of South Korea, on the other hand, fell by 5%. Which of the
following statements is/are true?
I. Foreign equity inflows to China were higher in 1998 than in 1997.
II. Foreign equity inflows to China were lower in 1998 than in 1997.
III. Foreign equity inflows to India were higher in 1998 than in 1997.
IV. Foreign equity inflows to South Korea decreased in 1998 relative to 1997.
V. Foreign equity inflows to South Korea increased in 1998 relative to 1997.
a. I, III and IV b. II, III and IV c. I, III and V d. II and v

Page 40 FLT – 06
149. China’s foreign equity inflows in 1998 were 10 times that of India. It can be concluded that
a. China’s GDP in 1998 was 40% higher than that of India
b. China’s GDP in 1998 was 70% higher than that of India
c. China’s GDP in 1998 was 50% higher than that of India
d. no inference can be drawn about relative magnitudes of China’s and India’s GDPs

Directions for questions 150 to 153: Answer the questions based on the table below.
The table shows trends in external transactions of Indian corporate sector during the period 1993-94 to
1997-98. In addition, following definitions hold good:

Salesi, Importsi, and Exportsi respectively denote the sales, imports and exports in year i.
Deficit for year i, Deficiti = Importsi – Exportsi
Deficit Intensity in year i, DIi = Deficiti / Salesi
Growth rate of deficit intensity in year i, GDIi = (DIi – DIi – 1 ) / DIi – 1
Further, note that all imports are classified as either raw material or capital goods.

Trends in External Transactions of Indian Corporate Sector


(All figures in per cent)

Year 1997-98 1996-97 1995-96 1994-95 1993-94

Export intensity* 9.2 8.2 7.9 7.5 7.3

Import intensity* 14.2 16.2 15.5 13.8 12.4

Imported raw material/Total cost


20.2 19.2 17.6 16.3 16
of raw material

Imported capital goods/Gross


17.6 9.8 11.8 16.3 19.5
fixed assets

* Ratio of Exports (or Imports) to sales.

150. The highest growth rate in deficit intensity was recorded in


a. 1994-95 b. 1995-96 c. 1996-97 d. 1997-98

151. The value of the highest growth rate in deficit intensity is approximately
a. 8.45% b. 2.15% c. 33.3% d. 23.5%

152. In 1997-98 the total cost of raw materials is estimated as 50% of sales of that year. The turnover of
gross fixed assets, defined as the ratio of sales to gross fixed assets, in 1997-98 is, approximately
a. 3.3 b. 4.3
c. 0.33 d. Not possible to determine

153. Which of the following statements can be inferred to be true from the given data?
a. During the 5-year period between 1993-94 and 1997-98 exports have increased every year.
b. During the 5-year period 1993-94 and 1997-98, imports have decreased every year.
c. Deficit in 1997-98 was lower than that in 1993-94.
d. Deficit intensity has increased every year between 1993-94 and 1996-97.

FLT – 06 Page 41
Directions for questions 154 to 159: Answer the questions based on the data given below.
The figures below present annual growth rate, expressed as the percentage change relative to the previous
year, in four sectors of the economy of the Republic of Reposia during the 9-year period from 1990 to 1998.
Assume that the index of production for each of the four sectors is set at 100 in 1989. Further, the four
sectors: manufacturing, mining and quarrying, electricity, and chemicals, respectively, constituted 20%,
15%, 10% and 15% of total industrial production in 1989.

Manufacturing

15

10

0
1990 1991 1992 1993 1994 1995 1996 1997 1998
-5

Mining and quarrying

10

0
1990 1991 1992 1993 1994 1995 1996 1997 1998
-5

Electrical

10
8
6
4
2
0
1990 1991 1992 1993 1994 1995 1996 1997 1998

Chem ical

15

10

0
1990 1991 1992 1993 1994 1995 1996 1997 1998

Page 42 FLT – 06
154. Which is the sector with the highest growth during the period 1989 to 1998?
a. Manufacturing b. Mining and quarrying
c. Electricity d. Chemical

155. The overall growth rate in 1991 of the four sectors together is approximately
a. 10% b. 1% c. 2.5% d. 1.5%

156. When was the highest level of production in the manufacturing sector achieved during the 9-year
period 1990-98?
a. 1998 b. 1995 c. 1990 d. Cannot be determined

157. When was the lowest level of production of the mining and quarrying sector achieved during the
9-year period 1990-98?
a. 1996 b. 1993 c. 1990 d. Cannot be determined

158. The percentage increase of production in the four sectors, namely, manufacturing, mining and
quarrying, electricity and chemicals, taken together in 1994, relative to 1989 is approximately
a. 25% b. 20% c. 50% d. 40%

159. It is known that the index of total industrial production in 1994 was 50% more than that in 1989.
Then, the percentage increase in production between 1989 and 1994 in sectors other than the four
listed above is
a. 57.5% b. 87.5% c. 127.5% d. 47.5%

Directions for questions 160 to 165: Answer the questions based on the following information.
ABC Ltd. produces widgets for which the demand is unlimited and they can sell all of their production. The
graph below describes the monthly variable costs incurred by the company as a function of the quantity
produced. In addition, operating the plant for the first shift results in a fixed monthly cost of Rs. 800. Fixed
monthly costs for second shift operation is estimated at Rs. 1,200. Each shift operation provides capacity
for producing 30 widgets per month.
Variable cost
700 0

600 0

500 0

400 0

300 0

200 0

100 0

0
1 6 11 16 21 26 31 36 41 46 51 56

FLT – 06 Page 43
Total monthly costs
Note: Average unit cost, AC = and marginal cost, MC is the rate of change in total
Monthly production
cost for unit change in quantity produced.

160. Total production in July is 40 units. What is the approximate average unit cost for July?
a. 3,600 b. 90
c. 140 d. 115

161. ABC Ltd. is considering increasing the production level. What is the approximate marginal cost of
increasing production from its July level of 40 units?
a. 110 b. 130
c. 150 d. 160

162. From the data provided it can be inferred that, for production levels in the range of 0 to 60 units,
a. MC is an increasing function of production quantity
b. MC is a decreasing function of production quantity
c. initially MC is a decreasing function of production quantity, attains a minimum and then it is an
increasing function of production quantity
d. None of the above

163. Suppose that each widget sells for Rs. 150. What is the profit earned by ABC Ltd. in July?
(Profit is defined as the excess of sales revenue over total cost.)
a. 2,400 b. 1,600
c. 400 d. 0

164. Assume that the unit price is Rs. 150 and profit is defined as the excess of sales revenue over total
costs. What is the monthly production level of ABC Ltd. at which the profit is highest?
a. 30 b. 50 c. 60 d. 40

165. For monthly production level in the range of 0 to 30 units,


a. AC is always higher than MC
b. AC is always lower than MC
c. AC is lower than MC up to a certain level and then is higher than MC.
d. None of the above

Page 44 FLT – 06
PAST CAT PAPER – 2002

ANSWERS and EXPLANATIONS

1 3 2 4 3 4 4 3 5 3 6 1 7 1 8 3 9 3 10 3
11 2 12 4 13 1 14 4 15 1 16 1 17 1 18 4 19 2 20 3
21 3 22 3 23 4 24 1 25 3 26 3 27 2 28 4 29 2 30 2
31 2 32 4 33 4 34 2 35 2 36 2 37 1 38 2 39 1 40 4
41 2 42 2 43 2 44 4 45 3 46 1 47 3 48 2 49 4 50 3
51 3 52 2 53 1 54 2 55 1 56 4 57 4 58 2 59 4 60 4
61 3 62 2 63 4 64 4 65 3 66 2 67 2 68 4 69 3 70 1
71 3 72 2 73 3 74 4 75 4 76 1 77 1 78 4 79 2 80 4
81 4 82 3 83 4 84 3 85 1 86 3 87 1 88 3 89 2 90 2
91 4 92 4 93 4 94 3 95 4 96 2 97 2 98 4 99 3 100 3
101 3 102 2 103 4 104 2 105 4 106 3 107 1 108 3 109 4 110 2
111 3 112 1 113 4 114 3 115 1 116 4 117 3 118 2 119 3 120 2
121 1 122 4 123 1 124 3 125 1 126 3 127 2 128 3 129 4 130 1
131 1 132 4 133 4 134 4 135 2 136 4 137 2 138 2 139 4 140 4
141 4 142 1 143 1 144 2 145 1 146 3 147 4 148 3 149 1 150 3

Scoring table

Section Question Total Total Total Total Net Time


number questions attempted correct incorrect score taken
DI 1 to 50 50

QA 51 to 100 50

EU + RC 101 to 150 50

Total 150

PAST CAT PAPER – 2002 Explanations Page 1


1. 3 Statement I tells us that 7. 1
(1) Ashish is not an engineer, (2) Ashish got more N
offers than the engineers.
Hence, Ashish did not have 0 offers. W E
After this the following table can be achieved.

Profession Names Offers S


FINISH Vth Sig nal
3 2 1 0 X Profession
@ 4 0 km ph
CA Ashish × × P × X Engineer @ 1 00 km p h
t = 1 5 m inu te s
t = 2 4 m inu te s 1 0 km
MD Dhanraj P × × × X Engineer s = 40 km ∴ s = 10 km
P
IIIrd
Economist Sameer × × × Sig nal IVT H
P
Engineer × × × @ 4 0 km ph 4 0 km
t = 3 0 m inu te s Sig nal

From statement IV, Dhanraj is not at 0 and 1. ∴ s = 20 km


2 0 km @ 4 0 km ph
2. 4 Option (3) is ruled out by statement VII. t = 1 5 m inu te s
Option (1) is ruled out by statements VII and VIII. ∴ s = 10 km
From statement IV, Sandeep had Rs. 30 to start and I Sig nal
1 0 km
Daljeet Rs. 20. IInd M oves @ 2 0 km p h,
From statement II, option (2) is not possible as Sandeep Sig nal 1 0 km t = ½ h r = 30 m in utes
was left with Re 1, he spent Rs. 29. But according to ∴ s = 20 × 3 0 = 1 0 km
(2) he spent Rs. 1.50 more than Daljeet. But Daljeet 60
S
had only Rs. 20. Hence option (4) is correct.
START
3. 4 Data insufficient, please check the question. Note: s = Distance covered; v = Velocity (km/hr)
t = Time taken; s = v × t
4. 3 Statements V and VI rule out options (1) and (2). Since The total distance travelled by the motorist from the starting
contestants from Bangalore and Pune did not come point till last signal is 10 + 10 + 20 + 40 + 10 = 90 km
first, school from Hyderabad can come first. Convent
is not in Hyderabad which rules out option (4). 8. 3
N
5. 3 The only two combinations possible are:
Younger Older W E
2 4 3 0 km
3 9 F S
T
Cubes of natural numbers are 1, 8, 27, 64, ... . Here 64
and above is not possible as the age will go above 10.
1 0 km
Only (2) and (3) satisfy the case of mother and father.
III
6. 1 Total seats in the hall 200 4 0 km IV
Seats vacant 20 4 0 km
Total waiting 180 2 0 km
Ladies 72
2 I
Seating capacity of flight × 180 = 120 II 1 0 km
3
Number of people in flight A = 100 1 0 km
For flight B = 180 – 100 = 80
80 S
Thus, airhostess for A = =4
20
Empty seats in flight B = 120 – 80 = 40
40 : 4 = 10 : 1

Page 2 PAST CAT PAPER – 2002 Explanations


Note: According to Pythagoras’ theorem, for a right- 10. 3 If the car heads towards south from here starting
angled triangle. point,
N
B

W E
A C
BC2 = AB2 + AC2 S S
S TA R T
BC = AB2 + AC2
1 0 km
SF = ST 2 + TF2 = 402 + 302 = 2500 = 50 km

I 1 0 km II
9. 3 For the case when 1st signal were 1 red and 2 green
lights, the surface diagram will be as given below. 4 0 km
N 2 0 km

W E 3 0 km
IV
F S 4 0 km III
5 0 km 1 0 km
T

1 0 km F
III FIN IS H
IV From the above we can conclude that option (3) is
4 0 km 4 0 km
correct.

2 0 km 11. 2 Total five lie between 10 E and 40 E.


Austria, Bulgaria, Libya, Poland, Zambia
I N N N N S
II
1 0 km 1
= 20%
5
1 0 km

12. 4 Number of cities starting with consonant and in the


S northern hemisphere = 10.
TF = 50 km; ST = 40 km Number of countries starting with consonant and in
Considering the above figure, option (3) is correct, the east of the meridien = 13.
50 km to the east and 40 km to the north. Hence option (4), the differencce is 3.

13. 1 Three countries starting with vowels and in southern


hemisphere — Argentina. Australia and Ecuador and
two countries with capitals beginning with vowels —
Canada and Ghana.

14. 4 Let us consider two cases:


(a) If 5 min remaining the score was 0 – 2. Then final
score could have been 3 – 3. [Assuming no other
Indian scored]
(b) But if the score before 5 min was 1 – 3, then final
score could have been 4 – 3.

15. 1 From A, if by adding 12 students, the total number of


students is divisible by 8, then by adding 4 students,
it will be divisible by 8.

PAST CAT PAPER – 2002 Explanations Page 3


23. 4 80% attendance = 80% of 25 = 20 days
 1 1 y + x Emp. numbers 47, 51, 72, 73, 74, 79, 80.
16. 1 From (A), (x + y)  x + y  = 4 or (x + y)  xy  = 4
    Thus, total = 7
⇒ (x + y)2 = 4xy
⇒ (x – y)2 = 0 24. 1 Em p. No. Earnings No. of days E/D
⇒x=y ... (i) E D
From (B), (x – 50)2 = (y – 50)2 (m edium ) (m edium )
On solving
2001151 159.64 13.33 11.97
x(x – 100) = y(y – 100) ... (ii)
This suggests that the values of x and y can either be 2001158 109.72 9.61 11.41
0 or 100. 2001164 735.22 12.07 60.91
2001171 6.10 4.25 -
17. 1 Statement:
A. Let the wholesale price is x. 2001172 117.46 8.50 13.81
Thus, listed prices = 1.2x 2001179 776.19 19.00 40.85
After a discount of 10%, new price = 0.9 × 1.2x 2001180 1262.79 19.00 66.46
= 1.08x
1.08 – x = 10$. Thus, we know x can be found.
B. We do not know at what percentage profit, or at From the above we can see that Emp. number 2001180
what amount of profit was the dress actually earns the maximum per day salary.
sold.
25. 3 If you have practised Vedic mathematics you can easily
18. 4 A gives 500 as median and B gives 600 as range. compare the fraction in your time. Thus, Emp. numbers
A and B together do not give average. Therefore, it 51, 58, 64, 71, 72 satisfy the condition.
cannot be answered from the given statements. [For emp. 64, you see 12 is not the double of 5. And
735 is not even double of 402.
19. 2 From statement A, we know that for all –1 < x < 1,
we can determine |x – 2| < 1 is not true. 402 735
Hence, > (3)
From B, –1 < x < 3, we cannot determine whether 5 12
|x – 2| < 1 or not. Therefore, statement B alone is
sufficient. 26. 3 Total revenue of 1999 = 3374
5
20. 3 From statement A, we cannot find anything. 5% of 3374 = 3374 × = 168.7
From B alone we cannot find. 100
From A and B, For 1999, revenue for Spain is 55, Rest of Latin
America is 115, North Sea is 140, Rest of the world is
3 00 91.
R So total four operations of the company accounted
F
for less than 5% of the total revenue earned in the
X 196 58 year 1999.

27. 2 For man the 200% growth. The value should be more
x + 196 + 58 = 300. Thus, x can be found. than three times.

21. 3 Jagdish (J), Punit (P), Girish (G) 28. 4 Four operations, as given below:
(1) North Africa and Middle-East
2 (2) Argentina
(A) J = [P + G]
9 (3) Rest of Latin America
P + G + J = 38500 (4) Far East
Thus, only J can be found. have registered yearly increase in income before taxes
(B) Similarly, from this only P can be found. and charges from 1998 to 2000.
Combining we know J, P and G can be found.
29. 2 Percentage increase in net income before tax and
22. 3 Emp. numbers 51, 58, 64, 72, 73 satisfy them. charges for total world (1998-99)
Total = 5 1375 − 248
= × 100 = 454.4%
248
Spain is making loss.

Page 4 PAST CAT PAPER – 2002 Explanations


Percentage increase for North Africa and Middle-East 36. 2 BC → AC → AAC = 0
341 − 111
× 100 = 207.2% 37. 1 BD → AE → AAB = 95.2
111 The least cost to reach to AAB is for AE. And that is
838 − 94 BD to AE is zero.
Percentage increase for Argentina = × 100
94
= 791.5% 38. 2 BB → AB → AAG = 311.1
From the table one can directly say that there is no Same as above.
operation other than Argentina, whose percentage
increase in net income before taxes and charges is 39. 1 First you check the minimum cost for receiving at AAA.
higher than the average (world). This is O for AE. But BB to AE is very high. Next is AC
[314.5] BB to AC is 451.1. After AC the others are
30. 2 Statement 1 is obviously wrong. high. Hence, 314.5 + 451.1 = 765.6
54 20
(b) > . Hence, (2) is correct. 40. 4 Number of refineries = 6
65 52 Number of depots = 7
500 61 Number of districts = 9
(c) > . Hence (3) is wrong.
1168 187 Therefore, number of possible ways to send petrol
from any refinery to any district is 6 × 7 × 9 = 378.
31. 2 Profitability of North Africa and Middle-East in 2000
356 41. 2 If you look for large figures you would find them in
= = 0.67 both tables in AE. Add them and we get
530
225 BE → AE → AAH
Profitability of Spain in 2000 = = 5.23
43
42. 2
169
Profitability of Rest of Latin America in 2000 = ,
252 Position
i.e. < 1.
of
189 Year
Profitability of Far East in 2000 = =<1 States
311
(Rank)
32. 4 Except Rest of Latin America and Rest of the World all 96-97 97-98 98-99 99-00 00-01
the operations are greater than 2. 1) mA mA mA mA mA
33. 4 I would rate this question as time-taking because to 2) TN TN TN TN TN
verify option (4) it takes time. But if you have ruled out 3) GU AP AP AP AP
options (1), (2) and (3), then you can straight away 4) AP GU GU GU UP changed
mark (4).
5) kA UP UP UP GU } tw ice
34. 2 It can be easily observed from the two charts that 6) UP kA kA kA kA
20 7) WB WB WB WB WB
Switzerland’s ratio of chart 1 to chart 2 is has the
11 From above, we can conclude that option (2) is correct.
highest price per unit kilogram for its supply. Finding
the ratio of the value and quantity is enough to reach 43. 2 On referring to the above table, we can see that UP is
the solution. the state which changed its relative ranking most
number of times.
35. 2 Total value of distribution to Turkey is 16% of 5760
million Euro. 44. 4 We can say directly on observing the graph that the
Total quantity of distribution to Turkey is 15% of 1.055 sales tax revenue collections for AP has more than
million tonnes. doubled from 1997 to 2001.
So the average price in Euro per kilogram for Turkey is

 16 
 5760 × 100 
  ; 5.6
 15 
 1055 × 100 
 

PAST CAT PAPER – 2002 Explanations Page 5


45. 3 Growth rate of tax revenue can be calculated as: 52. 2
(Sales tax revenue of correct year – Sales tax revenue
of previous year) A
7826 − 7290 3 0° 3 0°
For year 1997-98 = 0.068
7826 y
4 3
8067 − 7826
For year 1998-99 = 0.030
7826
10284 − 8067 B D C
For year 1999-2000 = 0.274
8067 Let BC = x and AD = y.
12034 − 10284 BD AB 4
For year 2000-01 = 0.170 As per bisector theorem, = =
10284 DC AC 3
4x 3x
46. 1 For this we have to check every option. Hence, BD = ; DC =
7 7
Sales tax revenue of the states 16x 2
For Karnataka, (4)2 + y 2 –
total sales tax 49
In ∆ABD, cos30° =
2× 4× y
3510
1996-97 ⇒ = 0.11
29870 3 16x 2
⇒ 2× 4× y× = 16 + y 2 –
2 49
3829
1997-98 ⇒ = 0.11
33168 16x 2
⇒ 4 3y = 16 + y 2 – ... (i)
4265 49
1998-99 ⇒ = 0.11
36068
9x 2
9 + y2 –
4839 49
1999-2000 ⇒ = 0.11 Similarly, from ∆ADC, cos30° =
42348 2×3× y

5413 9x 2
2000-01 ⇒
49638
= 0.11 ⇒ 3 3y = 9 + y 2 – ... (ii)
49
So Karnataka is the correct option and no further check Now (i) × 9 – 16 × (ii), we get
is required.
12 3
36 3y – 48 3y = 9y 2 – 16y 2 ⇒ y =
47. 3 On referring to the table prepared in the solution for 7
question (11), we can see that Tamil Nadu has been 53. 1
maintaining a constant rank over the years in terms of
its contribution to total tax collections. C
15 20
48. 2 Only R9
A B
49. 4 Statement (1) is not satisfied by R9. 25
Statement (2) is not satisfied R2 & R3
Statement (3) is incorrect as there are five such region
R1, R2, R3, R4 & R11. Let the chord = x cm
Statement (4) is correct.
1 1 x
∴ (15 × 20) = × 25 × ⇒ x = 24 cm
50. 3 All three R9, R10, R11. 2 2 2

51. 3 Total possible arrangements = 10 × 9 × 8


Now 3 numbers can be arranged among themselves  1+ x   1+ y 
54. 2 f(x) + f(y) = log  1– x  + log  1– y 
in 3! ways = 6 ways    
Given condition is satisfied by only 1 out of 6 ways.
Hence, the required number of arrangements  (1 + x) (1 + y) 
= log  
10 × 9 × 8  (1– x)(1– y) 
= = 120
6
 1 + x + y + xy 
= log  
 1 + xy – x – y 

Page 6 PAST CAT PAPER – 2002 Explanations


Section – III

Directions for questions 101 to 104: Answer the questions based on the table given below.
The following table describes garments manufactured based upon the colour and size for each lay. There
are four sizes: M – medium, L – large, XL – extra large and XXL – extra extra large. There are three colours:
yellow, red and white.

Lay Num ber of Garm ents


Yellow Red White
Lay No. M L XL XXL M L XL XXL M L XL XXL
1 14 14 7 0 0 0 0 0 0 0 0 0
2 0 0 0 0 0 0 0 0 42 42 21 0
3 20 20 10 0 18 18 9 0 0 0 0 0
4 20 20 10 0 0 0 0 0 30 30 15 0
5 0 0 0 0 24 24 12 0 30 30 15 0
6 22 22 11 0 24 24 12 0 32 32 16 0
7 0 24 24 12 0 0 0 0 0 0 0 0
8 0 20 20 10 0 2 2 1 0 0 0 0
9 0 20 20 10 0 0 0 0 0 22 22 11
10 0 0 0 0 0 26 26 13 0 20 20 10
11 0 22 22 11 0 26 26 13 0 22 22 11
12 0 0 2 2 0 0 0 0 0 0 0 0
13 0 0 0 0 0 0 0 0 0 0 20 20
14 0 0 0 0 0 0 0 0 0 0 22 22
15 0 0 10 10 0 0 2 2 0 0 22 22
16 0 0 0 0 1 0 0 0 1 0 0 0
17 0 0 0 0 0 5 0 0 0 0 0 0
18 0 0 0 0 0 32 0 0 0 0 0 0
19 0 0 0 0 0 32 0 0 0 0 0 0
20 0 0 0 0 0 5 0 0 0 0 0 0
21 0 0 0 18 0 0 0 0 0 0 0 0
22 0 0 0 0 0 0 0 26 0 0 0 0
23 0 0 0 0 0 0 0 0 0 0 0 22
24 0 0 0 8 0 0 0 1 0 0 0 0
25 0 0 0 8 0 0 0 0 0 0 0 12
26 0 0 0 0 0 0 0 1 0 0 0 14
27 0 0 0 8 0 0 0 2 0 0 0 12
Production 76 162 136 97 67 194 89 59 135 198 195 156
Order 75 162 135 97 67 194 89 59 135 197 195 155
Surplus 1 0 1 0 0 0 0 0 0 1 0 1

101. How many lays are used to produce yellow fabrics?


a. 10 b. 11 c. 12 d. 14

102. How many lays are used to produce XL fabrics?


a. 15 b. 16 c. 17 d. 18

103. How many lays are used to produce XL yellow or XL white fabrics?
a. 8 b. 9 c. 10 d. 15

FLT – 07 Page 25
104. How many varieties of fabrics, which exceed the order, have been produced?
a. 3 b. 4 c. 5 d. 6

Directions for questions 105 to 108: Answer the questions based on the table given below concerning the
busiest 20 international airports in the world.

International
No. Name Code Location Passengers
Airport Type

1 Hartsfield A ATL Atlanta, Georgia, USA 77939536

2 Chicago-O'Hare A ORD Chicago, Illinois, USA 72568076

3 Los Angeles A LA X Los Angeles, California, USA 63876561

4 Heathrow Airport E LHR London, United Kingdom 62263710

5 D FW A D FW Dallas/Ft. Worth, Texas, USA 60000125

6 Haneda Airport F HND Tokyo, Japan 54338212

7 Frankfurt Airport E FRA Frankfurt, Germany 45858315

8 Roissy-Charles de Gaulle E CDG Paris, France 43596943

9 San Francisco A S FO San Francisco, California, USA 40387422

10 Denver A DIA Denver, Colorado, USA 38034231

11 Amsterdam Schiphol E AMS Amsterdam, Netherlands 36781015

12 Minneapolis - St. Paul A MSP Minneapolis-St. Paul, USA 34216331

13 Detroit Metropolitan A DTW Detroit, Michigan, USA 34038381

14 Miami A MIA Miami, Florida, USA 33899246

15 Newark A EWR Newark, New Jersey, USA 33814000

16 McCarran A LA S Las Vegas, Nevada, USA 33669185

17 Phoenix Sky Harbor A PHX Phoenix, Arizona, USA 33533353

18 Kimpo FE SEL Seoul, Korea 33371074

19 George Bush A IAH Houston, Texas, USA 33089333

20 John F. Kennedy A JF K New York, New York, USA 32003000

105. How many international airports of type ‘A’ account for more than 40 million passengers?
a. 4 b. 5 c. 6 d. 7

106. What percentage of top ten busiest airports is in the United States of America?
a. 60% b. 80% c. 70% d. 90%

Page 26 FLT – 07
107. Of the five busiest airports, roughly, what percentage of passengers in handled by Heathrow Airport?
a. 30 b. 40 c. 20 d. 50

108. How many international airports not located in the USA handle more than 30 million passengers?
a. 5 b. 6 c. 10 d. 14

Directions for questions 109 to 114: Answer the questions based on the two graphs shown below.
Figure I shows the amount of work distribution, in man-hours, for a software company between offshore and
onsite activities. Figure 2 shows the estimated and actual work effort involved in the different offshore
activities in the same company during the same period. [Note: Onsite refers to work performed at the
customer’s premise and offshore refers to work performed at the developer’s premise.]

500

400

300
Offshore
Onsite
200

100

0
Design Coding Testing

Figure 1

500

400

300
Estimated
Actual
200

100

0
Design Coding Testing

Figure 2

109. Which work requires as many man-hours as that spent in coding?


a. Offshore, design and coding b. Offshore coding
c. Testing d. Offshore, testing and coding

FLT – 07 Page 27
110. Roughly, what percentage of the total work is carried out onsite?
a. 40% b. 20 %
c. 30 % d. 10 %

111. The total effort in man-hours spent onsite is nearest to which of the following?
a. The sum of the estimated and actual effort for offshore design
b. The estimated man-hours of offshore coding
c. The actual man-hours of offshore testing
d. Half of the man-hours of estimated offshore coding

112. If the total working hours were 100, which of the following tasks will account for approximately 50 hr?
a. Coding b. Design
c. Offshore testing d. Offshore testing plus design

113. If 50% of the offshore work were to be carried out onsite, with the distribution of effort between the
tasks remaining the same, the proportion of testing carried out offshore would be
a. 40% b. 30% c. 50% d. 70%

114. If 50% of the offshore work were to be carried out onsite, with the distribution of effort between the
tasks remaining the same, which of the following is true of all work carried out onsite?
a. The amount of coding done is greater than that of testing
b. The amount of coding done onsite is less than that of design done onsite
c. The amount of design carried out onsite is greater than that of testing
d. The amount of testing carried out offshore is greater than that of total design

Directions for questions 115 to 117: Answer the questions based on the pipeline diagram below.

The following sketch shows the pipelines carrying material from one location to another. Each location has
a demand for material. The demand at Vaishali is 400, at Jyotishmati is 400, at Panchal is 700, and at
Vidisha is 200. Each arrow indicates the direction of material flow through the pipeline. The flow from
Vaishali to Jyotishmati is 300. The quantity of material flow is such that the demands at all these locations
are exactly met. The capacity of each pipeline is 1,000.

Vaishali Jyotishm a ti P anchal

Avanti

Vid isha
115. The quantity moved from Avanti to Vidisha is
a. 200 b. 800 c. 700 d. 1,000

116. The free capacity available at the Avanti-Vaishali pipeline is


a. 0 b. 100 c. 200 d. 300

Page 28 FLT – 07
117. What is the free capacity available in the Avanti-Vidisha pipeline?
a. 300 b. 200 c. 100 d. 0

Directions for questions 118 to 120: Answer these questions based on the data given below:
There are six companies, 1 through 6. All of these companies use six operations, A through F. The
following graph shows the distribution of efforts put in by each company in these six operations.

100%
15.7 18.2 17.6
22.2 19.7
90% F F 23.4 F
F F
F

80%
23.5
21.8 23.6
70% E
E 28.6 E
25.9
% Distribution of Efforts

E 28.6 E
60% E
15.7 16.3 11.8
50% D 7.4 D 7.7 D
D D
11.2
9.8 9.3 13.8
40% 10.9 D 13
C C C
C 8.2 C

30% C
17.6 16.7
16.4 10.3 17.6
B B 16.1
B B B
20% B

10% 17.7 18.5 16.4 18.5 15.6


15.1
A A A A A
A

0%
1 2 3 4 5 6
Company

118. Suppose effort allocation is inter-changed between operations B and C, then C and D, and then D
and E. If companies are then ranked in ascending order of effort in E, what will be the rank of
company 3?
a. 2 b. 3 c. 4 d. 5

119. A new technology is introduced in company 4 such that the total effort for operations B through F
get evenly distributed among these. What is the change in the percentage of effort in operation E?
a. Reduction of 12.3 b. Increase of 12.3 c. Reduction of 5.6 d. Increase of 5.6

120. Suppose the companies find that they can remove operations B, C and D and redistribute the effort
released equally among the remaining operations. Then which operation will show the maximum
across all companies and all operations?
a. Operation E in company 1 b. Operation E in company 4
c. Operation F in company 5 d. Operation E in company 5

FLT – 07 Page 29
Directions for questions 121 to 127: Each question is followed by two statements, I and II.

Mark
a. if the question can be answered by one of the statements alone and not by the other.
b. if the question can be answered by using either statement alone.
c. if the question can be answered by using both the statements together, but cannot be answered
by using either statement alone.
d. if the question cannot be answered even by using both statements together.

121. What are the values of m and n?


I. n is an even integer, m is an add integer, and m is greater than n.
II. Product of m and n is 30.

122. Is Country X’s GDP higher than country Y’s GDP?


I. GDPs of the countries X and Y have grown over the past 5 years at compounded annual rate of
5% and 6% respectively.
II. Five years ago, GDP of country X was higher than that of country Y.

123. What is the value of X?


X
I. X and Y are unequal even integers, less than 10, and is an odd integer.
Y
II. X and Y are even integers, each less than 10, and product of X and Y is 12.

124. On a given day a boat ferried 1,500 passengers across the river in 12 hr. How many round
trips did it make?
I. The boat can carry 200 passengers at any time.
II. It takes 40 min each way and 20 min of waiting time at each terminal.

125. What will be the time for downloading software?


I. Transfer rate is 6 kilobytes per second.
II. The size of the software is 4.5 megabytes.

126. A square is inscribed in a circle. What is the difference between the area of the circle and that of
the square?
I. The diameter of the circle is 25 2 cm.
II. The side of the square is 25 cm.

127. Two friends, Ram and Gopal, bought apples from a wholesale dealer. How many apples did they
buy?
I. Ram bought one-half the number of apples that Gopal bought.
II. The wholesale dealer had a stock of 500 apples.

Page 30 FLT – 07
Directions for questions 128 to 130: Answer the questions based on the pie charts given below.
Chart 1 shows the distribution of 12 million tonnes of crude oil transported through different modes over a
specific period of time. Chart 2 shows the distribution of the cost of transporting this crude oil. The total
cost was Rs. 30 million.

Road Airfreight Road Airfreight


11%
Rail Ship
22% Ship 6% 7%
12%
9% 10%

Rail
9%

Pipeline
49% Pipeline
65%

Chart 1: Volume transported Chart 2: Cost of transportation

128. The cost in rupees per tonne of oil moved by rail and road happens to be roughly
a. Rs. 3 b. Rs. 1.5 c. Rs. 4.5 d. Rs. 8

129. From the charts given, it appears that the cheapest mode of transport is
a. road b. rail c. pipeline d. ship

130. If the costs per tonne of transport by ship, air and road are represented by P, Q and R respectively,
which of the following is true?
a. R > Q > P b. P > R > Q c. P > Q > R d. R > P > Q

Diretions for questions 131 to 134: Answer the questions independently.

131. At a village mela, the following six nautankis (plays) are scheduled as shown in the table below.

No. Nautanki Duration Show Times

1 Sati Savitri 1 hr 9 a.m. and 2 p.m.

2 Joru ka Ghulam 1 hr 10.30 a.m . and 11: 30 a.m.

3 Sundar Kand 30 min 10 am and 11 a.m.

4 Veer Abhimanyu 1 hr 10 a.m. and 11a.m.

5 Reshma aur Shera 1 hr 9.30 a.m., 12 noon and 2 p.m.

6 Jhansi ki Rani 30 min 11 a.m. and 1: 30 pm

You wish to see all the six nautankis. Further, you wish to ensure that you get a lunch break from
12.30 p.m. to 1.30 p.m. Which of the following ways can you do this?

FLT – 07 Page 31
a. Sati Savitri is viewed first; Sundar Kand is viewed third, and Jhansi ki Rani is viewed last
b. Sati Savitri is viewed last; Veer Abhimanyu is viewed third, and Reshma aur Shera is viewed first
c. Sati Savitri is viewed first; Sundar Kand is viewed third, and Joru ka Ghulam is viewed fourth
d. Veer Abhimanyu is viewed third; Reshma aur Shera is viewed fourth, and Jahansi ki Rani is
viewed fifth

132. Mrs Ranga has three children and has difficulty remembering their ages and months of their birth.
The clue below may help her remember.

. The boy, who was born in June, is 7 years old.


. One of the children is 4 years old but it was not Anshuman.
. Vaibhav is older than Suprita.
. One of the children was born in September, but it was not Vaibhav.
. Suprita’s birthday is in April.
. The youngest child is only 2-year-old.

Based on the above clues, which one of the following statements is true?
a. Vaibhav is the oldest, followed by Anshuman who was born in September, and the youngest is
Suprita who was born in April
b. Anshuman is the oldest being born in June, followed by Suprita who is 4-year-old, and the
youngest is Vaibhav who is 2-year-old
c. Vaibhav is the oldest being 7-year-old, followed by Suprita who was born in April, and the youngest
is Anshuman who was born in September
d. Suprita is the oldest who was born in April, followed by Vaibhav who was born in June, and
Anshuman who was born in September

133. The Bannerjees, the Sharmas, and the Pattabhiramans each have a tradition of eating Sunday
lunch as a family. Each family serves a special meal at a certain time of day. Each family has a
particular set of chinaware used for this meal. Use the clues below to answer the following
question.

. The Sharma family eats at noon.


. The family that serves fried brinjal uses blue chinaware.
. The Bannerjee family eats at 2 o’clock.
. The family that serves sambar does not use red chinaware.
. The family that eats at 1 o’clock serves fried brinjal.
. The Pattabhiraman family does not use white chinaware.
. The family that eats last likes makkai-ki-roti.

Which one of the following statements is true?


a. The Bannerjees eat makkai-ki-roti at 2 o’clock, the Sharmas eat fried brinjal at 12 o’clock and the
Pattabhiramans eat sambar from red chinaware
b. The Sharmas eat sambar served in white chinaware, the Pattabhiramans eat fried brinjal at
1 o’clock, and the Bannerjees eat makkai-ki-roti served in blue chinaware

Page 32 FLT – 07
c. The Sharmas eat sambar at noon, the Pattabhiramans eat fried brinjal served in blue chinaware,
and the Bannerjees eat makkai-ki-roti served in red chinaware
d. The Bannerjees eat makkai-ki-roti served in white chinaware, the Sharmas eat fried brinjal at
12 o’clock and the Pattabhiramans eat sambar from red chinaware

134. While Balbir had his back turned, a dog ran into his butcher shop, snatched a piece of meat off the
counter and ran out. Balbir was mad when he realised what had happened. He asked three other
shopkeepers, who had seen the dog, to describe it. The shopkeepers really did not want to help
Balbir. So each of them made a statement which contained one truth and one lie.

. Shopkeeper number 1 said: “The dog had black hair and a long tail.”
. Shopkeeper number 2 said: “The dog had a short tail and wore a collar.”
. Shopkeeper number 3 said: “The dog had white hair and no collar.”

Based on the above statements, which of the following could be a correct description?
a. The dog had white hair, short tail and no collar
b. The dog had white hair, long tail and a collar
c. The dog had black hair, long tail and a collar
d. The dog had black hair, long tail and no collar

Directions for questions 135 and 136: Answer the following questions based on the information given
below.
Elle is three times older than Yogesh. Zaheer is half the age of Wahida. Yogesh is older than Zaheer.

135. Which of the following can be inferred?


a. Yogesh is older than Wahida
b. Elle is older than Wahida
c. Elle may be younger than Wahida
d. None of these

136. Which of the following information will be sufficient to estimate Elle’s age?
a. Zaheer is 10-year-old
b. Both Yogesh and Wahida are older than Zaheer by the same number of years
c. Both (a) and (b)
d. None of these

Directions for questions 137 to 139: Answer the questions based on the passage below.

A group of three or four has to be selected from seven persons. Among the seven are two women: Fiza and
Kavita, and five men: Ram, Shyam, David, Peter and Rahim. Ram would not like to be in the group If
Shyam is also selected. Shyam and Rahim want to be selected together in the group. Kavita would like to
be in the group only if David is also there. David, if selected, would not like Peter in the group. Ram would
like to be in the group only if Peter is also there. David insists that Fiza be selected in case he is there in
the group.

FLT – 07 Page 33
137. Which of the following is a feasible group of three?
a. David, Ram and Rahim
b. Peter, Shyam and Rahim
c. Kavita, David and Shyam
d. Fiza, David and Ram

138. Which of the following is a feasible group in four?


a. Ram, Peter, Fiza and Rahim
b. Shyam, Rahim, Kavita and David
c. Shyam, Rahim, Fiza and David
d. Fiza, David, Ram and Peter

139. Which of the following statements is true?


a. Kavita and Ram can be part of a group of four
b. A group of four can have two women
c. A group of four can have all four men
d. None of these

Directions for questions 140 to 146: Answer the questions independently.

140. On her walk through the park, Hamsa collected 50 coloured leaves, all either maple or oak. She
sorted them by category when she got home, and found the following:
The number of red oak leaves with spots is even and positive.
The number of red oak leaves without any spot equals the number of red maple leaves without
spots.
All non-red oak leaves have spots, and there are five times as many of them as there are red spotted
oak leaves.
There are no spotted maple leaves that are not red.
There are exactly 6 red spotted maple leaves.
There are exactly 22 maple leaves that are neither spotted nor red.

How many oak leaves did she collect?


a. 22 b. 17 c. 25 d. 18

141. Eight people carrying food baskets are going for a picnic on motorcycles. Their names are A, B, C,
D, E, F, G, and H. They have 4 motorcycles M1, M2, M3 and M4 among them. They also have 4
food baskets O, P, Q and R of different sizes and shapes and each can be carried only on motorcycles
M1, M2, M3 and M4 respectively. No more than 2 persons can travel on a motorcycle and no more
than one basket can be carried on a motorcycle. There are 2 husband-wife pairs in this group of
8 people and each pair will ride on a motorcycle together. C cannot travel with A or B. E cannot travel
with B or F. G cannot travel with F, or H, or D. The husband-wife pairs must carry baskets O and P.
Q is with A and P is with D. F travels on M1 and E travels on M2 motorcycles. G is with Q, and B
cannot go with R. Who is travelling with H?
a. A b. B c. C d. D

Page 34 FLT – 07
142. In a family gathering there are 2 males who are grandfathers and 4 males who are fathers. In the
same gathering there are 2 females who are grandmothers and 4 females who are mothers. There is
at least one grandson or a granddaughter present in this gathering. There are 2 husband-wife pairs
in this group. These can either be a grandfather and a grandmother, or a father and a mother. The
single grandfather (whose wife is not present) has 2 grandsons and a son present. The single
grandmother (whose husband is not present) has 2 grand daughters and a daughter present. A
grandfather or a grandmother present with their spouses does not have any grandson or granddaughter
present.
What is the minimum number of people present in this gathering?
a. 10 b. 12 c. 14 d. 16

143. I have a total of Rs. 1,000. Item A costs Rs. 110, item B costs Rs. 90, item C costs Rs. 70, item D
costs Rs. 40 and item E costs Rs. 45. For every item D that I purchase, I must also buy two of item
B. For every item A, I must buy one of item C. For every item E, I must also buy two of item D and
one of item B. For every item purchased I earn 1,000 points and for every rupee not spent I earn a
penalty of 1,500 points. My objective is to maximise the points I earn.
What is the number of items that I must purchase to maximise my points?
a. 13 b. 14 c. 15 d. 16

144. Four friends Ashok, Bashir, Chirag and Deepak are out for shopping. Ashok has less money than
three times the amount that Bashir has. Chirag has more money than Bashir. Deepak has an
amount equal to the difference of amounts with Bashir and Chirag. Ashok has three times the
money with Deepak. They each have to buy at least one shirt, or one shawl, or one sweater, or one
jacket that are priced Rs. 200, Rs. 400, Rs. 600, and Rs. 1,000 a piece respectively. Chirag borrows
Rs. 300 from Ashok and buys a jacket. Bashir buys a sweater after borrowing Rs. 100 from Ashok
and is left with no money. Ashok buys three shirts. What is the costliest item that Deepak could buy
with his own money?
a. A shirt b. A shawl c. A sweater d. A jacket

145. In a ‘keep-fit’ gymnasium class there are 15 females enrolled in a weight-loss programme. They all
have been grouped in any one of the five weight-groups W1, W2, W3, W4, or W5. One instructor is
assigned to one weight-group only. Sonali, Shalini, Shubhra and Shahira belong to the same weight-
group. Sonali and Rupa are in one weight-group, Rupali and Renuka are also in one weight-group.
Rupa, Radha, Renuka, Ruchika, and Ritu belong to different weight-groups. Somya cannot be with
Ritu, and Tara cannot be with Radha. Komal cannot be with Radha, Somya, or Ritu. Shahira is in
W1 and Somya is in W4 with Ruchika. Sweta and Jyotika cannot be with Rupali, but are in a weight-
group with total membership of four. No weight-group can have more than five or less than one
member. Amita, Babita, Chandrika, Deepika and Elina are instructors of weight-groups with
membership sizes 5, 4, 3, 2 and 1 respectively. Who is the instructor of Radha?
a. Babita b. Elina c. Chandrika d. Deepika

FLT – 07 Page 35
146. A king has unflinching loyalty from eight of his ministers M1 to M8, but he has to select only four to
make a cabinet committee. He decides to choose these four such that each selected person
shares a liking with at least one of the other three selected. The selected persons must also hate at
least one of the likings of any of the other three persons selected.
M1 likes fishing and smoking, but hates gambling.
M2 likes smoking and drinking, but hates fishing.
M3 likes gambling, but hates smoking,
M4 likes mountaineering, but hates drinking,
M5 likes drinking, but hates smoking and mountaineering.
M6 likes fishing, but hates smoking and mountaineering.
M7 likes gambling and mountaineering, but hates fishing.
M8 likes smoking and gambling, but hates mountaineering.

Who are the four people selected by the king?


a. M1, M2, M5 and M6 b. M3, M4, M5 and M6
c. M4, M5, M6 and M8 d. M1, M2, M4 and M7

Directions for questions 147 to 150: Answer the questions based on the following information.
A and B are two sets (e.g. A = Mothers, B = Women). The elements that could belong to both the sets
(e.g. women who are mothers) is given by the set C = A . B. The elements which could belong to either A
or B, or both, is indicated by the set D = A ∪ B . A set that does not contain any elements is known as a
null set represented by ϕ (e.g. if none of the women in the set B is a mother, then C = A .B is a null set, or
C = ϕ ).
Let ‘V’ signify the set of all vertebrates, ‘M’ the set of all mammals, ‘D’ dogs, ‘F’ fish, ‘A’ alsatian and ‘P’,
a dog named Pluto.

147. Given that X = M .D is such that X = D. Which of the following is true?


a. All dogs are mammals b. Some dogs are mammals
c. X = ϕ d. All mammals are dogs

148. If Y = F . (D . V) is not a null set, it implies that


a. all fish are vertebrates b. all dogs are vertebrates
c. some fish are dogs d. None of these

149. If Z = (P . D) ∪ M, then
a. the elements of Z consist of Pluto, the dog, or any other mammal
b. Z implies any dog or mammal
c. Z implies Pluto or any dog that is a mammal
d. Z is a null set

150. If P . A = ϕ and P ∪ A = D, then which of the following is true?


a. Pluto and alsatians are dogs
b. Pluto is an alsatian
c. Pluto is not an alsatian
d. D is a null set

Page 36 FLT – 07
101. Count only those lays for which any size of yellow 111. From figure the total effort in man-hours spent on-site
coloured fabric is produced. is 320.
They are lay number It is nearest to actual man-hours of offshore testing
1, 3, 4, 6, 7, 8, 9, 11, 12, 15, 21, 24, 25, 27 which is 280 (approximately.)
Hence, 14 is the answer.
112. Total man-hours
102. Count those lays for which extra-extra large fabric is = (100 + 80) + (420 + 100) + (280 + 140) = 1120.
produced of any colours, i.e. count the lay numbers Total working hours = 100
for which at least one of XXL from 3 colours is non-
1120
zero. Total man working = = 11.2 or 11.
They are lay number 7, 8, 9, 10, 11, 12, 13, 14, 15, 21, 100
22, 23, 24, 25, 26, 27 . For 50 hr the total man-hours is 50 × 11 = 550 which
Hence, 16 is the answer. is near to coding (420 + 100)
Hence, (a) is the answer
103. Again count lay number for which at least one of the
XXL from yellow and white are non-zero. 113. Total offshore work = 100 + 420 + 280
Lay number 7, 8, 9, 10, 11, 12, 13, 14, 15, 21, 23, 24, = 800 man-hours.
25, 26, 27. 50% of offshore work are carried out on-site.
Hence, 15 is the answer. Distribution of effort are in ratio 180 : 520 : 420
9 : 26 : 21
104. The varieties for which there is surplus gives the Effort distributed to testing will be
answer. There are 4 such varieties.
21
× 400 = 147 man-hours.
105. Put a decimal after the first two digit in the passengers 56
column and it wil give the figure in millions. 280
In that case we have only 5 international airports of Offshore testing work is = 140
2
type A having more than 40 million passengers.
They are in serial number 1, 2, 3, 5, 9. ∴ Proportion of testing carried out offshore is
Rest all ‘A’ type is below 40 million. 140
× 100 = 30%
(140 + 140 + 147 )
106. There are only six airports of USA among the top 10
busiest airports. They are in serial number 1, 2, 3, 5,
9, 10. 114.
6 Design Coding Testing
Hence, × 100 = 60%
10
Initially 80 100 140
107. We have to calculate the percentage of passengers
handled at Heathrow Airport. 100 420 294
Now total number of passengers in the 5 busiest Finally 80 + = 130 100 + = 310 140 + = 287
2 2 2
airport is approximately
(77 + 72 + 63 + 62 + 60) million 115. We see flow from Vaishali to Jyotishmati is 300 where
= 334 million as demand is 400 so the deficient 100 would be met
At Heathrow it is 62 million. by flow from Vidisha. Again the demand of 700 in
The approximate percentage is Panchal is again to be met by flow from Jyotishmati
60 which can get it from Vidisha.
× 100 ~ 20% Thus, the quantity moved from Avanti to Vidisha is
300
200 + 100 + 700 = 1000
108. All the international airports handle more than 30 million
116. Free capacity at Avanti-Vaishali pipeline is 300, since
passengers. Among these only 6 airports are not
capacity of each pipeline is 1000 and demand at
located in USA. Hence, (b) is the answer.
Vidisha is 400 and 300 flows to Jyotishmati.
Thus, free capacity = {1000 – (400 + 300)} = 300
109. Man-hours spent in coding is 420 + 100 = 520.
Now going by options, we see (a) is the only option.
117. Free capactiy in Avanti-Vidisha is zero. Explanation is
similar as in previous answer.
110. Total work is approximately
(100 + 80) + (420 + 100) + (280 + 140) = 1120
118. On interchanging the effort allocation between
On-site work = 80 + 100 + 140 = 320
operations B and C, then C and D, and then D and E
Percentage of total work carried out on-site is
we find that B takes the E’s position.
320 Looking at the effort in B and then ranking in ascending
× 100 = 30% approxmately. order we find that the company 3 ranks third.
1120

Page 12 FLT – 07
119. Total effort for operation B through F is 81.5%. 126.
Even distribution will give effort allocation in each
81.5
operation = = 16.3%
5
∴ Change in E = 28.6 – 16.3 = 12.3% O

120. Since we are given about company 1, 4, 5 in options


so we will look for changes in these companies only.
We know that the diameter of circle will be the diagonal
Allocation of effort in B, C, D in companies 1 = 43.1 of the square.
43.1 Thus, from any of the two statements, we can find
Remaining operations gets = 14.4% each. out the areas of the circle and square.
3
Hence, (b) is the answer.
Allocation of effort in B, C, D operations of company
4 = 29.7 127. I gives a general figure of Ram and Gopal.
Remaining operation is allocated extra II does not give any idea of how much apples Ram
29.7 and Gopal purchased.
= 9.9% each. Both statements together also cannot give any result.
3
Allocation of effort in B, C, D operation of company
128. Cost in rupees of oil moved by rail and road is 18% of
5 = 36.8
30 million = 5.4 million.
36.8 Volume of oil transported by rail and road
Remaining operation is allocated = 12.3% each. = 31% of 12 million tonnes = 3.72 million tonnes.
3
We see that operation E in company 5 will then show 5.4
the maximum.
Cost in rupees per tonnes = = 1.5 approximately.
3.72

121. From II, m, n could be (2, 15) (5, 6), (3, 10) and (1, 30)
129. From the chart, we can make out the least among
but from I, we get m, n as (2, 15).
road, rail, pipeline, ship by looking at the ratio of cost
to volume.
122. From I nothing can be said since exact figures are not
given. 6
Road =
From II since X > Y (from B) we do not know how 22
much X is greater than Y, because if it is slightly greater 12
than it will be less than Y after 5 years whereas if the Rail =
9
difference is very high, then X will be greater than Y
even after 5 years. 65
Pipeline =
49
123. From I, unequal even integers less than 10 are 2, 4, 6, 10
8. Ship =
9
X Obviously road is the lowest and hence the cheapest.
is an odd integer is possible only if x = 6, y = 2
Y
130. Ship, air and road.
From II, even integers less than 10 are 2, 4, 6, 8. Like the previous answer again look at ratio of
XY = 12 ⇒ X = 6, Y = 2 or X = 2, Y = 6
10 7 6
Hence, question can be answered using either , ,
statement alone but not from statement B. 9 11 22

10 7 6
124. I gives the capacity of boat and is of no help in finding So > >
out the number of round trips. 9 11 22
From II round trips can be calculated since we know Hence, P > Q > R
the total time taken is 12 hr.
131. Sati-Savitri starts at the earliest.
125. I gives the rate and II gives the size. It is like I gives the So we view it first.
speed and II the distance and we are to find out time. (1) Sati-Savitri — 9.00 a.m. to 10.00 a.m.
So both statements are needed. (2) Veer Abhimanu — 10.00 a.m. to 11.00 a.m.
(3) Jhansi Ki Rani/Sundar Kand — 11.00 a.m. to
11.30 a.m.
(4) Joru Ka Ghulam — 11.30 a.m. to 12.30 p.m.
Now lunch break from 12.30 p.m. to 1.30 p.m.
At 1.30 p.m. he can takes the show of only Jhansi
Ki Rani so it cannot be viewed at 3rd.

FLT – 07 Page 13
(5) Jhansi Ki Rani — 1.30 p.m. to 2.00 p.m. 135. Elle is 3 times older than Yogesh and Zaheer is half
(6) Reshma aur Shera 2.00 p.m. to 3.00 p.m. the age of Wahida.
Hence, option (c) is best. If Wahida is 2x-year-old, then Zaheer is x.
Now Yogesh > Zaheer
132. Three children Vaibhav, Suprita and Anshuman. ⇒ Yogesh > x
Vaibhav > Suprita Elle is 3 times older than Yogesh.
↓ Which means Elle is older than Wahida as 3x > 2x.
(Born in April)
Alternative method:
One of children is born in September, but it is not E = 3y ...(i)
Vaibhav, so it has to be Anshuman. w
So Vaibhav is born in June and is 7-year-old. Vaibhav z= , or 2z = w ...(ii)
2
is 7-year-old and Anshuman is not 4-year-old.
y > z , implies 2y > 2z implies 2y > w from (ii)
So Suprita is 4-year-old.
Now, if 2y > w
Youngest child is 2-year-old and it has to be Anshuman.
3y > w, i.e. E > w from (i)
Vaibhav > Suprita > Anshuman
Hence, Elle is older than Wahida.
(June, 7 years) (April, 4 years) (Sept., 2-year-old)
Hence, (c) is the answer.
136. From (a) Zaheer is 10-year-old means Wahida is
20-year-old. From (b) Yogesh and Wahida are older
133. We can find out the time for lunch of respective families
than Zaheer by same number of years.
from the table below:
This means Yogesh is 20-year-old. Now Elle is 3 times
older than Yogesh.
Family/Time 12:00 1:00 2:00 Elle is 20 × 3 = 60-year-old.
Hence, we see that both (a) and (b) statements are
ü
needed so the answer is (c).
Sharma
137. Find out from the options.
Banerjee ü (a) David, Rama and Rahim
Ram would like to be in the group only if Peter is there,
Pattabhiraman ü so it is not feasible.
(b) Peter, Shyam and Rahim
Shyam and Rahim want to be selected together and
Fried brinjal → Chinaware
none of them have problem or any conditions, hence
Sambar → White Chinaware
feasible.
Makkai-ki-roti → Red Chinaware (c) Since Shyam is there, Rahim has to be but he is
The family that eats at 1 o’clock serves fried brinjal, not also Fiza is not there which David insists so not
hence Pattabhiraman serves fried brinjal. feasible.
The family that eats last like makkai-ki-roti so (d) Since Peter is not there and so Ram would not
Banaerjees like makkai-ki-roti. Sharmas are left with prefer that group, hence not feasible.
sambar.
Sharma - 12:00 - Sambar - White 138. Looking at options, we see (c) is best as Shyam and
Pattabhiraman - 1:00 - Fried brinjal - Blue Rahim is selected and Fiza is there when David is
Bannerjees - 2:00 - Makkai-ki-roti - Red selected.
Hence, (c) is the best option. In (a) we see Shyam is not there with Rahim.
In (b) Fiza is not there with David.
134. We start making one true and other false. In (d) Peter and David cannot go together as David
Case I would not like Peter in the group.
T F
Shopkeeper 1: Black hair Long tail 139. In Ist option — Kavita is in the group means David is
T F there and David would not like Peter in the group,
Shopkeeper 2: Short tail Wore a collar whereas Ram would like to be in the group if Peter is
T F there so the statement cannot be true.
Shopkeeper 3: White hair No collar 2nd option — If David is there, then only the group will
have both women Kavita and Fiza, but in that case
Case II we see none of the rest could be the fourth person
T F as Shyam and Rahim has to be together and Ram
Shop keeper 1: Black hair Long tail would be if Peter is there and David would not like
T F Peter in the group, hence statement is false.
Shop keeper 2: Short tail Wore a collar 3rd option — It is not possible as Ram cannot go with
T F Shyam and David with Peter.
Shop keeper 3: White hair No collar So none of the above statements are true.
Both the cases are correct, and hence we see only
option (b) is correct.

Page 14 FLT – 07
140. Let S = spotted, NS = Non-spotted 143. We have packages as follows:
3 item (D + 2B) = Rs. 40 + Rs.180 = Rs. 220 ... (i)
2 item (A + C) = Rs.180 ... (ii)
Oak Maple 4 item (E + 2D + B) = 45 + 50 + 90 = 215 ... (iii)
The combinations of purchase possible are
Red Non-red Red Non-red Case 1: Rs. 220 × 4 = Rs. 880
Points: 12 × 1000 – 120 × 1500 = – 1,68,000
S NS S NS S NS S NS
Case 2: Rs. 180 × 5 = Rs. 900
2n x 10n 0 6 x 0 22 Points: 10 × 1000 – 100 × 1500 = – 1,40,000

There are 50 coloured leaves and is given as red and Case 3: Rs. 215 × 4 = Rs. 860
non-red. Points:16 × 1000 – 140 × 1500 = – 1,94,000
We make the following table. Let 2n be number of red
oak leaves where n is any natural number. Case 4: 2(220 + 180) + 180 = Rs. 980
Points: 12 × 1000 – 20 × 1500 = – 18,000
Now we have 2n + x + 10n + 6 + x + 22 = 50
⇒ 12n + 2x = 22 Case 5: 2(220 + 215) = Rs. 890
It is possible for only n = 1, x = 5 Points : 14 × 1000 – 110 × 1500 = – 1,51,000
We cannot take n > 1
Hence, number of oak leaves are Case 6: 2(215 + 180) + 180 = Rs. 970
2 × 1 + 5 + 10 × 1 = 17 Points :14 × 1000 – 30 × 1500 = – 31,000
By seeing the above figure, we see that we maximize
141. O, P, Q and R carried on motorcycles M1, M2, M3 and M4 the point in last case when purchase is 14 item for
respectively. So Rs. 970.
OP Q R
M1 M2 M3 M4 144. Bashir < Chirag.
F E A+G C Now Chirag borrows Rs. 300 and Bashir Rs. 100
BD H from Ashok. Ashok buys 3 shirt so he must have at
Since B cannot be with R so it will go with O that is least Rs. 1,000.
only left. Bashir is left with no money after buying a sweater
Hence, C and H will go together in M4 with R. and he had to borrow Rs.100 from Ashok means he
had Rs. 500 with him.
Ashok must have less than Rs. 1,500.
142. Ashok has three times the money with Deepak.
G F1 Gm1 So Deepak cannot have Rs. 300 because Ashok must
have Rs.1,000, again Deepak cannot have Rs. 500
because Ashok should have less than Rs.1,500.
F1 M1 So Deepak has Rs. 400 for which he can purchase
the shawl which is costliest.

145. W1 W4
Rupa Radha Renuka Ruchika Ritu
GS1 G S2 GD1 GD2 Sonali Rupali Somya Tara
Shalini Komal Sweta
Shubhra Jyotika
G F 2+G M 2 Shahira Deepika

Amita Elina Chandrika Babita


Hence, Elina is instructor of Radha.
F2 M2

Thus, we have 2 grandfathers GF1, GF2


4 fathers GF1, GF2, F1 and F2
2 grandmothers GM1, GM2
4 mothers GM1, GM2, M1 and M2
Thus, minimum number will be 12.

FLT – 07 Page 15
146. 147. X = M.D = M ∩ D
X=D
M ∩D=D
Mount- ⇒D ⊂M
Gam-
Fishing Smoking Drinking aineer-
bling Thus all dogs are mammals.
ing

L i ke s
M1 M1 M2 M3 M4 M
M6 M2 M5 M7 M7 D
M8 M8

M5
M2 M3
Dislikes M4 M1 M6 148. Y = F ∩ (D ∩ V ) is not a null set means some F’s are
M7 M5
M8 D’s and sum D’s are V’s .
M6
That means some fish are dogs.

Now go by options. 149. Z = (P.D) ∪ M


(a) M does not hate at least one of the liking of any of
Z = (P ∩ D) ∪ M
the other 3 persons selected.
P ∩ D means pluto the dog.
(b) None of person shares the liking of at least one of
the other selected. P ∩ D ∪ M means pluto the dog or any other mammal.
(c) None of the person shares a liking with at least
one of the other three selected. 150. P.A = φ P ∪ A = D
(d) M1 shares liking with M2 and vice versa.
P ∩ A = φ means no alsations are pluto or pluto is not
M4 shares liking with M7 and vice versa.
M1, M2 dislikes M7 liking. an alsation where dogs are composed of alsation or
M4, M7 dislikes M2 liking. pluto or both.
Hence the answer is (d).

Page 16 FLT – 07
108. 117. (a) would only result in more cars per family. (b) and
(c) defeat Athens’ purpose as citizens devise
1 2 19 82 ingenious methods to maintain status quo both in terms
1 2 16 5 – 2 of number of cars and congestion.
1 2 13 – 9
1 – 1 118. The Central Bank can only express ‘reservations’ on
‘monetisation’ to the government. It cannot be the ‘boss’,
it only advises. (c) does not support the conclusion in
The answer is 1192. any way. (a) and (d) are specific observations, but
they do not contribute to our answer.
109. Work with options. If the cylinder has a capacity of
1,200 L, then the conical vessel shall have a capacity 119. (d) is a summary for the general words ‘varied use’,
of 700 L. Once 200 L have been taken out from the ‘common basis’ pertaining to the ‘symbol’ in the
same, the remaining volume in each of them shall be geographical and historical context. (c) does not
1000 and 500. present the complete picture. (a) refuses to divulge
the significance of the umbrella. (b) is wrong as the
Alternative method: ruler is regarded as the instrument of firmament of the
Let the volume of conical tank be x. supreme law.
Then the volume of cylindrical tank = x + 500
x + 300 = 2(x – 200) ⇒ x = 700 120. This answer goes without guessing. There are two
Volume of cylindrical tank = 700 + 500 = 1200. parties in the game, and each has its own strategy
and a guess on the opponent’s move. (a) involves
110. Work with options. Length of wire must be a multiple more of cooperation strategies than game plans. (b) is
of 6 and 8. Number of poles should be one more than competition involving more than two candidates. (d) is
the multiple.
about cartels.
111. If the number of students enrolled for a certain class
do not fit into that age interval, they are in excess and 121. Since yellow is between green and red, it should be
hence, unrepresentative, thus resulting in bloated house number 2 or 3. Also green is adjacent to blue
ratios. (a) is wrong because the definition of gross house, it should have blue and yellow house on either
enrollment ratio itself is flawed. Attendance is not the side. Hence, the following table can be constructed.
focus of our argument. We are also not concerned
House number 1 2 3 4
with demographic trends, but only with given data.
Colour Blue Green Yellow Red
Occupant X Z
112. The argument states that ‘clubs that spend more should Since X does not live adjacent to Z, it has to live in blue
finish at a higher ranking’. This is reflected in ‘highly house.
paid white players returned a low ranking’. (a) focuses
on clubs that recruited black players, a consquence 122. The ratio of points for carrying books of various
Szymanski is not immediately concerned with. (c) also subjects is:
throws no light on the relation ‘clubs that spend more Management : Mathematics : Physics : Fiction
should finish higher’. Nor does (d). =4:3:2:1
Since the points are to be maximized, the number of
113. Only (d) connects the recommendation directly to the books that Ramesh should carry in descending order
cause ‘rising tensions’ in prisons and not to any is management, mathematics, physics and fiction.
marginal political factors. (a), (b) and (c) may have The ratio which Ramesh has to maintain is:
contributed, but peripherally, to the minister’s decision. Management : Fiction < 1 : 2,
Mathematics : Physics < 1 : 2.
114. (a) and (d) cover the government’s honourable This means that a combination of management and
intentions, which look best on paper. (b) discusses fiction books in the ratio of 1 : 2 will give 6 points while
one feasiblilty factor. (c) is the best choice as it shows a combination of mathematics and physics books in
how the project has reached the implementation stage the ratio of 1 : 2 will give 7 points, hence, Ramesh
from the pilot stage. should carry the following combination of books to
maximize the points; management 1, mathematics 2,
115. ‘Manoeuvrability’ is linked to ‘flight direction changes’ physics 5 and fiction 2, a total of 22 points.
in (d). (c) just vaguely mentions ‘faster’. (a) makes no
inference, as such. It may or may not be true. There is
insufficient evidence to infer (d), it sounds rather far-
fetched.

116. (b) does not attack the argument, it helps the


Association’s cause. (c) is pointless, if there isn’t
adequate consumption. (d) has little to do with the
core issue in the argument. But (a) if true, would render
the cumulative efforts of the Association fruitless.

FLT – 06 Page 9
123. The following table can be created using the data Information Technology Industry in India
given. (Figures are in million US dollars)

Persons P M U T X 1994-95 1995-96 1996-97 1997-98 1998-99

Software
Blue Red
Colour
and Yellow and Black Domestic 350 490 670 950 1250
choice
Red Blue
Exports 485 734 1083 1750 2650
Hotel
Stays in
Hardware
Does not
Palace Palace Palace Domestic 590 1037 1050 1205 1026
stay in

Since X stays in a hotel and P or U or T cannot stay in Exports 177 35 286 201 4
a palace, M stays in palace.
Peripherals
124. The attendants of X, Y and Z are to be Mohan, Jack
Domestic 148 196 181 229 329
and Rita. The animals under Mohan’s care is given in
the data. Since Jack does not attend to deer, lion and Exports 6 6 14 19 18
bison, the following table can be created using the
data given. Training 107 143 185 263 302

Maintenance 142 172 182 221 236


Attendants Mohan Ja ck Rita
Networking
36 73 156 193 237
Lion and Bear and and others
Animals
Panther Panther
Total 2041 2886 3807 5031 6052

Enclosure X Y Z
126. Total exports
= Software export + Hardware export + Peripherals
export
Name Mohan Ja ck Rita Shalini Suman Hence, total export as a percentage of IT business for
1994-95
Li on B ear D eer Li on D eer 668
Animals and and and and and = × 100 = 32.7%
2041
panther panther b i so n bear b i so n
775
For 1995-96 = × 100 = 26.8%
C age X Y Z Q P 2886

1383
For 1996-97 = × 100 = 36%
The data for Mohan and Jack can be filled directly. 3807
Similarly, X, Y, Z can be filled directly from data given.
1970
The key after filling in these animals is that Z and P For 1997-98 = × 100 = 39%
have the same pair of animals, the only option is deer 5031
and bison. 2672
For 1998-99 = × 100 = 44 %
125. By trial and error, we can make different 6052
combinations and find the cost.
Like 20 kg × 2 + 10 kg × 4, the cost would be 127. Percentage growth for 1995-96 = 41%,
Rs.180. The minimum cost comes in the case of 1996-97 = 32%, 1997-98 = 32%, 1998-99 = 20%.
10 kg × 8, i.e. Rs.160.
128. (a) and (b) can be easily eliminated from the given
Direction for students: The table for questions 126 table.
to 130 in CAT 2001 Bulletin has some misprints and it
should be read like the following. 129. Total IT business in hardware (Export + Import) shows
a continuous increase from 1994-95 to 1997-98 and
then declines in 1998-99.

Page 10 FLT – 06
130. In this question there are two activities — hardware imply towards C being the mid-point of BD. The ratio
and peripherals, hence for year X to dominate year Y, of AC/CE will be one by using any statement.
at least one activity in year X has to be greater than
that in year Y and the other activity in year X cannot 139. Here, by combining the two statements, we get the
be in year Y. In (a), (b) and (c) while hardware duration of the flight.
dominates in one year, the peripherals dominate in the For the arrival time we should have information
other. regarding the time zone difference of Mumbai and
No-man’s-land.
131. Statement I implies X > Y, or X > Z, or X > Y and Z
Statement II implies Y > X, or Y > Z, or Y > X and Z. 140. Statement I implies X – Y = 6. .
Combining both statements, we can get Y > X > Z or Statement II implies XY is divisible by 6.
X > Y > Z. Hence, Z is the smallest. You can see that many values of X and Y can satisfy
statement I and II.
132. The first statement implies that X must lie between
0 and – 3. Hence, it gives the answer. But from the 141. If the total number of factories is 100, then the total
second statement, we have either X > 3 or X < 0. This number of employees
does not gives us an information about the modulus = 60 × 100 = 6000 of which 64.6% = 3876 work in
of X. wholly private factories. Since the number of wholly

133. The Venn diagram arrived at from both I and II clearly 3876
private factories = 90.3, the answer = = 43.
indicates that 500 people are watching programme P. 90.3

60 2
Short cut: 0.64 × <   × 60 = 45
P Q 0.903  3 

5 00 1 00 9 00 Value added
142. Value added per employee = Employment

134. For a given inradius and circumradius, there is only Gross output
one possible value of (PR + RQ). 143. Compound productivity = Fixed capital .
Hence, both the statements are required to answer
Hence, compound productivity for various sectors is:
the question.
Public sector = 0.6, Central Government = 0.725,
States/Local = 0.47, Central/States/Local = 1.07, Joint
135. Statement I implies that profit is 2.95%, but we want sector = 1.23 and wholly private = 1.36. Hence, the
the profit per rupee spent on buying the shares. The order should be: Wholly private, Joint, Central/State/
cost of buying the shares for Harshad is CP + 0.01 CP Local, Central Government, Public sector and State/
= 1.01 CP. The cost of selling is Local government.
SP – 0.01 SP = 0.99 SP
The difference of the two is profit, i.e. 144. Calculate the ratios: Value added/employment and
0.99 SP – 1.01 CP = 0.99 × 1.05 CP – 1.01 CP value added/fixed capital for the sectors mentioned in
= 0.0295 CP. Hence, profit = 2.95%. the choices. The respective values are:
Wholly private 0.9 and 1.25; Joint sector 1.59 and
136. We cannot work the questions individually through 1.19; Central/State/Local 1.8, 1.28; others 0.92 and
I or II. But combining the two statements, we get 0.75.
(2 ⊕ 0) = (0 ⊕ 2) = 0 and 0 ⊕ (– 5 ⊕ – 6) = 0.
145. The number of factories in joint sector is 1.8% = 2700,
137. Both the statements combined also do not tell us if thus the number of factories in Central Government
they are intersecting or not. The two lines can be = 1% of (2700 x 100/1.8) = 1500. Value added by
parallel also depending on the values of a, b, d, e. Central Government = 14.1% of 1,40,000 crore
= 19,740.
138. You can see from the following diagram that both
statements individually 19740
Hence, answer = = Rs. 13.1 crore.
1500

146. Find the difference between FEI in 1998 relative to its


FEI in 1997. Hence, for India it is 0.72 – 1.71 = –0.99.
For China it is 4.8 – 5.96 = – 1.16.
For Malaysia it is 9.92 – 10.67 = –0.75 and for Thailand
A B C D E it is 5.82 – 5.09 = 0.73.
Change in FEI in 1998 relative to its FEI in 1997.
−0.99
For India, percentage = × 100 = –57.89
1.71

FLT – 06 Page 11
153. Clear from the table.
−0.75
For Malaysia, percentage = × 100 = –7.02
10.67 154. Clear from the given graph.
For Thailand, change is 14.34%.
For China and Korea, changes are – 19.46% and 155. First find out the growth in 1990 of the all four sectors.
15.74% respectively. So manufacturing 9% of 20 = 1.8. Hence, 20 + 1.8
Hence, we can see that the country with the largest = 21.8. Similarly, for mining and quarrying it is 15.6.
change in FEI is India. For electrical, it is 10.85 and for chemical it is 16.1, now
in 1991 there is 1% negative growth in manufacturing.
147. Since the absolute values are not given, it cannot be So 1% of 21.8 becomes 0.218. Thus, 21.8 – 0.218
calculated. = 21.582. Similarly, for mining and quarrying it is 15.44.
For electrical it is 11.88 and for chemical it is 16.21.
148. Assume of GDP of India for 1997 to be x. Now we add the figures for 1991 of all the sectors
which comes to 21.582 + 15.75 + 11.88 + 16.21
0.72 × 102x
For 1998, India's FEI = = 0.7344x = 65.42. Now 65.42 – 64.35 = 1.07 which comes to
100 approximately 1.5% growth rate.
And foreign equity inflows for 1997 = 1.71x
For China, assume GDP as y. Then FEI in 1998 156. Clear from the graph.
107y
= × 4.8 = 5.136y. And FEI in 1997 = 5.96y. 157. In 1990, there is 4% growth. Hence, 4% of 15 = 0.6.
100 So weightage in 1990 becomes 15.6.
For South Korea, let GDP be z. Similarly, in 1991 it becomes 15.44, in 1992 it is 15.6,
in 1993 it is 14.97, in 1994 it is 16.16.
95z
FEI in 1998 = × 2.5 = 2.375z and FEI = 2.16z. Hence, it can be seen that the lowest level of
100 production was in 1993.
We can see that India and China were lower in 1998
than in 1997, and South Korea is higher in 1998 than 158. Find out the weightage for all the sectors for 1994.
in 1997. For manufacturing it is 25.54, for mining and quarrying
it is 16, for electrical it comes out to be 14.5 and for
149. Let x be the foreign equity inflow of India. Hence, chemical it is 19.5. The total comes to approximately
China’s foreign equity inflow is 10x. 77. In 1989, it was 60. Hence, 77 – 60 = 17 which is
Now in 1998, FEI in India was 0.72, therefore approximately 25% increase.
x
0.72 = 159. Since the index of total industrial production in 1994 is
GDP of India
50% more than in 1989, it becomes 150.
Similarly, FEI in China in 1998 was 4.8, therefore 4.8 Now total weightage for manufacturing, mining and
quarrying, electrical and chemical in 1994 is
10x
= GDP of China approximately 77. So 150 – 77 = 73.
In 1989, it was 100 – 60 = 40.
Hence, (GDP of China/GDP of India) So 73 – 40 = 33, which is approximately 87.5%.
= (10 × 0.72)/4.8 = 1.5.
Thus, China’s GDP is 50% higher than that of India. 160. Cost in shift operation = 800 + 1200 = Rs. 2,000
Variable cost for 40 units = Rs. 3,600
150. As from the table, the deficit intensity from 1993-94 to Approximate average unit cost for July
1997-98 are 5.1, 6.3, 7.6, 8 and 5.
3600 + 2000
Therefore, highest growth rate is 7.6 – 6.3 = 1.3 which = = Rs. 140 .
is in 1994-95. 40

161. The only change for change of production from 40 to


7.6 – 6.3
151. The highest growth rate = × 100 = 23.5% 41 is the variable cost which is
6.3 Rs. (3730 – 3600) = Rs. 130.

152. From the tables given, 162. The trend for MC is varying and is just the reverse
Import of raw material = 10.1 × Sales (S) import of condition as that stated in C. Take some values and
capital goods = 17.6 × Gross fixed assets (GFA) check.
Given imports = Raw materials + Capital goods
So import = 10.1 S + 17.6 GFA 163. Total sales revenue = Rs. (150 × 40) = Rs. 6,000
So imports = 14.2 S Total production cost = Rs. (3600 + 2000) = Rs. 5600.
Hence, 14.2 S = 10.1 S + 17.6 GFA So profit = Rs. 400.
S 17.6
Hence, = = 4.3 164. Profit is highest when there is no second shift.
GFA 4.1
165. (a) and (b) are definitely not true as the case is the
inverse of that mentioned in (c). Take some values
and check.

Page 12 FLT – 06
Section – III

Sub–Section III-A : Number of questions = 10

Note: Questions 61 to 70 carry one mark each.

Directions for questions 61 to 64: Answer the questions on the basis of the information given below:
A management institute was established on January 1, 2000 with 3, 4, 5, and 6 faculty members in the
Marketing, Organisational Behaviour (OB), Finance, and Operations Management (OM) areas respec-
tively, to start with. No faculty member retired or joined the institute in the first three months of the year
2000. In the next four years, the institute recruited one faculty member in each of the four areas. All these
new faculty members, who joined the institute subsequently over the years, were 25 years old at the time
of their joining the institute. All of them joined the institute on April 1. During these four years, one of the
faculty members retired at the age of 60. The following diagram gives the area-wise average age (in terms
of number of completed years) of faculty members as on April 1 of 2000, 2001, 2002, and 2003.

55

52.5
51.5
50.5 50.2
50 49.33 49
2000
47.8
2001
46 46 2002
45 45 45 45 2003
45 44 44
43

40
Marketing OB Finance OM

61. From which area did the faculty member retire?


(1) Finance (2) Marketing (3) OB (4) OM

62. Professors Naresh and Devesh, two faculty members in the Marketing area, who have been with the
Institute since its inception, share a birthday, which falls on 20th November. One was born in 1947
and the other one in 1950. On April 1 2005, what was the age of the third faculty member, who has
been in the same area since inception?
(1) 47 (2) 50 (3) 51 (4) 52
63. In which year did the new faculty member join the Finance area?
(1) 2000 (2) 2001 (3) 2002 (4) 2003

CAT PAPER – 2005 MBA Page 19


Test Prep
64. What was the age of the new faculty member, who joined the OM area, as on April 1, 2003?
(1) 25 (2) 26 (3) 27 (4) 28

Directions for questions 65 to 67: Answer the questions on the basis of the information given below:
The table below reports annual statistics related to rice production in select states of India for a particular
year.

State Total Area % of Area Under Production Population


(in million Rice Cultivation (in million (in millions)
hectares) tons)
Himachal Pradesh 6 20 1.2 6
Kerala 4 60 4.8 32
Rajasthan 34 20 6.8 56
Bihar 10 60 12 83
Karnataka 19 50 19 53
Haryana 4 80 19.2 21
West Bengal 9 80 21.6 80
Gujarat 20 60 24 51
Punjab 5 80 24 24
Madhya Pradesh 31 40 24.8 60
Tamilnadu 13 70 27.3 62
Maharashtra 31 50 48 97
Uttar Pradesh 24 70 67.2 166
Andhra Pradesh 28 80 112 76

65. Which two states account for the highest productivity of rice (tons produced per hectare of rice
cultivation)?
(1) Haryana and Punjab (2) Punjab and Andhra Pradesh
(3) Andhra Pradesh and Haryana (4) Uttar Pradesh and Haryana

66. How many states have a per capita production of rice (defined as total rice production divided by its
population) greater than Gujarat?
(1) 3 (2) 4 (3) 5 (4) 6

67. An intensive rice producing state is defined as one whose annual rice production per million of
population is at least 400,000 tons. How many states are intensive rice producing states?
(1) 5 (2) 6 (3) 7 (4) 8

Page 20 MBA CAT PAPER – 2005


Test Prep
Directions for questions 68 to 70: Answer the questions on the basis of the information given below:
The table below reports the gender, designation and age-group of the employees in an organization. It also
provides information on their commitment to projects coming up in the months of January (Jan), February
(Feb), March (Mar) and April (Apr), as well as their interest in attending workshops on: Business Opportu-
nities (BO), Communication Skills (CS), and E-Governance (EG).

Sl. Na m e Ge nde r De signa tion Age group Com m itte d to Inte re ste d in
No. proje cts w orkshop
during on
1 Anshul M Mgr Y Jan, Mar CS, EG
2 Bushkant M Dir I Feb, Mar BO, EG
3 Charu F Mgr I Jan, Feb BO, CS
4 Dinesh M Exe O Jan, Apr BO, CS, EG
5 Eashwaran N Dir O Feb, Apr BO
6 Fatima F Mgr Y Jan, Mar BO, CS
7 Gayatri F Exe Y Feb, Mar EG
8 Hari M Mgr I Feb, Mar BO, CS, EG
9 Indira F Dir O Feb, Apr BO, EG
10 John M Dir Y Jan, Mar BO
11 Kalindi F Exe I Jan, Apr BO, CS, EG
12 Lavanya F Mgr O Feb, Apr CS, EG
a
Jn
,
13 Mandeep M Mgr O Mar, Apr BO, EG
14 Nandlal M Dir I Jan, Feb BO, EG
15 Parul F Exe Y Feb, Apr CS, EG
16 Rahul M Mgr Y Mar, Apr CS, EG
17 Sunita F Dir Y Jan, Feb BO, EG
18 Urvashi F Exe I Feb, Mar EG
19 Yamini F Mgr O Mar, Apr CS, EG
20 Zeena F Exe Y Jan, Mar BO, CS, EG

M = Male, F = Female; Exe = Executive, Mgr = Manager, Dir = Director;


Y = Young, I = In between, O = Old
For each workshop, exactly four employees are to be sent, of which at least two should be Females and
at least one should be Young. No employee can be sent to a workshop in which he she is not interested in.
An employee cannot attend the workshop on
• Communication Skills, if he/she is committed to internal projects in the month of January.
• Business Opportunities, if he/she is committed to internal projects in the month of February.
• E-governance, if he/she is committed to internal projects in the month of March.

68. Assuming that Parul and Hari are attending the workshop on Communication Skills (CS), then
which of the following employees can possibly attend the CS workshop?
(1) Rahul and Yamini (2) Dinesh and Lavanya
(3) Anshul and Yamini (4) Fatima and Zeena

CAT PAPER – 2005 MBA Page 21


Test Prep
69. How many Executives (Exe) cannot attend more than one workshop?
(1) 2 (2) 3 (3) 15 (4) 16

70. Which set of employees cannot attend any of the workshops?


(1) Anshul, Charu, Eashwaran and Lavanya
(2) Anshul, Bushkant, Gayatri, and Urvashi
(3) Charu, Urvashi, Bushkant and Mandeep
(4) Anshul, Gayatri, Eashwaran and Mandeep

Page 22 MBA CAT PAPER – 2005


Test Prep
Sub–Section III-B : Number of questions = 20

Note: Questions 71 to 90 carry two marks each.

Directions for questions 71 to 74: Answer the questions on the basis of the information given below:
In the table below is the listing of players, seeded from highest (#1) to lowest (#32), who are due to play in
an Association of Tennis Players (ATP) tournament for women. This tournament has four knockout rounds
before the final, i.e., first round, second round, quarterfinals, and semi-finals. In the first round, the highest
seeded player plays the lowest seeded player (seed #32) which is designated match No. 1 of first round;
the 2nd seeded player plays the 31st seeded player which is designated match No. 2 of the first round, and
so on. Thus, for instance, match No. 16 of first round is to be played between 16th seeded player and the
17th seeded player. In the second round, the winner of match No. 1 of first round plays the winner of match
No. 16 of first round and is designated match No. 1 of second round. Similarly, the winner of match No. 2
of first round plays the winner of match No. 15 of first round, and is designated match No. 2 of second
round. Thus, for instance, match No. 8 of the second round is to be played between the winner of match
No. 8 of first round and the winner of match No. 9 of first round. The same pattern is followed for later rounds
as well.

Seed# Name of Player Seed# Name of Player Seed# Name of Player


1 Maria Sharapova 12 Mary Pierce 23 Silvia Farina Elia
2 Lindsay Davenport 13 Anastasia Myskina 24 Tatiana Golovin
3 Amelie Mauresmo 14 Alicia Molik 25 Shinobu Asagoe
4 Kim Clijsters 15 Nathalie Dechy 26 Francesca Schiavone
5 Svetlana Kuznetsova 16 Elena Bovina 27 Nicole Vaidisova
6 Elena Dementieva 17 Jelena Jankovic 28 Gisela Dulko
7 Justine Henin 18 Ana Ivanovic 29 Flavia Pennetta
8 Serena Williams 19 Vera Zvonareva 30 Anna Chakvetadze
9 Nadia Petrova 20 Elena Likhovtseva 31 Al Sugiyama
10 Venus Williams 21 Daniela Hantuchova 32 Anna-lena Groenefeld
11 Patty Schnyder 22 Dinara Safina

71. If there are no upsets (a lower seeded player beating a higher seeded player) in the first round, and
only match Nos. 6, 7, and 8 of the second round result in upsets, then who would meet Lindsay
Davenport in quarter finals, in case Davenport reaches quarter finals?
(1) Justine Henin (2) Nadia Petrova (3) Patty Schnyder (4) Venus Williams

CAT PAPER – 2005 MBA Page 23


Test Prep
72. If Elena Dementieva and Serena Williams lose in the second round, while Justine Henin and Nadia
Petrova make it to the semi-finals, then who would play Maria Sharapova in the quarterfinals, in the
event Sharapova reaches quarterfinals?
(1) Dinara Safina (2) Justine Henin (3) Nadia Petrova (4) Patty Schnyder

73. If, in the first round, all even numbered matches (and none of the odd numbered ones) result in
upsets, and there are no upsets in the second round, then who could be the lowest seeded player
facing Maria Sharapova in semi-finals?
(1) Anastasia Myskina (2) Flavia Pennetta (3) Nadia Petrova (4) Svetlana Kuznetsova

74. If the top eight seeds make it to the quarterfinals, then who, amongst the players listed below, would
definitely not play against Maria Sharapova in the final, in case Sharapova reaches the final?
(1) Amelie Mauresmo (2) Elena Dementieva (3) Kim Clijsters (4) Lindsay Davenport

Directions for questions 75 to 78: Answer the questions on the basis of the information given below:
Venkat, a stockbroker, invested a part of his money in the stock of four companies — A, B, C and D. Each
of these companies belonged to different industries, viz., Cement, Information Technology (IT), Auto, and
Steel, in no particular order. At the time of investment, the price of each stock was Rs. 100. Venkat
purchased only one stock of each of these companies. He was expecting returns of 20%, 10%, 30% and
40% from the stock of companies A, B, C and D, respectively. Returns are defined as the change in the
value of the stock after one year, expressed as a percentage of the initial value. During the year, two of
these companies announced extraordinarily good results. One of these two companies belonged to the
Cement or the IT industry, while the other one belonged to either the Steel or the Auto industry. As a result,
the returns on the stocks of these two companies were higher than the initially expected returns. For the
company belonging to the Cement or the IT industry with extraordinarily good results, the returns were
twice that of the initially expected returns. For the company belonging to the Steel or the Auto industry, the
returns on announcement of extraordinarily good results were only one and a half times that of the initially
expected returns. For the remaining two companies which did not announce extraordinarily good results,
the returns realized during the year were the same as initially expected.

75. What is the minimum average return Venkat would have earned during the year?
(1) 30% (2) 311/4% (3) 321/2% (4) Cannot be determined

76. If Venkat earned a 35% return on average during the year, then which of these statements would
necessarily be true?
I. Company A belonged either to Auto or to Steel Industry.
II. Company B did not announce extraordinarily good results.
III. Company A announced extraordinarily good results.
IV. Company D did not announce extraordinarily good results.
(1) I and II only (2) II and III only (3) III and IV only (4) II and IV only

Page 24 MBA CAT PAPER – 2005


Test Prep
77. If Venkat earned a 38.75% return on average during the year, then which of these statement(s)
would necessarily be true?
I. Company C belonged either to Auto or to Steel Industry.
II. Company D belonged either to Auto or to Steel Industry.
III. Company A announced extraordinarily good results.
IV. Company B did not announce extraordinarily good results.
(1) I and II only (2) II and III only (3) I and IV only (4) II and IV only

78. If Company C belonged to the Cement or the IT industry and did announce extraordinarily good
results, then which of these statement(s) would necessarily be true?
I. Venkat earned not more than 36.25% return on average.
II. Venkat earned not less than 33.75% return on average.
III. If Venkat earned 33.75% return on average, Company A announced extraordinarily good results.
IV. If Venkat earned 33.75% return on average, Company B belonged either to Auto or to Steel
Industry.
(1) I and II only (2) II and IV only (3) II and III only (4) III and IV only

Directions for questions 79 to 82: Answer the questions on the basis of the information given below:
The year is 2089. Beijing, London, New York, and Paris are in contention to host the 2096 Olympics. The
eventual winner is determined through several rounds of voting by members of the IOC with each member
representing a different city. All the four cities in contention are also represented in IOC.

1) In any round of voting, the city receiving the lowest number of votes in that round gets eliminated.
The survivor after the last round of voting gets to host the event.
2) A member is allowed to cast votes for at most two different cities in all rounds of voting combined.
(Hence, a member becomes ineligible to cast a vote in a given round if both the cities (s)he voted for
in earlier rounds are out of contention in that round of voting.)
3) A member is also ineligible to cast a vote in a round if the city (s)he represents is in contention in
that round of voting.
4) As long as the member is eligible, (s)he must vote and vote for only one candidate city in any round
of voting.
The following incomplete table shows the information on cities that received the maximum and
minimum votes in different rounds, the number of votes cast in their favour, and the total votes that
were cast in those rounds.

Round Total Maximum votes cast Eliminated


votes
City No. of City No. of
1 London 30 New York 12
2 83 Paris 32 Beijing 21
3 75

CAT PAPER – 2005 MBA Page 25


Test Prep
It is also known that:
• All those who voted for London and Paris in round, 1 continued to vote for the same cities in
subsequent rounds as long as these cities were in contention. 75% of those who voted for Beijing in
round 1, voted for Beijing in round 2 as well.
• Those who voted for New York in round 1, voted either for Beijing or Parish in round 2.
• The difference in votes cast for the two contending cities in the last round was a.
• 50% of those who voted for Beijing in round 1, voted for Paris in round 3.

79. What percentage of members from among those who voted for New York in round 1, voted for Beijing
in round 2?
(1) 33.33 (2) 50 (3) 66.67 (4) 75

80. What is the number of votes cast for Paris in round 1?


(1) 16 (2) 18 (3) 22 (4) 24

81. What percentage of members from among those who voted for Beijing in round 2 and were eligible
to vote in round 3, voted for London?
(1) 33.33 (2) 38.10 (3) 50 (4) 66.67

82. Which of the following statements must be true?


I. IOC member from New York must have voted for Paris in round 2.
II. IOC member from Beijing voted for London in round 3.
(1) Only I (2) Only II (3) Both I and II (4) Neither I nor II

Directions for Questions 83 to 86: Answer the questions on the basis of the information given below:
The table below presents the revenue (in million rupees) of four firms in three states. These firms, Honest
Ltd., Aggressive Ltd., Truthful Ltd. And Profitable Ltd. Are disguised in the table as A, B, C and D, in no
particular order.

States Firm A Firm B Firm C Firm D


UP 49 82 80 55
Bihar 69 72 70 65
MP 72 63 72 65

Further, it is known that:


• In the state of MP, Truthful Ltd. Has the highest market share.
• Aggressive Ltd.’s aggregate revenue differs from Honest Ltd.’s by Rs. 5 million .

Page 26 MBA CAT PAPER – 2005


Test Prep
83. What can be said regarding the following two statements?
Statement 1: Profitable Ltd. Has the lowest share in MP market.
Statement 2: Honest Ltd.’s total revenue is more than Profitable Ltd.
(1) If Statement 1 is true then Statement 2 is necessarily true.
(2) If Statement 1 is true then Statement 2 is necessarily false.
(3) Both Statement 1 and Statement 2 are true.
(4) Neither Statement 1 nor Statement 2 is true.

84. What can be said regarding the following two statements?


Statement 1: Aggressive Ltd.’s lowest revenues are from MP.
Statement 2: Honest Ltd.’s lowest revenues are from Bihar.
(1) If Statement 2 is true then Statement 1 is necessarily false.
(2) If Statement 1 is false then Statement 2 is necessarily true.
(3) If Statement 1 is true then Statement 2 is necessarily true
(4) None of the above.

85. What can be said regarding the following two statements?


Statement 1: Honest Ltd. Has the highest share in the UP market.
Statement 2: Aggressive Ltd. Has the highest share in the Bihar market.
(1) Both statements could be true.
(2) At least one of the statements must be true.
(3) At most one of the statements is true.
(4) None of the above.

86. If Profitable Ltd.’s lowest revenue is from UP, then which of the following is true?
(1) Truthful Ltd.’s lowest revenues are from MP.
(2) Truthful Ltd.’s lowest revenues are from Bihar.
(3) Truthful Ltd.’s lowest revenues are from UP.
(4) No definite conclusion is possible.

CAT PAPER – 2005 MBA Page 27


Test Prep
Directions for questions 87 to 90: Answer the questions on the basis of the information given below:
Help Distress (HD) is an NGO involved in providing assistance to people suffering from natural disasters.
Currently, it has 37 volunteers. They are involved in three projects: Tsunami Relief (TR) in Tamil Nadu, Flood
Relief (FR in Maharashtra, and Earthquake Relief (ER) in Gujarat. Each volunteer working with Help Dis-
tress has to be involved in at least one relief work project.

• A maximum number of volunteers are involved in the FR project. Among them, the number of
volunteers involved in FR project alone is equal to the volunteers having additional involvement in the
ER project.
• The number of volunteers involved in the ER project alone is double the number of volunteers involved
in all the three projects.
• 17 volunteers are involved in the TR project.
• The number of volunteers involved in the TR project alone is one less than the number of volunteers
involved in ER project alone.
• Ten volunteers involved in the TR project are also involved in at least one more project.

87. Based on the information given above, the minimum number of volunteers involved in both FR and TR
projects, but not in the ER project is
(1) 1 (2) 3 (3) 4 (4) 5

88. Which of the following additional information would enable to find the exact number of volunteers
involved in various projects?
(1) Twenty volunteers are involved in FR.
(2) Four volunteers are involved in all the three projects.
(3) Twenty three volunteers are involved in exactly one project.
(4) No need for any additional information.

89. After some time, the volunteers who were involved in all the three projects were asked to withdraw
from one project. As a result, one of the volunteers opted out of the TR project, and one opted out
of the ER project, while the remaining ones involved in all the three projects opted out of the FR
project. Which of the following statements, then, necessarily follows?
(1) The lowest number of volunteers is now in TR project.
(2) More volunteers are now in FR project as compared to ER project.
(3) More volunteers are now in TR project as compared to ER project.
(4) None of the above.

90. After the withdrawal of volunteers, as indicated in Question 89, some new volunteers joined the
NGO. Each one of them was allotted only one project in a manner such that, the number of volun-
teers working in one project alone for each of the three projects became identical. At that point, it
was also found that the number of volunteers involved in FR and ER projects was the same as the
number of volunteers involved in TR and ER projects. Which of the projects now has the highest
number of volunteers?
(1) ER (2) FR (3) TR (4) Cannot be determined

Page 28 MBA CAT PAPER – 2005


Test Prep
Section – 1I

Directions for Questions 26 to 29: Answer the following questions based on the information given below:
A health-drink company’s R&D department is trying to make various diet formulations, which can be used
for certain specific purposes. It is considering a choice of 5 alternative ingredients (O, P, Q, R, and S),
which can be used in different proportions in the formulations. The table below gives the composition of
these ingredients. The cost per unit of each of these ingredients is O: 150, P: 50. Q: 200, R: 500, S: 100.

Composition
Ingredient
Carbohydrate% Protein% Fat% Minerals%
O 50 30 10 10
P 80 20 0 0
Q 10 30 50 10
R 5 50 40 5
S 45 50 0 5

26. For a recuperating patient, the doctor recommended a diet containing 10% minerals and at least
30% protein. In how many different ways can we prepare this diet by mixing at least two ingredients?
(1) One (2) Two (3) Three (4) Four (5) None

27. Which among the following is the formulation having the lowest cost per unit for a diet having 10% fat
and at least 30% protein? (The diet has to be formed by mixing two ingredients).
(1) P and Q (2) P and S (3) P and R (4) Q and S (5) R and S

28. In what proportion P, Q and S should be mixed to make a diet having at least 60% carbohydrate at
the lowest cost per unit?
(1) 2:1:3 (2) 4:1:2 (3) 2:1:4 (4) 3:1:2 (5) 4:1:1

29. The company is planning to launch a balanced diet required for growth needs of adolescent children.
This diet must contain at least 30% each of carbohydrate and protein, no more than 25% fat and at
least 5% minerals. Which one of the following combinations of equally mixed ingredients is feasible?
(1) O and P (2) R and S (3) P and S
(4) Q and R (5) O and S

CAT PAPER – 2007 MBA Page 7


Test Prep
Directions for Questions 30 to 33: Each question is followed by two statements, A and B.
Answer each question using the following instructions:
Mark (1) if the question can be answered by using the statement A alone but not by using the statement
B alone.
Mark (2) if the question can be answered by using the statement B alone but not by using the statement
A alone.
Mark (3) if the question can be answered by using either of the statements alone.
Mark (4) if the question can be answered by using both the statements together but not by either of the
statements alone.
Mark (5) if the question cannot be answered on the basis of the two statements.

30. In a particular school, sixty students were athletes. Ten among them were also among the top
academic performers. How many top academic performers were in the school?
A. Sixty per cent of the top academic performers were not athletes.
B. All the top academic performers were not necessarily athletes.

31. Five students Atul, Bala, Chetan, Dev and Ernesto were the only ones who participated in a quiz
contest. They were ranked based on their scores in the contest. Dev got a higher rank as compared
to Ernesto, while Bala got a higher rank as compared to Chetan. Chetan’s rank was lower than the
median. Who among the five got the highest rank?
A. Atul was the last rank holder.
B. Bala was not among the top two rank holders.

32. Thirty per cent of the employees of a call centre are males. Ten per cent of the female employees
have an engineering background. What is the percentage of male employees with engineering
background?
A. Twenty five per cent of the employees have engineering background.
B. Number of male employees having an engineering background is 20% more than the number of
female employees having an engineering background.

33. ln a football match, at the half-time, Mahindra and Mahindra Club was trailing by three goals. Did it
win the match?
A. In the second-half Mahindra and Mahindra Club scored four goals.
B. The opponent scored four goals in the match.

Page 8 MBA CAT PAPER – 2007


Test Prep
Directions for Questions 34 to 37: Answer the following questions based on the information given below:
The following table shows the break-up of actual costs incurred by a company in last five years (year 2002
to year 2006) to produce a particular product:

Year 2002 Year 2003 Year 2004 Year 2005 Year 2006
Volume of production and sale (units) 1000 900 1100 1200 1200
Costs (Rs.)
Material 50,000 45,100 55,200 59,900 60,000
Labour 20,000 18,000 22,100 24,150 24,000
Consumables 2,000 2,200 1,800 1,600 1,400
Rent of building 1,000 1,000 1,100 1,100 1,200
Rates and taxes 400 400 400 400 400
Repair and maintenance expenses 800 820 780 790 800
Operating cost of machines 30,000 27,000 33,500 36,020 36,000
Selling and marketing expenses 5,750 5,800 5,800 5,750 5,800

The production capacity of the company is 2000 units. The selling price for the year 2006 was
Rs. 125 per unit. Some costs change almost in direct proportion to the change in volume of production,
while others do not follow any obvious pattern of change with respect to the volume of production and hence
are considered fixed. Using the information provided for the year 2006 as the basis for projecting the figures
for the year 2007, answer the following questions:

34. What is the approximate cost per unit in rupees, if the company produces and sells 1400 units in
the year 2007?
(1) 104 (2) 107 (3) 110 (4) 115 (5) 116

35. What is the minimum number of units that the company needs to produce and sell to avoid any
loss?
(1) 313 (2) 350 (3) 384 (4) 747 (5) 928

36. If the company reduces the price by 5%, it can produce and sell as many units as it desires. How
many units the company should produce to maximize its profit?
(1) 1400 (2) 1600 (3) 1800 (4) 1900 (5) 2000

37. Given that the company cannot sell more than 1700 units, and it will have to reduce the price by
Rs.5 for all units, if it wants to sell more than 1400 units, what is the maximum profit, in rupees, that
the company can earn?
(1) 25,400 (2) 24,400 (3) 31,400 (4) 32,900 (5) 32,000

CAT PAPER – 2007 MBA Page 9


Test Prep
Directions for Questions 38 to 41: Answer the following questions based on the information given below:
The proportion of male students and the proportion of vegetarian students in a school are given below.
The school has a total of 800 students, 80% of whom are in the Secondary Section and rest are equally
divided between Class 11 and 12.

Male (M) Vegetarian (V)


Class 12 0.6
Class 11 0.55 0.5
Secondary Section 0.55
Total 0.475 0.53

38. What is the percentage of male students in the secondary section?


(1) 40 (2) 45 (3) 50 (4) 55 (5) 60

39. In Class 12, twenty five per cent of the vegetarians are male. What is the difference between the
number of female vegetarians and male non-vegetarians?
(1) less than 8 (2) 10 (3) 12 (4) 14 (5) 16

40. What is the percentage of vegetarian students in Class 12?


(1) 40 (2) 45 (3) 50 (4) 55 (5) 60

41. In the Secondary Section, 50% of the students are vegetarian males. Which of the following
statements is correct?
(1) Except vegetarian males, all other groups have same number of students.
(2) Except non-vegetarian males, all other groups have same number of students.
(3) Except vegetarian females, all other groups have same number of students.
(4) Except non-vegetarian females, all other groups have same number of students.
(5) All of the above groups have the same number of students.

Directions for Questions 42 to 45: Answer the following questions based on the information given below:
The Table below shows the comparative costs, in US Dollars, of major surgeries in USA and a select few
Asian countries.
Comparative Costs in USA and some Asian countries
Procedure (in US Dollar)
USA India Thailand Singapore Malaysia
Heart Bypasss 130000 10000 11000 18500 9000
Heart Valve Replacement 160000 9000 10000 12500 9000
Angioplasty 57000 11000 13000 13000 11000
Hip Replacement 43000 9000 12000 12000 10000
Hysterectomy 20000 3000 4500 6000 3000
Knee Replacement 40000 8500 10000 13000 8000
Spinal Fusion 62000 5500 7000 9000 6000

Page 10 MBA CAT PAPER – 2007


Test Prep
The equivalent of one US Dollar in the local currencies is given below:

1 US Dollar equivalent
India 40.928 Rupees
Malaysia 3.51 Ringits
Thailand 32.89 Bahts
Singapore 1.53 S Dollars

A consulting firm found that the quality of the health services were not the same in all the countries above.
A poor quality of a surgery may have significant repercussions in future, resulting in more cost in correcting
mistakes. The cost of poor quality of surgery is given in the table below:

Procedure Comparative cost of poor quality in USA and some Asian


countries (in US Dollars '000)
USA India Thailand Singapore Malaysia
Heart Bypasss 0 3 3 2 4
Heart Valve Replacement 0 5 4 5 5
Angioplasty 0 5 5 4 6
Hip Replacement 0 7 5 5 8
Hysterectomy 0 5 6 5 4
Knee Replacement 0 9 6 4 4
Spinal Fusion 0 5 6 5 6

42. A US citizen is hurt in an accident and requires an angioplasty, hip replacement and a knee
replacement. Cost of foreign travel and stay is not a consideration since the government will take
care of it. Which country will result in the cheapest package, taking cost of poor quality into account?
(1) India (2) Thailand (3) Malaysia (4) Singapore (5) USA

43. Taking the cost of poor quality into account, which country/countries will be the most expensive for
knee replacement?
(1) India (2) Thailand (3) Malaysia
(4) Singapore (5) India and Singapore

44. Approximately, what difference in amount in Bahts will it make to a Thai citizen if she were to get a
hysterectomy done in India instead of in her native country, taking into account the cost of poor
quality? (It costs 7500 Bahts for one-way travel between Thailand and India).
(1) 23500 (2) 40500 (3) 57500 (4) 67500 (5) 75000

45. The rupee value increases to Rs.35 for a US Dollar, and all other things including quality, remain the
same. What is the approximate difference in cost, in US Dollars, between Singapore and India for a
Spinal Fusion, taking this change into account?
(1) 700 (2) 2500 (3) 4500 (4) 8000 (5) No difference

CAT PAPER – 2007 MBA Page 11


Test Prep
Directions for Questions 46 to 50: Answer the following questions based on the information given below:
A low-cost airline company connects ten Indian cities, A to J. The table below gives the distance between
a pair of airports and the corresponding price charged by the company. Travel is permitted only from a
departure airport to an arrival airport. The customers do not travel by a route where they have to stop at
more than two intermediate airports.

Sector Airport of Airport of Distance Price (Rs.)


No Departure Arrival between the
Airports (km)
1 A B 560 670
2 A C 790 1350
3 A D 850 1250
4 A E 1245 1600
5 A F 1345 1700
6 A G 1350 2450
7 A H 1950 1850
8 B C 1650 2000
9 B H 1750 1900
10 B I 2100 2450
11 B J 2300 2275
12 C D 460 450
13 C F 410 430
14 C G 910 1100
15 D E 540 590
16 D F 625 700
17 D G 640 750
18 D H 950 1250
19 D J 1650 2450
20 E F 1250 1700
21 E G 970 1150
22 E H 850 875
23 F G 900 1050
24 F I 875 950
25 F J 970 1150
26 G I 510 550
27 G J 830 890
28 H I 790 970
29 H J 400 425
30 I J 460 540

Page 12 MBA CAT PAPER – 2007


Test Prep
46. What is the lowest price, in rupees, a passenger has to pay for travelling by the shortest route from
A to J?
(1) 2275 (2) 2850 (3) 2890 (4) 2930 (5) 3340

47. The company plans to introduce a direct flight between A and J. The market research results
indicate that all its existing passengers travelling between A and J will use this direct flight if it is
priced 5% below the minimum price that they pay at present. What should the company charge
approximately, in rupees, for this direct flight?
(1) 1991 (2) 2161 (3) 2707 (4) 2745 (5) 2783

48. If the airports C, D and H are closed down owing to security reasons, then what would be the
minimum price, in rupees, to be paid by a passenger travelling from A to J?
(1) 2275 (2) 2615 (3) 2850 (4) 2945 (5) 3190

49. If the prices include a margin of 10% over the total cost that the company incurs, then what is the
minimum cost per kilometer that the company incurs in flying from A to J?
(1) 0.77 (2) 0.88 (3) 0.99 (4) 1.06 (5) 1.08

50. If the prices include a margin of 15% over the total cost that the company incurs, then which among
the following is the distance to be covered in flying from A to J that minimizes the total cost per
kilometer for the company?
(1) 2170 (2) 2180 (3) 2315 (4) 2350 (5) 2390

CAT PAPER – 2007 MBA Page 13


Test Prep
Section – 1I

Directions for Questions 26 to 28: Answer the following questions based on the statements given
below:

(i) There are three houses on each side of the road.


(ii) These six houses are labeled as P, Q, R, S, T and U.
(iii) The houses are of different colours, namely, Red, Blue, Green, Orange, Yellow and White.
(iv) The houses are of different heights.
(v) T, the tallest house, is exactly opposite to the Red coloured house.
(vi) The shortest house is exactly opposite to the Green coloured house.
(vii) U, the Orange coloured house, is located between P and S.
(viii) R, the Yellow coloured house, is exactly opposite to P.
(ix) Q, the Green coloured house, is exactly opposite to U.
(x) P, the White coloured house, is taller than R, but shorter than S and Q.

26. What is the colour of the house diagonally opposite to the Yellow coloured house?
(1) White (2) Blue (3) Green
(4) Red (5) none of these

27. Which is the second tallest house?


(1) P (2) S (3) Q
(4) R (5) cannot be determined

28. What is the colour of the tallest house?


(1) Red (2) Blue (3) Green
(4) Yellow (5) none of these

Page 6 MBA CAT PAPER – 2008


Test Prep
Directions for Questions 29 to 31: Answer the following questions based on the information given
below:

Telecom operators get revenue from transfer of data and voice. Average revenue received from transfer of
each unit of data is known as ARDT. In the diagram below, the revenue received form data transfer as
percentage of total revenue received and the ARDT in US Dollars (USD) are given for various countries.

P h ilip pine s ($3 .5 4% ) Japa n ($1 3,70 % )

In do ne sia ($ 2,4 2% )
UK
R e ve n ue fro m D a ta Tran sfer as a % o f To ta l R eve nu e

M alaysia
30 %

G erm a ny
C h in a S w itzerlan d S o uth K ore a
P o la nd Ire la nd

S ing ap ore N o rw ay
A u stria
20 %

R u ssia S w e de n
M exico
USA
H o ng K o ng Spain

Isra el
Th aila nd D e nm a rk C a na da
1 0%

In dia B ra zil

$5 $ 10 $1 5
A R D T (in U S D )

Legend: A S IA EURO PE A M E R IC A S

29. It was found that the volume of data transfer in India is the same as that of Singapore. Then which
of the following statements is true?
(1) Total revenue is the same in both countries.
(2) Total revenue in India is about 2 times that of Singapore.
(3) Total revenue in India is about 4 times that of Singapore.
(4) Total revenue in Singapore is about 2 times that of India.
(5) Total revenue in Singapore is about 4 time that of India.

CAT PAPER – 2008 MBA Page 7


Test Prep
30. It is expected that by 2010, revenue from data transfer as a percentage of total revenue will triple
for India and double for Sweden. Assume that in 2010, the total revenue in India is twice that of
Sweden and that the volume of data transfer is the same in both the countries. What is the
percentage increase of ARDT in India if there is no change in ARDT in Sweden?
(1) 400% (2) 550% (3) 800%
(4) 950% (5) cannot be determined

31. If the total revenue received is the same for the pairs of countries listed in the choices
below, choose the pair that has approximately the same volume of data transfer.
(1) Philippines and Austria
(2) Canada and Poland
(3) Germany and USA
(4) UK and Spain
(5) Denmark and Mexico

Directions for Questions 32 to 34: Answer the following questions based on the information given below:

For admission to various affiliated colleges, a university conducts a written test with four different sections,
each with a maximum of 50 marks. The following table gives the aggregate as well as the sectional cut-off
marks fixed by six different colleges affiliated to the university. A student will get admission only if he/she
gets marks greater than or equal to the cut-off marks in each of the sections and his/her aggregate marks
are at least equal to the aggregate cut-off marks as specified by the college.

Sectional Cut – off Marks Aggregate


Section A Section B Section C Section D Current Marks
College 1 42 42 42 176
College 2 45 45 175
College 3 46 171
College 4 43 45 178
College 5 45 43 180
College 6 41 44 176

32. Bhama got calls from all colleges. What could be the minimum aggregate marks obtained
by her?
(1) 180 (2) 181 (3) 196 (4) 176 (5) 184

33. Charlie got calls from two colleges. What could be the minimum marks obtained by him in a
section?
(1) 0 (2) 21 (3) 25 (4) 35 (5) 41

34. Aditya did not get a call from even a single college. What could be the maximum aggregate
marks obtained by him?
(1) 181 (2) 176 (3) 184 (4) 196 (5) 190

Page 8 MBA CAT PAPER – 2008


Test Prep
Directions for Questions 35 to 38: Answer the following questions based on the information given below:
In a sports event, six teams (A, B, C, D, E and F) are competing against each other Matches are sched-
uled in two stages. Each team plays three matches in State – I and two matches in Stage – II. No team
plays against the same team more than once in the event. No ties are permitted in any of the matches. The
observations after the completion of Stage – I and Stage – II are as given below.

Stage-I:
• Once team won all the three matches.
• Two teams lost all the matches.
• D lost to A but won against C and F.
• E lost to B but won against C and F.
• B lost at least one match.
• F did not play against the top team of Stage-I.

Stage-II:
• The leader of Stage-I lost the next two matches
• Of the two teams at the bottom after Stage-I, one team won both matches, while the other lost both
matches.
• Once more team lost both matches in Stage-II.

35. The two teams that defeated the leader of Stage-I are:
(1) F & D (2) E & F (3) B & D (4) E & D (5) F & D

36. The only team(s) that won both matches in Stage-II is (are):
(1) B (2) E & F (3) A, E & F (4) B, E & F (5) B & F

37. The teams that won exactly two matches in the event are:
(1) A, D & F (2) D & E (3) E & F (4) D, E & F (5) D & F

38. The team(s) with the most wins in the event is (are):
(1) A (2) A & C (3) F (4) E (5) B & E

CAT PAPER – 2008 MBA Page 9


Test Prep
Directions for Questions 39 to 42: Answer the following questions based on the information given below:
The bar chart below shows the revenue received in million US Dollars (USD), from subscribers to a particular
Internet service. The data covers the period 2003 to 2007 for the United States (US) and Europe. The bar
chart also shows the estimated revenues from subscription to this service for the period 2008 to 2010.

1000

900
Subscription Revenue in Million USD

800

700

600

500 US
400 Europe

300

200

100

0
03 04 05 06 07 08 09 10
Years

39. The difference between the estimated subscription in Europe in 2008 and what it would have been if
it were computed using the percentage growth rate of 2007 (over 2006), is closest to:
(1) 50 (2) 80 (3) 20 (4) 10 (5) 0

40. In 2003, sixty percent of subscribers in Europe were men. Given that women subscribers increase
at the rate of 10 percent annum and men at the rate of 5 percent per annum, what is the approximate
percentage growth of subscribers between 2003 and 2010 in Europe? The subscription prices are
volatile and may change each year.
(1) 62 (2) 15 (3) 78 (4) 84 (5) 50

41. Consider the annual percent change in the gap between subscription revenues in the US and
Europe. What is the year in which the absolute value of this change is the highest?
(1) 03 - 04 (2) 05 - 06 (3) 06 - 07 (4) 08 - 09 (5) 09 - 10

42. While the subscription in Europe has been growing steadily towards that of the US, the growth rate
in Europe seems to be declining. Which of the following is closest to the percent change in growth
rate of 2007 (over 2006) relative to the growth rate of 2005 (over 2004)?
(1) 17 (2) 20 (3) 35 (4) 60 (5) 100

Page 10 MBA CAT PAPER – 2008


Test Prep
Directions for Questions 43 to 47: Answer the following questions based on the information given below:

Abdul, Bikram and Chetan are three professional traders who trade in shares of a company XYZ Ltd. Abdul
follows the strategy of buying at the opening of the day at 10 am and selling the whole lot at the close of the
day at 3 pm. Bikram follows the strategy of buying at hourly intervals: 10 am, 11am, 12 noon, I pm. And 2
pm, and selling the whole lot at the close of the day, Further, he buys an equal number of shares in each
purchase. Chetan follows a similar pattern as Bikram but his strategy is somewhat different. Chetan’s total
investment amount is divided equally among his purchases. The profit or loss made by each investor is the
difference between the sale value at the close of the day less the investment in purchase. The “return” for
each investor is defined as the ratio of the profit or loss to the investment amount expressed as a percent-
age.

43. On a day of fluctuating market prices, the share price of XYZ Ltd. ends with a gain, i.e, it is higher
at the close of the day compared to the opening value. Which trader got the maximum return on that
day?
(1) Bikram
(2) Chetan
(3) Abdul
(4) Bikram or Chetan
(5) cannot be determined

44. Which one of the following statements is always true?


(1) Abdul will not be one with the minimum return
(2) Return for Chetan will be higher than that of Bikram
(3) Return for Bikram will be higher than that of Chetan
(4) Return for Chetan cannot be higher than that of Abdul
(5) none of the above

45. On a “boom” day the share price of XYZ Ltd. keeps rising throughout the day and dpeaks at the
close of the day. Which trader got the minimum return on that day?
(1) Bikram (2) Chetan (3) Abdul
(4) Abdul or Chetan (5) cannot be determined

One day, two other traders. Dane and Emily joined Abdul, Bikram and Chetan for trading in the shares of
XYZ Ltd. Dane followed a strategy of buying equal numbers of shares at 10 am. 11 am and 12 noon, and
selling the same numbers at 1 pm, 2 pm and 3 pm Emily, on the other hand, followed the strategy of buying
shares using all her money at 10 am and selling all of them at 12 noon and again buying the shares for all
the money at 1 pm and again selling all of them at the close of the day at 3 pm. At the close of the day the
following was observed.
i. Abdul lost money in the transactions.
ii. Both Dane and Emily made profits.
iii. There was an increase in share price during the closing hour compared to the price at 2 pm.
iv. Share price at 12 noon was lower than the opening price

46. Share price was at its highest at


(1) 10 am (2) 11 am (3) 12 noon
(4) 1 pm (5) cannot be determined

CAT PAPER – 2008 MBA Page 11


Test Prep
47. Which of the following is necessarily false?
(1) Share price was at its lowest at 2 pm
(2) Share price was at its lowest at 11 am
(3) Share price at 1 pm was higher than the share price at 2 pm
(4) Share price at 1 pm was higher than the share price at 12 noon
(5) none of the above

Directions for Questions 48 to 50: Answer the following questions based on the information given below:

There are 100 employees in an organization across five departments. The following table gives the depart-
ment-wise distribution of average age, average basic pay and allowances. The gross pay of an employee is
the sum of his/her basic pay and allowances.

Number of Average Average Basic Allowances


Department
Employees Age (Years) Pay (Rs.) (% of Basic Pay)
HR 5 45 5000 70
Marketing 30 35 6000 80
Finance 20 30 6500 60
Business
35 42 7500 75
Development
Maintenance 10 35 5500 50

There are limited numbers of employees considered for transfer/promotion across departments Whenever
a person is transferred/promoted from a department of lower average age to a department of higher average
age, he/she will get an additional allowance of 10% of basic pay over and above his/her current allowance.
There will not be any change in pay structure if a person is transferred/promoted from a department with
higher average age to a department with lower average age.

Questions below are independent of each other.

48. What is the approximate percentage change in the average gross of the HR department due to
transfer of a 40-year old person with basic pay of Rs. 8000 from the Marketing department?
(1) 9% (2) 11% (3) 13% (4) 15% (5) 17%

49. There was a mutual transfer of an employee between Marketing and Finance departments and
transfer of one employee from Marketing to HR. As a result, the average age of finance department
increased by one year and that of Marketing department remained the same. What is the new
average age of HR department?
(1) 30 (2) 35 (3) 40
(4) 45 (5) cannot be determined

50. If two employees (each with a basic pay of Rs. 6000) are transferred from Maintenance department
to HR department and one person (with a basic pay of Rs. 8000) was transferred from Marketing
department to HR department, what will be the percentage change in average basic pay of HR
department?
(1) 10.5% (2) 12.5% (3) 15% (4) 30% (5) 40%

Page 12 MBA CAT PAPER – 2008


Test Prep
Section – 1

Directions for questions 1 to 5: Answer the questions on the basis of the information given below:
K, L, M, N, P, Q, R, S, U and W are the only ten members in a department. There is a proposal to form a
team from within the members of the department, subject to the following conditions:

1. A team must include exactly one among P, R, and S.


2. A team must include either M or Q, but not both.
3. If a team includes K, then it must also include L, and vice versa.
4. If a team includes one among S, U, and W, then it must also include the other two.
5. L and N cannot be members of the same team.
6. L and U cannot be members of the same team.

The size of a team is defined as the number of members in the team.

1. Who cannot be a member of a team of size 3?


(1) L (2) M (3) N (4) P (5) Q

2. Who can be a member of a team of size 5?


(1) K (2) L (3) M (4) P (5) R

3. What would be the size of the largest possible team?


(1) 8 (2) 7 (3) 6 (4) 5 (5) Cannot be determined

4. What could be the size of a team that includes K?


(1) 2 or 3 (2) 2 or 4 (3) 3 or 4 (4) Only 2 (5) Only 4

5. In how many ways a team can be constituted so that the team includes N?
(1) 2 (2) 3 (3) 4 (4) 5 (5) 6

CAT PAPER – 2006 MBA Page 1


Test Prep
Directions for questions 6 to 10: Answer questions on the basis of the information given below:

In a Class X Board examination, ten papers are distributed over five Groups - PCB, Mathematics, Social
Science, Vernacular and English. Each of the ten papers is evaluated out of 100. The final score of a
student is calculated in the following manner. First, the Group Scores are obtained by averaging marks in
the papers within the Group. The final score is the simple average of the Group Scores. The data for the top
ten students are presented below. (Dipan's score in English Paper II has been intentionally removed in the
table.)

Social
Vernacular
Name of the PCB Group Mathematics Science English Group
Group
student Group Group Final
Phy. Chem. Bio. Hist. Geo. Paper I Paper II Paper I Paper II Score
Ayesha (G) 98 96 97 98 95 93 94 96 96 98 96.2
Ram (B) 97 99 95 97 95 96 94 94 96 98 96.1
Dipan (B) 98 98 98 95 96 95 96 94 96 ?? 96.0
Sagnik (B) 97 98 99 96 96 98 94 97 92 94 95.9
Sanjiv (B) 95 96 97 98 97 96 92 93 95 96 95.7
Shreya (G) 96 89 85 100 97 98 94 95 96 95 95.5
Joseph (B) 90 94 98 100 94 97 90 92 94 95 95
Agni (B) 96 99 96 99 95 96 82 93 92 93 94.3
Pritam (B) 98 98 95 98 83 95 90 93 94 94 93.9
Tirna (G) 96 98 97 99 85 94 92 91 87 96 93.7

Note: B or G against the name of a student respectively indicates whether the student is a boy or a girl.

6. How much did Dipan get in English Paper II?


(1) 94 (2) 96.5 (3) 97 (4) 98 (5) 99

7. Among the top ten students, how many boys scored at least 95 in at least one paper from each of
the groups?
(1) 1 (2) 2 (3) 3 (4) 4 (5) 5

8. Had Joseph, Agni, Pritam and Tirna each obtained Group Score of 100 in the Social Science Group,
then their standing in decreasing order of final score would be:
(1) Pritam, Joseph, Tirna, Agni (2) Joseph, Tirna, Agni, Pritam
(3) Pritam, Agni, Tirna, Joseph (4) Joseph, Tirna, Pritam, Agni
(5) Pritam, Tirna, Agni, Joseph

Page 2 MBA CAT PAPER – 2006


Test Prep
9. Students who obtained Group Scores of at least 95 in every group are eligible to apply for a prize.
Among those who are eligible, the student obtaining the highest Group Score in Social Science
Group is awarded this prize. The prize was awarded to:
(1) Shreya (2) Ram
(3) Ayesha (4) Dipan
(5) No one from the top ten

10. Each of the ten students was allowed to improve his/her score in exactly one paper of choice with
the objective of maximizing his/her final score. Everyone scored 100 in the paper in which he or she
chose to improve. After that, the topper among the ten students was:
(1) Ram (2) Agni (3) Pritam (4) Ayesha (5) Dipan

Directions for questions 11 to 15: Answer the questions on the basis of the information given below:
Mathematicians are assigned a number called Erdös number (named after the famous mathematician,
Paul Erdös). Only Paul Erdös himself has an Erdös number of zero. Any mathematician who has written
a research paper with Erdös has an Erdös number of 1. For other mathematicians, the calculation of his/
her Erdös number is illustrated below:
Suppose that a mathematician X has co-authored papers with several other mathematicians. From among
them, mathematician Y has the smallest Erdös number. Let the Erdös number of Y be y. Then X has an
Erdös number of y+1 . Hence any mathematician with no co-authorship chain connected to Erdös has an
Erdös number of infinity.
In a seven day long mini-conference organized in memory of Paul Erdös, a close group of eight mathema-
ticians, call them A, B, C, D, E, F, G and H, discussed some research problems. At the beginning of the
conference, A was the only participant who had an infinite Erdös number. Nobody had an Erdös number
less than that of F.
1 On the third day of the conference F co-authored a paper jointly with A and C. This reduced the
average Erdös number of the group of eight mathematicians to 3. The Erdös numbers of B, D, E, G
and H remained unchanged with the writing of this paper. Further, no other co-authorship among any
three members would have reduced the average Erdös number of the group of eight to as low as 3.
2 At the end of the third day, five members of this group had identical Erdös numbers while the other
three had Erdös numbers distinct from each other.
3 On the fifth day, E co-authored a paper with F which reduced the group's average Erdös number by
0.5. The Erdös numbers of the remaining six were unchanged with the writing of this paper.
4 No other paper was written during the conference.

11. How many participants in the conference did not change their Erdös number during the conference?
(1) 2 (2) 3 (3) 4 (4) 5 (5) Cannot be determined

CAT PAPER – 2006 MBA Page 3


Test Prep
12. The person having the largest Erdös number at the end of the conference must have had Erdös
number (at that time):
(1) 5 (2) 7 (3) 9 (4) 14 (5) 15

13. How many participants had the same Erdös number at the beginning of the conference?
(1) 2 (2) 3 (3) 4 (4) 5 (5) Cannot be determined

14. The Erdös number of C at the end of the conference was:


(1) 1 (2) 2 (3) 3 (4) 4 (5) 5

15. The Erdös number of E at the beginning of the conference was:


(1) 2 (2) 5 (3) 6 (4) 7 (5) 8

Directions for questions 16 to 20: Answer the questions on the basis of the information given below:

Two traders, Chetan and Michael, were involved in the buying and selling of MCS shares over five trading
days. At the beginning of the first day, the MCS share was priced at Rs 100, while at the end of the fifth day
it was priced at Rs 110. At the end of each day, the MCS share price either went up by Rs 10, or else, it
came down by Rs 10. Both Chetan and Michael took buying and selling decisions at the end of each
trading day. The beginning price of MCS share on a given day was the same as the ending price of the
previous day. Chetan and Michael started with the same number of shares and amount of cash, and had
enough of both. Below are some additional facts about how Chetan and Michael traded over the five trading
days.

1 Each day if the price went up, Chetan sold 10 shares of MCS at the closing price. On the other
hand, each day if the price went down, he bought 10 shares at the closing price.
2 If on any day, the closing price was above Rs 110, then Michael sold 10 shares of MCS, while if it
was below Rs 90, he bought 10 shares, all at the closing price.

16. If Chetan sold 10 shares of MCS on three consecutive days, while Michael sold 10 shares only once
during the five days, what was the price of MCS at the end of day 3?
(1) Rs 90 (2) Rs 100 (3) Rs 110 (4) Rs 120 (5) Rs 130

17. If Chetan ended up with Rs 1300 more cash than Michael at the end of day 5, what was the price of
MCS share at the end of day 4?
(1) Rs 90 (2) Rs 100 (3) Rs 110
(4) Rs 120 (5) Not uniquely determinable

18. If Michael ended up with 20 more shares than Chetan at the end of day 5, what was the price of the
share at the end of day 3?
(1) Rs 90 (2) Rs 100 (3) Rs 110 (4) Rs 120 (5) Rs 130

Page 4 MBA CAT PAPER – 2006


Test Prep
19. If Michael ended up with Rs 100 less cash than Chetan at the end of day 5, what was the difference
in the number of shares possessed by Michael and Chetan (at the end of day 5)?
(1) Michael had 10 less shares than Chetan.
(2) Michael had 10 more shares than Chetan.
(3) Chetan had 10 more shares than Michael,
(4) Chetan had 20 more shares than Michael.
(5) Both had the same number of shares.

20. What could have been the maximum possible increase in combined cash balance of Chetan and
Michael at the end of the fifth day?
(1) Rs 3700 (2) Rs 4000 (3) Rs 4700 (4) Rs 5000 (5) Rs 6000

Directions for questions 21 to 25: Answer the questions on the basis of the information given below:

A significant amount of traffic flows from point S to point T in the one-way street network shown below.
Points A, B, C, and D are junctions in the network, and the arrows mark the direction of traffic flow. The fuel
cost in rupees for travelling along a street is indicated by the number adjacent to the arrow representing the
street.

A
9 5
2
2 3 2
S B C T

7 1 6

Motorists travelling from point S to point T would obviously take the route for which the total cost of
travelling is the minimum. If two or more routes have the same least travel cost, then motorists are indiffer-
ent between them. Hence, the traffic gets evenly distributed among all the least cost routes.

The government can control the flow of traffic only by levying appropriate toll at each junction. For example,
if a motorist takes the route S-A-T (using junction A alone), then the total cost of travel would be Rs 14 (i.e.,
Rs 9 + Rs 5) plus the toll charged at junction A.

21. If the government wants to ensure that no traffic flows on the street from D to T, while equal amount
of traffic flows through junctions A and C, then a feasible set of toll charged (in rupees) at junctions
A, B, C, and D respectively to achieve this goal is:
(1) 1,5,3,3 (2) 1,4,4,3 (3) 1,5,4,2 (4) 0,5,2,3 (5) 0,5,2,2

CAT PAPER – 2006 MBA Page 5


Test Prep
22. If the government wants to ensure that all motorists travelling from S to T pay the same amount (fuel
costs and toll combined) regardless of the route they choose and the street from B to C is under
repairs (and hence unusable), then a feasible set of toll charged (in rupees) at junctions A, B, C, and
D respectively to achieve this goal is:
(1) 2,5,3,2 (2) 0,5,3, 1 (3) 1,5,3,2 (4) 2,3,5,1 (5) 1,3,5,1

23. If the government wants to ensure that the traffic at S gets evenly distributed along streets from S to
A, from S to B, and from S to D, then a feasible set of toll charged (in rupees) at junctions A, B, C,
and D respectively to achieve this goal is:
(1) 0,5,4,1 (2) 0,5,2,2 (3) 1,5,3,3 (4) 1,5,3,2 (5) 0,4,3,2

24. If the government wants to ensure that all routes from S to T get the same amount of traffic, then a
feasible set of toll charged (in rupees) at junctions A, B, C, and D respectively to achieve this goal
is:
(1) 0,5,2,2 (2) 0,5,4,1 (3) 1,5,3,3 (4) 1,5,3,2 (5)1,5,4,2

25. The government wants to devise a toll policy such that the total cost to the commuters per trip is
minimized. The policy should also ensure that not more than 70 per cent of the total traffic passes
through junction B. The cost incurred by the commuter travelling from point S to point T under this
policy will be:
(1) Rs. 7 (2) Rs. 9 (3) Rs. 10 (4) Rs. 13 (5) Rs. 14

Page 6 MBA CAT PAPER – 2006


Test Prep
CAT PAPER – 2004

ANSWERS and EXPLANATIONS

1 1 2 4 3 2 4 3 5 3 6 4 7 4 8 1 9 1 10 3
11 1 12 2 13 2 14 3 15 4 16 1 17 4 18 1 19 2 20 4
21 1 22 1 23 1 24 2 25 3 26 4 27 3 28 1 29 2 30 2
31 4 32 3 33 3 34 4 35 4 36 2 37 4 38 4 39 3 40 2
41 1 42 2 43 1 44 2 45 1 46 1 47 3 48 4 49 4 50 2
51 3 52 4 53 2 54 2 55 3 56 4 57 4 58 3 59 4 60 3
61 3 62 2 63 3 64 2 65 1 66 2 67 2 68 4 69 4 70 1
71 3 72 3 73 1 74 2 75 1 76 4 77 3 78 2 79 4 80 3
81 1 82 4 83 1 84 3 85 2 86 1 87 3 88 3 89 4 90 2
91 4 92 2 93 1 94 2 95 4 96 1 97 2 98 1 99 3 100 3
101 1 102 1 103 2 104 4 105 2 106 1 107 1 108 3 109 4 110 4
111 3 112 3 113 3 114 3 115 2 116 3 117 2 118 2 119 3 120 1
121 3 122 2 123 4

Scoring table
Section Question Total Total Total Total Net Time
number questions attempted correct wrong Score Taken
DI 1 to 38 38

Quant 39 to 73 35

EU + RC 74 to 123 50

Total 123

SUP-0001/09

CAT PAPER – 2004 Explanations MBA Page 1


Test Prep
1. 1 GPA of Preeti = 3.2 Solution for questions 9 to 12:
F+D+ X+D+ Y Comparing Table 1 and 2, university 4 corresponds to UK and
i.e. = 3.2
5 university 6 corresponds to USA (after as day 3 values are
0 + 2 + x + 2 + y = 16 concerned and university 8 corresponds to India and university
x + y = 12 3 to Netherlands now Indian or Netherlands can take university
So only combination possible is A, A. 1 or university 5. Now university 2 and 7 belongs to either UK
So Preeti obtained A grade in statistics. or Canada (only one)
UNIVERSITY DAY COUNTRY
2. 4 Tara received same grade in 3 courses. We already
know that Tara has got B grade in one of the subject 1 2 3
and GPA is 2.4. So in 3 courses in which he scored University 1 1 0 0 India / Netherlands
same grade is B.
So Tara has received the same grade as Manab. University 2 2 0 0 UK / Canada
University 3 0 1 0 Netherlands
3. 2 GPA of Gowri is 3.8
i.e. 3 + 3 + 6 + x + 4 = 3.8 × 5 University 4 0 0 2 UK
16 + x = 18 University 5 1 0 0 India/Netherlands
x=2
So in strategy, Gowri's grade is C. University 6 1 0 1 USA
Rahul's grade in strategy = (4.2 × 5) – 15 = 6, i.e., A. University 7 2 0 0 UK/Canada
Fazal's grade in strategy = (2.4 × 5) – 8 = 4, i.e., B.
Hence, Gowri's grade will be higher than that of Hari. University 8 0 2 0 India

4. 3 As Fazal GPA = 2.4


So D + F + B + P + D = 2.4 × 5 9. 1 10. 3 11. 1 12. 2
2 + 0 + 4 + P + 2 = 12
P=4 13. 2 In 1999, total number of Naya mixer-grinder = 124
So his grade in strategy is B. Number of Naya mixer-grinder disposed = 20% of 30
So Grade of Utkarsh in marketing is also B. =6
So for Utkarsh, x + B + F + C + A = 3 × 5 Number of mixtures bought
x + 4 + 0 + 3 + 6 = 15 124 = [50 + 24] 50
x=2
So grade of Utkarsh in finance = D. 14. 3 Number of Naya mixer-grinder disposed in 1999 ⇒ 6
Number of Naya mixer-grinder disposed in 2000 ⇒ 10
5. 3 Average income of Ahuja Total disposed by end of 2000 = 16
700 + 1700 + 1800 4200
= = 15. 4 Initial number of Purana mixer-grinder not available,
3 3
Average income of Bose hence cannot be determined.
800 + 1600 + 2300 4700
= = 16. 1 20 Purana mixer-grinder were purchased in 1999.
3 3
Average income of Coomar 17. 4 Thailand and Japan (Maximum difference of 4 ranks
300 + 1100 + 1900 3300 (5 – 1) = 4)
= =
3 3
Average income of Dubey 18. 1 China (Maximum difference between 2 parameter is
1200 + 2800 4000 2)
= =
2 2
It's clear that lowest average income is of Coomar. (It 19. 2 Japan (Maximum difference of 4)
is clear visually as well)
20. 4 Japan and Malaysia (Inferring from question 17)
6. 4 From the figure draw a line parallel to the expenditure
axis and midway between observations of each 21. 1 Statement A: 20% of Z > 25% of S
family's values. Z 5
> Cannot say.
S 4
7. 4 From figure the Ist member of Dubey family is on the Statement B: 13% of S > 10% of Z
line indicating income = expenditure. ⇒ 39% of S > 30% of Z. So 40% of S must be greater
The 2nd member is just above the line. than 33% of Z.
Hence statement B is sufficient to answer.
8. 1 Look at the leftmost member of Ahuja family.

Page 2 MBA CAT PAPER – 2004 Explanations


Test Prep
22. 1 Assume A, B, C, D get score 10, 8, 6, 4 reap. Solution for questions 27 to 30: Go through the following
A B C D table.
10 8 6 4
Statement A: Pakistan South Africa Australia
With the conditions A will give vote to B
K 28 51 < 48
With the conditions B will give vote to A
With the conditions C will give vote to A R < 22 49 55
Even if D gives to A/B/C - 2 situation arises. S < 22 75 50
Either A will win or there will a tie when D gives vote to
B. V 130 < 49 < 48
Even then A will win. Y 40 < 49 87
So we are getting the answer.
Top 3 batsman 198 175 192
Statement B: Can conclude anything.
Answer (1) first statement. India Total 220 250 240

23. 1 Statement A: Cannot say anything. 27. 3 28. 1 29. 2 30. 2


Statement B: Because amongst the Top 5 3 are boys,
2 are girls. And Rashmi is third among the girls and Solutions for questions 31 to 34: For solving these
Kumar is 6th. questions make a table like this:
We can conclude.
Answer (1) statement II is sufficient.
Africa America Australasia Europe
24. 2 Statement A: We can find, there are 12 Tails and 9 L 0 1 1 1 3
Heads.
H 1 1 6
After tosses he will reach at blue point. So statement
A is sufficient. P 2 1 6
Statement B: 3 more Tails greater than Heads. So he
R 1 1 6
will reach at blue point after tosses.
So statement B is also sufficient. 4 8 5 4
21
25. 3 Statement A: 2 kg potato cost + 1 kg gourd cost < 1 kg
potato cost + 1 kg gourd cost
⇒ 1 kg potato cost < 1 kg gourd cost. (i) As the labour expert is half of each of the other, so the
So statement A is not sufficient. only possible combination is
Statement B: 1 kg potato cost + 2 kg onion cost = 1 kg L –3
onion cost + 2 kg gourd cost 1 kg potato cost + 1 kg H
P
onion cost = 2 kg gourd cost. 6 ea ch
R
So statement B is also not sufficient.
Combining both statements we get
1 kg potato cost < 1 kg gourd cost …(i) (ii) Statement (d): If the number of Australasia expert is 1
1 kg potato cost + 1 kg onion cost less, i.e. total export are 20 American be twice as
= 2 kg gourd cost …(ii) each of other. The only combined possible is Americas
So the onion is costliest. 8.
Australasia 4 + 1 = 5
26. 4 Statement A: 13 currency notes will give diff. Values. Europe 4
Statement B: Multiple of 10 and by many. Africa 4
Even if you combine the statement, we can have Now, we need to workout the various options possible
various values. in the blank cells.
Answer is (4).
Africa America Australasia Europe
L 0 1 1 1 3
H 2 2 1 1 6
P 1 2 2 1 6
R 1 3 1 1 6
4 8 5 4
21

CAT PAPER – 2004 Explanations MBA Page 3


Test Prep
Africa America Australasia Europe 39. 3 The boats will be colliding after a time which is given
L 0 1 1 1 3 by;

H 1 3 1 1 6 20 4
t= = hours = 80 minutes .
5 + 10 3
P 1 2 2 1 6
After this time of 80 minutes, boat (1) has covered
R 2 2 1 1 6
5 20
4 8 5 4 80 × kms = kms ,
60 3
21 whereas boat (2) has covered
10 40
Africa America Australasia Europe 80 × kms = kms.
60 3
L 0 1 1 1 3
After 79 minutes, distance covered by the first boat =
H 1 3 1 1 6
 20 5 
d1 =  – kms
P 2 1 2 1 6  3 60 
R 1 3 1 1 6 After 79 minutes, distance covered by the second
4 8 5 4  40 10 
boat = d2 =  – kms
21  3 60 
So the separation between the two boats
1
31. 4 32. 3 33. 3 34. 4 = 20 – (d1 + d2 ) = kms
4
Solution for question 35 and 36: After 2nd round, team
goals for and goals against table can be drawn: Alternative method:
Team Round 1 and Round 2 combinations Relative speed of two boats = 5 + 10 = 15 km/hr
i.e. in 60 min they cover (together) = 15 km
Goals for → Goals Goals for → Goals
15 1
against against ∴ in 1 min they will cover (together) = km
60 4
Germany 2 →1 1 →0
40. 2 x
Argentina 1 →0 1 →0
4 →0 1 → 2 Not
Spain possible
5 →1 0 → 1 Possible
2
Pakistan 2 →0 0 →1
1 →2 0 → 4 Not
New
possible
Zealand
0 →1 1 → 5 Possible x/2
South 1 →2 0 →2
x
Africa In original rectangle ratio =
2

35. 4 36. 2 2
In Smaller rectangle ratio =
Solutions for question 37 and 38: From the statements x
2
from (a), (b), (c) given in the problem four teams (Spain, Ar-  
gentina, Germany, Pakistan) appear to win their matches in the x 2
Given = ⇒x=2 2
fifth round. However, there are only three matches per round 2 x
and hence only three teams can win their matches in any 2
round. Hence, the data set appears to be inconsistent.
x
37. 4 38. 4 Area of smaller rectangle = × 2 = x = 2 2 sq. units
2

Page 4 MBA CAT PAPER – 2004 Explanations


Test Prep
35. 2 DC is the mandatory pair, which makes 3 and 4 46. 3 According to the passage, Derrida is against
incorrect. E is the opening statement. A concludes the logocentrism and choices (1), (2) and (4) are pro
argument by substantiating the argument in EBCD. logocentrism which leaves option (3) which is differ-
Therefore, the analogy from the previous argument is ent from logocentrism.
being extended in ‘A’ (keyword – “similarly”)
47. 1 This is a fact based question. In paragraph 2, refers
36. 4 From the options, it can be ascertained that ‘B’ is the to line 5 “Rather, they exist … position”. Option (1)
opening statement. Also, B explains “greater interest… directly follows from this line.
than”, hence ‘C’ is the natural antecedent to ‘B’, wherein 48. 1 Answer choice (4) is contrary to what is being said.
“a similar neglect” has been talked, about. Hence (4) is Answer choice (3) is irrelevant. There can be a
the correct option. confusion between 1 and 2 but it must be noted that it
is not the meaning of the text which is based on binary
37. 2 After reading statement B the first question that comes opposites but the interpretation. This leaves us only
to mind is what does ‘it’ stand for. The question is with answer choice (1).
answered by statement (E) which should be the logical
antecedent. This makes EB a mandatory pair and that 49. 1 Option (1) is a logical corollary to the passage. The
is present only in option (2). paragraph is silent about the audience of sodoku,
therefore (2) and (3) cannot be answers. Option (4)
38. 2 Option (2) talks about a ‘near’ friend. There is nothing is of extreme nature and thus ruled out.
like a near friend. It should have been ‘close’ friend
50. 2 Option (1) is extreme. (3) is not talked about in the
39. 1 It should have been “I have my hands full”. passage 4 th is not true according to the passage,
making (2) correct.
40. 3 It should have been “I can’t bear her being angry”. 51. 3 Option (2) talks about humility which is not talked about
in the passage, option (3) is an extension of the
41. 2 Answer choice (4), says that the danger being talked concept of being a minnow.
about is ‘imminent’, which is not necessarily the case
as per the author in the passage, whereas the fact 52. 1 The passage talks about hubris of civilization and
that everyone is complacent about it, is being talked humility is a direct consequence thus … ‘humble’ is
about throughout the passage, which makes option correct. (2), (3) and (4) are not talked about in the
(2) correct. passage.
53. 2 The second sentence does not use the article. It should
42. 3 Options (1) and (4) are incorrect because these
be ‘As a/the project progresses” in sentence C there
choices are too narrow . Choice (3) is mentioned
should be the indefinite article ‘a’ before single-minded
directly in the passage in the last 3 paragraphs.
which leaves us with option (2) as the correct answer.
43. 1 This is the correct option as choice (2) is too narrow. 54. 3 Sentence B should have “making them break apart”.
Choice (3) is a universal truth which may not be the Sentence C should have “many offending chemicals”.
case. There could be a problem between 1 and 4 but
4 is ruled out because this option is one of the reasons 55. 2 B should be “rarely has …”
supporting the author’s argument but is not his key C should begin with ‘The’.
argument as such. Moreover, the author does not say
56. 1 Option B should be “since the Enlightenment.
that the crisis is imminent.
Option C should be “in the 1820’s”
44. 4 In the 2nd paragraph, the author is being sarcastic 57. 3 Resurrecting i.e. bring back to practice is the best
about the fact that the new production and refining choice. (1), (2) and (4) are negative options.
capacity will effortlessly bring demand and supply
58. 3 Sputtering is a light popping sound of a flame which is
back to balance. (line 2 onwards “the accepted …just
dying out. The ideas conveyed are dim and grim so
like that”) and he quotes Tommy Cooper to emphasize
‘shining’, bright and effulgent are out.
his sarcasm. It must be remembered that we have to
consider the author’s point of view, not Tommy 59. 4 Such a scene should be distressing to a sensitive
Cooper’s. Therefore option (4) is correct traveler. Irritating and disgusting are negative options.
1 can be clearly ruled out.
45. 4 Option (1) and (3) are contrary to what Derrida says
in the passage which makes them incorrect. There 60. 4 The one word reply conveys that it is terse. As it has
can be a confusion between 2 and 4. Option (2) could no element of humour we can easily rule out – “witty”.
have been an inference if the statement had been
“Language limits our interpretations of reality”. But the Questions 61 to 64:
word ‘construction’ is incorrect. Therefore only option Note 61-64:
(4) according to the passage, is correct. In any department in any given year; the average ages range
from 45 – 55 year.

Page 8 MBA CAT PAPER – 2005 Explanations


Test Prep
(1) When a 25 year old joins; the avg. dips by around 5 to 24
6 yrs. 66. 2 Gujarat → = 0.47
51
(2) When some 60 yrs old retires the avg. ages dips lesser Only per capita production of rice for Haryana, Punjab,
than in (1). Maharashtra and Andhra Pradesh are greater than
Marketing Total Age 0.47.
2000 49.33 × 3 = 148 67. 4 As seen from the table
2001 44 × 4 = 176 here one faculty joined, age 25. Haryana, Gujarat, Punjab, MP, Tamil Nadu, Maharashtra,
2002 45 × 4 = 180 UP and AP are intensive rice producing states.
2003 46 × 4 = 184
68. 1 Rahul and Yamini.
OB Total Age
2000 50.5 × 4 = 202 69. 2 Gayatri, Urvashi and Zeena, cannot attend atleast
2001 51.5 × 4 = 200 more than one workshop.
2002 52.5 × 4 = 210 70. 2 Anshul, Bushkant, Gayatri and Urvashi cannot attend
2003 47.8 × 5 = 239 One faculty joined, age 25. any of the workshops.
Finance Total Age
2000 50.2 × 5 = 251 71. 4 1 16
2001 49 × 4 = 196 Year 2001 one faculty retired age 60.
2 15
2002 45 × 5 = 225 One faculty joined age 25.
2003 46 × 5 = 230 3 14

OM Total Age 4 13
2000 45 × 6 = 270 5 12
2001 43 × 7 = 301 One faculty joined age 25. 6 11
2002 44 × 7 = 308
2003 45 × 7 = 315 7 10
8 9
61. 1 Clear from the data
62. 4 From the data of 2000: Let that person be X, on April 1, We will have to draw two tables in the Question number
2000 (age of Naresh) + (age of Devesh) + (age of X) 71. Winners after round two would be 1, 2, 3, 4, 5, 11,
= 49.33 × 3 = 148 yr. 10, 9 for 8 rounds respectively. As Lindsay is number
Now ages of Naresh/Devesh on 1 Apr. - 2000 ≡ two she will play Venus Williams in quarter final.
 52y + 4m + 10d  72. 3 Elena is at number 6, Serena is at number 8
+  If they loose then table would be
 49y + 4m + 10d 
_________________ 1 9
2 7
(101y + 8m + 20d) 3 11
⇒ the age of X on 1-Apr-2000 is 47y + 3m + 10d 4 5
⇒ the X’s age on 1st April 2005 is ≡ (52y + 3m + 10d) Maria is number 1 she will play
Number 9, i.e. Nadia Petrova
63. 3 Read the notes in the beginning, the average age dips
twice (from 2000–2001) & from (2001–2002). The dip 73. 1 1 32
is more when a 25 yr old joins and lesser when 2 31
somebody retires.
3 30
64. 4 New faculty joined in 2001, on 1 April, 2001 his age 4 29
was 25 yrs. So on April 1, 2003, his age is 28 years. 5 28
6 27
65. 1 State Productivity (Tons per hectare)
7 26
19.2
Haryana =6 8 25
3.2
9 24
24
Punjab =6 10 23
4
11 22
112
Andhra Pradesh =5 12 21
22.4
13 20
67.2
Uttar Pradesh =4 14 19
16.8 15 18
Hence, Haryana and Punjab have the highest
productivity. 16 17

CAT PAPER – 2005 Explanations MBA Page 9


Test Prep
Matches in bold letters had upsets. The only possible arrangement of 140 being
40 × 1.5 + 30 + 20 × 2 + 10.
Then from the table would be winners are: A = 20 × 2 (Cement or IT)
1, 31, 29, 5, 27, 7, 25, 9, 23, 11, 21, 13, 19, 15, 17 B = 10
So for next round table is C = 30
D = 40 (1.5) (Steel or Auto)
1 17 From the data given in the question we see that A has
to be Cement or IT.
31 15
D is Steel or Auto.
3 19 Hence statements (II) and (III) are correct.
29 13
5 21 77. 3 Total return is 38.75 × 4 = 155
The possible arrangement is
27 11 20 + 10 + 30 × 1.5 + 40 × 2
7 23 Hence
25 9 A = 20, B = 10, C = 30 (Steel or Auto)
D = 40 (Cement or IT)
Hence, statements (I) and (IV) are correct.
No upset in second round hence table in next round is
Hence (3).
1 9 78. 2 Given C … Cement or IT industry
15 7 C’s Return is 30 × 2 = 60%
3 11 Among the other values we see that the possible
arrangements can be
13 5 10 × 1.5 + 20 + 40, 10 + 20 × 1.5 + 40, 40 + 20 + 40 ×
1.5
We are given Maria is in semi-final. As we are not sure
The average returns will be in each case
what is the result of other games. Table is drawn as
below: 10 × 1.5 + 20 + 40 + 60
Table in next round could be:– (33.75%),
4
1 5/13
10 + 20 × 1.5 + 40 + 60
7/15 3/11 (35%),
4
Hence Anastasia will play with Maria Sharapova.
40 + 20 + 40 × 1.5 + 60
(45%).
74. 3 4
1 8 Considering 33.75% as the valid value, then B be-
2 7 longs to the Auto industry.
3 6 Hence (II) and (IV) are correct.
Hence (2)
4 5

In this case Kim Clijster will either not reach semi final Questions 79 to 82:
or she will play Maria in semi final. L = London, Paris = P, New York= NY, Beijing = B
Hence she cannot play Maria in final. In round III, one of the two cities, either London or Paris will get
38 votes and the other 37. Further:
75. 1 The minimum return will be gained if the extraordinary
performing stocks (double & 1.5 growth) are the ones 1) The persons representing London, Paris, Beijing and
whose expected returns are lowest (i.e. 10% & 20%). New York can not vote as long as their own cities are
Taking the minimum value of the expected returns as in contention. In round I, New York gets eliminated and
10. We have to see which of the two values of 10 and hence the representative from NY becomes eligible
20 multiplied by 2 and 1.5 and vice versa yields the for voting in the II round hence increasing the total
minimum value. votes by 1. This means the total votes in the first
Hence comparing the minimum value between round must be 83 – 1 = 82.
20 × 2 + 10 × 1.5 and 20 × 1.5 + 10 × 2, the 2nd one is 2) After round II, the representative from Beijing votes in
minimum. Hence the minimum average return is the III round. This should have increased the number
of total votes by 1 and the total votes must have
20 × 1.5 + 10 × 2 + 30 + 40 become 83 + 1 = 84.
= 30%
4 We are given that the total votes in round III are 75
only. We conclude that 84 – 75 = 9 people who voted
76. 2 If the average return is 35%, then the total return is in round I and II have become ineligible for voting in
35 × 4 = 140. round III.

Page 10 MBA CAT PAPER – 2005 Explanations


Test Prep
3) 9 people who have voted in round I and II become
ineligible for voting in round III. The reason of their New
ineligibility is that till round I and II, they have already Total
Round London (L) Paris(P) Beijing (B) York
voted for two different cities which are not available Votes
(NY)
for contention in round III. All of these 9 voters are
those who voted for NY in round I and then voted for
I 82 30 24 16 12
Beijing in round II.
4) Beijing’s vote in round II is 21. This includes 9 votes 32 = ( 24 + 4 + 3
from people who voted for NY in the first round. So 21 II 83 30 21 (12 + 9) X
+ 1 of NY-rep)
– 9 = 12 people voted for Beijing in both round I and II.
5) We are given that 75% of the people who voted for 37= (32 + 4 + 1
III 75 38 = (30 + 8) X X
Beijing in round I, voted again for Beijing in round II as of B-rep )
well. So, 16 people must have voted for Beijing in
round I. (The Data Shown in Bold was already provided in the prob-
6) In round I we have: lem. The other data is deduced from the solution.)
82 = L + P + B + NY 9
Or 79. 4 Required percentage = × 100 = 75%
12
82 = 30 + P + 16 + 12
80. 4 As seen from the table. Paris got 24 votes.
Giving P = 24
8
7) In round II we have: 81. 4 Required percentage = × 100 = 66.67%
12
83 = L + 32 + 21, giving L = 30
8) NY had 12 votes in round I. 9 of these votes went to 82. 1 Based on the table IOC members from New York must
B(see point 2 , again). The rest 3 went to P. have voted for Paris in round (2).
9) 16 votes for B in round I. 12 of them still vote for B. The Questions 83 to 86: The given information can be shown
rest 4 voted for either L or P. As L has the same below:
number of votes in both the rounds I and II. This means
in round II, these 4 votes must have gone to Paris only. States Firm A Firm B Firm C Firm D
10) The representative from NY did not vote in round I. But UP 49 82 80 55
has voted in round II. As L has the same people voting Bihar 69 72 70 65
for it (30 votes in both the rounds I and II) and we
MP 72 63 72 65
know the exact break up of B in II. This NY-represen-
tative vote must go to Paris only. Further, in order to Total 190 217 222 185
avoid ineligibility, this NY rep must vote for Paris only in
round III also. 83. 2 As Truthful Ltd. has highest market share hence
11) Paris (in round II) break up is: Truthful Ltd. can be A or C.
32 = 24 ( from round I, who voted for Paris ) From neutral statement either B and C are aggressive
+ 4 ( out of the 16, who voted for Beijing in round I) and honest or A and D are aggressive and honest.
+ 3( out of 12, who voted for NY in round I ) According to statement 1 of question 83, B is profitable,
+ 1 (NY -Rep) then A and D are aggressive and honest.
12) Beijing gets eliminated in round II. So the rep of Beijing Then honest total revenue cannot be more than that of
can vote in round III. profitable, hence statement 2 is false.
13) 12 People (out of 21) who voted for Beijing in round II
are still eligible for vote in round III. 84. 3 According to statement 1 aggressive is (B). Then
14) 50% of people who voted for Beijing in I ( i.e. 8 People) Honest Ltd. has to be C (as given in neutral statement).
voted for Paris in round III. These 8 People include 4 of Then statement 2 is also true have Honest Ltd’s. lowest
those who voted for Paris in round II also. Therefore revenue is from Bihar.
4(out of 12 who voted for Beijing in round II and are Hence answer (3).
still eligible for vote in round III ) people have voted for
Paris in round III. 85. 3 B is honest according to Statement 1
15 ) This implies that the rest 8(out of 12 who voted for Atmost only one statement can be true as both give
Beijing in round II and are still eligible for vote in round Aggressive and Honest as firm B.
III ) can vote for London only. This makes London’s Firm B cannot have two names.
vote = 30+ 8 or 38 in round III. Which implies that Paris
got 37 votes. 86. 3 Profitable can be either A or D. Then aggressive and
16) The Beijing Rep who is eligible to vote in round III must honest has to be B and C. Hence truthful is A or D.
have voted for Paris only. And for both A and D lowest revenue is from UP.
The following table sums up the Vote Pattern: Hence choice (3).

CAT PAPER – 2005 Explanations MBA Page 11


Test Prep
Questions 87 to 90:
FR ER
• 17 in TR
FR ER
(1 6 + x) 8 4 8
4
x y
(1 6 + y)

TR (17 ) TR (17 )
• 10 in TR also in at least one more ⇒ 7 in TR alone
Now, total number of FR is maximum
FR ER
⇒8+4+4+x>8+4+4+y
⇒ x > y and x + y = 6
as n(TR) = 17 

=x+y+4+7 
⇒ x = {4, 5, 6}
y = {0, 1, 2}
7
TR (17 ) 87. 3 Both FR and TR but not ER
• TR alone = one less than ER alone ⇒ ER alone = 8 =x
Minimum x = 4

8 88. 1 Option (2) and option (3) are superfluous. They are
• ER alone = double of all 3 ⇒ In all three = =4 not required.
2
Option (1), if given, would tell us the value of x = 4 and
hence y = 2.
FR ER
89. 2 Out of 4 who are in all three projects, 2 move out of FR
and one-one move out of ER and TR.
8
4 FR (14 + x) ER (15 + y)
x y

7 8 8 5 (= 1 + 4)
1 y+
TR (17 ) x+ 2

N ull
• FR alone = (FR and ER) 7

FR ER TR (16)

Minimum in FR = 14 + x = 14 + 4 = 18
p q 8 Maximum in ER = 15 + y = 15 + 2 = 17
4
x y  As 
 
 x = {4, 5, 6}
 y = {0, 1, 2} 
7  
Hence, option (2).
TR (17 )
90. 4 FR and ER = 5
⇒p=q+4 ...(1) ER and TR = y + 2
Total = 37 ⇒5=y+2
[7 + 8 + p + (x + y + q) + 4] ⇒ y = 3;
= 37 [p + q = 12] which is not a possible value as y is 0, 1, or 2 only.
⇒ p – a = 4 ⇒ p = 8 and q = 4 ⇒ option (4)
Inconsistent data.

Page 12 MBA CAT PAPER – 2005 Explanations


Test Prep
CAT PAPER – 2006

ANSWERS and EXPLANATIONS

1 1 2 3 3 4 4 5 5 5 6 3 7 1 8 1 9 4 10 5
11 4 12 2 13 2 14 2 15 3 16 3 17 2 18 1 19 5 20 4
21 5 22 2,3 23 1,4 24 4 25 3 26 5 27 1 28 2 29 3 30 4
31 2 32 1 33 4 34 5 35 3 36 3 37 2 38 5 39 1 40 4
41 3 42 1 43 4 44 2 45 4 46 3 47 5 48 4 49 2 50 1
51 1 52 2 53 1 54 2 55 5 56 2 57 2 58 4 59 2 60 4
61 2 62 4 63 1 64 4 65 4 66 3 67 2 68 5 69 1 70 5
71 5 72 5 73 2 74 5 75 3

Question Total Total Total Total Net Time


number questions attempted correct wrong Score Taken
LRDI 1 to 25 25

EU + RC 26 to 50 25

QA 51 to 75 25

Total 75

SUP-0003/09

CAT PAPER – 2006 Explanations MBA Page 1


Test Prep
For questions 1 to 5: 4. 5 If 'K' is included 'L' has to be included (statement three)
From statement one, team would include exactly one among P, If 'L' is chosen neither N nor U can be chosen
R, S (statements five and six)
⇒ P (or) R (or) S. ⇒ S, W are also not included because S, U, W have to
be always together. (Statement four)
From statement two, team would include either M, or Q
⇒ M but not Q Hence one of P (or) R would be selected (statement
(or) Q but not M one) and one of M (or) Q would be selected statement
(two)
From statement three, if a team includes K, it will include L or (K, L) and two of the above five have to be included.
vice versa.
⇒ K, L always accompany each other. 5. 5 If a team includes N, it cannot include 'L'.
And therefore not even 'K' (from statement five and
From statement four, if one of S, U, W is included, then the three)
other two also have to be included.
⇒ S, U, W are always together. According to statement one
One of P (or) R (or) S has to be included.
From statement five, L and N cannot be included together According to statement two
⇒ L, N are never together. One of M (or) Q has to be selected.

From statement six, L and U cannot be included together. So the following cases are possible
⇒ L, U are never together. P Q N,
RQN
1. 1 From statements one and two; P M N,
one of P, R, S and RMN
one of M, Q are to be selected. We require one more
member. If 'S' is selected
But from statement three; (K, L) are always together. SUWMN
Hence 'L' cannot be included in a team of 3 members. SUWQN

2. 3 Again, from statement one; are the only possible cases.


one of P, R, S has to be selected. Hence in all 4 + 2 = 6 ways can be constituted.

To make a team of '5' For questions 6 to 10:


'S' will be chosen (which leaves out P and R)
⇒ If 'S' is chosen 'U' and ‘W’ have to be chosen 6. 3 Let Dipan get x marks in paper II
(statement four) Dipan's average in PCB group = 98
⇒ If 'U' is chosen 'L' cannot be chosen (statement Maths group = 95
five) S.S. group = 95.5
⇒ K cannot be chosen (statement three) Vernacular group = 95
And from statement two; one of M (or) Q has to be
 96 + x 
chosen. English group =  
 2 
3. 4 From statements one and two Sum of all = 96 × 5
Two members are to be selected.
x
Of the remaining seven; So 95.5 + 96 × 3 + 48 + = 96 × 5
To maximize the size of the team. 2
We would chose S,
x
⇒ U and W are included in the team (statement four) ⇒ = 96 × 2 − 95.5 − 48
We cannot include K (or) L because we would then 2
have to leave out N and U (from statements five and x = 2 (96.5 − 48 ) = 2 × 48.5 = 97
six)
So, answer is (3)

7. 1 The only boy getting 95 in atleast one of the subjects


of the group among all the groups is Dipan.
So answer is option (1).

Page 2 MBA CAT PAPER – 2006 Explanations


Test Prep
8. 1 A group score of 100 in Social Science would have For questions 11 to 15:
Increased the scores as follows: As only Paul Erdös was having an Erdös number of zero so
the minimum Erdös number among A, B, C, D, E, F, G, H should
be 1 or greater than one. At the end of the third day F co-
authored a paper with A and C. F had the minimum Erdös
Score Group Final Final
number among the 8 people. So if F's Erdös number is y, then
Increase Score Score group
A and C's Erdös number should change to (y + 1) after third
Increase Score
day. As A and C decreased the average by maximum possible
Pritam 22 11 11 96.1
= 2.2 extent, it means C had the second-height Erdös number among
5 all eight, as A had an Erdös number of infinity. Suppose Erdös
Joseph 9 4.5 4.5 95.9 numbers of A, B, C, D, E, F, G, H are y + 1, b, y + 1, c, d, e, y,
= .9 g, h respectively at the end of third day.
5
Tirna 21 10.5 95.8 ∴ (y + 1 + b + y + 1 + c + d + e + y + g + h) = 24 = (3 × 8)
10.5
= 2.1 3y + 2 + b + c + d + g + h = 24
5 When E co-authored with F, the average Erdös number reduced
again, it means, E's Erdös number was not the same with A &
Agni 9 4.5 4.5 95.4 C initially. As at the end of third day, 5 people had same Erdös
= .9
5 number, they should be A, C and any 3 out of B, D, G, H.
Suppose those 3 people are B, D, G. Then
(3y + 2 + y + 1 + y + 1 + h) = 24
So, the order is Pritam > Joseph > Trina > Agni. 6y + h + e = 19 …(i)
Option (1). On the fifth day E co-authored a paper with F and hence Erdös
number of E changed to (y + 1). Also the average decreased
9. 4 The student having atleast 95 in every group is Dipan, by 0.5 that means the total decreased by
so the answer is Dipan, option (4). Hence, e - (y + 1) = 4
⇒e–y=5
10. 5 Let us increase the score in one of the subjects of the Putting the value of e in equation (i), we get
following candidates 6y + h + (5 + y) = 19
7y + h = 14
Least Scores Contributio Final Score Only possible value of y = 1 as h cannot be zero.
n in net So after 3rd round Erdös number of A, C, E, F were 2, 2, 2, 1
Score respectively.
Ram 94 in group of 2 3 in 5 96.1 + .6 =
groups 96.7 11. 4 Only A, C, E changed their Erdös number, rest 5 did not
Agni 82 in group of 2 9 in 5 94.3 + 1.8 = change their Erdös number.
groups 96.1
Pritam 83 in group of 2 8.5 in 5 93.9 + 1.7 = 12. 2 At the end of conference 6 people including E were
groups 95.6 having an Erdös number of 2 and F was having 1 as
Ayesha 93 in group of 2 3.5 in 5 96.2 + .7 = Erdös number. So 8th person was having an Erdös
groups 96.9 number of [20 – (2 × 6 + 1)] = 7
Dipan 95 in group of 1 5 in 5 96 + 1 =
groups 97.0 13. 2 As at the end of 3rd round 5 people were having
same Erdös number. A and C changed their Erdös
number after coauthoring with F. So, the other 3 would
So, Dipan will end with a highest total.
have same Erdös number in the beginning.
So the answer is option (5).
14. 2 2

15. 3 After co-authoring with F, E was having Erdös number


of 2, which was 4 less than initial Erdös number of E.
So answer is 2 + 4 = 6.

CAT PAPER – 2006 Explanations MBA Page 3


Test Prep
For questions 16 to 20:
19. 5 Day Day Day Day Day
1 2 3 4 5
Day Day Day Day Day Start
16. 3 1 2 3 4 5 100 90 100 110 120
price
Start
100 90 100 110 120 End price 90 80 110 120 110
price

End price 90 100 110 120 110


Let initial amount with Chetan and Michael is Y.
In the above table Chetan sold shares on Day 2, Day 3 Total money with Chetan = Y - 900 + 1000 + 1100 +
and Day 4 whereas Michael sold shares on Day 4 1200 - 1100 = Y + 1300
only. Total money with Michael = Y + 1200
Therefore at the end of day 3 the price of Share is Rs. Therefore difference between Chetan and Michael is
110. Rs. 100 and Number of shares with Michael and Chetan
is same.
17. 2 Day Day Day Day Day
1 2 3 4 5 20. 4 Day Day Day Day Day
Start 1 2 3 4 5
100 90 100 110 100
price Start
100 110 120 130 120
price
End price 90 100 110 100 100
End price 110 120 130 120 110

Let initial amount with Chetan and Michael is y.


Total Money with Chetan = y - 900 + 1000 + 1100 + Let initial amount with Chetan and Michael is Y.
1200 - 1100 = y + 1300 Chetan sold shares on Day 1, Day 2, Day 3 whereas
Total money with Michael = y buys shares on Day 4 and Day 5.
Therefore Chetan ended up with Rs.1300 more cash Total Money with Chetan is = Y + 110 × 10 + 120 × 10
than Michael. + 130 × 10 - 120 × 10 - 110 × 10 = Y + 1300
Therefore at the end of day 4 the price of Share is Total money with Michael = Y + 1200
Rs. 100. Total money with Michael = Y + 120 × 10 + 130 × 10 +
120 × 10 = Y + 3700
Total money with Michael & Chetan = 2Y + 5000.
18. 1 Day Day Day Day Day Therefore maximum possible increase is 5000.
1 2 3 4 5
Start For questions 21 to 25:
100 90 80 90 100 In this set the fuel cost for each of the path is given. In addition
price
there are four toll collection junctions.
End price 90 80 90 100 110
21. 5 No traffic flows on the street from D to T
Now we have fuel cost on different paths as
Assume initial number of share with Chetan and Michael SAT 9 + 5 = Rs. 14 + toll at junction A
is x. In the above table Chetan buy 10 share each on SBAT 2 + 2 + 5 = Rs. 9 + toll at junction B and D
day 1, day 2 and sold 10 share on day 3, day 4 and SBCT 2 3 + 2 = Rs. 7 + toll at junction B and C
day 5. SDCT 7 + 1 + 2 = Rs. 10 + toll at junction D and C
Now checking the options we find that toll at junction
∴ Total shares with Chetan is x - 10. A is 0 or 1.
In the above table Michael buy shares only on day 2. When toll is 0, fuel cost on SAT = 14 + 0 = Rs. 14
∴ Total shares with Michael is x + 10. When toll is 1, fuel cost on SAT = 14 + 1 = Rs. 15
∴ Michael had 20 shares more then Chetan. The fuel cost on all the paths should be equal.
Therefore at the end of day 3 the price of share is Rs. Option, (1), (2), (3) can be ruled out as in all these
90. options toll at C and D adds up to more than Rs. 5. As
fuel cost on SDCT is Rs. 10 without toll, so with toll it
cannot exceed Rs. 15 (i.e. toll of path SAT).
Option (4) is ruled out as in this option SAT comes out
to be Rs. 14 and SDCT sums up to 15.
Correct answer is option (5).

Page 4 MBA CAT PAPER – 2006 Explanations


Test Prep
22. 2 & 3 Option (3) can be the answer, when toll at junction B
Available routes are is 4 and toll at junction C is 0. Then SDCT will have toll
SAT → Rs. 14 equal to Rs. 10.
SBAT → Rs. 9 As Rs. 10 is lesser than Rs. 13 so option (4) is also
SDCT → Rs. 10 ruled out.
SDT → Rs. 13 Answer is option (3).
Now fuel cost of SAT - fuel of SDT = 14 – 13 = Rs. 1.
Hence toll at junction D should be 1 more than the toll at 26. 5 The paragraph stresses on the relationships between
A. So option (1), (4) and (5) are ruled out. the factories, dealers and the consumers. Every entity
Now fuel cost of SAT - fuel cost of SBAT = 14 – 9 = has certain short-term expectations from each other.
Rs. 5. So toll at junction B should be Rs. 5. So answer This makes these relationships strenuous. This strain
could be either (2) or option (3). leads to feelings of mistrust and lack of commitment.
So the longer this continues, the more the chances of
23. 1 & 4 everyone succumbing to this vicious trap and they
would soon realize that they have sacrificed long-
Note: Both the options (1) and (4) are correct. term stability and gain for short-term benefits. Hence
Option (5). Option (4) is too specific to industry (at the
Available paths considering no toll are cost of the other players – dealers and customers),
SAT → Rs. 14 option (2) suffers from the same short-comings
SBCT → Rs. 7 together with throwing the technical (unexplained)
SBAT → Rs. 9 jargon ‘supply chain’ to us. Option (1) takes into account
SDCT → Rs. 10 only 2 players and repeats what is stated in the
SDT → Rs. 13 passage about “dealers adjusting prices and making
Fuel cost on path SAT - fuel cost on SDT = 14 - 13 = deals” in the term ‘Deal making’; option (3) seems close
Rs. 1, toll at junction D should be 1 more than the toll at but can be eliminated as the word ‘adversary’ is too
junction A. strong. The passage implies that everyone tries to
So option (2), (3) and (5) are ruled out. maximize his benefits, not that they ‘oppose’ one
Checking options (1) and (4). another.
When A = 0, paths SAT, SBAT and SDT are equally
likely to be taken by a motorist. 27. 1 The passage heads towards describing the functions
When A = 1, toll at B and C should be equal to Rs. 5 that bad / good maps (and therefore theories) serve.
and Rs. 3 respectively. Just as a ‘Bad theory’ does not help us understand a
problem, a ‘good theory’ is invaluable to us, though it
24. 4 Available routes are may be simplified. ‘Simplified’ here implies that less
SAT → Rs. 14 valuable information is left out. According to this logic,
SBAT → Rs. 9 option (2), (3), (4), get eliminated. Option (5) is close
SBCT → Rs. 7 but more negative in tone than required. The word
SDCT → Rs. 10 ‘limitation’ here indicates a short coming whereas the
SDT → Rs. 13 passage implies that it is a simplification as it would
Fuel cost on path SAT - fuel cost on path SDT = 14 - 13 not be of practical use otherwise.
= Rs. 1.
So the toll at junction D should be 1 more than toll at 28. 2 Going with the direction of the passage, the last line is
junction A. So option 1 and 3 are ruled out. stating ‘now all players “profess” to seek only peace’.
Fuel cost on path SAT - fuel cost on path SBCT = 14 - Profess means to mask or to pretend. Thus option (2)
7 = Rs. 7. which talks about the veil being lifted is the most logical
So sum of toll at junction B and C should be 7 more statement that completes the passage. More so this
than the toll at A. Hence only option (4) matches. also follows from the source of the text.

25. 3 We have to find a path on which minimum cost is 29. 3 The answer is very direct. With every statement of
incurred and such that total traffic through B does not his, the author seeks to show how foolish those people
exceed 70% are who call his advice ‘rules’. After his first statement
So, option (5) is ruled out because we can send al he has posed the rhetorical question “Call that a rule?”
traffic through SDCT or SDT and meet all conditions. The same should follow after his second “scarcely a
Option (1) is also ruled out as in that case all traffic will rule!”
be passed through SBCT [not possible as traffic at B
can't be more than 70%]
Option (2) is also ruled out as it is possible only when
toll at junction C is 2. In that case also all traffic will
pass through B.

CAT PAPER – 2006 Explanations MBA Page 5


Test Prep
For questions 24 and 25: 30. 1 From statement A, it is clear that 40% of top academic
Using the given expressions — performers are athletes and that is equal to 10. So
total number of academic performers can be calculated.
a1 = p b1 = q Statement B does not provide any relevant information.
So the answer is (1).
a2 = pq b2 = q2
a3 = p 2 q b3 = pq2 31. 4 Statement A and B alone are not sufficient but if both
are combined, then we can form the following
a4 = p2q2 b4 = pq3 sequence:
a5 = p3q2 b5 = p2q3 1 2 3 4 5
3 3 2 4 D E B C A
a6 = p q b6 = p q
So the answer is (4).
and so on
32. 3 Statement A alone is sufficient because 10% of the
female employees have engineering background, 70%
n n n n
–1 +1 of the employees are females, so 7% of the employees
24. 1 an + bn (n is even ) = p 2 q 2 + p 2 q 2 are female and having engineering background. Hence,
18% of the employees are male and having
n–1
engineering background. From statement B, we know
= q (pq ) 2 (p + q) the number of male employees having engineering
background. So, the percentage of male employees
n +1 n −1 n −1 n +1
having engineering background can be calculated. So,
the answer is (3)
25. 4 an + bn (n is odd ) = p 2 q 2 + p 2 q 2
33. 5 Statement A alone is not sufficient because it is not
n–1
= (p + q )(pq ) 2 giving any information about the opponent. Statement
B alone also not sufficient because it is not giving any
1 2 information regarding the performance of Mahindra &
Substituting p = and q = Mahindra in the second half. Even if both the
3 3
statements are used we will have two cases.
n −1 M&M 0 1
2
an + bn =   2 Opponent 3 4
9
So in one case match is drawn and in another case it
Substituting n = 7, an + bn > 0.01 is won by Mahindra & Mahindra.
Hence the answer is (5)
Substituting n = 9, an + bn < 0.01
Hence smallest value of n is 9 For questions 34 to 37:
Looking at the values in the table one can easily conclude that
26. 1 As the diet should contain 10% minerals and only two the costs which are directly proportional to the change in
ingredient contains 10% minerals i.e. O and Q. volume of proportion are ‘Material’, ‘Labour’ and ‘Operating
Hence only by mixing O and Q a diet with 10% minerals cost of machines’. Rest of the costs are all fixed costs. If ‘x’ is
can be formed. the number of units produced in 2007 then the total cost of
Hence, there is only one way. production would be
C = 9600 (Fixed cost) + 100x (Variable cost)
27. 4 The required diet can be formed by mixing P and S or Variable cost = 100x because as the number of units for 2006
Q and S only but the lowest cost per unit can be is 1200 and variable cost for that is 120000 i.e. 100 times the
achieved by Q and S only. number of units.

28. 5 To make a diet with atleast 60% carbohydrates we 34. 2 Total cost = 9600 + 100 × 1400 = 149600
can use option (2) or (5) only but the lowest cost per
149600
unit can be achieved when P, Q and S are mixed in the Cost per unit = = 107 (approx.)
ratio 4 : 1 : 1. 1400

29. 5 As the ingredients are fixed in equal amounts, so we


can take the average of the constituent percentage of
the elements used.
Only option O and S satisfies all the conditions.

CAT PAPER – 2007 Explanations MBA Page 5


Test Prep
35. 3 To avoid any loss the total selling price should be 40. 1 Percentage of vegetarian students in Class 12 =
equal to the total cost price. If ‘x’ units are produced
32
and selling price of each unit is 125 Rs. × 100 = 40%
Therefore, 125x = 9600 + 100x 80
25x = 9600
⇒ x = 384 41. From the main table
Hence, 384 units should be produced. M F V
Male Female Total
Veg Veg
36. 5 Same as in question number 36 profit would be Class 12 48 32 32 80
maximum if the number of units are maximum i.e. 2000 Class 11 44 36 40 80
Secondary 288 352 352 320 320 640
37. 1 If the company sells a maximum of 1400 units, the Section
selling price is fixed at Rs. 125 per unit. If more than Total 380 420 424 800
1400 units are sold, the selling price is reduced to
Rs. 120 per unit. The company cannot sell more than
This question is wrong because the number of Male
1700 units.
vegetarian cannot be greater than 288.
To earn maximum profit at a unit selling price of
Rs. 125, the company must sell 1400 units. The
42. 3 For Malaysia, total cost
maximum profit earned, denoted by P0, is calculated
= (11,000 + 6,000) + (10,000 + 8,000) + (10,000 +
as below:
8,000)
= US$ 47,000 (minimum)
Profit = (Selling Price) – (Cost Price)
Hence, the answer is Malaysia
P0 = 125 × 1400 – (9600 + 100 × 1400) = Rs. 25400
43. 1 In India, total cost in US$ = 8,500 + 9,000 = 17,500
Now if the company sells an x number of units
which is maximum
(x > 1400) then the profit earned will be:
Hence, the answer is India.
Px = 120 × x – (9600 + 100 × x) = 20 × x – 9600

The minimum value of x for which Px will be more than 1500


P0 must satisfy the following inequality: 44. 4 In India, total cost in US$ = 3000 + 5000 +
32.89
(transportation cost)
20 × x – 9600 > 25400 = 8456.06
⇒ x > 1750 In Thailand, total cost in US$ = 4500 + 6000 = 10,500
As only a maximum of 1700 units can be sold, Px will Difference in amount is 10,500 – 8456.06 = US$ 2044
never be more than P0. Hence the maximum profit that ≈ 67,500 Bahts
can be earned is Rs. 25400 only.
Hence (1) is correct. 45. 2 In India, total cost for spirial fusion in US$ =

For questions 38 to 41: 5500 × 40.928


= 6431.5
From the given information the following table can be formed: 35
In Singapore, total cost for spirial fusion in US$ = 9000
M F V NV Total
Difference (in US$) is 9000 – 6431.5 = 2568.5 ≈ 2500
Class 12 48 32 32 48 80
Class 11 44 36 40 40 80
46. 4 For the shortest route we have to consider the path
Secondary 288 352 352 288 640
A-C-F-J.
Section
Following table compiles the distance and the
Total 380 420 424 800
corresponding price.
38. 2 From the above table Path Distance Price
Percentage of male students in the secondary section A-C 790 1350
288 C-F 410 430
= × 100 = 45%
640 F-J 970 1150
Total 2170 2930
39. 5 From the above table
Male vegetarians = 8 Hence the price for travelling by the shortest route is
Female vegetarians = 24 Rs. 2930.
So, their difference is 16.

Page 6 MBA CAT PAPER – 2007 Explanations


Test Prep
47. 2 For the lowest price we have to consider the path 54. 1 C and E
A-H-J. In (A) ‘to’ is redundant.
Following table compiles the distance and the In (B) ‘hands on about Israel’.
corresponding price. In (D) a Shaliach, a sort of recruiter to Minneapolis.
Path Distance Price 55. 5 B only
A-H 1950 1850 In (A) ‘into’ should be used in place of ‘in’
H-J 400 425 In (C) the article is missing before the word ‘slump’
Total 2350 2275 In (D) the singular form ‘stimulus’ should be used in
place of ‘stimuli’
If the company charges 5% below the minimum price
In (E) ‘effect’ should be used in place of ‘affect’
of Rs. 2275 then it should charge 0.95 × 2275
= Rs. 2161
56. 3 B and D
In (A) ‘said’ should be used instead of ‘told’
48. 3 If airports C, D and H are closed, then the passenger
In (C) ‘handed down to’ should be used in place of
must follow the path A-F-J for minimum price.
‘handed to’
Following table compiles the distance and the
In (E) a subject is required after ‘hence’
corresponding price.
Path Distance Price 57. 2 ‘The difference between two artistic interpretations’
A-F 1345 1700 Refer the last three lines of the 1st para where the
F-J 970 1150 author talks about the gap between the two artistic
Total 2315 2850 interpretations within the depth of the creative power
and doesn’t mention width.
So the corresponding minimum price paid by a
passenger is Rs. 2850. 58. 1 ‘Define the place of the poet in his culture’.
The lines starting with “But suddenly I understood
49. 2 For minimum cost per km, we have to consider the …..”define the position of the poet in his culture.
path A-H-J
From solution of question 47, we know the distance 59. 4 Refer to the 5th line of the 2nd para. Here the term
of path A-H-J is 2350 km and the price is Rs. 2275. "adventures of experience" refers to the poet & artists
The price include a margin of 10%. who over vitalize and enrich the past for us.
So, the minimum cost per km =
60. 5 “The personification of a whole organization is a textual
10 1
2275 × × = 0.88 device …” is the choice which continues the theme in
11 2350 the last two lines of the paragraph in the best possible
way. The latter half of the paragraph is talking about
50. 4 For minimum cost per km, again we have to consider personification of whole organizations and this is the
the path A-H-J as illustrated in the solution of question choice which completes the paragraph satisfactorily.
49. The choice justifies why whole organizations are
The distance of path A-H-J is 2350 km. personified even though they cannot speak as
characters.
51. 5 ‘Reciprocal roles determine normative human behaviour
in society’. 61. 1 ‘Yet despite these technical developments…… …care
This is the main idea of the passage that is carried about’.
throughout. Note that ‘role of biology’ is negated and In the first line of the paragraph the author speaks
‘reciprocal roles’ are affirmed in paragraph 1 and 2. about the magical allure still retained by photographs.
Then he goes on to describe the negative effects of
52. 2 ‘We would not have been offended by the father technological advancements on the same. Thus, in
playing his role ‘tongue in cheek’’. line with the first idea (1) describes how photographs
All the other options would have been false if biological are still holding out against these negative effects.
linkages would have structured human society.
62. 2 ‘No inventory would ever include those, ofcourse’.The
53. 4 The last para where the author mentions the examples para starts with listing out ‘the inventory’. The option
of a waitress and clergyman, and driver refers to the which says ‘what she lacked………..natural
alignment of self with the rules being performed and shrewdness’ is beyond the scope of the argument. All
society preventing manifestation of the true self. the options can be easily eliminated. ‘Those’ in the
option ‘No inventory would ever include those,
ofcourse’, refers to human intuition and intelligence of
Mma Ramotswe.

CAT PAPER – 2007 Explanations MBA Page 7


Test Prep
101. 1 Is a sitter, its just simple addition and subtraction, FRG 110. 4 Again for timber and wages experienced declined
+ CZE = 43.01 and US Total = 42.83 only once for the given period.
Hence difference in time = 43.01 – 42.83 = 0.18
For questions 111 to 114:
102. 2 The first two rankers of final score are 8905 and The table consists of sorting the different nations according to
8897. The third ranker is carrying a score of 8880. So the birth rate and death rate. The only thing to remember is that
he needs to score 8881 to get a bronze, whereas his if two countries have same rank (3), then next country will get
sum is 582 + 3003 = 3585. (5) rank. So
Least score required = 8881 – 3585 = 5296 rank of Philippines is 33,
rank of Spain is 17,
103. 2 Based on score 2 – Michael Smith is 9th which means rank of Taiwan is 28.
that if we exclude Daley Thompson there are 9 people In-between total 9 countries.
below him. Based on High-Jump alone Michael would In consolidated list 37th country is Venezuela.
be 3rd last and on Pole Vault he would be 9th again. In last question, we have to check last country of South America
Which means that Michael needs to ensure that he is (42, 11) and last country of Africa (50, 28). In-between there
ahead of the same two guys in the Long Jump to keep are 8 countries of Asia.
the same position.
111. 2
104. 2 Here you need to compare the ratio as which of the
112. 1
189 209
following is the highest , and rest of the
561 587 113. 4
option have equal value.
209 189 114. 1
Now you can see that is greater than .
587 561
115. 3 March rainfall is lower than September rainfall in every
105. 1 Population of the Chaidesh is lowest for lowest per location. Just check the bar graph.
capita availability of Tea wrt production, i.e. for 1995.
116. 2 Peak rainfall occurs in April. Just check the bar graph.
106. 1 From 96 to 99 in each year the production has increased
but the area has decreased. Therefore, the production For questions 117 to 119: Based on observation only.
in unit per area is highest in 1999.
117. 3 4 and 2
107. 3 Cement Limestone Power Wages
118. 4 5
93 − 94 100 20 25 15
02 − 03 104 21 27 15.8 119. 4 Egg and onion.

104 − (21 + 27 + 15.8) 120. 4 According to statement (1), A which has the third
So percentage profit = × 100
104 highest profitability has the lowest operating income
in 2002-03.
40.2 According to statement (2), company D has the highest
= × 100 = 38.65%
104 combined operating income and has the lowest
operating profit.
108. 2 According to statement (3), only company B has higher
operating income in 2001-02 than 2002-03, and has
Steel Power Wages Iron Ore higher profitability in 2002-03 than 2001-02.
93 − 94 100 30 10 25 According to statement (4), the companies having
02 − 03 105.5 32.4 10.53 26.5 profitability between 10-20% are A, C, E but C has
operating income equal to 200 crore.
So percentage profit So the answer is 4.
105.5 − (32.4 + 10.53 + 26.5)
= × 100 121. 3 Operating profit of companies is:
105.5
A = 15
= 34.18% B = 7.75
C = 30
109. 1 You only need to see the the particular row in the table D=3
for the given options and for Power it experienced E = 35
continuous rise. F = 20
So the answer is E.

CAT PAPER – 2003(Re-Test) Explanations Page 23


122. 1 The companies excluded are B and D. As both of them 132. 1 F + n = 4 (k + n) … (i)
have –ve profitability, the answer is –ve. So the M + n = 3 (k + n) …(ii)
answer is (1). From the above equations
F – M = (k + n)
123. 4 Companies exceeding profitability of 10% are C and E. From A
F – M = 10 ⇒ k + n = 10
(30 + 35 ) = 32.5 F + n = 40
So the average is .
2 M + n = 30
⇒ F + M + 2n = 70
For questions 124 to 126: Hence, from A alone, we can get the answer.
Increase of HP is from 884 to 970, i.e. 86
Increase of Kerala is from 1004 to 1058, i.e. 54 For questions 133 to 137:
Increase of Punjab is from 832 to 874, i.e. 42 If a person copies from one source, he must have the same
Increase of Assam is 919 to 932, i.e. 13 blank answers as the source and exactly one wrong answer
Increase of J & K is 882 to 900, i.e. 18 more than the source (as the copier has introduced one wrong
answer on his own). If the person copies from two sources,
124. 2 HP and Kerala are highest. the distinct wrong answers from the two sources would be
left blank by the copier and the same wrong answers in the
125. 3 Goa (1091 to 960) two sources would be copied as it is.
Tamil Nadu (1044 to 986)
Bihar (1061 to 921) 133. 2 A and D have only one source since they have only
Orissa ( 1037 to 972) one wrong answer each.
Bihar just by inspection C may have copied from only one source i.e. I and
hence only B has two sources
126. 3 Females outnumbering males means that the sex ratio
is more than 1000. 134. 3 I did it before C since 27 is the wrong choice for I,
So option (3), i.e. the number remains unchanged for similarly A,D,E made keys before C. Hence 4 people
4 years. made their keys before C.
For questions 127 and 128: 135. 4 Both G and H can’t be sources to any of F,B or I and
Congress – Thursday BJP – Friday hence (4) is the correct option.
SP – Thursday BSP – Friday
CPM – Friday 136. 3 F introduced wrong answer to question 14. because
nobody else has done question 14 wrong.
127. 1 Congress procession can only be allowed on
Thursday. 137. 4 A, D and G have one distinct wrong answer and no
blank answers. So, they must have the same source.
128. 4 According to the given table, statement (4) is not true. E and H also have a common wrong answer and no
blanks.
129. 4 From statement A, both Aakash and Biplab have the Hence, both groups had identical sources.
same scores. But we cannot find the man of match.
From statement B only, we cannot find the man of the 138. 3 VCD = 70% Microwave = 75% ACS = 80% Washing
match. M/c = 85%
Combining both statements we can find the man of the Least percentage of employees having both VCD and
match i.e. Aakash. Microwave = 70 + 75 – 100 = 45%
Least percentage having all 3 – VCD, Microwave,
130. 3 From statement A, their ranks will be A - 4, B -1, C - 2, ACS = 45 + 80 – 100 = 25%
D - 3. Least percentage having all 4 = 25% + 85% – 100%
From statement B, also their ranks will be A - 4, B -1, = 10%
C - 2, D - 3.

131. 1 From statement A, the number of members are 20.


So Each member will pay Rs. 30.
From statement B, we cannot find the each payable
amount.

Page 24 CAT PAPER – 2003(Re-Test) Explanations


For questions 139 to 142: For questions 143 to 146:
4 Families 7 Faculty
The key to cracking this question is to follow the simple JC came in first and the next 2 people were SS and SM. When
fundamentals in Analytical Reasoning of going 1 line at a time he left, DG left with him. JP and VR stayed behind.
and making a simple table
Entry Exit Met
Arrival Order Husband Wife Kids JC à SS à SM JC& DG JP&VR
1 Joya 2
2 Shanthi 0 JP&VR together JC& one more person
3 Sridevi
4 Sunil 1 SS left immediately after SM
PK only met JP & DG
Sentence 1 – Family with 2 kids before no kids The key to this question is that when JP & VR entered apart
Sentence 2 – Shanthi with no kids came before Sridevi from JC there was ONLY 1 other person. This could not have
Sentence 3 – Sunil and wife came last with only kid been SS or SM as they came and left together. Hence, this
Sentence 4 – Anil and Joya not husband and wife would have to be DG.
Sentence 5 – Anil and Raj are fathers – hence cannot Hence, DG came 4th, before JP and is the answer to both 143
be the family with no kids and 144.
Sentence 6 – Sridevi and Anita cannot be the persons Now for Qs 145 we need to see how many people did VR
with no kid meet. Both SS and SM had already left and JC and DG were
Sentence 7 – Anil and Joya not husband and wife sitting. He entered with JP eliminating 2 and 5 from our answer
Sentence 8 – Joya before Shanti and Anita was already choices. Since, PK did not meet VR – the answer is 3 and not
present. 4.

Using the above info – Anil and Raj cannot be married 143. 2
to Shanthi as Shanthi has no kids! Hence, Sunil has to
be married to Sridevi (not with Joya already stated) 144. 3
and Raman with Shanthi.
145. 2
Arrival Order Husband Wife Kids
1 Anil Anita 1 146. 4
2 Raj Joya 2
3 Raman Shanthi 0 For questions 147 to 150:
4 Sunil Sridevi 1
D A F
139. 1
Entrance Corridor
140. 2 C E B

141. 3
147. 3 If E faces the corridor, person to his left is C.
142. 2 So the answer is (3).

148. 4 According to figure, E faces A’s office.


So the answer is (4).

149. 1 According to figure, F’s neighbour is A.


So the answer is (1).

150. 2 According to figure, B’s room is last on the right.


So the answer is (2).

CAT PAPER – 2003(Re-Test) Explanations Page 25


Section – 1II
Directions for questions 101 to 103: Answer the questions on the basis of the following information.
In a Decathlon, the events are 100 m, 400 m, 100 m hurdles, 1,500 m, High jump, Pole vault, Long jump,
Discus, Shot put and Javelin. The performance in the first four of these events is consolidated into
Score-1, the next three into Score-2, and the last three into Score-3. Each such consolidation is obtained
by giving appropriate positive weights to individual events. The final score is simply the total of these three
scores. The athletes with the highest, second highest and the third highest final scores receive the gold,
silver, and the bronze medals respectively. The table below gives the scores and performance of 19 top
athletes in this event.
Final High Pole-
Name Country Score-1 Score-2 Score-3 100m
Score jump vault
Eduard Hämäläinen BLS 8802 491 5322 2989 10.74 2.08 4.8
Michael Smith CAN 8855 174 5274 3407 11.23 1.97 4.9
Tomas Dvorak CZE 8796 499 5169 3128 10.63 1.91 4.7
Uwe Freimuth DDR 8799 441 5491 3124 11.06 1.97 4.8
Torsten Voss DDR 8880 521 5234 2868 10.69 2.1 5.1
Erki Nool EST 8768 408 5553 2808 10.71 1.99 5.4
Christian Plaziat FRA 8775 563 5430 2781 10.72 2.1 5
Jürgen Hingsen FRG 8792 451 5223 3033 10.95 2 4.9
Siegfried Wentz FRG 8856 470 5250 3137 10.85 2.05 4.8
Guido Kratschmer FRG 8861 575 5308 3064 10.58 2 4.6
Daley Thompson GBR 582 3003 10.55 2.11 4.6
Frank Busemann GER 8905 568 5392 2945 10.6 2.04 4.8
Alexandr Apaichev SOV 8803 492 5370 3115 10.92 1.95 4.8
Grigory Degtyarov SOV 8823 339 5196 3114 11.05 2.08 4.9
Robert Zmelik TCH 8832 494 5455 2883 10.78 2.06 5.1
Dave Johnson USA 8811 366 5370 3114 10.78 2.1 5
Steve Fritz USA 8827 427 5163 3119 10.75 2.04 5
Bruce Jenner USA 8846 483 5280 3200 10.94 2.03 4.8
Dan O’Brien USA 8897 408 5331 3120 10.36 2.09 4.8

101. The athletes from FRG and USA decided to run a 4 × 100 m relay race for their respective countries
with the country having three athletes borrowing the athlete from CZE. Assume that all the athletes
ran their stretch of the relay race at the same speed as in Decathlon event. How much more time did
the FRG relay team take as compared to the USA team?
1. 0.18 2. 0.28 3. 0.78 4. 0.00
102. What is the least that Daley Thompson must get in Score-2 that ensures him a bronze medal?
1. 5309 2. 5296 3. 5271 4. 5270

103. At least how many competitors (excluding Daley Thompson) must Michael Smith have out-jumped
in the long jump event?
1. One 2. Two 3. Three 4. Four

Page 66 CAT PAPER – 2003 (Re-Test)


Directions for questions 104 to 106: Answer the questions on the basis of the following charts.

Per Capita Availability of Tea (gm ) in Chaidesh

544 566
600 487 510
464
500
400
300
200
100
0
1995 1996 1997 1998 1999
Year

(Note: Availability is defined as production less export.)

Production and export of Tea (Chaidesh)

1995 207
421

1996 189
561

1997 209
587

1998 215
645

1999 220
660

0 100 200 300 400 500 600 700

Production (million Kg) Export (million Kg)

104. In which year during the period 1996-1999 was Chaidesh’s export of tea, as a proportion of tea
produced, the highest?
1. 1996 2. 1997 3. 1998 4. 1999

105. In which of the following years was the population of Chaidesh the lowest?
1. 1995 2. 1996 3. 1997 4. 1999

106. The area under tea cultivation continuously decreased in all four years from 1996 to 1999, by 10%,
7%, 4%, and 1%, respectively. In which year was tea productivity (production per unit of area) the
highest?
1. 1999 2. 1998 3. 1997 4. 1996

CAT PAPER – 2003 (Re-Test) Page 67


Directions for questions 107 to 110: Answer the questions on the basis of the following information.

The following is the wholesale price index (WPI) of a select list of items with the base year of 1993-94.
In other words, all the item prices are made 100 in that year (1993-94). Prices in all other years for an item
are measured with respect to its price in the base year. For instance, the price of cement went up by 1%
in 1994-95 as compared to 1993-94. Similarly, the price of power went up by 3% in 1996-97 as compared
to 1993-94.

1993-94 1994-95 1995-96 1996-97 1997-98 1998-99 1999-2000 2000-01 2001-02 2002-03

All items 100 102.0 102.5 104.0 103.0 105.0 106.0 108.0 107.0 106.0
Cement 100 101.0 100.5 103.0 102.5 103.5 103.1 103.8 103.7 104.0
Limestone 100 102.0 102.5 102.75 102.25 103.0 104.0 105.0 104.5 105.0
Power 100 101.5 102.5 103.0 103.5 104.0 106.0 107.0 107.5 108.0
Steel 100 101.5 101.0 103.5 104.0 104.25 105.0 105.5 106.0 105.5
Timber 100 100.5 101.5 102.0 102.5 102.0 103.0 103.5 104.0 104.5
Wages 100 101.5 103.0 103.5 104.0 104.25 104.0 104.75 104.9 105.3

107. Let us suppose that one bag of cement (50 kg) consumes 100 kg of limestone and 10 units of
power. The only other cost item in producing cement is in the form of wages. During 1993-94,
limestone, power and wages contributed, respectively, 20%, 25% and 15% to the cement price per
bag. The average operating profit (per cent of price per cement bag) earned by a cement manufacturer
during 2002-03 is closest to
1. 40% 2. 39.5% 3. 38.5% 4. 37.5%

108. Steel manufacturing requires the use of iron ore, power and manpower. The cost of iron ore has
followed the All Items index. During 1993-94 power accounted for 30% of the selling price of steel,
iron ore for 25%, and wages for 10% of the selling price of steel. Assuming the cost and price data
for cement as given in the previous question, the operating profit (per cent of selling price) of an
average steel manufacturer in 2002-03
1. is more than that of a cement manufacturer
2. is less than that of a cement manufacturer
3. is the same as that of a cement manufacturer
4. Cannot be determined

109. Which item experienced continuous price rise during the ten-year period?
1. Power 2. Cement 3. Wages 4. Limestone

110. Which item(s) experienced only one decline in price during the ten-year period?
1. Steel and limestone 2. Steel and timber
3. Timber 4. Timber and wages

Page 68 CAT PAPER – 2003 (Re-Test)


Directions for questions 111 to 114: Answer the questions on the basis of the following table.

Below is a table that lists countries region-wise. Each region-wise list is sorted, first by birth rate and then
alphabetically by name of country. We now wish to merge the region-wise list into one consolidated list
and provide overall rankings to each country based first on birth rate and then on death rate. Thus, if some
countries have the same birth rate, then the country with a lower death rate will be ranked higher. Further,
countries having identical birth and death rates will get the same rank. For example, if two countries are
tied for the third position, then both will be given rank 3, while the next country (in the ordered list) will be
ranked 5.

Birth Death Birth Death


Rank Country Region Rank Country Region
Rate Rate Rate Rate
1 South Africa 36 12 Africa 1 Germany (FRG) 10 12 Europe
2 Egypt 39 13 Africa 2 Austria 12 13 Europe
3 Cameroon 42 22 Africa 3 Belgium 12 12 Europe
4 Mozambique 45 18 Africa 4 Germany (DRG) 12 14 Europe
5 Zaire 45 18 Africa 5 Sweden 12 11 Europe
6 Ghana 46 14 Africa 6 Switzerland 12 9 Europe
7 Angola 47 23 Africa 7 U.K. 12 12 Europe
8 Madagascar 47 22 Africa 8 Netherlands 13 8 Europe
9 Morocco 47 16 Africa 9 France 14 11 Europe
10 Tanzania 47 17 Africa 10 Italy 14 10 Europe
11 Ethiopia 48 23 Africa 11 Greece 16 9 Europe
12 Ivory coast 48 23 Africa 12 Bulgaria 17 10 Europe
13 Rhodesia 48 14 Africa 13 Hungary 18 12 Europe
14 Uganda 48 17 Africa 14 Spain 18 8 Europe
15 Nigeria 49 22 Africa 15 USSR 18 9 Europe
16 Saudi Arabia 49 19 Africa 16 Yugoslavia 18 8 Europe
17 Sudan 49 17 Africa 17 Czech. Rep. 19 11 Europe
18 Algeria 50 16 Africa 18 Portugal 19 10 Europe
19 Kenya 50 14 Africa 19 Romania 19 10 Europe
20 Upper Volta 50 28 Africa 20 Poland 20 9 Europe

CAT PAPER – 2003 (Re-Test) Page 69


Birth Death Birth Death
Rank Country Region Rank Country Region
Rate Rate Rate Rate
1 Japan 16 6 Asia 1 U.S.A. 15 9 N. America
2 Korea (ROK) 26 6 Asia 2 Canada 16 7 N. America
3 Sri Lanka 26 9 Asia 3 Cuba 20 6 N. America
4 Taiwan 26 5 Asia 4 Mexico 40 7 N. America
5 Malaysia 30 6 Asia 1 Australia 16 8 Pacific
6 China 31 11 Asia 2 Philippines 34 10 Pacific
7 Thailand 34 10 Asia 3 Indonesia 38 16 Pacific
8 Turkey 34 12 Asia 1 Argentina 22 10 S. America
9 India 36 15 Asia 2 Chile 22 7 S. America
10 Burma 38 15 Asia 3 Colombia 34 10 S. America
11 Iran 42 12 Asia 4 Brazil 36 10 S. America
12 Vietnam 42 17 Asia 5 Venezuela 36 6 S. America
13 Korea
43 12 Asia 6 Guatemala 40 14 S. America
(DPRK)
14 Pakistan 44 14 Asia 7 Peru 40 13 S. America
15 Nepal 46 20 Asia 8 Ecuador 42 11 S. America
16 Bangladesh 47 19 Asia
17 Syria 47 14 Asia
18 Iraq 48 14 Asia
19 Afghanistan 52 30 Asia

111. In the consolidated list, what would be the overall rank of the Philippines?
1. 32 2. 33 3. 34 4. 35

112. In the consolidated list, how many countries would rank below Spain and above Taiwan?
1. 9 2. 8 3. 7 4. 6

113. In the consolidated list, which country ranks 37th?


1. South Africa 2. Brazil 3. Turkey 4. Venezuela

114. In the consolidated list, how many countries in Asia will rank lower than every country in South
America, but higher than at least one country in Africa?
1. 8 2. 7 3. 6 4. 5

Page 70 CAT PAPER – 2003 (Re-Test)


Directions for questions 115 and 116: Answer the questions on the basis of the data presented in the
figure below.

Rainfall at Selected Locations in Certain Months

350
300
Rainfall (in cm.)

250

200

150
100

50

0
1 2 3 4 5 6 7
Locations

March April September November

115. Which of the following statements is correct?


1. November rainfall exceeds 100 cm in each location
2. September rainfall exceeds 50 cm in each location
3. March rainfall is lower than September rainfall in each location
4. None of these

116. Locations 6 and 7 differ from all the rest because only in these two locations,
1. April rainfall exceeds March rainfall
2. Peak rainfall occurs in April
3. November rainfall is lower than March rainfall
4. April rainfall is less than 200 cm

CAT PAPER – 2003 (Re-Test) Page 71


Directions for questions 117 to 119: Answer the questions on the basis of the data presented in the
figure below.

Mid-year Prices of Essential Commodities


80
70
60
Prices (Rs.)
50
40
30
20
10
0
1996 1997 1998 1999 2000 2001 2002
Year

Rice (kg) Dal (kg) Edible oil (kg)


Egg (dozen) Chillies (kg) Onioin (kg)

117. During 1996-2002, the number of commodities that exhibited a net overall increase and net overall
decrease, respectively, were
1. 3 and 3 2. 2 and 4 3. 4 and 2 4. 5 and 1

118. The number of commodities that experienced a price decline for two or more consecutive years is
1. 2 2. 3 3. 4 4. 5

119. For which commodities did a price increase immediately follow a price decline only once in this
period?
1. Rice, edible oil and dal 2. Egg and dal
3. Onion only 4. Egg and onion

Page 72 CAT PAPER – 2003 (Re-Test)


Directions for questions 120 to 123: Answer the questions on the basis of the following charts.
The profitability of a company is defined as the ratio of its operating profit to its operating income, typically
expressed in percentage. The following two charts show the operating income as well as the profitability of
six companies in the financial years (F.Ys.) 2001-02 and 2002-03.

Operating Incom e Profitability


Operating Income (crores)

25%
300
20%
250
200 15%
150 10%
100 5%
50
0%
0
-5% A B C D E F
A B C D E F Com pany
Com pany
Fy 01-02 Fy 02-03 Fy 01-02 Fy 02-03

The operating profits of four of these companies are plotted against their respective operating income
figures for the F.Y. 2002-03, in the third chart given below.

O p erating P ro fit vs O p erating Incom e


40
35
30
O perating P rofit

25
20
15
10
5
0
100 150 200 250 300
O p erating Inco m e
120. Which of the following statements is NOT true?
1. The company with the third lowest profitability in F.Y. 2001-02 has the lowest operating
income in F.Y. 2002-03
2. The company with the highest operating income in the two financial years combined has the
lowest operating profit in F.Y. 2002-03
3. Companies with a higher operating income in F.Y. 2001-02 than in F.Y. 2002-03 have higher
profitability in F.Y 2002-03 than in F.Y. 2001-02
4. Comanies with profitability between 10% and 20% in F.Y. 2001-02 also have operating incomes
between 150 crore and 200 crore in F.Y. 2002-03

CAT PAPER – 2003 (Re-Test) Page 73


121. Which company recorded the highest operating profit in F.Y. 2002-03?
1. A 2. C 3. E 4. F

122. What is the approximate average operating profit, in F.Y 2001-02, of the two companies excluded
from the third chart?
1. –7.5 crore 2. 3.5 crore 3. 25 crore 4. Cannot be determined

123. The average operating profit in F.Y. 2002-03 of companies with profitability exceeding 10% in F.Y.
2002-03, is approximately
1. 17.5 crore 2. 25 crore 3. 27.5 crore 4. 32.5 crore

Directions for questions 124 to 126: Answer the questions on the basis of the table given below:
Sex Ratio (Number of females per 1,000 males) of Selected States in India : 1901-2001

1901 1911 1921 1931 1941 1951 1961 1971 1981 1991 2001
AP 985 992 993 987 980 986 981 977 975 972 978
Assam 919 915 896 874 875 868 869 896 910 923 932
Bihar 1061 1051 1020 995 1002 1000 1005 957 948 907 921
Goa 1091 1108 1120 1088 1084 1128 1066 981 975 967 960
Gujarat 954 946 944 945 941 952 940 934 942 934 921
Haryana 867 835 844 844 869 871 868 867 870 865 861
HP 884 889 890 897 890 912 938 958 973 976 970
J&K 882 876 870 865 869 873 878 878 892 896 900
Karnataka 983 981 969 965 960 966 959 957 963 960 964
Kerala 1004 1008 1011 1022 1027 1028 1022 1016 1032 1036 1058
MP 972 967 949 947 946 945 932 920 921 912 920
Maharashtra 978 966 950 947 949 941 936 930 937 934 922
Orissa 1037 1056 1086 1067 1053 1022 1001 988 981 971 972
Punjab 832 780 799 815 836 844 854 865 879 882 874
Rajasthan 905 908 896 907 906 921 908 911 919 910 922
TN 1044 1042 1029 1027 1012 1007 992 978 977 974 986
UP 938 916 908 903 907 998 907 876 882 876 898
WB 945 925 905 890 852 865 878 891 911 917 934
India 972 964 955 950 945 946 941 930 934 927 933

124. The two states which achieved the largest increases in sex ratio over the period
1901-2001 are
1. Punjab and HP 2. HP and Kerala 3. Assam and J & K 4. Kerala and J & K

125. Among the states which have a sex ratio exceeding 1000 in 1901, the sharpest decline over the
period 1901-2001 was registered in the state of
1. Goa 2. TN 3. Bihar 4. Orissa

Page 74 CAT PAPER – 2003 (Re-Test)


126. Each of the following statements pertains to the number of states with females outnumbering
males in a given census year. Which of these statements is NOT correct?
1. This number never exceeded 5 in any census year.
2. This number registered its sharpest decline in 1971.
3. The number of consecutive censuses in which this number remained unchanged never
exceeded 3.
4. Prior to the 1971 census, this number was never less than 4.

Directions for questions 127 and 128: Answer the questions on the basis of the following information.
Shown below is the layout of major streets in a city.
E

D
A
B
Two days (Thursday and Friday) are left for campaigning before a major election, and the city
administration has received requests from five political parties for taking out their processions along the
following routes.

Congress: A-C-D-E BJP: A-B-D-E SP: A-B-C-E


BSP: B-C-E CPM: A-C-D

Street B-D cannot be used for a political procession on Thursday due to a religious procession. The district
administration has a policy of not allowing more than one procession to pass along the same street on the
same day. However, the administration must allow all parties to take out their procession during these two
days.

127. Congress procession can be allowed


1. only on Thursday 2. only on Friday
3. on either day 4. only if the religious procession is cancelled

128. Which of the following is NOT true?


1. Congress and SP can take out their processions on the same day.
2. The CPM procession cannot be allowed on Thursday.
3. The BJP procession can only take place on Friday.
4. Congress and BSP can take out their processions on the same day.

CAT PAPER – 2003 (Re-Test) Page 75


Directions for questions 129 to 132: Each question is followed by two statements, A and B. Answer
each question using the following instructions:

Choose 1 if the question can be answered by using statement A alone but not by using B alone.
Choose 2 if the question can be answered by using statement B alone but not by using A alone.
Choose 3 if the question can be answered by using either statement alone and
Choose 4 if the question can be answered using both the statements together but not by either statement
alone.

129. In a cricket match, the ‘Man of the Match’ award is given to the player scoring the highest number
of runs. In case of a tie, the player (out of those locked in the tie) who has taken the higher number
of catches is chosen. Even thereafter if there is a tie, the player (out of those locked in the tie) who
has dropped fewer catches is selected. Aakash, Biplab, and Chirag who were contenders for the
award dropped at least one catch each. Biplab dropped two catches more than Aakash did, scored
50, and took two catches. Chirag got two chances to catch and dropped both. Who was the ‘Man of
the Match’?
A. Chirag made 15 runs less than both Aakash and Bipla2.
B. The catches dropped less by Biplab are 1 more than the catches taken by Aakash.

130. Four friends — A, B, C and D got the top four ranks in a competitive examination, but A did not get
the first, B did not get the second, C did not get the third, and D did not get the fourth rank.
Who secured which rank?
A. Neither A nor D were among the first 2.
B. Neither B nor C was third or fourth.

131. The members of a local club contributed equally to pay Rs. 600 towards a donation. How much did
each one pay?
A. If there had been five fewer members, each one would have paid an additional Rs. 10.
B. There were at least 20 members in the club, and each one paid not more than Rs. 30.

132. A family has only one kid. The father says, “After ‘n’ years, my age will be 4 times the age of my
kid.” The mother says, “After ‘n’ years, my age will be 3 times that of my kid.” What will be the
combined ages of the parents after ‘n’ years?
A. The age difference between the parents is 10 years.
B. After ‘n’ years the kid is going to be twice as old as she is now.

Page 76 CAT PAPER – 2003 (Re-Test)


Directions for questions 133 to 137: Answer the questions on the basis of the following information.
Recently, the answers of a test held nationwide were leaked to a group of unscrupulous people.
The investigative agency has arrested the mastermind and nine other people A, B, C, D, E, F, G, H and I in
this matter. Interrogating them, the following facts have been obtained regarding their operation. Initially the
mastermind obtains the correct answer-key. All the others create their answer-key in the following manner.
They obtain the answer-key from one or two people who already possess the same. These people are
called his/her ‘sources’. If the person has two sources, then he/she compares the answer-keys obtained
from both sources. If the key to a question from both sources is identical, it is copied, otherwise it is left
blank. If the person has only one source, he/she copies the source’s answers into his/her copy. Finally,
each person compulsorily replaces one of the answers (not a blank one) with a wrong answer in his/her
answer key.
The paper contained 200 questions; so the investigative agency has ruled out the possibility of two or more
of them introducing wrong answers to the same question. The investigative agency has a copy of the
correct answer key and has tabulated the following data. These data represent question numbers.

Name Wrong Answer(s) Blank Answer(s)

A 46 —
B 96 46, 90, 25
C 27, 56 17, 46, 90
D 17 —
E 46, 90 —
F 14, 46 92, 90
G 25 —
H 46, 92 —
I 27 17, 46, 90

133. Which one among the following must have two sources?
1. A 2. B 3. C 4. D

134. How may people (excluding the mastermind) needed to make answer-keys before C could make his
answer-key?
1. 2 2. 3 3. 4 4. 5

135. Both G and H were sources to


1. F 2. B 3. I 4. None of the nine

136. Which of the following statements is true?


1. C introduced the wrong answer to question 27.
2. E introduced the wrong answer to question 46.
3. F introduced the wrong answer to question 14.
4. H introduced the wrong answer to question 46.

CAT PAPER – 2003 (Re-Test) Page 77


137. Which of the following two groups of people had identical sources?
I. A, D and G II. E and H
1. Only I 2. Only II 3. Neither I nor II 4. Both I and II

Directions for question 138: Answer the question on the basis of the following information.

138. Seventy percent of the employees in a multinational corporation have VCD players, 75% have
microwave ovens, 80% have ACS and 85 have washing machines. At least what percentage of
employees has all four gadgets?
1. 15 2. 5 3. 10 4. Cannot be determined

Directions for questions 139 to 142: Answer the questions on the basis of the following information.

Four families decided to attend the marriage ceremony of one of their colleagues. One family has no kids,
while the others have at least one kid each. Each family with kids has at least one kid attending the
marriage. Given below is some information about the families, and who reached when to attend the
marriage.

The family with two kids came just before the family with no kids.
Shanthi who does not have any kids reached just before Sridevi’s family.
Sunil and his wife reached last with their only kid.
Anil is not the husband of Joya.
Anil and Raj are fathers.
Sridevi’s and Anita’s daughters go to the same school.
Joya came before Shanthi and met Anita when she reached the venue
Raman stays the farthest from the venue.
Raj said his son could not come because of his exams.

139. Who among the following arrived third?


1. Shanthi 2. Sridevi 3. Anita 4. Joya

140. Name the correct pair of husband and wife.


1. Raj and Shanthi 2. Sunil and Sridevi 3. Anil and Sridevi 4. Raj and Anita

141. Of the following pairs, whose daughters go to the same school?


1. Anil and Raman 2. Sunil and Raman 3. Sunil and Anil 4. Raj and Anil

142. Whose family is known to have more than one kid for certain?
1. Raman’s 2. Raj’s 3. Anil’s 4. Sunil’s

Page 78 CAT PAPER – 2003 (Re-Test)


Directions for questions 143 to 146: Answer the questions on the basis of the following information.
Seven faculty members at a management institute frequent a lounge for strong coffee and stimulating
conversation. On being asked about their visit to the lounge last Friday we got the following responses.

JC: I came in first, and the next two persons to enter were SS and SM. When I left the lounge, JP and
VR were present in the lounge. DG left with me.
JP: When I entered the lounge with VR, JC was sitting there. There was someone else, but I cannot
remember who it was.
SM: I went to the lounge for a short while, and met JC, SS and DG in the lounge that day.
SS: I left immediately after SM left.
DG: I met JC, SS, SM, JP and VR during my first visit to the lounge, I went back to my office with JC.
When I went to the lounge the second time, JP and VR were there.
PK: I had some urgent work, so I did not sit in the lounge that day, but just collected my coffee and left.
JP and DG were the only people in the lounge while I was there.
VR: No comments.

143. Based on the responses, which of the two, JP or DG, entered the lounge first?
1. JP 2. DG
3. Both entered together 4. Cannot be determined

144 . Who was sitting with JC when JP entered the lounge?


1. SS 2. SM 3. DG 4. PK

145. How many of the seven members did VR meet on Friday in the lounge?
1. 2 2. 3 3. 4 4. 5

146. Who were the last two faculty members to leave the lounge?
1. JC and DG 2. PK and DG 3. JP and PK 4. JP and DG

CAT PAPER – 2003 (Re-Test) Page 79


Directions for questions 147 to 150: Answer the questions on the basis of the following information.

E ntra nce
C orrid or

The plan above shows an office block for six officers — A, B, C, D, E and F. Both B and C occupy offices
to the right of the corridor (as one enters the office block) and A occupies an office to the left of the corridor.
E and F occupy offices on opposite sides of the corridor but their offices do not face each other. The offices
of C and D face each other. E does not have a corner office. F’s office is further down the corridor than A’s,
but on the same side.

147. If E sits in his office and faces the corridor, whose office is to his left?
1. A 2. B 3. C 4. D

148. Whose office faces A’s office?


1. B 2. C 3. D 4. E

149. Who is/are F’s neighbour(s)?


1. A only 2. A and D 3. C only 4. B and C

150. D was heard telling someone to go further down the corridor to the last office on the right. To whose
room was he trying to direct that person?
1. A 2. B 3. C 4. F

Page 80 CAT PAPER – 2003 (Re-Test)


Section – 1
Sub-Section I – A: Number of Questions = 26

Note: Questions 1 to 26 carry one mark each.

Directions for questions 1 to 4: Answer the questions on the basis of the information given below.
The Dean's office recently scanned student results into the central computer system. When their character
reading software cannot read something, it leaves the space blank. The scanner output reads as follows:

Name Finance Marketing Statistics Strategy Operations GPA


Aparna B F 1.4
Bikas D D F F
Chandra D A F F 2.4
Deepak A B D D 3.2
Fazal D F B D 2.4
Gowri C C A B 3.8
Hari B A D 2.8
Ismet B A
Jagdeep A A B C 3.8
Kunal F A F F 1.8
Leena B A B F 3.2
Manab A B B
Nisha A D B A F 3.6
Osman C B B A 4.6
Preeti F D D 3.2
Rahul A C A F 4.2
Sameer C F B
Tara B 2.4
Utkarsh F C A 3
Vipul A C C F 2.4

In the grading system, A, B, C, D, and F grades fetch 6, 4, 3, 2, and 0 grade points respectively. The Grade
Point Average (GPA) is the arithmetic mean of the grade points obtained in the five subjects. For example
Nisha's GPA is (6 + 2 + 4 + 6 + 0) / 5 = 3.6. Some additional facts are also known about the students'
grades. These are
(a) Vipul obtained the same grade in Marketing as Aparna obtained in Finance and Strategy.
(b) Fazal obtained the same grade in Strategy as Utkarsh did in Marketing.
(c) Tara received the same grade in exactly three courses.

1. What grade did Preeti obtain in Statistics?


(1) A (2) B (3) C (4) D

CAT PAPER – 2004 MBA Page 1


Test Prep
2. In operations, Tara could have received the same grade as
(1) Ismet (2) Hari (3) Jagdeep (4) Manab

3. In Strategy, Gowri's grade point was higher than that obtained by


(1) Fazal (2) Hari (3) Nisha (4) Rahul

4. What grade did Utkarsh obtain in Finance?


(1) B (2) C (3) D (4) F
Directions for questions 5 to 8: Answer the questions on the basis of the information given below.
The data points in the figure below represent monthly income and expenditure data of individual members
of the Ahuja family ( ), the Bose family ( ), the Coomar family ( ), and the Dubey family ( ). For
these questions, savings is defined as

Savings = Incom e – Expenditure


In co m e

Lin e indicating
Incom e = E xpend itu re
300 0

200 0

100 0

0 E xpenditure
100 0 200 0 300 0

5. Which family has the lowest average income?


(1) Ahuja (2) Bose (3) Coomar (4) Dubey

Page 2 MBA CAT PAPER – 2004


Test Prep
6. Which family has the highest average expenditure?
(1) Ahuja (2) Bose (3) Coomar (4) Dubey

7. Which family has the lowest average savings?


(1) Ahuja (2) Bose (3) Coomar (4) Dubey

8. The highest amount of savings accrues to a member of which family?


(1) Ahuja (2) Bose (3) Coomar (4) Dubey

Directions for questions 9 to 12: Answer the questions on the basis of the information given below.
Prof. Singh has been tracking the number of visitors to his homepage. His service provider has provided
him with the following data on the country of origin of the visitors and the university they belong to:

Number of visitors Number of visitors


DAY DAY
COUNTRY 1 2 3 UNIVERSITY 1 2 3
Canada 2 0 0 University 1 1 0 0
Netherlands 1 1 0 University 2 2 0 0
India 1 2 0 University 3 0 1 0
UK 2 0 2 University 4 0 0 2
University 5 1 0 0
USA 1 0 1
University 6 1 0 1
University 7 2 0 0
University 8 0 2 0

9. To which country does University 5 belong?


(1) India or Netherlands but not USA (2) India or USA but not Netherlands
(3) Netherlands or USA but not India (4) India or USA but not UK

10. University 1 can belong to


(1) UK (2) Canada
(3) Netherlands (4) USA

11. Which among the listed countries can possibly host three of the eight listed universities?
(1) None (2) Only UK
(3) Only India (4) Both India and UK

12. Visitors from how many universities from UK visited Prof. Singh's homepage in the three days?
(1) 1 (2) 2
(3) 3 (4) 4

CAT PAPER – 2004 MBA Page 3


Test Prep
Directions for questions 13 to 16: Answer the questions on the basis of the information given below.
Purana and Naya are two brands of kitchen mixer-grinders available in the local market. Purana is an old
brand that was introduced in 1990, while Naya was introduced in 1997. For both these brands, 20% of the
mixer-grinders bought in a particular year are disposed off as junk exactly two years later. It is known that
10 Purana mixer-grinders were disposed off in 1997. The following figures show the number of Purana and
Naya mixer-grinders in operation from 1995 to 2000, as at the end of the year.

250 236 236


222

200 182
162
150 134
120 124

100 80

50 30

0 0
0
1995 1996 1997 1998 1999 2000

Purana Naya

13. How many Naya mixer-grinders were purchased in 1999?


(1) 44 (2) 50
(3) 55 (4) 64

14. How many Naya mixer-grinders were disposed off by the end of 2000?
(1) 10 (2) 16
(3) 22 (4) Cannot be determined from the data

15. How many Purana mixer-grinders were disposed off in 2000?


(1) 0 (2) 5
(3) 6 (4) Cannot be determined from the data

16. How many Purana mixer-grinders were purchased in 1999?


(1) 20 (2) 23
(3) 50 (4) Cannot be determined from the data

Page 4 MBA CAT PAPER – 2004


Test Prep
Directions for questions 17 to 20: Answer the questions on the basis of the information given below.
A study was conduced to ascertain the relative importance that employees in five different countries
assigned to five different traits in their Chief Executive Officers. The traits were compassion (C), decisive-
ness (D), negotiation skills (N), public visibility (P), and vision (V). The level of dissimilarity between two
countries is the maximum difference in the ranks allotted by the two countries to any of the five traits. The
following table indicates the rank order of the five traits for each country.

Country
Rank India China Japan Malaysia Thailand
1 C N D V V
2 P C N D C
3 N P C P N
4 V D V C P
5 D V P N D
17. Which of the following pairs of countries are most dissimilar?
(1) China and Japan (2) India and China
(3) Malaysia and Japan (4) Thailand and Japan

18. Which of the following countries is least dissimilar to India?


(1) China (2) Japan (3) Malaysia (4) Thailand

19. Which amongst the following countries is most dissimilar to India?


(1) China (2) Japan (3) Malaysia (4) Thailand

20. Three of the following four pairs of countries have identical levels of dissimilarity. Which pair is the
odd one out?
(1) Malaysia and China (2) China and Thailand
(3) Thailand and Japan (4) Japan and Malaysia

Directions for questions 21 to 26: Each question is followed by two statements, A and B. Answer each
question using the following instructions.
Choose (1) if the question can be answered by using one of the statements alone but not by using the
other statement alone.
Choose (2) if the question can be answered by using either of the statements alone.
Choose (3) if the question can be answered by using both statements together but not by either state-
ment alone.
Choose (4) if the question cannot be answered on the basis of the two statements.

21. Zakib spends 30% of his income on his children's education, 20% on recreation and 10% on
healthcare. The corresponding percentage for Supriyo are 40%, 25%, and 13%. Who spends more
on children's education?
A. Zakib spends more on recreation than Supriyo.
B. Supriyo spends more on healthcare than Zakib.

CAT PAPER – 2004 MBA Page 5


Test Prep
22. Four candidates for an award obtain distinct scores in a test. Each of the four casts a vote to
choose the winner of the award. The candidate who gets the largest number of votes wins the
award. In case of a tie in the voting process, the candidate with the highest score wins the award.
Who wins the award?
A. The candidates with top three scores each vote for the top score amongst the other three.
B. The candidate with the lowest score votes for the player with the second highest score.

23. In a class of 30 students, Rashmi secured the third rank among the girls, while her brother Kumar
studying in the same class secured the sixth rank in the whole class. Between the two, who had a
better overall rank?
A. Kumar was among the top 25% of the boys merit list in the class in which 60% were boys.
B. There were three boys among the top five rank holders, and three girls among the top ten
rank holders.

24. Tarak is standing 2 steps to the left of a red mark and 3 steps to the right of a blue mark. He tosses
a coin. If it comes up heads, he moves one step to the right; otherwise he moves one step to the left.
He keeps doing this until he reaches one of the two marks, and then he stops. At which mark does
he stop?
A. He stops after 21 coin tosses.
B. He obtains three more tails than heads.

25. Ravi spent less than Rs. 75 to buy one kilogram each of potato, onion, and gourd. Which one of the
three vegetables bought was the costliest?
A. 2 kgs potato and 1 kg gourd cost less than 1 kg potato and 2 kg gourd.
B. 1 kg potato and 2 kgs onion together cost the same as 1 kg onion and 2 kgs gourd.

26. Nandini paid for an article using currency notes of denominations Re. 1, Rs. 2, Rs. 5, and Rs. 10
using at least one note of each denomination. The total number of five and ten rupee notes used was
one more than the total number of one and two rupee notes used. What was the price of the article?
A. Nandini used a total of 13 currency notes.
B. The price of the article was a multiple of Rs. 10.

Page 6 MBA CAT PAPER – 2004


Test Prep
Sub-Section I-B: Number of Questions = 12

Note: Questions 27 to 38 carry two marks each.

Directions for questions 27 to 30: Answer the questions on the basis of the information given below.
Coach John sat with the score cards of Indian players from the 3 games in a one-day cricket tournament
where the same set of players played for India and all the major batsmen got out. John summarized the
batting performance through three diagrams, one for each game. In each diagram, the three outer triangles
communicate the number of runs scored by the three top scores from India, where K, R, S, V, and Y
represent Kaif, Rahul, Saurav, Virender, and Yuvraj respectively. The middle triangle in each diagram
denotes the percentage of the total score that was scored by the top three Indian scorers in that game. No
two players score the same number of runs in the same game. John also calculated two batting indices for
each player based on his scores in the tournaments; the R-index of a batsman is the difference between
his highest and lowest scores in the 3 games while the M-index is the middle number, if his scores are
arranged in a non-increasing order.

Y (40) K (51) R (55)

90% 70% 80%

V (130) K (28) S (75) R (49) Y (87) S (50)

P akistan S outh A frica A ustralia

27. For how many Indian players is it possible to calculate the exact M-index?
(1) 0 (2) 1 (3) 2 (4) More than 2

28. Among the players mentioned, who can have the lowest R-index from the tournament?
(1) Only Kaif, Rahul or Yuvraj (2) Only Kaif or Rahul
(3) Only Kaif or Yuvraj (4) Only Kaif

29. How many players among those listed definitely scored less than Yuvraj in the tournament?
(1) 0 (2) 1 (3) 2 (4) More than 2

30. Which of the players had the best M-index from the tournament?
(1) Rahul (2) Saurav (3) Virender (4) Yuvraj

CAT PAPER – 2004 MBA Page 7


Test Prep
Directions for questions 31 to 34: Answer the questions on the basis of the information given below.
Twenty one participants from four continents (Africa, America, Australasia, and Europe) attended a United
Nations conference. Each participant was an expert in one of four fields, labour, health, population studies,
and refugee relocation. The following five facts about the participants are given.
(a) The number of labour experts in the camp was exactly half the number of experts in each of the
other three categories.
(b) Africa did not send any labour expert. Otherwise, every continent, including Africa, sent at least one
expert for each category.
(c) None of the continents sent more than three experts in any category.
(d) If there had been one less Australasian expert, then the Americas would have had twice as many
experts as each of the other continents.
(e) Mike and Alfanso are leading experts of population studies who attended the conference. They are
from Australasia.

31. Which of the following combinations is NOT possible?


(1) 2 experts in population studies from the Americas and 2 health experts from Africa attended the
conference.
(2) 2 experts in population studies from the Americas and 1 health expert from Africa attended the
conference.
(3) 3 experts in refugee relocation from the Americas and 1 health expert from Africa attended the
conference.
(4) Africa and America each had 1 expert in population studies attending the conference.

32. If Ramos is the lone American expert in population studies, which of the following is NOT true about
the numbers of experts in the conference from the four continents?
(1) There is one expert in health from Africa.
(2) There is one expert in refugee relocation from Africa.
(3) There are two experts in health from the Americas.
(4) There are three experts in refugee relocation from the Americas.

33. Alex, an American expert in refugee relocation, was the first keynote speaker in the conference.
What can be inferred about the number of American experts in refugee relocation in the conference,
excluding Alex?
i. At least one
ii. At most two
(1) Only i and not ii (2) Only ii and not i (3) Both i and ii (4) Neither i nor ii

34. Which of the following numbers cannot be determined from the information given?
(1) Number of labour experts from the Americas.
(2) Number of health experts from Europe.
(3) Number of health experts from Australasia.
(4) Number of experts in refugee relocation from Africa.

Page 8 MBA CAT PAPER – 2004


Test Prep
Directions for questions 35 to 38: Answer the questions on the basis of the information given below.
The year was 2006. All six teams in Pool A of World Cup hockey, play each other exactly once. Each win
earns a team three points, a draw earns one point and a loss earns zero points. The two teams with the
highest points qualify for the semifinals. In case of a tie, the team with the highest goal difference (Goal For
- Goals Against) qualifies.
In the opening match, Spain lost to Germany. After the second round (after each team played two matches),
the pool table looked as shown below.

Pool A

Teams Games Won Drawn Lost Goals Goals Points


Played For Against
Germany 2 2 0 0 3 1 6
Argentina 2 2 0 0 2 0 6
Spain 2 1 0 1 5 2 3
Pakistan 2 1 0 1 2 1 3
New 2 0 0 2 1 6 0
Zealand
South 2 0 0 2 1 4 0
Africa

In the third round, Spain played Pakistan, Argentina played Germany, and New Zealand played South
Africa. All the third round matches were drawn. The following are some results from the fourth and fifth
round matches
(a) Spain won both the fourth and fifth round matches.
(b) Both Argentina and Germany won their fifth round matches by 3 goals to 0.
(c) Pakistan won both the fourth and fifth round matches by 1 goal to 0.

35. Which one of the following statements is true about matches played in the first two rounds?
(1) Germany beat New Zealand by 1 goal to 0.
(2) Spain beat New Zealand by 4 goals to 0.
(3) Spain beat South Africa by 2 goals to 0.
(4) Germany beat South Africa by 2 goals to 1.

36. Which one of the following statements is true about matches played in the first two rounds?
(1) Pakistan beat South Africa by 2 goals to 1. (2) Argentina beat Pakistan by 1 goal to 0.
(3) Germany beat Pakistan by 2 goals to 1. (4) Germany beat Spain by 2 goals to 1.

37. If Pakistan qualified as one of the two teams from Pool A, which was the other team that qualified?
(1) Argentina (2) Germany (3) Spain (4) Cannot be determined

38. Which team finished at the top of the pool after five rounds of matches?
(1) Argentina (2) Germany (3) Spain (4) Cannot be determined

CAT PAPER – 2004 MBA Page 9


Test Prep
Since, the three integers are positive, the value of ‘n’ Surface area of the cylinder PQSR
cannot be equal to 1, therefore the value of ‘n’ = 4 or
 2 5x 
= 2π  x + hx  = 2π  x +
m = n – 1 = 3. 2 (4 − x 
Hence, the three consecutive positive integers are 3,    2 
4 and 5.
 2 5 2   3 2
Hence, option (1) is the correct choice. = 2π  x − x + 10x  = 2π 10x − x 
 2   2 
24. 1
 3 10 
2
50 
1 1 1 1 1 1 = 2π  −  x −  + 
S = 1+ + + 1+ + + ... + 1+ +  2  3  3
12 22 22 32 20072 20082 
Maximum value of surface area of the cylinder will
1 1
Tn = 1 + + 10
n2 (n + 1)2 be at x = .
3
Hence, option (1) is the correct choice.
n4 + 2n3 + 3n2 + 2n + 1
=
n2 (n + 1)2 For questions 26 to 28:
The given information can be depicted as follows.
(i)
n2 + n + 1 1
= = 1+
2 2
n +n n +n
R o ad 3 ho use s on ea ch side o f th e roa d
2007 2007 1
1 1 
S = ∑ Tn = 2007 + ∑  −  = 2008 − 2008
n =1 n =1  n n + 1

(ii) Six houses – P, Q, R, S, T, U


Hence, option (1) is the correct choice. (iii) Colours – Red, Blue, Green, Orange, Yellow, White
(iv) Different heights
25. 1 (v) T = tallest & opposite to Red
(vi) Shortest opposite to Green
A (vii) U = orange & the position of U is: P/S U S/P
(viii) R = yellow & opposite to P
(ix) Q = Green & opposite to U
(x) P = White & (S, Q) > P > R (in height)
P N Q From (iv), (v), (vi), (ix) & (x), T > (S, Q) > P > R > U
in terms of height
From (iv), (vii), (viii), (ix) & (x), we get the following
h two cases.

(Talle st)
M 5 2/3 1 1 2/3 5
B C
R S
x R Q T T Q R
Ye llow G re en Blue Blue G re en Ye llow
Let, the height of the cylinder be ‘h’ cm and radius
be ‘x’ cm. R O AD OR R O AD
∆ANQ is similar to ∆QSC W h ite O ra ng e Red Red O ra ng e W h ite

AN QS 10 − h h P U S S U P
⇒ = ⇒ =
NQ SC x 4−x 4 (S ho rte st) 3 /2 3 /2 6 4
6
10 x 10 4
⇒ −1= ⇒ =
h 4−x h 4−x 26. 4 Diagonally opposite to yellow is red.
5 27. 5 Second tallest house is either Q or S. So, we can not
∴h = (4 − x)
2 determine.

Page 6 MBA CAT PAPER – 2008 Explanations


Test Prep
28. 2 Tallest house is T whose colour is Blue. 31. 4 For UK:

29. 5 Let volume of data transfer in India = Volume of data Re venue from Data transfar
× 100 = 30% (approx )
transfer in Singapore = X Total Re venue

For INDIA: 30
Revenue from Data transfer = × Total Re venue
ARDT for India ≈ $1 (approx) 100
∴ Revenue from data transfer = $X (approx) ARDT = $ 13 (approx)
30 Total Re venue
Re venue from data transfer
× 100 = 9% (approx)
∴ Volume of Data transfer =
100
×
13
Total Re venue
3
x ≈ × Total Re venue
⇒ Total Re venue ; × 100 (approx) 130
9
For Spain:
For SINGAPORE:

ARDT = $9 (approx) Re venue from Data transfar


× 100 = 15% (approx )
∴ Revenue from data transfer = $9X (approx) Total Re venue
ARDT = 6.5 (approx)

Re venue from data transfer


× 100 = 20.5% (approx) ∴ Volume of Data transfer =
15 Total Re venue
×
Total Re venue 100 6.5

9x 3
⇒ Total Re venue = × 100 (approx) ≈ × Total Re venue
20.5 130
Similarly, we can check the other options and easily
9x see that the volume of data transfer is NOT the same
× 100
Total Re venue for Singapore 20.5 for given pair countries.
= ≈ 4 (approx)
Total Re venue for India x
× 100
9 32. 2 Since Bhama got calls from all colleges, she has to
score marks in each section equal to at least the
So, the statement given in option (5) is true.
maximum of the cut-offs across colleges which means
45, 45, 46 & 45 in section A, B, C, & D respectively.
30. 3 Let total Revenue of Sweden in 2010 = x
This makes her total to be 181 with which she will
Therefore total Revenue of India in 2010 = 2x
clear the overall cut-offs of all institutes also.
For Sweden in 2010:
33. 3 Since we have to minimise the marks in a particular
ARDT = $6 section, we will maximise the marks in other 3 sections.
Revenue from data transfer = 2 × 18% of x Let us assume that marks obtained in each of the
three sections in which we are going to maximize the
2 × 18% of x
∴ Volume of data transfer = 6
score, is equal to 50. Now, the lowest overall cut-off
is 171 & second lowest is 175. Hence Charlie must
have scored at least 175 – (50 + 50 + 50) = 25 marks
For India in 2010: in the remaining section.
Let ARDT = y
Lets confirm whether he can clear sectional cut-offs
Revenue from data transfer = 3 × 9% of 2x also with such a distribution. On seeing the sectional
cut-offs, we conclude what they can be cleared with
3 × 9%of 2x
∴ Volume of data transfer = 50 marks each in section A, B & C and 25 marks in
y section D, which may enable Charlie to clear the
sectional cut-off of section D for college 1, 2, 3 or 5.
2 × 18%of x 3 × 9% of 2x
Therefore = ⇒ y = $9 Hence answer is 25.
6 y
Therefore % change in ARDT of India
9 –1
= × 100 = 800%
1

CAT PAPER – 2008 Explanations MBA Page 7


Test Prep
34. 3 Since we have to maximize Aditya’s marks, let use (To be read from Rows)
take the base values of 50 marks in each section and
try to reduce that by minimum values to ensure he
doesn’t get any call. We notice that by reducing the A B C D E F
marks obtained in section C to 41, we ensure colleges A X L L
1, 2, 3 & 5 are ruled out. Now for colleges 4 & 6,
B X W W
reducing the marks obtained in section D to 43, ensures
these colleges are also ruled out. Please note that we C L X L
are reducing the score to 1 less than the minimum cut-
D L X L
off across all colleges for that particular section.
In the other two sections A and B, Aditya may score 50 E W W X
each. So, the maximum possible aggregate marks = 50 F W W X
+ 50 + 41 + 43 = 184.
35. 2 E & F defeated A. [Please note that in this
For questions 35 to 38:
question option (1) and (5) were the same]
The given basic information can be collated as below:
(i) Six teams – A, B, C, D, E, F
36. 4 B, E & F won both the matches in Stage-II.
(ii) Matches scheduled in two stages – I & II.
(ii) No team plays against the same team more than once.
37. 5 D & F won exactly two matches in the event.
(iv) No ties permitted.
As per the instructions given for stage – I, we can
38. 5 B & E has most wins, 4 each.
reach the following conclusions:
(a) As B lost at least one match, hence A won all the 3
39. 1 Subscription in Europe in 2006 = 380 Mn USD
matches.
Subscription in Europe in 2007 = 500 Mn USD
(b) The two teams who lost all the matches cannot be A
(as explained above), cannot be B (E lost to B), cannot 500 – 380
be D (D won against C & F). Hence, the two teams % change in 2007 = × 100 ≈ 30%
380
must be C and F. Therefore subscription (based upon the growth rate
(c) F did not play against the top team (i.e. A).
of 2007 over 2006) in 2008 should have been
We get the following table for stage – I. = 500 × 1.3 = 650 Mn USD (approx)
Therefore difference from the estimated subscrip-
(To be read from rows)
tion = 650 – 600 = 50 Mn USD (approx)
[Please note that the unit is mentioned neither
A B C D E F in the question, nor in the options]
A X W W W
40. 1 Let total number of subscribers = 100x
B L X W W
Number of men = 60x
C L X L L Therefore number of men in 2010 = 60x × (1.05)7
D L W X W = 84.42x (approx)
E L W X W Number of women = 40x
Therefore number of women in 2010 = 40x × 1.17
F L L L X = 77.94x (approx)
Therefore total number of subscribers = 84.42x +
As per the instructions given for Stage-II, we can reach the 77.94 = 162.36x
following conclusions. % growth of subscribers
(d) A lost both its matches against E and F.
(e) F won against A, hence is the bottom team 162.36x – 100x
= = 62.36 (approx)
(out of C & F) which won both the matches 100x
⇒ F won against C as well.
This also means that C lost both its matches against 41. 4 Gap in 2008 = 780 – 600 = 180 Mn USD
B and F. Gap in 2009 = 810 – 700 = 110 Mn USD
(f) Apart from A and C, one more team lost both the
matches in Stage-II. 110 – 180
Annual % change = × 100 = –39%
That team can neither be E (A lost to E), nor B 180
(as C lost to B), nor F (as F won both its matches). Absolute change = 39% which is the highest.
Hence, the team must be D. Among the other options, option (3) ’06-07’ is closest,
We get the following table for Stage-II. but it will lead to only 22% change in gap.

Page 8 MBA CAT PAPER – 2008 Explanations


Test Prep
48. 3 Avg. Gross pay of HR department before transfer
500 – 380
42. 3 Growth rate of 2007 = × 100 = 31.58% = Rs. 5000 × 1.7 = Rs. 8500
380 Basic pay of the transferred person = Rs. 8000
New allowance of the transferred person = (80 + 10)
280 – 190
Growth rate of 2005 = × 100 = 47.37% = 90% of the basic pay
190 New Gross pay of the transferred person
Therefore % change in growth rate of 2007 relative = Rs. 8000 × 1.9 = Rs. 15,200
to growth rate of 2005 is New average gross pay of HR
47.37 – 31.58  15200 − 8500 
× 100 ≈ 35% dept. = Rs. 8500 +  
47.37  6 
43. 5 Since we do not know what are the share prices = Rs. (8500 + 1116)
during different times of the day we cannot come to
Percentage change = 1116 × 100 ≈ 13%
any conclusion. Hence (5) 8500
44. 5 Suppose the prices of shares remains same
49. 3 Since increase in average age of the Finance dept. is
throughout the day then all the four given statements
one year, the age of the person moving from Mktg. to
would not hold true. Hence (5).
Finance is more than that moving from Finance to
45. 1 As the prices are rising continuously, the earlier a Marketing, by 1 × 20 = 20 years.
person invests, the more profitable it would be. Hence, due to this transfer, cumulative age of Marketing
Abdul invested in the beginning only and hence reaped department has gone down by 20 yrs. But since the
in maximum return. average age of Mktg. dept. remaining unchanged, the
Between Bikram and Chetan, Chetan always invested person moving from Marketing to HR has age = (Avg.
the same amount but Bikarm invested more and more age of Marketing) – 20 = 15 years.
amount towards the end. Hence Bikram got the mini-
mum return.
New average age of HR dept. =
(5 × 45 ) + (1× 15 )
For questions 46 and 47: 5 +1
Let the share price are 10 am, 11 am, 12 noon, 1 pm, 2 pm and = 40 yrs.
3 pm be a, b, c ,d, e & f rupees respectively.
50. 2 Total basic pay of HR
From information (i) we get, a > f …(I) = 5 × 5000 (existing) + 2 × 6000 (from Maintenance)
+ 1 × 8000 (from Marketing) = Rs. 45,000
From information (ii) we get
a) Dane made profit i.e. (a + b + c) < (d + e + f) …(II) 45,000
New average = = Rs. 5,625
d 8
And, b) Emily made profit i.e. ×f >a …(III)
c 625
Percentage change = = 12.5%
Or, d × f > a × c …(IV) 5000
From information (iii) we get, e > f …(V)
From information (iv) we get, a > c …(VI) 51. 1 Sentence A is incorrect as the spelling of ‘imigrant’ is
not correct , should be ‘immigrant’. Sentence D is
incorrect because of a missing article and should be ‘
On combining in-equations (I) & (III) we get, c > d …(VII)
On combining in-equations (I) and (VIII) we get, the owner of a dry goods ….’. Sentence E is incorrect
a+c>d+f …(VIII) and should be ‘….. would later be known as…..’.
Sentence C is incorrect. We require a comma between
On combining in-equations (II) and (VIII) we get,
e>b …(IX) ‘brother-in-law’ and ‘David Stern’.
Hence, we get the sequence as (a > f > e > b) and (c > d).
And also we know that ‘a’ is greater than both ‘c’ and ‘d’, 52. 4 Sentence B should be’….its labour policy’ because
therefore ‘a’ is the highest among the six mentioned variables. the subject is Nike and we can’t substitute it with the
plural pronoun ‘their’. Sentence C should be ‘Perhaps
46. 1 The price at 10 am i.e. ‘a’ is greater than prices at sensing that the rising tide…’ as without ‘that’ the
any other time. sentence structure is incomplete. Sentence E should
be ‘ ….an industry..’ as the word industry begins with
47.1 and 4 a vowel so the appropriate article is ‘an’.
Since the price at 11 am i.e. ‘b’ is less than the price at
2 pm (i.e. ‘c’), hence statement (1) is necessarily false. 53. 3 Sentence B should be ‘…few millions…. ’Sentence D
From in-equation (VII) we know that c > d. should be … reach the hundreds who are marooned..
Hence statement (IV) is also necessarily false. Sentence E is incorrect as per subject verb agreement
and should be ‘…death count has begun’.

CAT PAPER – 2008 Explanations MBA Page 9


Test Prep
DATA INTERPRETATION QUESTION BANK

EXERCISE 1
Directions for Qs. 1 to 2: Refer to the following data and answer the following questions.

A B C
D
E F G
H
I

Each of the digits 1, 2, 3, 4, 5, 6, 7, 8 and 9 is represented by a different letter in the figure.


A + B + C, C + D + E, E + F + G and G + H + I is equal to 13.

1. Which of the digits does E represent?


(1) 9 (2) 4 (3) 7 (4) 1

2. Which of the digits does D represent?


(1) 8 (2) 7 (3) 3 (4) 7 or 8

Directions for Qs. 3 to 7: Refer to the following information and answer the following questions.
In a group of 200 people, number of people having at least primary education : number of people having at
least middle school education : number of people having at least high school education :: 7 : 3 : . 90 of
these play football and 60 play hockey. 5 people in category III (defined as people having high school
education) and one fourth each in category I and II (defined as people having primary school education
only and people having middle school education but not high school education, respectively) do not play
any game. In each of the above category the number of people playing only hockey equal the number of
people playing only football. 2 people each in categories I and II and 1 person in category III play both the
games. 2 people playing both games are uneducated (category IV). 5 people in category III play only
hockey.

Assume middle school education can be had only after completing primary school and high school
education can be had only after completing middle school. Also all people in the group fall under the four
categories described above.

3. How many people have middle school education?


(1) 16 (2) 32 (3) 48 (4) 64

4. How many high school educated people do not play football?


(1) 6 (2) 8 (3) 10 (4) 12

5. How many people having middle school, but not high school, education play only football?
(1) 2 (2) 7 (3) 11 (4) 15

6. How many people who completed primary school could not finish middle school?
(1) 48 (2) 64 (3) 80 (4) 96

7. How many uneducated people play neither hockey nor football?


(1) 15 (2) 20 (3) 23 (4) 28
Direction for Qs. 8 - 10: Refer to the following data and answer the following questions.

A simple coding system, using cryptograms, is designed as below. Three concentric wheels each having all
the alphabets and the number 0 to 9 are written around the rims of the wheels.

9 A
B
8 9 A
B C
8
7 9A C
B D
7 8 C D
7 D

8. Suppose that a cryptogram system is made by rotating the inner wheel 2 alphabets to the right and
rotating the outer wheel 2 alphabets to the left. Starting with the inner wheel, what would APPLE
be coded as?
(1) BRRC7 (2) 8RNJC (3) 8CCNR (4) 8RNNC

9. How many cryptogram system can be totally made from this device? (Starting inner wheel)
(1) 2315 (2) 1295 (3) 676 (4) Indeterminate

10. Assume that both the inner and outer wheels have been rotated one step, but you don’t know
whether left or right. What would the rotations be if HOCJ9 has to decode to a proper English
word? (Starting inner wheel)
(1) Inner right, Outer right (2) Inner left, Outer left
(3) Inner right, Outer left (4) Inner left, Outer right

Directions for Qs. 11 to 13: Each of these questions contains six statements followed by 4 sets of
combinations of 3. Choose the set in which the statements are most logically related.

11. A. All pianos are large B. All flutes are well turned
C. All pianos are loud D. All flutes are loud
E. All pianos are flutes F. All pianos are well tuned
(1) FBE (2) CDF (3) BEF (4) BCF

12. A. All acetones are corrosive substances.


B. All corrosive substances are acidic.
C. All acetones are alkaline.
D. Some acetones are alkaline and corrosive.
E. All acetones are acidic.
F. No oxide corrodes.
(1) ACB (2) BDE (3) BCD (4) BAE
13. A. Some oils are refined
B. Some oils are not brown.
C. Some refined oils are acceptable.
D. Only brown oils are acceptable.
E. Some oils are not acceptable.
F. Only refined oils are brown.
(1) BDE (2) CFE (3) AEB (4) ABE

Directions for Qs. 14-17: Refer to the following information and answer the following questions.
In a village far far away, by name Foolistan, there are four kinds of people: Dumbos, Idiots, Fools and
Morons. The village has each of these people in equal numbers, people in the village belong to exactly one
of the four kinds. Everyday, people in this village have the habit of making assess of themselves. However,
some rules are followed. A person may not make an ass of himself (or herself) at all. Idiots commit twice
the number of blunders as do Dumbos, Fools thrice as many as Idiots and Morons four times as many as
Fools. However, by a strange fact of the tribe, if even one person of a kind commits zero blunders, no one
in the group commits a blunder. Every Dumbo commits the same number of blunders as every other
Dumbo. Use these facts to answer the next four questions.

14. On a particular day, 66,000 blunders were committed. Which of the following is not possible?
(Assume the total population is 400)
(1) A Fool did not commit any blunder
(2) An Idiot did not commit any blunder
(3) A Moron did not commit any blunder
(4) None of these

15. The population of the village is 400. On a given day every group committed blunders, and Fools
as a group commit 1200 blunders. The total number of blunders committed on that day is:
(1) 5000 (2) 6600 (3) 66000 (4) 6000

16. On a particular day, the Morons don’t commit any blunders, and the other groups may or may not
have committed blunders. Which of the following is the least number of blunders, which certainly
satisfy the above the criteria? (Assume total population = 400)
(1) 600 (2) 100 (3) 2700 (4) 900

17. On a day, all four groups commit blunders and a Dumbo commits 3 blunders. If the population is
400, the total number of blunders on the day is:
(1) 99 (2) 9900 (3) 9000 (4) 990

Directions for Qs. 18 to 20: Refer to the following information and answer the following questions.

Shyam Kumar multiplier is a smart mathematician. While negotiating has salary with his prospective
employer he worked out and proposed a novel salary plan, which his employer accepted as he was one of
the smartest salesman around. According to the plan, his salary would increase every day of his
employment such that on any day his income would be two rupees more than the square of the number of
the days he has been employed for with the company.
Back home his wife Renu congratulated him on working out an excellent deal and then together they
planned their expenses in tune with the new salary plan. As they expected their income to be increasing on
a daily basis — they planned their expenses in a way that the expenses of any day would be met by the
day’s income itself, moreover they planned to make a net saving (which would be the difference of income
and the expenses of the day) out of the day’s income.

Thus they expected their expenses to be one rupee more than twice the number of days Shyam Kumar
would have been employed for on that day, with the company.
18. On which day of his employment did Shyam Kumar Multiplier’s savings of a day exceed Rs. 100?
(1) 10th (2) 11th (3) 12th (4) 13th

19. Shyam Kumar Multiplier wanted to get into Limca Book of Records. He eyed the category which
listed the person holding the record for the maximum salary increase in a single day. The person
currently holding the record is Salim Khan who was given a raise of Rs. 12,000 in a single day. If
Shyam Kumar were to beat his record he will have to remain in this job for a minimum of,
(1) 5999 days (2) 6000 days (3) 6001 days (4) 6002 days

20. Shyam and Renu planned to put each day’s savings into their savings bank account. They also
decided that they would regularly purchase Fixed Deposits in multiples of
Rs. 300 from their savings in the account. They would purchase their first Fixed Deposit on
(1) 11th day (2) 12th day (3) 13th day (4) 14th day

Directions for Qs. 21 – 25: Refer to the following information and answer following questions.
Alord received a large order for stitching school uniforms from Mayflower school and Little Flower school.
He has two cutters who will cut the fabric, five tailors who will do the stitching, and two assistants to stitch
the buttons and button holes. Each of these nine persons will work for exactly 10 hours a day. Each of the
May-flower uniforms requires 20 min. for cutting the fabric, one hour for stitching, and 15 min. for
stitching buttons and button holes, whereas the Little Flower uniform requires 30 min., 1 hour, and 30 min.
respectively for these activities.

21. What is the maximum number of little Flower uniforms that A lord can complete in a day?
(1) 50 (2) 20 (3) 40 (4) 30

22. On particular day, Alord decided to complete 20 Little Flower uniforms. How many Mayflower
uniforms can he complete on that day?
(1) 30 (2) 40 (3) 20 (4) 0

23. If Alord decides to complete 30 Little Flower uniforms only and no other on a particular day, how
many total man-hours will be idle?
(1) 20 (2) 30 (3) 5 (4) 25

24. If he hires one more assistant, what is the maximum number of Mayflower uniforms that he can
complete in a day?
(1) 40 (2) 50 (3) 60 (4) 30

25. A lord has the option to hire one more employee of any category. Which category should he hire
to get maximum increase in production capacity, assuming that he needs to stitch only Mayflower
uniforms on that day?
(1) Tailor (2) Cutter (3) Assistant (4) Cannot be determined

Directions for questions 26 to 35: Each questions is followed by two statements, A and B. Answer each
question using the following instructions.
Choose 1: if the question can be answered by using one of the statements alone, but cannot be
answered by using the other statement alone.
Choose 2: if the question can be answered by using either statement alone.
Choose 3: if the questions can be answered by using both the statements together, but cannot be
answered by using either statement alone.
Choose 4: if the question cannot be answered even by using both the statements together.

26. What is the value of the two digit positive integer N?


A. Four times N is 48 less than the square of the smallest two digit number.
B. N is a prime number whose square lies between 150 and 250.
27. ABCD is a cyclic quadrilateral. Is ABCD a rectangle?
A. AB⏐⏐ CD.
B. ∠ A + ∠ C = 1800
28. What is the value of the positive integer n?
A. The product of the numbers a, and b, which are respectively three less and two less than n,
is 0.
n!
B. = (n − 2)!
2

29. What is the perimeter of the triangle ABC? One of its side is 10√3 units.
A. ABC is the hypotenuse of the right angle triangle ABC.
B. The sum of the areas of the semicircles described on the three sides of the triangle ABC is
100 π sq. units.

30. The cost price of an article is 100. Find the profit made by selling it.
A. Ten percent discount was given on the list price and the profit percentage made is 25
percentage points more than the discount percent.
B. List price is Rs.180 and profit percent is 1/5th of the mark up percentage.

31. The length of trains of traveling at 90km/hr crosses train B in 32 seconds.


A. Train A traveling at 90 km/hr crosses train B in 32 seconds.
B. Train A and B are not traveling in the opposite directions.

32. Find the value of the real number N, where N > 0?


A. N is a two digits prime number less than 15, whose square and cube have the digit 1
occurring more than once.
B. N is a composite number less than 10, whose square and cube have their sum of digits equal
to the number itself or a multiple of it.

n
10 + 5
33. Is an integer?
3
A. n is an integers.
B. n is and natural integer.

34. Will A and B take more than 14½ days to complete the work working together?
A. If they work on alternate days with A starting the work, they take 28½ days to complete the
work.
B. If they work on alternate days with slower person among A and B starting the work, they
take 29 days to complete the work.

35. The length of train A and B are 4000m and 350 respectively. What is the speed of the train B?
A. Train B crosses train A which is traveling at 60km/hr in 22 seconds.
B. The speed of trains B is more than the speed of trains A.
Directions questions for 36 to 40: Study the following graph and answer the questions that follow.
PERCENTAGE SHARE OF NUMBER OF CARDS SOLD BY
VARIOUS MAKES OF CARS IN MARUTI (in '000s)
100 THE MARKET
90 5000
80
70 4000
60
50
3000
40
30
20 2000
10
0 1000
1994 1995 1996 1997 1998
0
Maruti Fiat Ambassador Ford Escort 1994 1995 1996 1997 1998

36. What is the number of Fiat cars sold in 1994 (in ‘000s)?
(1) 750 (2) 500 (3) 1500 (4) can’t say

37. Which of the following statements is false?


(1) The number of Fiat cars sold registered a decrease from 1995 to 1996.
(2) The number if Fiat cars sold registered an increase from 1996 to 1997.
(3) The number of Ambassador cars sold registered an increase from 1997 to 1998.
(4) None of these.

38. Which of the following statements is/are true?


(A) The percentage decrease in the number of cars sold by Fiat from 1994 to1998 is 75%
(B) The number of cars sold by Ford Escort in 1997 and 1998 is same.
(C) The percentage increase in the number in the number of cars produced by Maruti in 1996,
when compared to that of 1995 in more than 100%
(1) A and C (2) B and C (3) A and B (4) None of these

39. In which of the following years, did the umber of ford Escorts sold register a decrease compared
to that in the previous year?
(1) 1998 (2) 1996 (3) 1997 (4) Both1996 and 1998

40. The total number of cars sold is maximum in the year


(1) 1995 (2) 1996 (3) 1997 (4) 1998

Directions questions for 41 to 45: These questions are based on the following data.
50,000 Units of brand X are being sold in the market at a price of Rs.1o/- unit. A competitive brand, Y, enter the
market. The course of actions available for the company marketing brand X are
(1) Cut the price of X by 50 %, which would result in an increase in the number of units sold of X by
30 % with a probability of 0.5 and 20% with a probability of 0.5
((2) Advertise, which would cost Rs.2,00,00, but would result in an increase in the number of units
sold of X by 50% with a probability of 0.1, by 20% with a probability of 0.5 and 10% with a
probability of 0.4.
(3) Remain silent, in which case, the probability of losing the market by 40 % is 0.5 and the
probability of retaining its market is 0.5

41. Which is the best course of action to follow?


(1) cut down the prices (2) Advertising
(3) remain silent (4) Cannot be determined

42. In which of the cases, does the market of X (number of units sold) increase the most?
(1) cut down the prices (2) Advertising
(3) Remaining silent (4) None of these

43. What is the net loss if course 1 is followed ?


(Net Loss = Money realised originally – Money realized now)
(1) Rs.87,500 (2) Rs.1,00,000 (3) Rs.1,87,500 (4) Rs.2,00,000

44. What is the revenue realised (net of advertising expenses) if Course 2 is followed?
(1) Rs.4,00,000 (2) Rs.3,95,000 (3) Rs.3,12,500 (4) Rs.3,00,000

45. How much more/less in the net revenue realized by following course 2 than that realised by
following course 2 than that realised by following course 3?
(1) Rs.5,000 less (2) Rs.5,000 more (3) Rs.50,000 (4) Rs.50,000 more

DIRECTIONS for questions 46 to 50: Refer to the data below and answer the questions that follow.
ABC limited produces a product P for which demand is unlimited and they can sell all they produce. They
want to control costs and have three production plans from which to choose. The fixed cost incurred by the
company is Rs.1lac, Rs.1.5lacs and Rs.1lac from plan I, II and III respectively. Given below is the graph of
marginal cost of production virus number of units produced. It is also give that it can produce P in a batch
of 1000 only.
7
6
5
Rs. lacs

4
3
2
1
0
1 2 3 4 5 6
Unit productions (in '000)

Plan 1 Plan 2 Plan 3

46. If production = 2000, then which of the three is best plan?


(1) Plan 1 (2) Plan 2 (3) Plan 3 (4) Plan 1 or Plan 2

47. If production = 3000 then which of the three is the best plan?
(1) Plan 1 (2) Plan 2 (3) Plan 3 (4) Plan 1 or Plan 2

48. If production < 3000 then which of the three is the best plan?
(1) Plan 1 (2) Plan 2 (3) Plan 3 (4) Plan 1 or Plan 2

49. If production > 4000 then which of the three is the best plan?
(1) Plan 1 (2) Plan 2 (3) Plan 3 (4) Plan 1 or Plan 2

50. For some reasons, the company cannot choose plan1, then 2 is best plan if;
(1) Production≤4000 (2) Production<4000
(3) Production=3000 or Production=4000 (4) Non of these
EXERCISE 2
Direction for Qs. 1to5 : Refer to the following information and the answer the following questions.
People Power Corporation presently employs three Managers (A, B and C) and five recruitment agents (D,
E, F, G and H). The company is planning to open a new office in San Jose to manage placement of
software professionals in the US. It is planning to relocate two of the three managers and three of the five
recruitment agents to the office at San Jose. As it is an organization which is highly people oriented the
management wants to ensure that the individuals who do not function well together should not be made as a
part of the team going to the US. The following information was available to the HR department of People
Power Corporation.

Managers A and C are at each others throat and therefore cannot be sent as a team to the new
office.
C and E are excellent performers in their own right. However, they do not function together as a
team. They should be separated.
D and G have had a major misunderstanding during the last office picnic. After the picnic these
two have not been in speaking terms and should therefore not be sent as a team.
D and F are competing for a promotion which is due in another 3 months. They should not be a
team.

1. If D goes to the new office which of the following is (are) true?


I. C cannot go II. A cannot go III. H must also go
(1) I only (2) II and III only (3) I and III only (4) I, II and III

2. If A is to be moved as one of the Managers, which of the following cannot be a possible working
unit?
(1) ABDEH (2) ABFGH (3) ABEGH (4) ABDGH

3. If C and F are moved to the new office, how many combinations are possible?
(1) 4 (2) 1 (3) 3 (4) 5

4. Given the group dynamics of the Managers and the recruitment agents, which of the following is
sure to find a berth in the San Jose office?
(1) B (2) H (3) G (4) E

5. If C is sent to the San Jose office which member of the staff cannot go with C?
(1) B (2) D (3) G (4) F

Directions for Qs. 6 - 10: Refer to the following data and answer the following questions.
It is a game based on the position you take in a clock. You are at the 1 O’clock position. You can move one
step clockwise, 1 step anti clockwise or to a place that is diametrically opposite yours. For example, from 1
O’clock if you move clockwise you will be at 2 O’clock. As you start the game, you are at 1 O’clock
position and your score is 1. If you move a step clockwise, add the value of the time in that position to your
score to give you the new score. If you move a step anticlockwise, add the value of the time in that position
and subtract 2 from your score. If you move a step diametrically opposite, add the value of the time in that
position to your score and subtract 4 from your score to get the new score. You cannot get back to a
position that you have already visited.

6. What will be your minimum score after the third move?


(1) 10 (2) 7 (3) 11 (4) None of these
7. What will be your maximum score after the second move?
(1) 16 (2) 18 (3) 20 (4) 24
8. If you had moved a step anticlockwise in the first move, you could not have reached one of the
following positions in the third move.
(1) 10 O’clock (2) 5 O’clock (3) 7 O’clock (4) 6 O’clock

9. What is the shortest number of moves that you require to reach the 5 O’clock position When you
start from 1 O’clock position?
(1) 4 (2) 3 (3) 5 (4) 2

10. A man said to a lady, “Your mother’s husband’s sister is my aunt.” How is the lady related to the
man?
(1) Mother (2) Aunt (3) Sister (4) Grandmother

11. If P + Q means P is the brother of Q; P – Q means P is the mother of Q and P * Q means P is the
sister of Q. Which of the following means M is the maternal uncle of R, if you can assume a third
person K to be involved in establishing the relationship?
(1) M-K*P (2) M+K*R (3) M+K-R (4) M+K+R

Directions for Qs. 12 - 13: Refer to the following information and answer the following questions.
A, B, C and D are four ladies who are friends of Elizabeth. On one Saturday the four of them visited
Elizabeth at her weekend getaway.
I. The time of each visit was as follows: A at 8 O’clock, B at 9 O’clock, C at 10 O’clock and D at 11
O clock.
II. At least one woman visited Elizabeth between A and B.
III. At least one of C or D visited Elizabeth before A.
IV. C did not visit Elizabeth between B and D.

12. Who visited Elizabeth first?


(1) A (2) B (3) C (4) D

13. Who visited Elizabeth last?


(1) A (2) B (3) C (4) Insufficient data

Directions for Qs. 14 - 18 : Refer to the following information and answer the following questions.
Each of the questions is followed by two statements. You have to decide whether the information provided
in the statements is sufficient for answering the question.

Mark (1), If the question can be answered by using one of the statements alone, but cannot be
answered by using the other statement alone.
Mark (2), If the question can be answered by using either statement alone.
Mark (3), If the question can be answered by using both the statements together, but cannot be
answered using either statement alone.
Mark (4), If the question cannot be answered even by using both the statements together.

14. Is one of X or Y a fraction?


A. X2Y3 = 8 B. X is rational

15. A is a prime number. Is B a prime number?


A. B = 9A + 7 B. AB is even

16. Is 3x + 10y even?


A. x is even B. y is odd

17. Which amongst the three x, y and z (all real) is the greatest?
A. x : y : z : : 6 : 8 : 11 B. xyz – y2 is positive
18. What is the speed at which L is driving?
A. The ratio of the speeds of L and M is 3 : 5
B. L and M start from city X to Y simultaneously and as soon M reaches city Y he returns and
meets L on the way exactly 2 horus from the time they both left city X.

Directions for Qs. 19 – 23: Refer to the following information and answer the following questions.

Each question contains two statements in the question followed by four statements. Choose the alternative
from the four statements which is logically related and follows the statements in the question.

19. No M are P. Some M are S


(1) Some S are P 2. Some P are S (3) No S are M (4) Some S are not P

20. All M are P. All S are M


(1) All P are S (2) All M are S (3) All S are P (4) None of the above

21. Some M are not P. All M are S


(1) No S are P (2) All S are P (3) Some S are not P (4) No S are P
22. No P are M. Some S are not M.
(1) No S are P (2) All P are S (3) Some M could be S (4) All S are P

23. All Critical Thinking classes are full. John found a class that wasn’t full
(1) John might have found a Critical Thinking class
(2) The class John found was not a Critical Thinking class
(3) All Critical Thinking classes were not found
(4) John was a dumb guy

Directions for Qs.24– 25: Refer to the following information and answer following questions.

Speaker: The great majority of people in this city have access to the best medical care available any where
in the world.
Opposition: There are thousand of poor in this city who cannot afford to pay to see a doctor.

24. Which of the following is true of the opposition’s comment?


(1) It constitutes a hasty generalization on few examples
(2) It cities statistical evidence which tends to corroborate the views of the speaker
(3) It tries to compare two unrelated data and draws a contrarian conclusion
(4) It is not necessarily inconsistent with the speaker’s remarks.

25. A possible objection that opposition could have fielded to the spearker’s comments would be to
point to the existence of
(1) a city which has more doctors than this city
(2) a city in which people are given better medical care than this city
(3) a city which has a higher per capita hospital bed than this city
(4) the amount spent on medical insurance for people of this city
DIRECTIONS for questions 26 to 30: Refer to the data below and answer the questions that follow.

Figure 1

600

500
Property Crimes

400

300

200

100

0
75 80 85 90 95 2000
Year

Figure 2
60

50
Violent Crimes

40

30

20

10

0
75 80 85 90 95 2000
Year

The two graph above gives me crime statistics for the USA. Fig 1 give the variation of the number of
property crimes per 1000 households with time. Property crimes consist of motor vehicle theft, theft and
burglary. Fig 2 gives the number of violent crimes per 1000 population with time. Violent following can be
divisible into three categories – aggravated assault, simple assault and robbery. The following facts are also
given and may be used in answering the questions that follow.

1. The population of the USA between 1975 and 2050 is by the equation P= 2.3 (T-1950) + 157,
where P is the population in millions in the year T.
2. The number of persons per household can assumed to remain constant for the period 1975 to 2050.

26. Let x1 and x2 be the number of property crimes in 1975 and in 2000 expressed as a percentage of
the population, respectively. What is the ratio of x1 to x2?
(1) 3 : 1 (2) 27 : 10 (3) 1 : 3 (4) 18 : 5

27. Assume that the total number of property crimes per year follow the following trend after 2000.
The total number of property crimes per year at the end of every 25 year is 0.71 times the number
at the beginning. What is the number of property crimes per 1000 households in the year 2050?
(1) 63 (2) 90 (3) 129 (4) 180
28. In 2000, the number of aggravated assaults/1000 population was greater the number of
robberies/1000 population by 1.8 and the number of simple assaults was three time the number of
aggravated assaults. What was the total number of robberies in 2000?
(1) 1.1×106 (2) 1.1×103 (3) 3.9 (4) 3.9×103

29. Let y1 and y2 be the total number of violent crimes per year in 1975 and 2000, respectively. What
is y2 – y1
(1) 22 (2)-22 (3) 4.2 (4) none of these

30. The total number of property rimes in the year 200 was 1.45 times the total number of violent
crimes in the year 1975. what is the average number of person per household?
(1) 2.1 (2) 3.2 (3) 4.2 (4) 4.9

DIRECTIONS for questions 31 to 35: Refer to the data below and answer the questions that follow.

ABC is a firm which deals with furniture. Manufacturing of table requires three levels of assembly. The
finished table is at first level. The leg assembly and table top are second level. The pieces that go into the leg
assembly are at the third level which consist of shortrails, longrails and legs. One unit of table requires one
unit of tabletop and one unit of leg assembly. One unit of leg assembly requires 2 units of shortrails, 2 units of
longrails and 4 units. Orders are placed just in time to minimize storage.

The lead time for activities are (Lead time is waiting time required to complete one activity)
Parts Weeks
Assamble table 1
Finished leg assembly 1
Purchase legs 1
Purchase shortrails 1
Purchase longrails 1
Purchase table top 2

The availability of part at present time

Parts Units
Table 50
Leg assembly 100
Legs 150
Shortrails 50
Longrails 0
Table top 50

Demand of finished Tables

Details Week 4 Week5 Week6


Demands (units) 200 150 100

31. For meeting the demand of 200 units of finished table of week 4, when would the first order of
tabletops be placed?
(1) Week 1 (2) Week 3 (3) Week 4 (4) Week 5

32. What is the net requirement of legs for meeting the demand of week 4 finished table?
(1) 200 (2) 50 (3) 400 (4) 800
33. When and how many units of shortrail would be placed for meeting the demand of finished table
of week 6?
(1) 100 units in week 1 (2) 200 units in week 3
(3) 300 units in week 6 (4) Data insufficient

34. If in-hand units of legs are increased from 150 to 300, then what would be the net requirements of
legs for meeting the demand of finished table of week 5?
(1) 1800, 900 (2) 2200, 1100 (3) 1600, 800 (4) 800, 400

35. The supplier of longrails has shiftedhis manufacturing unit to its new location. Because of this the
delivery time of long rail has been increased by 1 week. When would the order of longrails be
placed to meet the week 5 demand of finished table?
(1) Week 1 (2) Week 4 (3) Week 5 (4) None of above

DIRECTIONS for questions 36 to 40: Each of the following questions is by two statements.
Mark [1], if the question can be answered by using any of the statements alone but not by using the
other statement alone.
Mark [2], if the question can be answered only by using either of the statements alone.
Mark [3], if the question can be answered only by using both the statements together.
Mark [4], if the question cannot be answered.

36. a, b, c, d are positive integer. Which is the second smallest of these?


a b b c a d c d
I. < , < II. < , <
3 4 2 3 3 6 4 3

37. LCM of two numbers A & a B is & is and HCF is 12. What is the number B?
I. A is not a factor B.
II. B is greater than A.

38. What is the age of Ram?


I. Sum of the ages of Ram and Shyam was 60 five years back.
II. Sum of the ages of Ram and Shyam would be 100 fifteen years from now.

39. What is the ratio of volume of sphere to of the cone?


I. Radius of the cone is twice that of the sphere.
II. Height of the cone is equal to the radius of the sphere.

40. Who is/are the tallest among A, B, C, D and E?


I. D is the tallest among C, D and E.
II. B, who is not shorter than D, is not the shorter of A and B.

Questions 41 to 43: are based on the following City College is selecting a four-person debate team. There are
seven candidates of equal ability. X, Y and Z, who belong to Group A; and L, M, N and P who belong to
Group B. The team must have two members from each group. Also, the members must be able to work well
with the all the members of the other team. Debaters Y & L, Z & N, L & M are incompatible pairs.

41. If debater Y is rejected and M is selected, the team will consist of


(1) L, M, X and Z (2) M, N, X and Z (3) M, N, P and X (4) M, P, X and Z.

42. If debater L is on the team, which other debaters must be on the team as well?
(1) M, X and Z (2) N, X and Z (3) P, N and Z (4) P, X and Z
43. If both Y and Z are selected, which of the other debaters are thereby assured of a place on the
team?
(1) Both L and M (2) Both M and P (3) Only N (4) Both N and P

Directions: Q 44-50: The following table gives the number of AIDS cases over 2 years of selected
countries.

Country 1996 1996 1997


(In ‘000 cases) (Rate) (In ‘000 cases)
Argentina 51 0.1 43
Australia 342 2.1 143
Austria 85 1.1 37
Bahamas 78 33.9 25
Belgium 85 0.8 25
Brazil 1361 0.9 206
Burundi 652 13.0 235
Canada 513 1.9 232
Chile 34 0.2 13
Denmark 97 1.8 25
Dominican Republic 256 3.9 152
Ethiopia 19 0.0 18
France 1852 3.3 555
French Guyana 45 56.2 10
Greece 53 0.5 18
Haiti 332 5.0 231
Honduras 58 1.2 38
Israel 13 0.3 11
Italy 888 1.5 231
Jamaica 37 1.4 13
Japan 34 0.0 7
Mexico 499 0.6 14
Netherlands 215 1.4 75
New Zealand 30 0.9 21
Norway 35 0.8 11
Portugal 44 0.4 35
South Africa 46 0.1 19
Sweden 73 0.8 34
Switzerland 163 2.4 84
United Kingdom 653 1.1 239
United States 21846 8.9 6442
Wet Germany 873 1.4 222
Yugoslavia 18 0.0 12
Zambia 286 4.0 218

All countries that have reported more than five hundred AIDS cases to the WHO in 1997 are listed here.
The left column gives the total number of cases reported by each country for 1996, the middle column
gives the 1996 rate (AIDS cases per 10000 population) and the last column shows the number of cases
reported in early 1997.
Most 1997 reports were for only the first quarter or a third of the year. Owing to reporting delays of six
months or more, cases reported in 1997 actually were diagnosed in 1996.

44. Which country has reported the maximum number of AIDS cases to WHO during 1996?
(1) Brazil (2) United States (3) France (4) Italy

55. How many countries have reported 25000 AIDS cased in the early 1997?
(1) Two (2) One (3) Three (4) None.
46. The ratio of AIDS cases reported in early 1997 is 2 : 5 for
(1) South Africa & Portugal (2) Israel & Denmark
(3) Yugoslavia & Netherlands (4) West Germany & France

47. The number of countries for which 1996 (rate) [per 100,000] is more than five is
(1) five (2) four (3) three (4) six

48. What is the population of Zambia on the basis of the reported cases of AIDS in 1996 (in
thousands)?
(1) 650000 (2) 700000 (3) 72000 (4) 715000

49. Which of the following are true from the table?


I. The reported AIDS cases of Ethiopia, Japan and Yugoslavia as compared to the
population is negligible.
II. The 1996 (rate) is highest for French Guyana though the reported cases are only 45000.
III. The population of Haiti is 66400000
IV. France reported more than 2000 cases of AIDS in 1996.
(1) I & II (2) II & III (3) I, II & III (4) I, II & VI

50. The countries which have reported less than 2000 cases both in 1996 and early 1997 are
(1) Japan, Ethiopia and Israel (2) Jamaica, Yugoslavia & Japan
(3) Ethiopia, Chile & French Guyana (4) Ethiopia, Israel and Yugoslavia.
EXERCISE 3
DIRECTIONS for questions 1 to 10: Each question is followed by two statements.
You have to decide whether the information provided in the statements is sufficient for answering the
question.

Mark 1 if the question can be answered by using one of the statements alone, but cannot be answered by
using the other statement alone.
Mark 2 if the question can be answered by using either statement alone.
Mark 3 if the question can be answered by using both statements together, but cannot be answered by
using either statement alone.
Mark 4 if the question cannot be answered even by using both the statements together.

1. One side of the triangle ABC is 5cm long. Is the largest angle in triangle ABC greater than 90°?
I. Triangle ABC is a right angled triangle 4,
II. Two sides of the triangle ABC are 3 cm and 4 cm long.

2. What is the number of institutes in Hyderabad that have an annual income of over Rs.5, 00, 000
and own a Laser Printer?
I. 30% of the institutes in Hyderabad have an annual income of over Rs.5, 00, 000
II. 25% of the institutes in Hyderabad which have an annual income of over Rs.5,00,000
own a Laser Printer.

3. Is a >b?
I. 8 - (a - b)3 is a positive number.
II. 4 - (a-b)2 is a negative number.

4. There are two identical cubical boxes A and B which contain 8 and 27 balls respectively. The balls
are made of the same material. Which box is heavier?
I. The balls are of different sizes.
II. The boxes are not made of same material.

5. Is a/b defined where a and b are integers?


I. a is neither a positive number nor a negative number.
II. b is neither a positive number nor a negative number.

6. What time does the clock show?


I. The angle between the hours hand and the minutes hand is 80°.
II. The time is between 3 O'clock and 4 O'clock.

7. A ladder is leaning against a wall at a height of 9m at 8:00 a.m. and it started slipping. What is the distance
between the point at which the ladder is contacting the wall and point at which the ladder is contacting the
floor at 8:30 a.m.?
I. The length of the ladder is 15 m.
II. The rate at which the top of the ladder is slipping is 2 cm per minute.
8. What is the speed of a particle at position x?
I. The speed of the particle was 5 cm/sec at point X0 which is 5 cm away from x.
II. The speed of the particle is increasing at the rate of 0.5 cm/sec every 1 cm it travels.
9. What is the ratio of two numbers p and q?
I. p is 50% greater than q.
II. p and q are respectively 87.5% and 25% greater than a third number r.

10. If each pencil costs Rs.2 and each pen costs Rs.3, then how many pens did I buy given that I
bought at least one pencil and at least one pen?
I. I bought one pencil.
II. I spent a total amount of Rs.8 buying pens and pencils.

DIRECTIONS for questions 11 to 14: These questions are based on the following information regarding
the price changes that a certain pharmaceutical company is considering for its products.

Product Existing Price (Rs.) Revised Price (Rs.)


Antacid 1.50 2.50
Anti Hypertensive 10.00 12.50
Expectorant 18.00/bottle 24.00/bottle
Anti Asthmatic 20.00 26.00
Anti Pyretic 5.00 8.00

The prices for all the products except Expectorant are the prices of 10 tablets.

11. If a family has a hypertensive and an asthmatic patient, where the person with hypertension has to
consume three tablets of Anti Hypertensive per day and the asthmatic patient has to take two tablets of
Anti Asthmatic every alternate day, what will be the increase in expenditure on two patients for 30
days?
(l) Rs.40.50 (2) Rs.42.75 (3) Rs.46.50 (4) Rs.38.50

12. What is the percentage increase in the expenditure of a person for one year if he consumes 32
tablets of Antacid in one week?
(1) 7½% (2) 6 ½ % (3) 62/3% (4) None of these

13. A person is prescribed to take two spoons of Expectorant thrice everyday for a period of, 20
weeks. Assuming that each bottle of Expectorant contains 90 spoons. Find the expenditure
according to the existing prices.
(1) Rs.200 (2) Rs.180 (3) Rs.168 (4) Rs.240

14. A person is prescribed a combination of antipyretic and anti-asthmatic such that he has to take one of
these before breakfast, the other after lunch and the one he had at breakfast at dinner also; if he
consumed an antipyretic at the end of the dinner after the 7th day of the course, he started the course
with
(1) Anti-asthmatic (2) Antipyretic
(3) Not possible to determine (4) None of the above
DIRECTIONS for questions 15 to 19: These questions are based on the pie diagrams given below.
Shefali, a first year student of management from a well known institute of management in western India, was
doing her internship with a leading public sector bank in India. Her project involved analyzing the market shares
of various Indian companies that manufacture and sell fuels and lubes. Halfway through her project she managed
to collect the following information from the sales figures of various companies.

Percentage shares of various companies in total Percentage shares of various companies in


sales of fuel (by value) total sales of lubes (by value)

PRL PRL
10% OTHERS 10%
OTHERS
15% 15%
PHCL PHCL
25% 25%

OICL OICL
30% 30%
PBCL
PBCL
20% 20%

Total Sales of fuels for the year Total Sales of lubes for the year
2000-2001 = Rs. 22, 400crores 2000-2001 = Rs. 22, 400crores
Shefali's project guide, after reviewing the above information, pointed out the fact that the above figures
were inclusive of the considerable volumes of inter-company sales that occur every year. Therefore the
correct market shares of the companies should be arrived at after deducting the inter company sales figures
from the present figures. Shefali then further collected the following information regarding the inter-
company sales.

BUYER Sale value as a percentage of the total sales of the selling company
SELLER PRL OICL PHCL PBCL OTHERS
Fuels - 50 10 - 40
PRL
Lubes - 30 - 10 60
Fuels — — 15 20 10
OICL
Lubes - - 20 - 40
Fuels - 20 - 20 20
PHCL
Lubes - 10 - 25 40
Fuels - 30 10 - 35
PBCL
Lubes - 15 5 - 25
Fuels - 10 5 10 -
OTHERS
Lubes - 15 5 15 -
15. By approximately what percentage did Shefali overestimate the correct value of the total sales of
fuels?
(1) 135% (2) 200% (3) 110% (4) 180%

16. If the correct sales figures are considered, then which of the following has the largest percentage
share by value of the sales of fuels and lubes put together?
(1) PRL (2) OTHERS (3) OICL (4) PBCL

17. If for any company, Sales - Purchases = Profit, then neither fuels nor lubes were profitable for
(1) PRL (2) PHCL (3) OTHERS (4) OICL
18. Assuming the information given in the above question as true, which of the following had the
maximum profitability for fuel?
(1) OICL (2) PHCL (3) PBCL (4) OTHERS

19. Which of the following had the second largest percentage share by value when the correct sales
figures of fuels and lubes put together are considered?
(1) OTHERS (2) PHCL (3) OICL (4) PBCL

DIRECTIONS for questions 20 to 23: These questions are based on the data given below.
A paddy processing unit purchases 10,000 kg of paddy. After subjecting it to milling operation, the husk
and rice are separated. The husk forms 5% of the total paddy weight. The rice so extracted from the paddy
contains 6% of brokens or defective material in it. Cost of purchase of paddy is Rs.3.50/kg and the selling
price of rice is Rs.8.00/kg. The broken material is sold at Rs.2.00/kg. The husk is sold atRs.0.50/kg.

20. What is the amount of paddy to be processed, such that the sale of the rice extracted from it excluding
brokens and husk fetches an amount of Rs.14, 288?
(1) 2000 kg (2) 1786kg (3) 2500 kg (4) 1886 kg

21. Assuming that all the rice produced by processing 10,000 kg of paddy is sold, what would be the
decrease in profits if the brokens in the operations increased to 8%?
(1) Rs. 1,000 (2) Rs. 1,100 (3) Rs. 1, 140 (4) Rs. 1,200

22. Due to the evaporation of moisture, there is a 5% loss of weight before the processing of the.
Paddy began. As a result the decrease in the weight of rice obtained (in kg) is
(1) 450 kg (2) 500 kg (3) 447 kg (4) 475 kg

23. What is the amount realised on the sale of by products i.e., broken and husk on processing 5,000
kg of paddy
(l) Rs.200 (2) Rs.180 (3) Rs.695 (4) Rs.190

DIRECTIONS for questions 24 to 27: These questions are based on the data given below.
In a zoo, twelve animals, Elephant. Wolf Deer, Bear. Gorilla, Tiger. Antelope, Giraffe, Lion, Chimpanzee, Horse
and Zibra are locked in twelve different cages from 1 through 12 (not necessarily in that order.) These cages are
on either side of the path. Cages 1 through 6, in that order, are on the left side of the path and the cages 7 through
12, in that order, are on the right side of the path. Cage 1 is opposite Cage 7, Cage 2 is opposite Cage 8 and so on.
The arrangement of the animals is as follows.

1. The Elephant is in the P1 cage and is diagonally opposite to the Wolf which is opposite to the
Chimpanzee.
2. The Bear is opposite to the Giraffe which is exactly in between the Gorilla and the Antelope.
3. The Deer is opposite to the Lion which is exactly in between the Wolf and the Tiger.
4. The Horse is diagonally opposite to the Chimpanzee.

24. If the Tiger is not opposite to the Gorilla, then which animal is adjacent to the Elephant?
(1) Antelope (2) Horse (3) Gorilla (4) Cannot be determined

25. In how many ways can the animals be arranged in the cage?
(l) One (2) Two (3) Three (4) Four

26. Which animal is exactly in between the Giraffe and the Deer?
(1) Antelope (2) Giraffe (3) Gorilla (4) Cannot be determined

27. If the Elephant is shifted to 12th cage and the Deer is adjacent to the Gorilla then which animal is
opposite to the Tiger?
(1) Antelope (2) Lion (3) Gorilla (4) Deer
28. Five items A, B, C, D and E are kept in five different boxes numbered 1 to 5. Exactly one item is
kept in each box. C's box number is as much less than E's box number as E's box number is less 1
than A's box number. D is in box number 4 but not next to E. Then B is in box number
(1) 3 (2) 1 (3) 5 (4) 4

Directions Q. 29 to 33: Study the following table and answer the question given below it.
Number of different types of tyres of tyres sold by a company over the years (Numbers in lakh)

Year A B C D E
1989 35 20 40 14 25
1990 40 15 55 20 35
1991 30 25 45 25 30
1992 25 30 50 30 35
1993 45 35 25 38 32
1994 42 28 34 42 30
1995 36 34 38 48 25

29. What was the approximate percentage increase in the number of D type tyres sold from 1992 to
1993?
(1) 26 (2) 30 (3) 35 (4) 40

30. In which of the following years was the percentage of D type tyres sold to total number of tyres
sold the maximum among the given years?
(1) 1992 (2) 1994 (3) 1990 (4) 1995

31. In the case of which of the following types of tyres was there continuous increase in sales over the
years?
(1) B (2) E (3) D (4) C

32. What was the difference in the number of C type tyres sold in 1990 and 1994?
(1) 21,000 (2) 2,10,000 (3) 2,50,000 (4) 21,00,000

33. The share of B type tyres in total sales of the company in 1994 was:
(1) 13% (2) 14% (3) 15% (4) 16%

DIRECTIONS for questions 34 to 36: These questions are based on the data given below.
In the recently held Asian Games, an apartment in the games village was allotted to five sportsmen -
Pranay, Qureshi, Rasool, Surendar and Tarif. They were practising for the games in which they were
participating. The games in which they are participating are as follows.

1. No two players play the same number of games.


2. Exactly three players participate in swimming.
3. Tennis was the only game common among Pranay, Qureshi and Tarif.
4. Weightlifting is the most common game.
5. Qureshi and Rasool are the only chess players and when Surendar joins them they practise
weightlifting.
6. Cycling was the common game of Rasool and Tarif.

34. Which player plays all the five games?


(1) Pranay (2) Qureshi (3) Rasool (4) Surendar
35. Which among the following statements is definitely true?
(1) The only common game among Pranay, Qureshi and Rasool is swimming.
(2) The only common game between Qureshi and Tarif is cycling.
(3) Qureshi and Rasool participate in ail the games except Tennis.
(4) The number of players participating in each game is unique.

36. How many players participate in weightlifting?


(1) five (2) three (3) two (4) four

DIRECTIONS for questions 37 to 38: Select the correct alternative from the given choices.

37. Eighty players numbered I through 80 are standing in a row, one behind the other, in the
increasing order of their numbers. The physical director of the players performed eight successive
inspections of the players and in each of the respective inspections he sent the first 10, 20, 30, 40,
50, 60, 70 and 80 players, from the front of the row to the end of the row. Each time the players
being sent back one after another. After these eight rounds of inspections what is the position of
the player numbered 5?
(1) 46th (2) 45th (3) 35th (4) None of these

38. Five friends A, B. C. D and F wear five different coloured shirts Red. White, Green, Blue, and
Black (not necessarily in that order) and five different coloured Trousers Red, White, Green, Blue
and Black (not necessarily in that order) such that no person wears the shirt and the trouser of
same colour. It is known that neither B nor E wears the Blue trouser while A wears a Black shirt.
A. person has to wear only a White or Blue shirt with a White or Blue trouser. Which of the
following gives a correct combination of the colour of A's trouser and the person wearing the Blue
trouser given that D wears the Green shirt but not the Black trouser?
(1) Green, D (2) Green, C (3) Red, D (4) cannot be determined

DIRECTIONS for questions 39 to 42: These questions are based on the pie charts given below
SECTORAL OUTLAY
1991- Total Outlay Rs. 2, 00, 000 1992- Total Outlay Rs. 2,50, 000

Medicine
10% Educations
Industrial 10% Medicine
Industrial
10% 15%
15%
Educations
25%

Agriculture
15%

Rural
Developme
Rural
nt Agriculture Developmen
30% 20% t
50%

39. In 1992, the amount allocated to the sector recording a decrease in outlay of 8 percentage points
over the one year period exceeds the amount allocated to the sector recording an increase in outlay
of 5 percentage points by an amount of
(l) Rs.l0, 000 (2) Rs.12, 500 (3) Rs. 15, 000 (4) Rs. 17, 500

40. In the year in which the allocation to four sectors equals the allocation to Rural Development, the
outlay of the sector which has the maximum allocation out of these four sectors, exceeds that of
the sector to which there was least allocation by
(l) Rs.5, 000 (2) Rs. 10.000 (3) Rs.15, 000 (4) Rs.12, 500
41. The year in which allocation to Rural Development and other sectors are equal, the two sectors
whose combined aggregate exceeds 25% but is less than 50% of the total allocation are
(1) Agriculture and Medicine (2) Agricultural and Industrial
(3) Medical and Education (4) None of the above

42. For the sector which records maximum percentage increase in allocation as compared to the
previous year, the difference between the outlay for the two years is
(1) Rs.40, 000 (2) Rs. 5, 000 (3) Rs.50, 000 (4) None of these

DIRECTIONS for questions 43 to 46: These are based on the bar graph given below.

PRODUCTION OF COTTON
In thousands bales
102
90
84
76
72

54

42 45

18
16

0.6 0.6
Manipur Sikkim Nagaland Tripura Assam W. Bengal
1993-94 1994-95

43. The total production in 1994-95 of the states that individually show a growth of more than 10%
from 1993-94 to 1994-95 forms what proportion of the total production for all the states for that
year?
(1) 82.1% (2) 91.7% (3) 93.6% (4) 85.8%

44. If x denotes maximum percentage increase in 1994-95 for any state and y denotes the least
percentage increase for any state in 1994-95, then which of the following relations between x and
y is the most appropriate?
(1) (x-y)2 = 0 (2) (x –y)2 = 5621
(3) x2/y2 is undefinable (4) y2/x2 is undefinable

45. By what percentage does the combined production in 1993-94 of the two largest cotton producing
states exceed the production of the state whose production was the closest to the average
production of all the states that year?
(1) 360% (2) 255% (3) 160% (4) None of these

46. The state which records 33.33% increase in production produces approximately what percentage
of total production for 1993-94?
(1) 20 (2) 21 (3) 19 (4) 22
DIRECTIONS for questions 47 to 50: These questions are based on the following pie chart which
represents the percentage of various trees in a fruit orchard consisting of 21, 600 trees and the table which
represents percentages of fruit-yielding trees in each category of trees.

Apricot Type of tree Percentage of fruit yielding


Avocado trees in the category
Fig
Avocado 93
Apricot 87
Cherries 61
Peaches Fig 86
Cherries
Plums 77
Plums
Peaches 78

47. The number of fruit yielding Cherry trees in the orchard are
(1) 1701 (2) 4941 (3) 3402 (4) 5751

48. The number of fruit yielding apricot trees exceed the number of fruit yielding peach trees by
(1) 327 (2) 364 (3) 357 (4) 347

49. Which among the following has the greatest number of fruit yielding trees?
(1) Avocado (2) Apricot (3) Peach (4) Plum

50. The fruit yielding plum trees from what percentage of fruit yielding peach trees?
(1) 112 (2) 110 (3) 107 (4) 101

DIRECTIONS for questions 51 to 55: These questions are based on the table given below.

PERFORMANCE OF EIGHT STUDENTS IN MOCK CAT


Student code Verbal Ability Quantitative Reading Data Interpretation
Ability Comprehension
√ x net √ x net √ x Net √ x Net
S1 20 4 19 31 12 29 35 10 32.5 14 4 12
S2 42 8 40 18 4 16 40 10 37.5 20 5 18.75
S3 36 14 32.5 30 12 27 34 16 30 18 4 17
S4 7 4 6 12 8 10 32 12 29 14 4 13
S5 20 30 12.5 11 12 8 18 4 17 10 12 7
S6 40 10 37.5 30 20 25 30 20 25 15 5 12.5
S7 34 10 31.5 24 4 23 35 12 32 18 6 16.5
S8 28 20 23 32 16 28 30 16 26 20 4 19

√ → Number of questions correct


x → Number of questions wrong
net → Net score in the section
Cut-off Marks

Area Total number of questions Cutoff


in the section Marks
Verbal Ability 50 23
Quantitative Ability 50 18
Reading Comprehension 50 22
Data Interpretation 25 10

Students qualify in the test if they score more than or equal to cut off mark in all the sections. The sum of
the net scores in all four sections together is called Grand Total.

51. Among the students who have qualified what is the maximum Grand Total?
(1) 106.5 (2) 108.5 (3) 100 (4) 103

52. What is the least Grand Total among the students who have qualified?
(1) 100 (2) 96 (3) 94 (4) 96.5

53. Who among the following committed maximum number of mistakes?


(1) S1 (2) S2 (3) S3 (4) S7

54. Which of the following statements is true?


I. The person who scored the maximum mark in Reading Comprehension did not qualify at
the Exam.
II. The person who had the maximum Grand Total did not get maximum in any of the
individual sections.
III. S8 attempted maximum number of questions among all the students.
(1) Only I (2) Only I and ill
(3) All three statements are true (4) Only I and II

55. Who among the following answered maximum number of questions correctly?
(1) S8 (2) S7 (3) S5 (4) S6
EXERCISE 4

DIRECTIONS for questions 1 to 5: Refer to data below and answer the questions that follow.
In the two Tables below some statistics about consumption of electricity in India are given.
Table 1 gives the electricity consumed per (1) 1000 population, (20 1000 sq. kms of area and (3) per1000
consumers, for the period 1950 to 1950. note that all the population in India does not consume
electricity- only a fraction are actual consumers. The data is given in terms of GWH (Giga watt
Hour) and 1 GWH = 109 WH.
Table 2 gives the number of consumers as well its break up into various categories for the same period.
TABLE 1
Electricity Consumption in GWH

Year to 1950 1970 1980 1990 1993 1994


Per 1000 Population 0.016 0.09 0.253 0.253 0.299 0.32
Per 1000 Sq. kms area 1.7 15.4 25.1 64.4 80.8 88
Per 1000 Consumers 3.5 3.3 2.5 2.7 3.2 3.4

TABLE 2
Electricity of Consumers in Thousand

Year to 1950 1970 1980 1990 1993 1994


Domestic 1157 10165 22338 50389 60193 63406
Commercial 259 3206 4582 8002 9209 9558
Industrial 63 553 1150 2077 2337 2423
Agriculture 19 1571 4233 8631 9971 10372
Others 3 70 268 534 602 637
Total 1501 14665 32571 69633 82312 86399

1. Total electricity consumed by all the consumers in India in 1980 was


(1) 0.82×105 KWH (2) 0.82 ×107 KWH
9
(3) 0.82×10 KWH (4) None of above

2. India’s total land area in 1994 was


(1) 3.3×106 sq. kms (2) 3.3×107 sq. kms
8
(3) 3.3× 10 sq. kms (4) 3.3×109 sq. kms

3. Let f1 andf2 be the fraction of the total population who were electricity
f2
consumers in 1970 and 1980, respectively. What is the value of the ratio
f1
(1) 0.51 (2) 1.97 (3) 1.32 (4) Data insufficient

4. Consider the percentage rate of growth for the period 1980 to 1990 in the numbers of the four
types consumers Domestic, Commercial, Industrial and Agricultural. How many of these
percentage rates of growth were more than the percentage rate of growth of the total number of
consumers for the same period?
(1) 0 (2) 1 (3) 2 (4) 3

5. What was the percentage increase in the number of Domestic consumers from 1970 to 1994?
(1) 223% (2) 374% (3) 524% (4) 612%
DIRECTIONS for questions 6 to10: Each of the following questions is followed by two statements.
Mark [1], if the question can be answered by using any of the statements alone but not by using the
other statement alone.
Mark [2], if the question can be answered by using either of the statements alone.
Mark [3], if the question can be answered by using both the statements together.
Mark [4], if the question can not be answered.

6. Is y a perfect square where x and y are natural numbers?


I. y is divisible by (x+1) and (x+2)
II. y <200

7. Is a bag there are less than 40 marbles which are red, blue and black in colour. What is the
probability of picking a black marble?
I The probability of drawing a blue marble is 1/3.
II. The probability of drawing a red marble is 1/24.

8. If 102 is perfectly divisible by a natural number x, what is the value of x?


I. x + 17 is divisible 3. II. X + 2is divisible by 4

9. The sum of the prices of 2 shares X and Y is Rs.160. what is the price of Y?
III. When there is an increase of 10% in one of these of the shares and a decrease of 55 in the
other, there is a change of Rs.6 in Y.
IV. 50 < x < 130

10. In ∆ABC. ∠A =700. What is the value of ∠C?


I. ∠A -∠C < ∠B - ∠C II. B2 + C2 = 7300

DIRECTIONS for questions 11 to 14: Refer to the data and answer the questions that follow.
Three leading drug manufacturers, ajit Pharma, and Chinku pharma each launch an atipyretic drug. Each
company uses all or some of the 5 components A, B, C,D and E. the table below gives the composition of
these components in their drug.

A B C D E
Ajit Pharma 12 15 20 28 25
Bittu Pharma 37 15 13 15 20
Chinku Pharam 26 15 10 34 15
Above value are in percentage of composition. All manufactures produce their drug as a tablet of 300 mg.

11. .The effectiveness of components A, B, C, D are in the ratio 5 : 6 : 4 : 3, while E is just a sweetner
and does not have any therapeutic. Which of the following company’s tablet is most effective?
(1) Ajit Pharma (2) Bittu Pharma
(3) ChinkuPharma (4) Data Insufficient

12. If the side effects caused by drugs are proportional to the ratio of ‘s composition to the sum of B’s
and E’s composition, which drug has least side effect.
(1) Ajit Pharma (2) Bittu Pharma
(3) ChinkuPharma (4) Data Insufficient

13. Ajit Pharma finds that its sales are dipping, because it drug’s taste is too sweet. So, it decides to
decrease the composition of E and increase that C, keeping other composition unchanged. It also
wants to make sure that the side effect is not more than double its previous value. What could be
the maximum percentage of C’s composition in Ajit’s Drug? (Use data from the previous
questions if necessary).
(1) 25% (2) 30% (3) 34% (4) 37%
14. If in all, million tables are produced by these 3 companies, per annum, the consumption of
component C, annually would be:
(1) 60 kg (2) 90 kg (3) 105 kg (4) Data Insufficient

DIRECTIONS for questions 15 to 19: Refer to the data below and answer the questions that follow.

JET AIRWAYS

1ST august, 2002 to 31st October, 2002

(To & From)

Special Festival Fares


From Current Festival From Mumbai to Current Festival
Mumbai to fares (R.) fares (Rs) fares (Rs) fares (Rs)
Delhi 6095 3920 Udaipur 4405 2213
Kolkata 6855 3320 Bhopal 4125 2371
Bangalore 4485 2839 Mangalore 4280 2446
Chennai 4905 2744 Nagpur 4200 2555
Hyderabad 4140 2443 Jodhpur 4935 2684
Ahedabad 3255 2017 Jaipur 5530 2917
Pune 2215 1295 Calicut 5060 3018
Indore 3360 1965 Cochin 6215 3248
Aurangabad 2745 2098 Thiruvananthapuram 6700 3312

To and From fares are same and the rates are effective from 1st August,2002 to 31st October,2002.

15. Ramesh, an executive of a company at Mumbai visits Delhi, Nagpur and Indors every month and
Mr. Ganesh visits Kolkata, Manglore and Hyderabad every mnth from Mumbai. Find the
difference in th percentager benefit to the company between the rips of Ramesh and Ganesh in the
festival month.
(1) 11 (2) 25 (3) 35 (4) none of these

16. With the discounts offered for Kolkata and Delhi tickets, how many different kinds of tickets can
be bought respectively utilizing the festival benefit for other visits also
(1) 11 & 5 (2) 12 & 6 (3) 17 & 4 (4) 16 & 3

17. The largest absolute discount is what percentage of the lowest absolute discount?
(1) 3.87% (2) 216% (3) 464% (4) 546%

18. What is the average discount in the air fare approximately?


(1) Less than 10% (2) More than 50%
(3) Less than 50% (4) None of these

19. Four executives of a company visited one of the four cities Delhi, Bhopal. Ahemdabad and Indore.
All of them left on 29th July, 2002 and came after a weeks visit, by the return flight of the same air
lines. What would have been percentage saving in tickets expenditure if they had postponed their
visit by one week, and returned after a week.
(1) 12% (2) 24% (3) 38% (4) 42%

DIRECTONS for questions 20 to 24: Refer to the data below and answer the questions that follow.
The table provides the percentage of total revenues accounted for the top 2 firms, top 4 firms and the top 190
firms in various industries. However, one of the data points in one of the industries is incorrect which is then
corrected by adding or subtracting 4% from the incorrect data.
Industry Top2 form (% of Top 4 Firms (% of Top 10 Firms(% of
total industry) total industry) total industry)

Steel 60 75 100
Oil & gas 50 80 100
Shipping 40 50 100
Leather 4 6 10
Textiles 8 18 30
Sp. Chem 3 5 7
Elevators 100 100 100
Refrigerators 25 40 60

20. Which is the incorrect data point and what is the correct data?
(1) Top4firmsforOil & Gas, 75% (2) Top 4 firms for Leather,55
(3) Top 4 firms for Textiles,14% (4) Top 4 firms for Refrigerators, 45%

21. if the size of the largest steel firms is Rs5000 cr, which of the following cannot be the total size of
the steel industry?
(1) 10,000 cr (2) 12,500 cr (3)15,000 cr (4) 17,000 cr

22. How many industry definitely have at least 20 firms in all?


(1) 4 (2) 3 (3) 5 (4) 6

23. An industry faces restrictions if the top 2 firms form more than 40% of the industry and the top 4
firms more than 60% of the industry. An exception to this is if the largest firm in the industry is
owned firms by the government. Which of the following is true?
(1) Government owned firms can exist in a maximum of 3 industries
(2) 4 industries face restrictions.
(3) The ratio of the number of industries that face restrictions to the number of industries that do
not face restrictions isles than 0.5.
(4) None of above.

24. If were least 2 mergers in the steel industry last year (the year before the data pertains to), how
many firms definitely existed in that industry last year?
(1) 8 (2) 10 (3) 9 (4) 7

DIRECTIONS for Qns. 25 to 29: Consider the following statements where every person gets exactly one
different dish:
1. Ria will not get soup unless Janet gets hot coffee.
2. Gia will not get gums unless Veena gets soup.
3. Veena will not get tea unless Gia gets soup.
4. Janet will not get gums unless Ria gets hot coffee.
5. Janet will not get hot coffee unless Veena gets gums.
6. Gia will not get hot coffee unless Ria gets tea.
7. Gia will not get tea unless Ria gets hot coffee.
8. Ria will not get hot coffee unless Gia gets soup.
9. Veena will not get gums unless Ria gets the hot coffee.
10. Janet will not get tea unless Ria gets gums.
11. Gia will not get soup unless Ria gets gums.

25. Who gets gums?


(1)Ria (2)Gia (3) Janet (4) Veena
26. Who gets soup?
(1)Janet (2)Veena (3) Gia (4) Ria

27. Who gets hot coffee?


(1)Gia (2)Veena (3) Ria (4) Janet

28. Who does Janet get?


(1)Hot coffee (2)Soup (3) Gums (4) Tea

29. Dileep, Martin and Salman married Ranjana, Vidisha and Karuna (not necessarily in that order).
Each of the couples has a son; their names being Saumitra, Shyam and Subhash. Further
i. Ranjana married six months before Karuna did.
ii. Salman was first to marry & Dileep, the last. Al the marriages took place in 1998
between February (month of first marriage) and September (month of last marriage).
iii. None of the couples had a child within one year of their marriage.
iv. Saumitra was born within 16 months of his parents’ wedding. He was not born between
August & Janauary both months inclusive
v. Karuna’s son was born within 16 months of her marriage and Vidisha’s exactly 24
months after the marriage.
vi. Subash was born an American citizen in Janaury.

Who are Saumitra’s parents


(1)Dileep-Ranjana (2)Salman-Karuna
(3) Martin-Ranjana (4) Martin-Karuna

30. Sangt Kripalchand had been preaching daily how important it was not to tell a lie. At last, Seth
Jhuthamal decided to heed Sant’s teaching. So, henceforth, he would not tell a lie on Monday,
Wednesday, Thursday and Saturday; on other days he would continue to tell lies only.
Presently, a customer comes to his jewellery shop and Seth Juthamal ties to close a sales deal.
“But what is the guarantee that the jewellery is of specified parity” the customer asks.
For today is Tuesday, the Santji’s beloved day, when I don’t speak a lie.
“What if I make purchase tomorrow”, the customer enquires.
“Tomorrow may be too late as being Saturday I may lie that day” insisted Sethji.
So, this is how the conversation took place.

What could be the day of this conversation?


(1)Friday (2)Tuesday (3) Sunday (4) Any one of these

31. Every month Chess Federation of India publishes ranking of Indian Chess Players. They actually
complement the FIDE lists which are brought out at longer intervals. It was seen, observing
monthly lists for last tear, that top six players in the list remained same throughout the year but
there was considerable mutual change of rankings among these six. Thus ranking for January 2003
as follows:

January 2003 was as follows:


1. P. Harikrishna 2. D. Barua 3. K. Humpy
4. S. Chanda 5. K. Ramesh 6. S.S Ganguli

The list of rankings for Feb 2003 had an entirely different look with each of the six ranked in a
position from the previous one. The following facts are known :

1. No one else had his/her ranking changed by as many places as D. Barua, whose change in
ranking was the greatest of the six.
2. The product of Chanda’s ranking for the two months was the same as product of Ganguli’s
ranking for the two months.
Who was ranked 5th in the list for February 2003.
(1)P. Harikrishna (2)D. Barua (3) K. Humpy (4) None of these

32. In the following sum


EE E EEE
FF F XXX
+GGG + YYY
J K LM JKLM

where each of the different letters stand for a different digits, E stands for
(1)2 2 .3 (3) 4 (4) None of these

DIERCTIONS for Questions 33 - 35: The coach of Indian Snooker and Billiards team has a novel way of
providing them the required practice and improving the individual’s skill set. He does this through forming
practice teams from among the team members after carefully considering relative strengths and
weaknesses. Presently, he aims to reduce the number of practice teams from 4 – A1, A2, A3, A4 – to 3 to
be called B1, B2 and B3. The players on team A1 are: A, B and C. The players on Team A2 are: D, E, F
and G. The players on team A3 are: H and I. The players on team A4 are: J, K and L. A new team must, of
course, have a minimum of two players. Other conditions are:

1. Each of the new teams (B teams) must contain at least one member from the four original teams
(A teams)
2. K and L have to be in separate teams
3. Any of the B teams cannot contain all the members of any of the A teams.

33. What could be the maximum strength of a B team?


(1) 8 (2) 6 (3) 7 (4) 5
34. If one of the B teams is constituted of C, D, E, F and J only, which of the following groups cannot
completely represent one of the other B teams?
(1) A, B, I (2) B, H, K (3) G, I, L (4) B, G, I, L

35. If a B team is constituted of A, C, D and K only, which of the following groups does not
completely represent one of the other B teams?
(1) B, G, L (2) B, E, I (3) B, F, G, H (4) E, H, J, L

DIRECTIONS for Q 36 - 37: Refer to the following data


There are four bags on a shelf all in a straight horizontal line. Each bag contains a pair of socks and a tie. No
bag contains a pair of socks and tie the same colour as the bag or each other. All four bags, pairs of socks and
ties are either red, green, blue or yellow. No two bags are the same colour, no two ties are the same colour
and no two pairs of socks are the same colour.
The red tie is in the bag next to the bag containing the pair of green socks. The yellow socks are in the bag
next to the green bag which is next to the bag containing the green tie. The bag on the far left is red. The
blue socks are in the bag next to the bag containing the blue tie. The yellow bag is n’t next to the blue bag
which is next to the bag containing the red socks. The green tie is in the blue bag or the yellow bag. The
yellow tie is not in the red bag which is not, and is not next to the bag containing the yellow socks.

36. Which bag is the right most?


(1) Yellow (2) Green (3) Blue (4) Yellow or Blue
37. Which combination of bag tie and socks (in that order) is in the extreme left?
(1) Red- Blue-Green (2) Red–Green-Yellow
(3) Red-Yellow-Red (4) Red-Yellow-Yellow
DIRECTIONS FOR Q. 38-39: Refer to the following data
In a city state, government officials never tell the truth and those who are not government officials always
tell the truth. A visitor meets three residents of the city state and asks one of them,” Are you a government
official?”
The first resident answer the question. The second native then reports that the first resident denied being a
government official. The third resident says that the first resident is a government officials.

38. How many of these three residents are not government officials?
(1) 1 (2) 3 (3) 2 (4) Insufficient data

39 What is the order in which the three residents statements are true/false
(1) True, True, False (2) False, False, True
(3) True, True, True (4) Insufficient data

40. In a batch of 120 postgraduate History students each student has to select at least one subject out
of American History, Ancient Indian History. Modern Indian History and History of Modern
Europe. 90 students selected History of Modern Europe and an equal number. American History.
105 students selected Ancient Indian History and an equal number. Modern Indian History. AT
least how many students selected all the four subjects.
(1) 75 (2) 45 (3) 30 (4) Insufficient data
EXERCISE 5
Directions: Q 1 to 4 are based on the following bar charts:

LIQUOR SUPPLIED IN TAMIL NADU BY DISTILLERIES A, B, C, D, E (in lakh cases).

50 The values of C, D and E to


45 1986 are 1.64, 1.05 and
40 12.89 2.45 respectively.
35
10.78
30 12.07
25
9.35
The order of
20
15 11.92 distilleries is A to E
6.41 7.26
10
5.79
from top to bottom.
3.15 5.75
5
3.57 4.21
0
1986 1987 1988

Q.1 Which of the five states manufactured liquor at the lowest cost?
(1) Tamil Nadu
(2) Delhi
(3) The state which has the lowest value for (Wholesale Price – Excise duty)
per litre
(4) Cannot be determined

Q.2 If Excise duty is levied before the goods leave the factory, then which of the following choices
shows the excise duty in ascending order for the year 1986?
(1) ECABD (2) ADEBC (3) DCEBA (4) Cannot be determined

Q.3 By what per cent do the excise duty rates between the closest among the five rates nearly differ?
(1) 2 (2) 13 (3) 1/3 (4) Cannot be determined

Q.4 If the Tamil Nadu Co., with the least simple annual growth in sales in the given period had shown
the same rate of growth as the one which grew fastest, what would that company’s sale have been
in 1988, in lakh cases?
(1) 13 (2) 15.4 (3) 130 (4) Cannot be determined

Directions Q 5 to 7 : Refer to the following Bar-chart (values are in Rs crore):

160 141 143.9


140
120 100.5
100
80 67 65
60
40
20
0
1984 1985 1986 1987 1988
Q.5 What is the average value of the contract secured during the years shown in the diagram?
(1) Rs. 103.48 crore (2) Rs. 105 crore
(3) Rs. 100 crore (4) Rs. 125.2 crore

Q.6 Considering 1985 as the performance base of 100% to which other performances may be referred,
which of the following most closely represents the set of Project Export Performance indices for
’84, ’85, ‘86’, ’87, ’88?
(1) 150, 100, 211, 216, 97 (2) 100, 67, 141, 144, 65
(3) 150, 100, 200, 215, 100 (4) 120, 100, 220, 230, 68

Q.7 Which is the year in which the highest percentage decline is seen in the value of contract secured
compared to the preceding year?
(1) 1985 (2) 1988 (3) 1984 (4) 1986

Directions Q 8 to 13 :The table below shows the estimated cost (in Rs. lakh) of a project of laying a railway
line between two places.

1988 1989 1990 1991


1. Surveying 41.5 7.5 2.2 0.5
2. Cement - 95.0 80.0 75.0
3. Steel - 70.0 45.0 60.0
4. Bricks - 15.0 12.0 16.0
5. Other building - 25.0 18.0 21.0
material
6. Labour 2.1 25.0 20.0 18.0
7. Administration 7.5 15.0 15.0 14.0
8. Contingencies 1.0 15.0 4.2 5.0
Total 52.1 267.5 196.4 209.5

Q.8 If the cost of materials rises by 5% each year from 1990 onwards, by how much w ill the
estimates fall short?
(1) Rs. 11.4 lakh (2) Rs. 16.4 lakh
(3) Rs. 21.4 lakh (4) Rs. 26.4 lakh

Q.9 What ratio does cost of material bear to labour cost approximately?
(1) 4 : 1 (2) 8 : 1 (3) 12 : 1 (4) 16 : 1

Q.10 The total expenditure is required to be kept within Rs. 700 lakh by making a cut equally in all the
years, on expenditure of administration. What will be the percentage cut for 1989?
(1) 22.6 (2) 32.6 (3) 42.6 (4) 52.6

Q.11 If the length of line to be laid each year is in proportion to the provision for material and labour
cost, what fraction of the total length is proposed to be completed in the third year?
(1) 0.9 (2) 0.7 (3) 0.6 (4) 0.3

Q.12 After preparing the estimate, the provision for contingencies is felt inadequate and is therefore
doubled. By what percent does the total estimate increase?
(1) 3.47 (2) 2.45 (3) 1.50 (4) 3.62

Q.13 It is found at the end of 1990, that the entire amount estimated for the project has been spent. But
for 1991, the actual amount spent is equal to that which was estimated. By what per cent
approximately has the actual expenditure exceeded the estimate?
(1) 39 (2) 29 (3) 19 (4) 9
Directions for Q. 14 to 18: The first table gives the number of saris (of all the eight colours) stocked in six
regional showrooms. The second gives the number of saris (of all the eight colours) sold in these six regional
showrooms. The third table gives the percentage of saris sold to saris stocked for each colour in each region.
The fourth table gives the percentage of saris of a specific colour sold within that region. The fifth table gives
the percentage of saris of a specific colour sold across all the regions.

Study the tables and for each of the following questions, choose the best alternative.

Table 1
Region Blue Green Magent Brown Orange Red Violet Yellow Total
a
1 267 585 244 318 132 173 195 83 1994
2 341 480 99 199 234 119 200 109 1781
3 279 496 107 126 100 82 172 106 1468
4 198 307 62 221 65 96 124 91 1164
5 194 338 120 113 82 60 125 124 1156
6 158 261 133 104 71 158 128 82 1095
Total 1437 2454 765 1081 684 688 944 595 8658

Table 2
Region Blue Green Magent Brown Orange Red Violet Yellow Total
a
1 122 164 71 165 40 84 97 45 788
2 124 200 37 78 67 47 73 50 676
3 21 57 7 24 9 14 20 11 163
4 79 85 22 164 18 46 43 54 511
5 29 36 22 17 9 18 19 16 166
6 1 3 2 2 1 3 2 4 18
Total 376 545 161 450 144 212 254 180 2322

Table 3
Region Blue Green Magent Brown Orange Red Violet Yellow Total
a
1 46 28 29 52 30 49 50 54 40
2 36 42 37 39 29 39 37 46 38
3 8 11 7 19 9 17 12 10 11
4 40 28 35 74 28 48 35 59 44
5 15 11 18 15 11 30 15 13 14
6 1 1 2 2 1 2 2 5 2
All 26 22 21 42 21 31 27 30

Table 4
Region Blue Green Magent Brown Orange Red Violet Yellow Total
a
1 15 21 9 22 4 11 12 6 100
2 18 30 5 12 10 7 11 7 100
3 13 35 4 15 6 9 12 7 100
4 15 17 4 32 4 9 8 11 100
5 17 22 13 10 5 11 11 10 100
6 6 14 11 11 6 17 11 22 100
Table 5
Region Blue Green Magent Brown Orange Red Violet Yellow
a
1 32 30 44 37 28 40 38 25
2 33 37 23 17 47 22 29 28
3 6 10 4 5 6 7 8 6
4 21 16 14 36 13 22 17 30
5 8 7 14 4 6 8 7 9
6 0 1 1 0 1 1 1 2
Total 100 100 100 100 100 100 100 100

Q.14 In which region is the maximum percentage of blue saris sold?


(1) 2 (2) 3 (3) 1 (4) 4

Q.15 Out of its total sates, which region sold the minimum percentage of green saris?
(1) 1 (2) 6 (3) 4 (4) 2

Q.16 Which region sold the maximum percentage of magenta saris out of the total sales of magenta
saris?
(1) 3 (2) 4 (3) 2 (4) 1

Q.17 Which colour is the most popular in region 1?


(1) Blue (2) Brown (3) Green (4) Violet

Q.18 Which region-colour combination accounts for the highest percentage of sales to stock?
(1) (1, Brown) (2) (2, Yellow) (3) (4, Brown) (4) (5, Red)

Directions for Q. 19 to 22 : The table below give the achievements of Agricultural Development
Programmes from 1983-84 to 1988-89.

Programme 83-84 84-85 85-86 86-87 87-88 88-89

Irrigation (Cumulative in Million Hectares)

Major & Medium 22.05 22.70 23.20 24.0 24.60 25.32


Minor 28.60 32.77 32.77 34.20 34.20 35.14

High yielding varieties (Million Hectares)

1. Paddy 16.90 18.20 19.70 18.70 21.70 22.80


2. Wheat 15.90 16.10 16.80 17.80 19.40 19.10
3. Jowar 3.10 3.50 3.90 4.40 5.30 5.10
4. Bajra 2.90 3.60 4.60 4.70 5.40 5.20
5. Maize 1.40 1.60 1.60 1.70 1.90 2.00
Consumption of Chemical fertilisers (Million tons)

1. Nitrogen 3.42 3.68 4.07 4.22 5.20 5.49


2. Phosphate 1.11 1.21 1.32 1.44 1.73 1.89
3. Potash 0.59 0.62 0.67 0.73 0.78 0.84

Q.19 How much area, in million hectares, was brought under irrigation during the year 1986-87?
(1) 58.20 (2) 1.43 (3) 0.80 (4) 2.23

Q.20 The consumption of chemical fertiliser per hectare of gross cropped area is lowest for the year
(1) 1984-85 (2) 1985-86 (3) 1986-87 (4) 1987-88

Q.21 It is possible that a part of the minor irrigated area is brought under major and medium areas. In
which year has this definitely happened?
(1) 1984-85 (2) 1985-86 (3) 1986-87 (4) 1987-88

Q.22 In which year does the area cropped under high yielding varieties show a decline for the maximum
number of crops?
(1) 1988-89 (2) 1985-86 (3) 1986-87 (4) None of these

Directions for Q. 23 to 30: Each of the following questions is followed by two statements.
Mark 1, if the question can be answered with the help of statement 1 alone.
Mark 2, if the question can be answered with the help of statement 2 alone.
Mark 3, if both statement 1 and statement 2 are needed to answer the question, and
Mark 4, if the question cannot be answered even with the help of both the statements.

Q.23 How long did Mr. X take for the 5000 km journey with 10 stopovers?
(1) The ith stopover lasted i2 minutes.
(2) The average speed between any two stopovers was 66 kmph.

Q.24 A man distributed 43 chocolates to his children. How many of his children are older than five
years?
(1) A child older than five gets 5 chocolates.
(2) A child 5 years or younger gets 6 chocolates.

Q.25 If R is an integer between 1 & 9, and P - R = 2370, what is the value of R?


(1) P is divisible by 4. (2) P is divisible by 9.

Q.26 Is (x-1 - y-1)/x-2 - y-2) > 1?


(1) x + y > 0.
(2) X and y are positive integers and each is greater than 2.

Q.27 x, y, and z are three positive odd integers. Is x + z divisible by 4?


(1) y - x = 2 (2) z - y = 2

Q.28 Ramu went by car from Calcutta to Trivandrum via Madras, without any stopping breaks. The
average speed for the entire journey was 40 kmph. What was the average speed from Madras to
Trivandrum?
(1) The distance from Madras to Trivandrum is 0.30 times the distance from Calcutta to
Madras.
(2) The average speed from Madras to Trivandrum was twice that of the average speed from
Calcutta to Madras.
Q.29 X is older than Y, Z is younger than W and V is as old as Y. Is Z younger than X?
(1) W may not be older than V (2) W is not older than V

Q.30 The unit price of product P1 is non-increasing and that of product P2 is decreasing. Five years
hence, which product will be costlier?
(1) Current unit price of P1 is twice that of P2.
(2) 5 years ago, unit price of P2 was twice that of P1.

Directions Q. 31 to 35: Each of these items has a question followed by two statements. As the answer,
mark

1, if the questions can be answered with the help of both the statements but not with the help of
either statement itself.
2, if the question can not be answered even with the help of both the statements.
3, if the question can be answered with the help of statement II alone
4, if the question can be answered with the help of statement I alone

Q.31 What is value of x, if x and y are consecutive positive even integers?


I. (x - y)² = 4. II. (x + y)² < 100

Q.32 What is the profit percent?


I. The cost price is 80% of the selling price. II. The profit is Rs. 50.

Q.33 What is the length of the rectangle ABCD?


I. Area of the rectangle is 48 square units.
II. Length of the diagonal is 10 units

Q.34 What is the price of bananas?


I. With Rs.84, I can buy 14 bananas and 35 oranges.
II. If price of bananas if reduced by 50% then we can buy 48 bananas in Rs.12.

Q.35 What is the first term of an arithmetic progression of positive integers?


I. Sum of the squares of the first and second term is 116.
II. The fifth term is divisible by 7.
EXERCISE 6
Directions: Q. 1 to 4: are based on the table and information given below.

Bankatal works x hours a day and rests y hours a day. This pattern continues for 1 week, with an exactly
opposite pattern next week, and so on for four weeks. Every fifth week he has a different pattern. When he
works longer than he rests, his wage per hour is twice of what he earns per hour when he rests longer than
he works.

The following are his daily working hours for the weeks numbered 1 to 13:

1st week 5th week 9th week 13th week


Rest 2 3 4 -
Work 5 7 6 8

A week consists of six days and a month consists of 4 weeks.

Q.1 What is his salary for first month?


(1) 1440 (2) 2040 (3) 1320 (4) 1680

Q.2 Referring to the data given in Q. 187, Bankatlal’s average monthly salary at the end of the first
four months will be:
(1) 1760 (2) 2040 (3) 1830 (4) 1680

Q.3 The new manager Kushaldas stipulated that Rs. 5 be deducted for every hour of rest and Rs.25 be
paid per hour starting the 9th week, then what will be the change in Bankatlal’s salary for the 3rd
month? (Hourly deductions and salaries are constant for all weeks starting 9th week).
(1) 540 (2) 480 (3) 240 (4) 0

Q.4 Using the data in the previous questions, what will be the total earning of Bankatlal at the end of
sixteen weeks.
(1) 7320 (2) 7800 (3) 8400 (4) 7680

Directions: Q 5 to 9 are based on the table given below:

Machine M1 as well as Machine M2 can independently produce either Product P or Product Q. The times
taken by machines M1 and M2 (in minutes) to produce one unit of product P and Q are given in the table
below: (Each machine works 8 hours per day).

Product M1 M2
P 10 8
Q 6 6

Q.5 What is the maximum number of units that can be manufactured in one day?
(1) 140 (2) 160 (3) 120 (4) 180

Q.6 If the number of units of P is to be 3 times that of Q, what is the minimum idle time for maximum
total units manufactured?
(1) 0 minutes (2) 24 minutes
(3) 1 hour (4) 2 hours

Q.7 If equal quantities of both are to be produced, then out of the four choices given below, the least
efficient way would be:
(1) 48 of each with 3 minutes idle (2) 64 of each with 12 minutes idle
(3) 53 of each with 10 minutes idle (4) 71 of each with 9 minutes idle.
Q.8 If M1 works at half its normal efficiency, what is the maximum number of units produced, if at
least one unit of each must be produced?
(1) 96 (2) 89 (3) 100 (4) 119

Q.9 What is the least number of machine hours required to produce 30 pieces of P and 25 pieces of Q?
(1) 6 hours 30 minutes (2) 7 hours 24 minutes
(3) 5 hours 48 minutes (4) 4 hours 6 minutes

Directions: Q. 10 to 14 are based on the information given below:

A company produces five types of shirts - A, B, C, D, E - using cloth of three qualities - High, Medium and
Low - using dyes of three qualities - High, Medium, and Low. The following tables give, respectively:
1. The number of shirts (of each category) produced, in thousands.
2. The percentage distribution of cloth quality in each type of shirt, and
3. The percentage distribution of dye quality in each type of shirt.

Note: Each shirt requires 1.5 meters of cloth.

Distribution of cloth (%) Distribution of dye (%)


Shirt Number Shirt High Medium Low Shirt High Medium Low
Type (000) Type Type
A 20 A 80 20 - A 70 15 15
B 30 B 30 40 30 B 20 50 30
C 30 C - 70 30 C - 60 40
D 10 D - 60 40 D - 40 60
E 10 E - 10 90 E - 20 80

Q.10 What is the total requirement of cloth?


(1) 150,000 m (2) 200,000 m
(3) 225,000 m (4) 250,000 m

Q.11 How many metres of high quality cloth is consumed by A-shirts?


(1) 8,000 m (2) 112,000 m
(3) 24,000 m .(4) 30,000 m

Q.12 What is the ratio of low-quality dye used for C-shirts to that used for D-shirts?
(1) 3 : 2 (2) 2 : 1 (3) 1 : 2 (4) 2 : 3

Q.13 How many metres of low-quality cloth is consumed?


(1) 22,500 (2) 46,500 (3) 60,000 (4) 40,000

Q.14 What is the ratio of the three qualities of dyes in high-quality cloth?
(1) 2 : 3 : 5 (2) 1 : 2 : 5 (3) 7 : 9 : 10 (4) None of the above.
For 15 to 19 use the following data:
A manufacturer can choose from any of the three types of tests available for checking the quality of his
product. The graph gives the relatives costs for each of these tests for a given percentage of defective pieces.
Relative Cost
0.3
0.25
0.2 Test 1
Test 2
0.15
Test 3
0.1
0.05
0
0 0.05 0.1 0.15 0.2 0.25 0.3
% defective
Q.15 dopting Test-2 will be feasible if the percentage of defective pieces (p) lies between:
(1) 0.10 to 0.020 (2) 0.20 to 0.30
(3) 0.05 to 0.20 (4) 0.00 to 0.05

Q.16 If p is equal to 0.2, then which test will be feasible?


(1)either 1 or 2 (2) 2 only (3) 3 only (4) either 2 or 3

Q.17 When will Test-3 be feasible?


(1) p > 0.2 (2) 0.1 < p < 0.2
(3) 0.05 < p < 0.1 (4) p < 0.05

Q.18 When is Test-1 feasible?


(1) p < 0.05 (2) 0.0 < p < 0.2
(3) 0.1 < p < 0.2 (4) 0.05 to 0.2

Q.19 If p < 0.2, then the best alternative will be:


(1) Test-2 (2) Test-3 (3) Test-1 (4) Not Test-3

DIRECTIONS for Q. 20 to 26: The table below provides the data for production of ferrous and non-
ferrous metals in million dollars for the year 1990 and % change for the three periods for the entire world
and regionwise.
Value Non-Ferrous Metals Region Value Ferrous Metals
1990 Annual %change 1990 Annual %change
1980-90 1989 1990 1980-90 1989 1990
8660 6 4 11 World 11350 6 4 12
2308 8 7 15 Asia 2590 8 10 23
670 5 8 14 Japan 665 5 11 20
348 15 6 18 China + HK + Taiwan 393 15 18 28
908 9 8 19 Asian 1075 8 11 25
1408 7 5 11 North America 2632 9 11 17
502 9 7 18 Latin America 678 12 -3 15
3415 4 0 3 Western Europe 4490 4 1 5
3151 4 0 2 European Union (15) 4133 4 2 5
378 7 0 24 C./E Europe/ Baltic 423 5 -12 13
States/USSR
162 8 1 14 Central Europe 255 10 -1 13
147 - 1 34 USSR 80 - -23 12
203 3 10 27 Africa 297 4 -3 7
368 7 28 46 Middle East 300 6 1 14
Q.20 In which of the following regions production of Non-ferrous metals exceeded that of ferrous
metals in 1989?
(1) Asean (2) China + HK + Taiwan
(3) Japan (4) Middle East

Q.21 What was the following regions production of Ferrous metals in million dollars by developing
economies of Asia in 1990 (In Asia Japan is the only developed economy)?
(1) 1256 (2) 1568 (3) 1925 (4) 1638

Q.22 Which of the following values is closest to world production of ferrous metals in the year 1988
(million dollars)?
(1) 13200 (2) 9000 (3) 7500 (4) 9750

Q.23 What percentage of the world production of non-ferrous metals was contributed by non-EU
countries of Western European in 1980?
(1) 6% (2) 36% (3) 3% (4) Indeterminate

Q.24 What was the CAGR of ferrous metals production between the year 1988-90 in USSR,
Central/East Central/East Europe, European Union, Batlic States and non-EU-Western Europe
economies?
(1) 2% (2) 4.1% (3) 3% (4) 0.40%

Q.25 Which of the following is not true?


(1) None ferrous metals production as a percentage of ferrous metals production in the year 1990
was highest for USSR.
(2) China accounted for less than 4 percent of all metals production in 1990.
(3) Africa contributed to less than 2 percent of all metals production in 1990.
(4) USSR witnessed the second highest annual rate of growth on non-ferrous metals production in
1990.

Q.26 What is value of total metals production of Western of Western Europe in 1988 (million dollars)?
(1) 7730 (2) 7550 (3) 7900 (4) 8300

DIRECTIONS for Q 27 - 13: Each question is followed by two statements A and B. Answer each question
using the following instructions.

Choose (1) if the question can be answered by using statement A alone but not by using B alone.
Choose (2) if the question can be answered by using statement B alone but not by using A alone.
Choose (3) if the question can be answered by using both the statements together, but cannot be
answered using either statement alone.
Choose (4) if the question cannot be answered even by using both the statements together.

Q.27 What is the ratio of radii of the circumcircle and the incircle of a polygon of 12 sides
(A) The polygon is a regular one of side 8 cm.
(B) One angle measures 150°

Q.28 A newspaper boy has 3 TOI, 2 ET and 3 HT in his bag (kept at random). He takes out 2
newspapers at random, without replacement. Does he retain at least one ET?
(A) He takes out 5 more newspapers at random of which there is at least one TOI, one ET and
one HT
(B) He takes out 5 more newspapers at random and there are 2 TOI and 2 HT among them
Q.29 Determine the value of the function f(x) = aloga x
(A) a is a positive real number such that twice of ‘a’ is 8 more than ‘a’ itself
(B) x is 12 less than half of itself

Q.30 If a sequence of numbers a1, a2, a3, …. is given by the rule an2 = an+1, does a multiple of 3 appear in
the sequence?
(A) a1 (B) an

Q.31 Which of the three bowlers in the series of test matches took most wickets?
(I) The first and the third bowlers took twice as many wickets as the second bowler
(II) The second and the third bowlers took three times as many wickets as the first bowler.

Q.32 What is the price of mangoes per kg?


I. Ten kg of mangoes and two dozens of oranges cost Rs. 252.
II. Two kg of mangoes could be bought in exchange for one dozen oranges.

Q.33 Is x + y - z + t even?
I. x + y + t is even. II. tz is odd.

Q.34 What is the area of the triangle?


I. Two sides are 41 cm each
II. The altitude to the third side is 9 cm long.
DATA INTERPRETATION ANSWER KEY
Ex Ex Ex Ex Ex
Q.No 1 Q.No 2 Q.No 3 Q.No Ex 4 Q.No 5 Q.No 6
1 2 1 3 1 2 1 4 1 4 1 1
2 4 2 4 2 4 2 1 2 4 2 3
3 3 3 2 3 3 3 2 3 4 3 2
4 3 4 1 4 4 4 2 4 4 4 2
5 3 5 2 5 1 5 3 5 1 5 2
6 2 6 4 6 4 6 4 6 2 6 1
7 4 7 3 7 1 7 3 7 2 7 3
8 4 8 4 8 4 8 3 8 2 8 4
9 2 9 2 9 2 9 3 9 2 9 1
10 3 10 3 10 1 10 3 10 3 10 1
11 3 11 3 11 1 11 2 11 4 11 3
12 4 12 2 12 4 12 3 12 1 12 2
13 1 13 3 13 2 13 2 13 2 13 2
14 4 14 3 14 3 14 4 14 1 14 3
15 2 15 1 15 1 15 4 15 2 15 3
16 2 16 3 16 3 16 3 16 4 16 4
17 2 17 4 17 3 17 4 17 2 17 1
18 3 18 4 18 2 18 3 18 3 18 1
19 3 19 4 19 1 19 2 19 4 19 4
20 1 20 3 20 1 20 3 20 1 20 3
21 3 21 3 21 3 21 4 21 4 21 3
22 1 22 3 22 4 22 2 22 1 22 4
23 2 23 2 23 3 23 4 23 3 23 3
24 2 24 4 24 3 24 2 24 3 24 3
25 1 25 2 25 2 25 1 25 2 25 3
26 2 26 1 26 4 26 3 26 2 26 2
27 4 27 1 27 3 27 2 27 3 27 1
28 1 28 1 28 3 28 4 28 3 28 4
29 3 29 3 29 1 29 3 29 2 29 2
30 2 30 2 30 4 30 3 30 4 30 1
31 1 31 1 31 3 31 2 31 2 31 3
32 2 32 2 32 4 32 4 32 4 32 3
33 1 33 2 33 4 33 2 33 1 33 3
34 2 34 2 34 2 34 1 34 3 34 3
35 4 35 1 35 3 35 1 35 4
36 2 36 3 36 4 36 3
37 4 37 3 37 2 37 1
38 4 38 4 38 2 38 3
39 4 39 3 39 2 39 4
40 2 40 4 40 4 40 3
41 3 41 4 41 1
42 1 42 4 42 4
43 3 43 2 43 4
44 2 44 2 44 3
45 1 45 3 45 4 Q.No Ex 3
46 1 46 4 46 3 51 1
47 4 47 2 47 2 52 2
48 1 48 4 48 1 53 3
49 4 49 1 49 2 54 1
50 1 50 4 50 3 55 4
EXPLANATIONS
EXERCISE 1
Direction for Qs. 1 to 2 : Refer to the following data for the following solutions.

9 3 1
A B (2) C
(6) (5) (7) D 8
(4) E 4 (8) F 7 (1) G 2
(3) H 5
(9) I 6
∴E represents 4 and D represents 7 or 8.
1. (2)
2. (4)

Directions for Qs. 3 to 7: Refer to the following data for the following solutions.
Let ‘x’ people have high school education.
∴ 3x have middle school education and 7x have primary school education.
Also as all and middle school educational people have primary school education and all high
school educated people have middle school education, number of people in Category I = 4x,
Category II = 2x, Category III = x.
∴Number of people Category I and II, who do not play any game = x and x/2 respectively.
∴Number of people playing only hockey = Number of people playing only football.
3x
−2
3x − 2 3x − 4
= and 2 = respectively
2 2 4
x−6
Also number of people in category III playing only hockey = = 5 = ⇒ x = 16.
2
3. (3) 3x = 48 people have middle school education.
x−6
4. (3) Number of high school education who do not play football = 5 + = 10.
2
5. (3) Number of people having middle school education but not high school education who
x−6
play only football = = 11.
2
6. (2) Number of such people = Number of people having primary school education
- No. of people having middle school education = 7x – 3x = 4x = 64.

7. (4) Number of educated people playing football only = 39.


Number of educated people playing hockey only = 39.
∴Number of an uneducated people playing football = 90 – (39 + 5) = 46 and number of
uneducated people playing hockey
= 60 – (39 + 5) = 16. Out of these 2 play both games.
∴ No. of uneducated people playing at least one game = 46 + 16 - 2 = 60.
No. of uneducated people = 200 – 7x = 200 – 112 = 88.
∴ 88 – 60 = 28 uneducated people do not play any game.

8. (4) Right turns: Actual letter – 2, 2 left turns : Actual letter + 2


A (inner) : 8; P (outer) : R ; P (inner): N; L (outer) : N; E (inner) : C
∴ APPLE would be coded as 8RNNC.

9. (2) Total alphabets + number on a wheel = 26 + 10 = 36 per wheel


Total combinations = (36 x 36) – 1 = 1295 (one is where all wheels are aligned)
10. (3) 4 combinations possible :
Inner L, Outer L, Inner R, Outer R, Inner L, Outer R and Inner R, Outer L.
Translate HOCJ9 back in each of the four above, and the only one which would reproduce an
English an English word (INDIA) is inner R, outer L.

11. (3) From the given options, only (3) i.e. BEF are logically related.

12. (4) Statements BAE are logically related. Hence this is the answer.

13. (1) By combining Statements B and D, we can deduce the statements E. Hence BDE are logically
related.
14. (4)

15. (2) It is given that, Blunders committed by Idiots (I) = 2 (Dumbos) ……(i)
Blunders committed by Fools (F) = 3 (Idiots) and ………. (ii)
Blunders committed by Morons (M) = 4 (Fools) ………… (iii)
As, Fools commit 1200 blunders, hence Moron commit 4800 blunders, Idiots, commit 400
blunders & Dumbos commit 200 blunders from (i) , (ii) & (iii).
Hence total numbers of blunders committed = 1200 + 4800 + 400 + 200 = 6600.
400
16. (2) The population in each group = = 100
4
Let us assume that if Dumbos commit the lest number of blunders i.e. 1, hence blunders
committed by Dumbos group = 1 x 100 = 100.
Note: Others may & may not commit blunders. Hence (2).
400
17. (2) As the population is same in each groups, hence Dumbo’s population = = 100.
4
As every Dumbo commits the same number of blunders as every other Dumbo, so total blunders
committed by Dumbos = 3 x 100 = 300.
Now using equations as shown in Q. 135,
Blunders committed by Idiots, Fools & Morons are respectively 600, 1800 and 7200.
∴Total number of blunders of the day = 300 + 600 + 1800 + 7200 = 9900.

Direction for Qs. 18 to 20: Refer to the following data for the following solutions.
Equation
Salary = n2 + 2 where n > 1 ………….. (1)
Expenses = 2n + 1 where n > 1 ……….. (2)
Saving = n2 – 2n + 1 where n > 1 ………….. (3)

18. (3) Putting the value of savings in equation (3), 100 = n2 – 2n + 1 ⇒ n2 – 2n – 99 = 0, we get n = 11.
Hence the savings would exceed Rs. 100 on 12th day.

19. (3) Putting the value of n in eq.n (1)


Salary = n2 + 2
When n = 6000
Salary = Rs. 36000002
When n = 6001
Salary = Rs. 36012003
∴Increase in salary = 36012003 – 36000002 = Rs. 12001. Thus he will have to remain in his job
for minimum 6001 days to break the record.

20. (1) Putting different values of n starting from 1 in eqn 3 and adding all the savings up to 11th day,
Total saving = Rs. 385, so on 11th day they will buy a first fixed deposit.
Direction for Qs. 21 to 25 : Refer to the following table for the following solutions.

MAYFLOWER LITTLEFLOWER
CUTTING 20 mins 30 mins
STITCHING 60 mins 60 mins
STITCHING BUTTON 15 mins 30 mins
AND BUTTON HOLES

The time given in the table is for per uniform to be stitched.

21. (3) No. of working hours for each person = 10 hour


∴Total time available for cutters in a day = 20 hours = 1200 mins.
Max. No. of uniform that 2 cutters can cut for Little Flower in a day = 1200/30 = 40
Time required for stitching 40 uniform = 40 x 60 = 2400 min and Alord has 5 tailors for stitching
that means the Total available time is (5 X 10 x 60) = 3000 mins.
Hence maximum number of Little Flower uniforms can be completed in a day = 40.

22. (1) To complete 20 Little Flower uniforms,


Time consumed by Cutters = 20 x 30 = 600 mins
Time consumed by Tailors = 20 x 60 = 1200 mins
Time consumed by Assistants = 20 x 30 = 600 mins
Time available for Cutters = (1200 – 600) = 600 mins
Time available for Tailors = (3000 – 1200) = 1800 mins
Time available for Assistant = (1200 – 600) = 600 mins
Max. No. of uniform that 2 cutters can cut for May Flowers uniform in a day = 600/20 = 30
Time required to stitch 30 uniforms = (30 x 60) = 1800 min and is exactly equal to the time
available for tailors. Hence No. of Mayflower uniform that can be completed on that day is 30.

23. (2) (1) To complete 30 Little Flower uniforms,


Time consumed by Cutters = (30 x 30) = 900 mins
Time consumed by tailors = (30 x 60) = 1800 mins
Time consumed by assistants = (30 x 30) = 900 mins
Time available for cutters = (1200 – 900) = 300 mins
Time available for tailors = (3000 – 1800) = 1200 mins
Time available for Assistant = (1200 – 900) = 300 mins
As they are not working in this available time it is the idle time.
Hence total man hours that are idle = (300 + 1200 + 300) = 1800 mins = 30 hours.

24. (2) Hiring one more assistance increases the time available for stitching buttons and button holes.
Max. No. of uniform those cutter can cut for Mayflower in a day = 1200/20 = 60
Time required by Tailors to stitch 60 uniforms = (60 x 60) = 3600 mins, whereas time available is
3000 mins only.
Hence maximum No. of uniform that can be stitched in a day = 3000/60 = 50
Time consumed by assistant to stitch buttons to 50 uniform = (50 x 50) = 750 mins
Time available for assistant = (3 x 10 x 60) = 1800 mins.
Hence 50 uniforms can be completed in a day.

25. (1) Max. number of uniforms that can be cut by those 2 cutters in a day for MayFlower is 60. And the
buttons and buttonholes can be easily stitched by the 2 assistant for these 60 uniforms in a day in
the time available for them. While 5 tailors can stitch only 50 uniforms in the available time.
Hence, A lord will hire 1 tailor to get maximum increase in production capacity in a day.
26. (2) From statement A alone
4n + 48 = 102
4n = 52
n= 13
A alone is sufficient.
From statement B alone:
Numbers whose squares lie between 150 and 250 are 13, 14 and 15
Among these A, only 13 is a prime number.
:. B alone is sufficient.

27. (4) Even upon using both the statements we can only conclude that two of the opposite sides are
parallel and sum of 2 angles is 180°.
In order to conclude that ABCD is a rectangle which is also a cyclic quadrilateral we also need to
know whether the opposite sides are equal in length.

28. (1) From A


(n - 3) (n - 2) = 0
n = 2 or 3
∴A alone is not sufficient
From B
Only for n = 2
n! 2!
= (n − 2)! i.e. = (2 − 2)!
2 2
1 = 0! = 1
B alone is sufficient.

29. (3) From A Form B


B B

x
y

A C C
A z
Given, sum of areas of semi-circles x, y and z is 100π
If BC = a cm, AC = b cm, AB = c cm.
Area of semi-circle x is π/2 a2/4 cm2
Area of semi circle y is π/2 c2/4 cm2
Area of semi circlezx is π/2 b2/4 cm2
½ π/4 (a2 + b2 +c2) =100π
a2 + b2 + c2 = 400
We know that in ∆ABC
a2 = b2 +c2 = ∴2 (b2 + c2) = 800
b2 + c2 = 400
b2 + c2 = a2 = 400 ⇒ a = 20
⇒ b or c = 10√3 and ⇒ the sides are 10, 10√3, 20
∴ the perimeter = 30 + 10√3 cm

30. (2) From A


profit % is 35% (i.e. 10 +25)⇒ profit = Rs.35
From B Mankup% = 80%
⇒ profit % = 1/5 x 80 = 16%
profit = Rs.16
∴ Either statement is sufficient

31.(1) From A alone.


Tram A takes 30 seconds to cross tram B if it were stationary. Either the trains were moving in the
same directions where the speed of train A is greater than the speed of the tram B or the trains are
moving in the opposite direction.
If they the trains were moving in the opposite direction, A would have taken less than 30
seconds to cross B.
⎛ 750 18 ⎞
Therefore the trains are moving in same direction and the speed of the train B = 90 − ⎜ × ⎟
⎝ 32 5 ⎠
B alone is not sufficient.

32.(2) From statement A


only 11 is the two digit prime which satisfies the condition.
i.e. 112= 121
113 = 1331
From statement B
only 9 is such number
when 92= 81 and 8 + 1 = 9
93 = 729 and 7 + 2 + 9 = 18

33. (1) From A


n
10 + 5
n can take negative values for which
3
is not an integer and for positive values of n, 10n + 5 will result in a number which has the sum of
its digits equal to 6 which is a multiple of 3,
n
10 + 5
∴ is an integer if n is positive
3
From A, n can be negative or positive
n
10 + 5
From B, n can be only positive and hence is an integer.
3
34. (2) From statement A alone,
A worked for 14 ½ days and B worked for 14 days, so they together will be able to finish the work
in less than 14½ .
So A alone is sufficient.
From statement B alone,
When the slower person starts the work, the work takes 29 days. So, when the faster person starts
the work, the work would be completed in less than 29days.
So, they together need less than 29/2 days to complete the work.
So B alone is sufficient.

35. (4) From statement A alone, two cases are possible


Case 1
The two trains are travelling in the same direction. In this case, the speed of train B must be more
than that of train A.
Case 2
The two trains are travelling in the opposite direction In this case, if speed of train B = 60 km/hr
time taken to cross each other
750
= = 22.58
5
(60 + 60) ×
18
∴Speed of train B must be greater than 60 km/hr i.e. greater than speed of train A.
Since both the cases are possible, nothing can be concluded.
even after using statement B, both cases remain.

36.(2) Number of Maruthi cars sold in 1 994 = 40% =1000,000 Number of Fiat cars sold in 1994 » 20%
= 500,000

⎛ 30 ⎞
37.(4) Fiat in1995 → 1125⎜ × 1500⎟
⎝ 40 ⎠
⎛ 15 ⎞
Fiat in 1996 → 818⎜ × 3000 ⎟
⎝ 55 ⎠
⎛ 35 ⎞
Fiat in 1997 → 1167⎜ × 1500⎟
⎝ 45 ⎠
Ambassador in 1997 → 334
Ambassador in 1998 → 423
On observation we can see that all the statements are true.

38.(4) Choice A is false


Number of cars sold by Fiat in 1994 (in thousands)
20 100
× × 1000
100 40
Number of cars sold by Fiat in 1 996 (in thousands)
5 100 100
× × 2000 =
100 70 7
∴% decrease is close to 71 % Choice (2) is false
Although percentage share is same for the two years,
the total production is different. Choice (3) is false.
The increase in total production is from 1500 to 3000, which is exactly 100%

39.(4) Ford Escort in 1995 → 750


Ford Escort in 1996 → 272.7
Ford Escort in 1997 → 333.3
Ford Escort in 1998 → 285.7

40.(2) Total number of cars sold in


1995 → 1500 x 100/40
1996 → 3000 x 100/55 → Maximum
1997 → 1500 x 100/45
1998 → 2000 x 100/70

Solutions for questions 41 to 45:


Originally money realised by sales = 10 x 50000 = 5,00,000
Money realised from (3)
= 5.00,000 - 0.5 x 0.4 x 5,00,000 = 4,00,000
Money realised from (2)
= 5,00.000 [0.1 x 0.5 + 0.2 x 0.5 + 0.1 x 0.4] - 2,00,000 + 5,00,000 = 3,95,000 Money realised
from (1)
= 0.5 x 50000 [1 + 0.5 x 0.3 + 0.5 x 0.2 ] = 3,12,500
41. (3) Hence the most profitable option would be to remain silent.
42. (1)
43. (3) Loss = 5,00,000-3,12,500= 1,87.500.
44. (2) As calculated above, it is Rs.3.95,000
45. (1) Net revenue from course 2 = 3.95 lakhs
Net revenue from course 3 = 4.00 lakhs
∴Rs.5000 less

For answer to questions 46 to 50:

Production Total cost= Fixed cost+ Variable cost


Plan 1 Plan 2 Plan 3
1000 2 3 4
2000 3 3.5 4.5
3000 4 4 5
4000 5 5 5.5
5000 5.5 6.5 6
6000 6.5 7.5 6

Now all the questions can be answered.

46. (1) 47. (4) 48. (1) 49. (4) 50. (1)
EXERCISE 2
Direction for Qs. 1 to 5: Refer to the following information the following solutions.
From clues I & III we can conclude that B is always present and one only one of A & C is always present.
Also note that when D goes than the only combination possible agents is D-E-G and from clues IV, in such
a case A should be included.
1. (3)
2. (4)
3. (2) From clues IV & VI we conclude that F – G – H are the recruitment agents that should be
included. Hence only one combination is possible.
4. (1)
5. (2) From previous solutions we know that F, G and B all can go with C,

Directions for Qs. 6 to10 : Refer to the following table for the following solutions.
The following table gives the moves that can be made for the mentioned conditions. The underlined
positions indicate your position after nth move.

Move Minimum Score Maximum Score Reaching 5 O’clock


th
0 1=1 1=1 1
st
1 7–4=3 12 – 2 = 10 12
nd
2 1–4=3 11 – 2 = 9 11 / 6
rd
3 2=2 - 5
th
4 - - -
Total 3 20 -

6. (4)
7. (3)
8. (4) By moving a step anticlockwise in the first move, you reach at 12 O’clock. From here you can
reach
(1) 10’O Clock through 12 – 11 - 10
(2) 5 ‘O clock through 12 – 6 – 5
(3) 7 ‘O Clock through 12 – 6 – 7.
But you cannot reach 6 ‘O Clock.

9. (2)
10. (3) Your mother’s husband ⇒ your father. Your father’s sister ⇒ your aunt. So, the lady’s aunt is the
man’s aunt ⇒ the man and the lady are brother and sister.

11.(3) M is the maternal uncle of R means m is the brother of R‘s mother (say K) i.e., M + K – R.

Direction for Qs. 12 to 13 : Refer to the following information for the following solutions.
We are given that A visited at 8 O’clock. Now from III we conclude that A visited at 8 p.m. Now from I we
concluded that B has to visit at 9 a.m. otherwise nobody will be able to visit in between A & B. Now if D
were to visit at 11 p.m. then condition IV will get violated hence we concluded that D visited at 11 a.m. and
C visited at 10 p.m. From here all the questions are answered.

12. (2)
13. (3)
14. (3) As X2Y3 = 8, either X or Y or both are fractional. From statement B, we get that X is rational.
8
Therefore, X2 is either and integer or a fraction. Therefore, X2 = 3
will either be an integer or
γ
a fraction, So either X or Y is a fraction.

15. (1) Statement A alone is sufficient to say that B is not prime. However, statement B alone is not
sufficient.

16. (3) From statement A we can conclude that 3x is even. As y is an integer (by virtue of it being odd)
from statement B, we can conclude that 10y is even. The sum of two even numbers will be even.

17. (4) From statement A we cannot conclusively state which of the three is largest as they could all be
positive or negative. From statement B we know that xyz is positive as xyz – y2 (as y2 will always be
positive for real values of y) is positive. This is possible in two cases, I. All of x, y and z are positive
in this case z is the greatest II. Y is negative and one of x and z is negative. Hence we cannot
conclude even if we combine the two statements.

18. (4) From both the statements we know the ratio of the speeds and the times at which they met.
Without knowing the distance between the two cities X and Y, we will not be able to find out the
speed of L or M.

19. (4) Those S which are M are not P. Hence some S are not P.

20. (3)
21. (3) Those ‘M’ are not ‘P’ are ‘S’ because all ‘M’ are ‘S’

22. (3) Important link here is ‘could’.

23. (2)
24. (4) Because the minority might consist of thousand of people, the opposition might not be inconsistent
with the speaker’s remarks.

25. (2)
26. (1) Since the number of persons per household can be assumed to be a constant, the ratio of x1 to x2 is
the same as the ratio of the number of property crimes per1000 households in 1975 to that in 2000.
560
This value is = 3 :1 .
180

27. (1) Let the number of households in thousands be T1 in 2000 and T2 in 2050. The total number of
property crimes in 2000 is, therefore, T1 × 180. This will reduce to T1 × 180 × 0.71 × 0.71. The
number of property crimes per thousands households in 2050 is, therefore, T1 × 180 × 0.71 ×
0.71/T2/ Since the number of persons per households remains constant, the ratio of T1/ T2 is the
same as the ratio of the populations. The ratio of the populations is (157+ 2.3 × 50)/ (157 +2.3 ×
100). The required answer comes out to be approximately 63.

28. (1) Let x be the number of aggravated assaults per thousand population. The number of robberies is x
– 1.8 and the number of simple assaults is 3x. From the graph, we get the sum of all the three as
27. Hence x – 1.8 + x+{ 3x = 27, giving x = 5.76 and x – 1.8 = 3.96. The total number of robberies
is, therefore, (3.96/1000) × (1.57 + 2.3 × 50) ×106 = 1.1 × 106.

29. (3)

30. (2) The number of property crimes in 2000 was 180 × H where H is the number of households in the
thousands in 2000.
The number of violent crimes in 1975 was 49 × P1 where P1 is the population in 1975 in
H P1 3.67
thousands. Therefore, 180 × = 1.45 which gives =
49 × P1 H 1.45
P2
The quantity required is where P2 is the population in 2000.
H
157 + 2.3 × 50 272 272 3.67
But P2/ P1= = . Therefore P2/ H= × ≈ 3.2 .
157 + 2.3 × 25 214.5 214.5 1.45

31. (1) Week 1, for detailed solution refer the table below.
32. (2) 50, for detailed solution refer the table below.
33. (2) 200 units’ week 3, for detailed solution refer the table below.
The gross requirement of leg is 200 and 600 in week 3 out of 300 in – hand units of legs, 200 units
would be used for wek2 requirement and the rest units would be used for meeting the requirements
of week 3. Therefore net requirement of week would be 600 – 100 = 500 units of legs for meeting
the demand of finished table of week 5.

34. (2) For meeting the additional demand of 200 tables


Shortrails = 4 ×200 = 800
Legs = 8 × 200 = 1600
Total shortrails = 300 + 800 = 1100
Total legs = 600 + 1600= 2200.

35. (1) Details of components available on a particular week.


Figure in bracket represent the week to which components belong.

Details Week1 Week 2 Week 3 Week 4 Week 5 Week 6


Table 50 50+50 100 200 150
Leg assembly 100 100 – 50 = 50(4) 100(4) 150(6) 100(6)
Legs 150 150 + 50=200(6) 600(5) 400(6)
Shortrails 50 50 + 50 = 100(4) 300(5) 200(6)
Longrails 0 100 (4) 300(5) 200(6)
Top 50 50 – 50 = 0 100(4) 150(5) 100(6)

Details of component ordered on a particular week.

Details Week1 Week 2 Week 3 Week 4 Week 5 Week 6


Table
Leg assembly
Legs 50(4) 600(5) 400(6)
Shortrails 50(4) 300(5) 200(6)
Longrails 100(4) 300(5) 200(6)
Top 100(4) 150(5) 100(4)

Week 1
As lead – time increased by 1 week so planned order release would be by one week ahead.

a b b c
36. (3) I. < <
3 4 2 3
⇒ 4a < 3b ⇒ 3b < 2c
⇒a<b ⇒ b<c
(as both a & b are positive) ( ∴b & c are positive)
But, this is not sufficient as nothing is known about d till now
a d c d
II. < <
4 6 4 3
⇒ 6a < 4d ⇒ 3c < 4d
⇒a<d
This means a < d, but nothing can be concluded about the value of d compared to b or c.
When we combine I & II,
We get 3b < 2c or 9b < 6c & 3c < 4d or 6c < 8d
Which can be represented as 9b< 6c < 8d ⇒ 9b < 8d ⇒ b < d
We know that b < c & a < b. this shows that a is smallest and b the second smallest number, even
thought which is greater between c & d cannot be decided.

37. (3) LCM & HCF of A & B are 72& 12 respectively.


∴ LCM × HCF = A × B ⇒ 72 × 12 = A × B
Values (A, B) can take so that HCF will be 12 & LCM 72 are (12, 72) and
(24, 36)
I. A is not a factor of B, which means A and B can take either 123 and 72 or24 and 36 as
together values but not necessarily in the same order i.e.,
A = 72 and B = 12 OR
A = 36 and B = 24 OR
A = 24 and B = 36
II. B is greater than A. This means B can be either 72or 36.
Hence II alone is not sufficient. Combining, I & II, we get A = 24 & B = 36. Hence, the
answer is (3).
38. (4) I. Let R & S be the ages of Ram & Shyam respectively.
Age of Ram 5 years back = R – 5 & Shyam = S - 5
∴(R - 5) + (S – 5) = 60 or R + S = 70 …(1)
Hence, I alone is not sufficient.
II. Ages of Ram & Shyam fifteen years hence will be R + 15 & S + 15
∴(R + 15) + ( S + 15) = 100
∴ R + S = 70 …..(2)
Hence, II alone is not sufficient .
Combining I & II,
We are getting the same equation in both cases, hence we cannot we conclude what are
the ages of Ram & shyam respectively.

39. (3) Let r & R be the radius of sphere & cone respectively.
Let h be the height of the cone.
4 3
Volume of sphere = πr
3
1 2
Volume of cone = πR h
3
I. Radius of cone, R = 2r
4 3 1 2
∴Volume of sphere = πr & volume of cone = π × 4r h
3 3
4 3
πr
3 r
Ratio of volume of sphere to that of cone = = . Hence, I is not sufficient.
1 2 h
π × 4r h
3
II. Height of the cone, h = r.
But, this alone cannot give us the ratio. Hence, II is also not sufficient.
Combining I & II
r
Ratio = = 1 (∴r = h)
h

40. (4) I. D is the tallest among C, D & E. But, we don't know anything about
A & B. Hence, I is not sufficient.
II. B is not shorter than D, means B is either taller than or equal to D, But, this is not
sufficient, as nothing is known about C & E. Combining I & II,
We get that D is the tallest of C, D & E. And B not shorter of A & B. Between B & D it is opt
known whether they are equal or B is taller, which means either B is the tallest in the group or A;
B & D both are the tallest in the group hence we cannot decide.

41. (4) It y is dropped, XZ must be selected. Since Z is there, N cannot be taken hence M and P must be
taken.

42. (4) It L is taken, we cannot take Y hence XZ must be taken and also P.

43. (2) If Y and Z are taken, L and N cannot be taken hence M and P must be taken

44. (2) Visually, we see US has reported more than 21,000 cases.

45. (3) Bahamas, Belgium, Denmark

46. (4) West Germany = 222, France 555 Reqd ratio = 222 : 555 = 2 : 5

47. (2) Bahamas, Burundi, French Guyana and US.

48. (4) 4% of x = 286,000 hence x = 715,000.

49. (1) Only I and II are true.

50. (4) Ethiopia, Israel and Yugoslavia.


EXERCISE 3
1. (2) Using I above since ABC is a right angled triangle and the largest angle
= 90⇒ I alone is sufficient.
Using II above since 32 + 42 = 52 ABC is a right angled triangle and the largest angle = 90". => II
alone is sufficient.

2. (4) Since the actual number of institutes in Hyderabad is not mentioned, even both the statements are
insufficient.

3. (3) From I 8- (a-b)3>0


⇒ 8 > (a - b)3
⇒ (a-b)3 < 8 ⇒ a-b < 2 - (1)
Nothing can be said about this.
From II 4- (a-b)2 < 0
⇒ 4 < (a - b)2
⇒ (a-b)2 > 4
⇒ | (a - b)| > 2
⇒ a - b > 2 or ⇒ a-b<-2- (2)
⇒ From equation (1) and (2) we can see that a-b < -2 ⇒ a < b

4. (4) Since the weights of the balls mentioned are not given, ft is not possible to determine the heavier
box.

5. (1) B alone is sufficient because the statement implies b = 0


∴ a/b is not defined.

6. (4) Since the angle given is 80' in (I) and the time between 3' clock and 4" clock in (II), we can see
that there can be two values, hence no unique answer possible.

7. (1) From (I) the dotted line


is the ladder at say 8:30 A.M. h 9m
since we want the height h
of the ladder (i.e the point
from where ladder touches the wall to the point the ladder touches the floor) and since the length of
the ladder is given in statement (I). Answer can be got from (I) alone. Statement (II) will not give the
length of the ladder.

8. (4) From statement I, we cannot find the speed at x.


Similarly from statement II also, nothing is given about point x. But from I and II, two
possibilities arise, i.e. one case
5cm
X0 X
if the particle is moving from x0 to x then the particles speed at x would be 7.5 cm/s

second case 5 cm/sec


5cm
X X0
If the particle is going from x to x0, the speed of the particle at x would be 2.5 cm/sec, Since no
unique answer can be found.

9. (2) As per statement alone if q = 100 then p = 150


∴ p/q = 150/100 = 3/2 As per Statement II alone if r = 100
then p =187.5 and q= 125
so, p/q = 187.5/125
∴the question can be answered statement I alone or statement II alone.

10. (1) As per statement II alone, a sum of Rs.8 can be arrived at by adding 2 and 3 in the form 2 + 3x2 =
8 i.e. one pencil and two pens

11. (1) Three tablets/day would mean the hypertensive has to be taken 90 times in 30 days
The increase 2.50……> 10 tablets
hence for 2.50 x 9 = 22.50 for 90 tablets Asthmatic has to consume…> 30 tablets 6.00x3= 18.00
∴The total increase = 22.50 +18 = 40.50

1.00 2 2
12. (4) × 100 = × 100 = 66 %
1.50 3 3

13. (2) He has to consume = 2 x 3 x 7 x 20 = 840


spoons 90 spoons …….. 1 bottle
840 spoons …… 10 bottles
1
since he cannot buy 9 bottles;
3
hence expenditure according to the existing prices
= 10x18.00 = Rs. 180

14. (3) Since he may change the order in which he takes the tablets it is not possible to determine

15. (1) To arrive at the correct value of the total sales of Fuels the inter company sales figures should be
subtracted from the present total sales. To be subtracted from 100%
PRL ⇒ 50 + 10 + 40 = 100% i.e. has sold all its sales to other companies only ⇒ correct sales =
0%
OICL⇒ 15 + 20 + 10 = 45
⇒ (100 - 45)% of 30% = 16.5%
PHCL ⇒ 20 + 20 + 20 = 60
⇒ (100 - 60)% of 25% = 10%
PBCL ⇒ 30 + 10 + 35 = 75
⇒ (100 - 75)% of 20% = 5%
OTHERS ⇒ 10 + 5 + 10 = 25
⇒ (100 - 25)% of 15% = 11.25%
∴ Actual sales:
= (0 + 16.5 + 10 + 5 + 11.25) = 42.75% of given sales
∴% by which total sales of fuels were over estimated
(100 − 42.75)
× 100 ≅ 135%
42.5

16. (3) The correct sales figures


= (100 - % sales to other companies) x % share of total sales given = (100-M) x p (say)
∴M should be minimum and p maximum. By observation this is true for OICL in case of Fuels
and for PBCL for lubes.
(Note that OTHERS and PRL are eliminated) Calculations between PBCL and OICL
OICL:
= (100 - 45) x 30% x 25 + (100 - 60) x 15% x S
(where S = sales of lubes and since sales of fuels
= 25)
≅ .45
For PBCL
= (100 - 75) x 20% x 25 + (100 - 45) % x 30% x S
≅ 0.275 S
∴Clearly it is maximum for OICL

17. (3) We need to find that company for which Total sales are less than total purchases = S-P is
minimum (and -ve) i.e. S is minimum and P is maximum S is from the pie chart and P is the sum
of purchases obtained from the columns in the table. By observation alone
OTHERS have purchased far more than any of the rest.
And its sales are also minimal for both lubes and fuels.

18. (2) For maximum Profitability. Sales - Purchases should be maximum.


∴ S - P must be maximum
For ICL, the total Purchases are 17.5% and
∴ Profitability = 30 - 17.5 = 12.5% [i.e. 50% of PRL + 20% of PHCL + 30% of PBCL + 10% of
OTHERS]
Similarly, for PHCL Profitability = 25 - 8.25 = 16.75 for PBCL Profitability = 20 - 12.5 = 8/5%
and for others, there is Negative Profitability as we have seen in 2127.

19. (1) The correct sates figures for Fuels have already been calculated in solution (1) and in similar
manner we calculate those of lubes
OTHERS ⇒ [100 - (15+5+15)] x 25% = 16.25%
PHCL ⇒ [100 - (10+25+40)] x 20% = 5%
OICL ⇒ [100-(20+40)] x 15% = 6%
PBCL ⇒ [100 - (15+5+25)] x 30% = 16.5%
Now total sales of lubes =% x that of fuels ⇒ effect values for above are OTHERS ⇒ 8.125
PHCL ⇒ 2.5% (of total sales of fuels)
OICL ⇒ 3%
PBCL ⇒ 8.25%
The total correct sales value of fuels and lubes put together maximum is for OTHERS ⇒ 11.25 +
8.125 =19.375
PBCL⇒ 5+ 8.25 =13.25%
OICL ⇒ 16.5 + 3 = 19.5%
PHCL⇒ 10 + 2.5 = 12.5%
∴OTHERS is second in terms of (correct) total value of sales of fuels and lubes put together.

20. (1) Let the amount of paddy to be processed = q kg. the amount of rice it yields = q x 0.95 = 0.95 q
out of this if you exclude 6% broken
⎛ 6 ⎞
0.95q⎜ 1 − ⎟ = 0.893q
⎝ 100 ⎠
∴So the rice you get = 0.893q ∴0.893q x 8 = 14.288
0.893q = 1786 q = 2000 kg.

21. (3) The rice produced by processing 10.000 kg of paddy = 9,500 kg.
if the brokens increase by 2% of 9,500 ⇒ 190 kg
of additional brokens 190 kg of fine nee less ⇒190 x 8 = 1520 but this is offset by 190 x 2 = 380
profit over sale of additional brokens 1, 520 – 380 = Rs.1, 140

22. (4) 5% loss of weight would imply = 0.05 x 10,000 = 500kg.


95
the rice extracted will be = 9,500 x = 9025
100
the decrease will be = 9, 500 - 9025 = 475 kg.
23. (3) Proceedings from sale of husk =
1
5000 x 0.05 x = Rs.125
2
proceedings from the sale of brokens =
6
5000 x 0.95 x x 2 = 570
100
hence amount realised = Rs.570 + Rs 125 = Rs.695

Solutions for questions 24 to 27:


‘It-is given that the cages are as follows.
Antelope Gorilla
Elephant Gorilla Giraffe Antelope Deer
1 2 3 4 5
Chimpanzee Horse Zebra Bear Tiger Lion
6 7 8 9 10 11
Wolf
12
From statement (1) we get elephant is in 1st cage, wolf is in
12th cage and chimpanzee is in 6th cage.
From statement (3) we get lion is in 11th cage, deer is in 5th cage and tiger is in 10th cage.
From statement (4) we get horse is in 7th cage
From statement (2) we get bear is in 9th cage, and zebra is
in 8th cage and giraffe is in 3rd cage.
Either Antelope or Gorilla is in the 2nd cage or 4th cage.

24. (3) If tiger is not opposite to Gorilla, then the order of the animals are as follows.
Elephant Gorilla Giraffe Antelope Deer
Chimpanzee Horse Zebra Bear Tiger
Lion Wolf
Then Gorilla is adjacent to Elephant.

25. (2) The animals can be arranged in two ways.

26. (4) Either Gorilla or Antelope is in between Giraffe and deer.

27. (3) If elephant is shifted to the 12tH cage and deer is adjacent to gorilla, then the order in which the
animals are arranged in the cages is
Wolf Lion Tiger Bear Zebra Horse
Chimpanzee Deer Gorilla Giraffe Elephant
Gorilla is opposite to tiger.

28. (3) 1 2 3 4 5
C E A D B

C, E, A are in that order as C's number is less than E's as E's is less than A's. Hence C, D and A
can only take box numbers 1, 2 and 3. Therefore B has to be in box number 5.

29. (1) (38 - 30)/30 = 26%


30. (4) Visually, we see D is more in 1995 while total is more or less constant
31. (3)
32. (4) 55 - 34 = 21lakh.
33. (4) 28/176 = 16%.
Solutions for questions 34 to 38:

It is given that the number of games played by each player is different, i.e. one player plays one game,
another plays two games and so on. With the given information we can conclude that

Chess Swimming Weightlifting Cycling Tennis


Pranay × √ √
Qureshi √ √ √
Rasool × √ √
Surendar × √
Tarif × × √ √

From (6) Tarif does not plays weightlifting because cycling is the only common of Rasool and Tarif.
3 players participates in swimming and the most common game is weightlifting. Hence 4 players
participates in weightlifting,
From (6), Rasool does not play Tennis
All the players play at least two games except Surender. From Statement (1) we get Surender plays only
one game and
Qureshi plays all the five games and Pranay do not play cycling,
From (3) Tarif do not play swimming.
Hence, the final arrangement is as follows.

Chess Swimming Weightlifting Cycling Tennis


Pranay × √ √ × √
Qureshi √ √ √ √ √
Rasool √ √ √ √ ×
Surendar × × √ × ×
Tarif × × × √ √

34. (2) Qureshi plays all the five games.

35. (3) The common game among Pranay, Qureshi and Rasool are swimming and weight lifting. Hence
(1) is not true.
The common game among Qureshi and Tarif is cycling and tennis. Hence (2) is not true. All the
four games except Tennis are played by Qureshi and Rasool is true.

36. (4) Four players participate in weight lifting.

37. (2) When the first 10, 20, (and so on) players are sent back in that order the row is gradually "and
completely reversed after every 80 players who are sent back. Total number of players sent back = 10 +
20 +.... 80 = 360
i.e. (80 x 4) * 40 ^ effectively only the first 40 players have been sent to the end of the row and hence
the player numbered 5 will be in the 40 + 5 = 45th position.

38. (2) It is given that A wears Black shirt means A cannot wear black trouser, blue trouser, white trouser.
Hence he wears either Red or Green trouser
Similarly, D wears Green shirt means he cannot wear green trouser, black trouser, white trouser.
Hence he wears either Red or Black trouser. It is also given that D does not wear Black trouser.
Hence D weans Red trouser and A wears Green trouser. It was also given that neither B nor E
wears the Blue trouser. Hence, C wears the Blue trouser.
Solutions for questions39 to 42:
39. (2) The sector recording a decrease of 8 percentage points is medicine the sector recording a 5
percentage of points increase is education the difference in amounts allocated in 1992 will be
15 10
2,50,000× − 2,50,000× = 12,500
100 100

40. (4) The year in 4 sectors = Rural Development is 1992; the difference between maximum and
minimum allocations is
⎛ 15 10 ⎞
2,50,000⎜ − ⎟
⎝ 100 100 ⎠
5
= = 2,50,000× = Rs.12,500
100

41. (1) In the year 1992 medicine + agriculture is 15% + 15% = 30%

42. (4) The maximum percentage increase is for the rural development
∴the difference in outlay for two years 1, 25, 000 - 80, 000 = Rs.45, 000

43. (4) The states with a growth greater than 10% are Sikkim, Tripura, Assam, West Bengal.
Total production = 0.6 + 18 + 45 + 72 + 84 + 102 = 321.6
Production of states which show more than 10% increase
= 321 .6 - (0.6 + 45) = 276
276/321 = 85.8%

44. (3) The maximum % increase in 1994-95 is for the state of Tripura which is 33.33%. Where as the
least percentage decrease is zero in case of Manipur
∴ x2/y2 is undefinable.

45. (4) The average production for 1993-94


0.6 + 16 + 42 + 54 + 76 + 90
= 46
6
and the state which is closest to that is Nagaland which is 42 (closest to average production) The
average production of all the states for year 1995-94 =
90
76
54
42
16
0.6
278.6
278
= 46.4 = average production
6
The state is Nagaland with 42 (closest to average production)
∴The production of Assam + West Bengal = 166
166 − 42
× 100 = 300%
42

46. (3) The state which records 33% increase is Tripura has 19% of the lot at production for the year
1993-94.
47. (2) Total trees = 21.600
Cherries are 37.5% of total = 3/8 x 21600 - 8100
Fruit yielding cherry tree are 61% of 8100
= 3/5x8100+ 1% of 8100
= 4860 + 81 = 4941

48. (1) Apricot fruit yielding trees = 12.5% of 87% of 21600. = 10.87% of 21600
Peach fruit yielding trees = 12% of 78% of 21600 = 9.36% of 21600
∴10.87% of 21600- 9.36% of 21600 = 1.51% of 21600 = 327

49. (2) Avercado = 11%x93%of2l600


= 10.23% of 21600
Apricot = 12/5% of 87% of 21600
= 10.87% of 21600.
Peach = 12% of 78% of 21600 = 9.36% of 21600
Plum = 13% of 77% of 21600 = 10.01% of 21600
By observation Apricot has the greatest number of fruit yielding trees.

50. (3) The percentage of fruit yielding plum trees as a percentage of fruit yielding peach trees is
10.01
× 100 = 107%
9.36

51. (1) Only S3, S6, S7 and S8 have qualified and among these it is dear (by addition) that 106.5 marks is
the highest for S3.

52. (2) The four people who have qualified are 1


S3 (106.5); S6(105)
S7 (103) and S8 (96)

53. (3) Adding the total number of mistakes.


S1 = 30 mistakes
S2 = 27 mistakes
S3 = 46 mistakes
S7 = 30 mistakes

54. (1) (a) Maximum score in RC is 37.5 and was secured by S2. yet he did not
qualify. Hence statement I is true.
(b) 82 is the person with the maximum grand total and he did get the maximum in VA and
RC sections arid hence statement II is false.
(c) S6 has made maximum attempts. Not S8. Hence statement III is false.
Hence only statement I is true.

55. (4) Maximum number of questions answered correctly by


S8 is 110
S7 is111
S5 is 59
S6 is 115
EXERCISE 4
1. (4) From Table 2, we see that in 1980, the total number of consumers ere 32571 ×103 and from Table
1, the average energy consumption per 1000consumerswas2.5 GWH. Hence the total energy
2.5
consumption would be 32.6 × 106 × GWH ≈ 0.82 × 1011 KWH. Hence (4)
1000
2. (1) The total electricity consumed in 1994 (obtained from the number of consumers and the average
energy consumption per consumer) is 0.86 × 105 × 3.5 GWH. This is also equal to the area
average consumption per sq. km.
3.4
Hence the area is equal to 0.86× 105 = 3.3 × 106 sq. kms. Hence (1)
⎛ 88 ⎞
⎜ ⎟
⎝ 1000 ⎠
3. (2) Let C1 and C2 be the number of consumers in 1970 and 1980 respectively, let P1 and P2 be the
⎛ N1 ⎞ ⎛ N1 ⎞
⎜ ⎟ ⎜ ⎟
populations in 1970 and 1980 respectively. Then
⎝ C1 ⎠ 3.3
= and
⎝ P1 ⎠ = 0.09 . Since f1
⎛ N2 ⎞ 2.5 ⎛ N2 ⎞ 0.134
⎜ ⎟ ⎜ ⎟
⎝ C2 ⎠ ⎝ P2 ⎠
C1 C2
= and f2 = , the required
P1 P2
⎛ 3.3 ⎞
⎜ ⎟
ratio
f2
=
⎝ 2.5 ⎠ ≈ 1.97 . Hence (2)
f1 ⎛ 0.09 ⎞
⎜ ⎟
⎝ 0.134 ⎠
4. (2) We use Table 2 to obtain the ratios of the number (1990 to 1980) of consumers for Domestic,
Commercial, Industrial, agricultural and Total. The values are 2.26, 1.75, 1.81, 2.04, and 2.14
respectively. Hence (2)
5. (3) The number of Domestic consumers in 1970 and were 10165 and 63406 thousands respectively
giving a percentage increase of approximately 524%. Hence (3)
6. (4) Both the statements individually or combined are not sufficient 62 = 36 divisible by 2 & 3 or 52
=25 not divisible by two consecutive numbers. Hence (4)
1
7. (3) Since the probability of drawing a red marble is and the number of marbles < 40, ∴the number
24
of marbles in the bag is 24, of which one is red.
1
From I The probability of drawing a blue marble is ,
3
∴the number of blue marbles is 8
∴number of black marbles 24 – (1 + 8) = 15
15 5
∴The probability of drawing a black marbles is = .
24 8
∴Both statements are necessary.

Alternatively,
⎛ 1 1⎞
Using both the statements, the probability of drawing a black marbles 1 - ⎜ + ⎟
⎝ 24 3 ⎠
(since there are only red, blue and black marbles). Hence (3)
8. (3) Possible values of x can be 1, 2, 3, 6, 17, 34, 51, 102
Form I : x + 17 is divisible by 3
∴x + 2 is also divisible by 3 which is possible when x = 1or 34
From II : x + 2 is divisible by 4 which is possible when x = 2, 6, 34
∴Using both statements, we have only one common value of x, i.e., 34. Hence (3)
9. (3) From I : Y can either be Rs.60 or Rs.120
From Y = 60, X = 100
When Y = 120, X = 40
From II, x > 50, therefore the 2nd case does not apply.
∴x =100; y =60
∴Both statements are required. Hence (3)
10. (3) From I : ∠B > ∠A
From II : We know B + C = 1100
∴(B + C)2 =12,100
B2 + 2BC + C2 = 12,100
∴BC= 2400
2400
∴B + = 110 0
B
B – 110B + 2400 = 0
B = 80 OR B = 30
B>A
∴∠B = 1800 – (70 + 80)0
∠C = 300
∴Both statements are necessary. Hence (3)
11. (2) Let the effectiveness of A, B, C and D be 5x, 6x, 4x and 3x respectively then, total effectiveness
of Ajit’s drug ⇒ 12 × 5x + 15 × 6x + 20× 4x + 28× 3x = 314x
that of Bittu’s and drug ⇒ 37 × 5x + 15 × 6x + 13× 4x + 15× 3x = 372x
that of Chinku’s drug ⇒ 26 × 5x + 15 × 6x + 10× 4x + 34× 3x = 362x

Alternatively,

The most effective drug B is in equal proportion in all drugs. Now the second most effective is A,
Ajit‘s pharma can be easily eliminated as it has very less proportion of A compared to Bittu’s and
Chinku’s drug.
Again, Ajit pharma: 37× 5+ 13 × 4+ 15 × 3 = 82
Bittu’s pharma : 26 × 5 + 10 × 4 + 34 × 3 = 272 Hence (2)
C
12. (3) Side effect
B+ E
20 1
For Ajit’s drug, it I, =
15 + 25 2
13
For Bittu’s drug, it is, = 0.37
15 + 20
10 1
Fro Chinku’s drug, it is, . = Hence (3)
15 + 15 3
13. (2) Let the new composition of E be 25 – x and that of be 20 + x.
20 + x
then, ≤1
15 + 25 − x
20 + x ≤ 20
x ≤ 10
∴Maximum composition of C is 20 + 10 = 30%. Hence (2)
14. (4) We cannot answer this, unless the production of individual companies are given. Hence (4)
15. (4)
16. (3) The discount in Kolkata ticket is Rs.3535
and the discount in Delhi ticket is Rs. 2175
So with Rs.3535 ticket for 17 destination can be bought (any one of them except Delhi0 and with
Rs.2175 tickets for 4 destinations i.e. Ahemedabad, Pune, Indore and Aurangabad can be bought
(any one of them 0. So answer is 17 & 4. Hence (3)
17. (4) Maximum discount in numerical terms is in Kolkata ticket.
It is 6855 – 3320 = Rs. 3535
Lowest discount same way for Aurangabad is 647 Rs.
So Rs. 3535 is almost 546% of Rs. 647. Hence, (4)
18. (3) less than 50%
19. (2)
Case I Case II
To From To From
Delhi 6095 3920 3920 3920
Bhopal 4125 2371 2371 2371
Ahemedabad 3255 2017 2017 2017
indore 3360 1965 1965 1965
16835 10273 10273 10273
So Expenditure in I case =27108
So Expenditure in II case = 20546
The solving in the second case = 6562it is approx. 24%. Hence (2)
20. (3) From the options, it is obvious that only the top 4% numbers must be evaluated.
The average size of the 3rd and 4th largest firms must be less than or equal to the average size of
the first two firms.
Based on an inspection, the jump in textiles appears to be very large. The average fro the 3rd and
(18 − 8) 8
4th firms is = 5 . However, the first two firms’ average is only = 4 . Hence, it is
2 2
incorrect and needs to be corrected by subtracting 4%. Hence (3)
Only option (3) shows the correction by 4%.
21. (4) The solution lies in estimating the size of the largest firms as a proportion of the total industry.
(75 − 60)
Since the 3rd largest firms ≥ average of the 3rd and 4th firms, The 3rd largest firms ≥ or
2
7.5%
Hence, the 2nd largest firms ≥ 7.5% or the largest firms ⇐ 60 – 7.5 = 52.5%
Also, largest firms ≥ average of the top 2 firms
Hence, the largest firms ≥ 60/2 or 30%.
5000 5000
Hence, the size of the steel industry is between (about 9500) and (16667) . Hence, (4)
0.525 0.3
is not possible Hence (4)
22. (2) It is obvious that steel, oil & gas shipping and Elevators have less than 20 firms.
Now, the average size of the top 10 firms ≥ the average size of all the remaining firms. Consider
10
the leather industry. The average size of the top 10 firms = = 1% . Hence, each of the
10
90
remaining firms cannot be larger than 1% and there must be at least = 90 such firms. In all,
1
then there must be at least 100 firms.
Similarity, Textiles has at least 17 firms. Hence, leather, textiles and chemicals definitely have at
least 20 firms. Hence (2)
23. (4) Option (1) is a red herring and is false. Applying these rules, it can be seen that a maximum of 3
industries can face restrictions (Steel, Oil & Gas and Elevator). We do not know the number of
industries where the largest firms is government owned. Hence, options (2) and (3) cannot be
evaluated. Hence (4)
24. (2) Consider the current year. Maximum possible size of the 4th largest firms = average of 3rd and 4th
(75 − 60)
largest firms= = 7.55 of the industry. Assuming the remaining firms (the firms other
2
100 − 75)
than the top 4 firms) are the same size (7.5%), there would have been or 3.33 or 4 firms
7.5
other than the top 4. Hence, a total of at least 8 firms in this year. There were at least 2 mergers
last year. Hence, last year’s minimum = 8 + 2 =10. Hence (2)
25-28.
For this caselet consider possibilities one by one. E.g. first suppose Ria gets soup then Janet gets
hot coffee (statement 1), Venna gets gums (statement 5) and Ria gets hot coffee (statement 9)
which contradicts our supposition; hence, rejected.
Next suppose Gia gets soup then Ria gets gums (statement 11). Then if Janet get hot coffee, Veena
gets gums (statement 5), which is not possible. So, Janet gets tea and remaining Veena gets hot
coffee.

25. (1) 26. (3) 27. (2) 28. (4) 29. (3) 30. (3) 31. (2) 32. (4)
33. (2) Two other teams each must have at least 3 members.
34. (1) This team contains members from two of the original teams only.
35. (1) Since neither of two new teams has any member from team A3, both members of A3 will have to be
accommodated in the third team which, however, is not allowed.

36. (3) 37. (1)


38-39 The second resident always speaks truth (so, not a govt. official) First speaker may speak truth (IS
not & denies being a govt. official) or may tell a lie (is a govt. official but denies being one) – in
either case denying being a govt. official. If first resident speaks truth the third one tells a lie and
vice-versa.

38. (3) 39. (4)


40. (3) History of Modern Europe (HME) + American History (AH) = 90 + 90 = 180, but there are only
120 students. Thus, at least 60 students selected both of the above subjects. HME and AM +
Ancient Indian History (AIH) = 60 + 105 = 165. Thus, again, as there are only 120 students, at
least 45 would have taken all three of above. Using the same logic, (45 + 105) – 120 = 30 students
at least would select all the four subjects.
EXERCISE 5

1. (4) The data is insufficient as the manufacturing cost of the liquor is not mentioned anywhere for any
of the states.
2. (4) The data is insufficient, as the excise duty per litre for the year 1986 is not mentioned.
3. (4) The data is insufficient, as the amount of liquor supplied to four states is not mentioned. The
amount of liquor supplied in Tamil Nadu is only mentioned and moreover for the first bar graph
the year is not mentioned.
4. (4) The data is insufficient as the sales for the period is not mentioned in the question. Therefore, the
answer is 4.
5. (1) The average value of the contract secured during the years = (100.5 + 67 + 141 + 143.9 + 65)/5 =
103.48
6. (2) Considering 1985 as the performance base of 100% i.e. 67 = 100% the figure for the indices for
different years are shown in the table below:
Index is calculated as 67 = 100 (Base year 1985)
Index for 1984 is therefore = 100/67 × 100.5 = 150 (contract secured is 100.5)
7. (2) There is decline in contract secured only in the year 85 and 88 compared to the previous year. The
highest % decline in the value of contract secured is for year 1988 = {(143.9 - 65)/143.9} ×
100 = 54.82%

8. (2) The estimated cost of material in 1990 = 80 + 45 + 12 + 18 = 155.


The estimate cost in 1991 = 75 + 60 + 16 + 21 = 172
Cost of material rises by 5% and hence, the estimated cost will rise by the amount =.05 × (155 +
172) = 16.35.
9. (2) The ratio of cost of materials to labour cost = Total cost of material for the years/Total labour cost
for the years Using the above formulae, the ratio comes as 8 : 1
10. (3) The total expenditure is required to be kept within Rs. 700 lakh.
The estimated total expenditure = Rs. (52.1 + 267.5 + 196.4 + 209.5) = Rs. 725.5 lakh
The amount to be cut down = Rs. (725.25 - 700) = Rs. 25.5 lakh
This amount is cut equally in all the years on expenditure of administration and hence cut in each
year = Rs. 25.5/4 = Rs. 6.375 lakh.
% cut for year 1986 = (6.375/15) × 100 = 42.5%.
11. (4) Let the total length of the railway line to be laid is x km in the given years.
Length of line is proportional to the provision for material and labour cost.
That means x kms is laid for Rs. (sum of material and labour cost for the given years).
For Rs. 597.1 lakh, x km line is laid
Therefore by Rs. 175 lakh, (x/597.1) X 175 = 0.3x
12. (1) Total estimate = Rs. 725.5 Lakh; Estimate of contingencies = Rs. 25.2 lakh
Now the estimate of contingencies is doubled as it is felt inadequate = Rs. 50.4 % increase in total
estimate = (25.2/725.5) × 100 = 3.47%
13. (2) At the end of 1990, the entire amount for the project has been spent = 725.5 lakh. For 1991, the
amount spent = Rs. 209.5. The total amount spent on the project is Rs. (725.5 + 209.5) = Rs. 935
lakh. % by which the actual expenditure exceeds the estimated = {(209.5/725.5)} × 100 = 28.8%

14. (1) Referring table 5, we can directly say that % of blue sari sold is maximum in the region 2 = 33%
15. (2) Referring table 4, we can say directly say that minimum % of green sari is sold in region 6 = 14%
16. (4) Referring table 5, it is quite obvious that % of magenta saris sold is maximum for region 1 = 44%
17. (2) The popularity of any colour in any region can be checked by looking the % sale of that colour in
that region i.e. referring table 4. The most popular colour in region 12 is brown
18. (3) Refer to table 3 to know % of saris sold to saris stocked for each colour in each region.
The region-colour combination that accounts for the highest % of sales to stock is (4, Brown) =
74%
19. (4) The area which was brought under irrigation in 1986-87 = (24 - 23.2) + (34.2 - 32.77) = 2.23
million hectares
20. (1) Consumption of chemical fertilisers per hectare of gross cropped area is given by {Total
consumption of chemical fertiliser/gross cropped area}
The consumption of chemical fertiliser per hectare of gross cropped are is lowest for the year
1984-85 = (3.68 + 1.21 + 0.62)/173.1 = 0.0318 ton per hectare
21. (4) In year 1987-88 a part of minor irrigated area is brought under major and medium areas as it was
the cumulative figures and hence data for minor irrigation for 1987-88 must be greater than the
previous year, but it is not so.
22. (1) Refer to the table showing High yielding varieties. In year 1988-89, wheat jowar and bajra shows
a decline from previous year and that is the maximum number of crop.
23. (3) The travelling time was 5000/66 hours. The stopping time was 12 + 22 + 32 + … + 10² minutes.
24. (3) If the number of children older than 5 years of age is x and the number of children younger than 5
years of age is y, then 5x + 6y = 43. The only values of x and y satisfying the equation are x = 5
and y = 3.
25. (2) P lies between 2371 and 2379. If P is divisible by 4, it could be 2372 or 2376. But if it is divisible
by 9, it can only be 2376.
26. (2) [(x-1 - y-1)/(x-2 - y-2)] = (1/x - 1/y)(1/x² - 1/y²) = 1/(1/x + 1/y). If x and y are both greater than 2, 1/x
and 1/y are both less than 0.5. Hence 1/x + 1/y is less than 1, or its reciprocal is greater than 1.
27. (3) x + z = 2x + (y - x) + (z - y) = 2x + 2 + 2 = 2x + 4. Since x is odd (dos not have a factor 2), 2x + 4
is not divisible by 4.
28. (3) Let the average speed from C to M be s, and the distance be x. Then from M to T, the speed is 2s
and the distance is 0.3x. Average speed of the entire journey = distance / time = (x + 0.3x) / (x/s
+ 0.3x/2s) = 2.6s/2.3 = 40.
29. (2) If w is not older than V, he is younger than or the same age as Y. Hence Z is younger than X.
30. (4) The change in the prices of the two products over the next five years cannot be determined.
31. (2) x - y will always be 2. x + y < 10 does not specify the numbers.
32. (4) If CP is 80% of SP, then SP is 125% of CP, or the profit is 25%.
33. (1) If the length is I, the breadth is 48/1, and the diagonal is √[i² + (48/l)²] = 10. Hence I can be found.
34. (3) If a price reduction of 50% enables me to buy 48 bananas in Rs. 12, I can buy 48 bananas for Rs.
24 at the original price. ∴ The price of a banana is 50 paise.
35. (4) The only two positive numbers whose squares add up to 116 are 4 and 10.
EXERCISE 6

Direction for 1 to 4: First of all derive a general formula for Bankatlal’s monthly salary

1 week 2 week 3 week 4 week


Hours of rest X Y X Y
Working hrs. Y X Y X

Thus for every month, 1st & 3rd week and 2nd week must be identical.
Salary per day in 1st & 3rd week = KY, where k = wage per hour
Salary per day in 2nd & 4th week = kX/2
Total monthly salary = 1{ky + kX/2} = 6k(2Y + X), where
x = hours of rest per day, Y = working hours per day in the 1st week of the month.

Direction for 1 to 4

1. (1) Fro the 1st month X = 2, Y = 5, so total salary = 6x 20 (2x 5 + 2) = Rs. 1440.
2. (3) Total salary for 4 months = [1440 + 120 {(2 × 7 + 3) + (2x 6 + 4) + (2x 8)}] = Rs. 7320
7320
Average Salary = = Rs.1830.
4
3. (2) Under New Scheme salary per day in 9th & 11th week = 25 × 6 - 5x4 = Rs. 130
Under New Scheme salary per day in 10th & 12th week = 25 × 4 - 4x6 = Rs. 70
Under New Scheme total salary for 3rd month = 12 (130 + 70) = Rs. 2400
Under previous scheme total salary for 3rd month = 120 (2 × 6 + 4) = Rs. 1920
Difference = 2400 - 1920 = Rs. 480.
4. (2) As per the conditions salary for first 3 months = (!440 + 2040 + 2400) = Rs. 5880
For 4th month salary per day in 13th & 15th week = 8 × 25 = Rs. 200
For 4th month salary per day in 14th & 16th week = - 8 × 5 = (- 40)
Total salary for 4th month = 12 (200 - 40) = Rs. 1920
Total salary for 16 weeks = {1440 + 2040 + 2400 + 19

5. (2) In order to manufacture maximum number of units, make Q on both the machines as time taken
for manufacturing one unit of Q is less on both the machines. And maximum number of units
obtained = {(8 + 8) × 60}/6 = 160.

6. (1) As per the given condition number of units of P should be three times that of Q. One unit of P
takes less time on M2 as compared to that on M1. So manufacture P on all the time available on
M1, number of units of P obtained = (8 × 60)/8 = 60. Now number of units of Q required to fulfil
the given condition = (60/3) = 20. Time taken to manufacture 20 units of Q on M2 = 20 × 6 = 120
min. Remaining time available on machine M2 = (8 × 60) - (120) = 360 min. In this time
manufacture 3 units of P followed by 1 unit of Q & continue till all the available time is utilised.
Time taken to manufacture 3 units of P & 1 unit of Q on M1 = (3 × 8 + 6) = 30 min. Thus in 360
min 36 units of P & 12 units of Q will be manufactured & idle time will be zero.

7. (3) Here four different method is given and to compare each method we will bring them into common
platform so that we can compare them. Taking the LCM of 48, 64, 53 and 71 which will give total
number quantities to be manufactured and hence we will see which method is taking maximum
idle machine hour to manufactured this number of quantities. LCM of 48, 64, 53 and 71 = 722496
By method a) Total idle machine hours (722496 × 3)/48 = 45156 min
b) Total idle machine hours (722496 × 12)/64 = 135468 min
c) Total idle machine hours (722496 × 10) = 136320 min
d) Total idle machine hours (722496 × 9)/71 = 91584
From above it is very clear that method c) is maximum idle hours hence it is the least efficient
method. Answer is 3).

8. (4) If M1 works at half of its normal efficiency, time taken by M1 to manufacture 1 unit of P = 20
min. and Q = 12 min. And now maximum number of units obtained = (8 × 60)/12 + (8 × 60 - 8)/6
+ 1 = (41 + 472/6) = 119 (consider only complete units).

9. (1) In order to take minimum time manufacture P on M2 & Q on M1. Number of machine hours
required = (30 × 8 + 25 × 6)/60 = 6.50 hours.

10. (1) Total requirement of cloth = Total number of shirts x cloth required per shirt = (20 + 30 + 30 + 10
+ 10) 1000 × 1.5 = 150,000 m.

11. (3) Total quantity of high quality cloth consumed by A shirts = (80% of 20000) × 1.5 = (4/5) × 30000
= 24,000 m.

12. (2) Required Ratio = (40% of 30000)/(60% of 10,000) = 2 : 1.

13. (2) Total low quality cloth consumed = 1.5 {(3/10)x60 + (2/5) × 10 + (9/10) × 10} 1000 = 46,500 m.

14. (3)

15. (3) One can logically assume that a test will be feasible if its relative cost is lower than that of any other
test. Test -2 has the lowest relative cost in the range 0.05% to 0.2%, so the answer is (c).

16. (4) For p = 0.2, both, Test-2 and Test - 3 have the same relative cost, so both of them are feasible.

17. (1) Test-3 has the lowest relative cost for the range p > 0.2, so Test - 3 will be the best option to adopt
for p > 0.2.
18. (1) In the range 0.00 < p < 0.05, Test - 1 has the lowest relative cost, so the answer is (a), i.e. p < 0.05.
19. (4) If p < 0.2, then initially Test-1 is more feasible, while thereafter, Test -2 is
a better option. While we cannot say from the data given in the question, which of these two tests
is better, we can definitely say that Test-3 is the most expensive test for the range p < 0.2.

20. (3) 24. (3) 28. (4)


21. (3) 25. (3) 29. (2)
22. (4) 26. (2) 30. (1)
23. (3) 27. (1) 31. (3)

32. (3) dozen organs are equivalent to 4 kg mangoes, hence 14 kg mangoes cost Rs. 252.

33. (3) If tz is odd, both t and z are odd. If x + y + t is even, x + y + t - z odd.


34. (3) If the altitude to the base of an isosceles triangle is known, the base can be found, and hence the
area.
1

Solid DI Set 1
Question
For Questions 1 to 5 read the following instructions:

Nine horses participated in a horse tournament. Five races were held in this tournament. The winner of a race
gets 5 points, 2nd gets 3 and the 3rd gets 1. The table gives the points tally.

One race is held between five horses. The first race is held among the first 5 horses from the left in the above
table. Out of these horses, A drops out of the race and a new horse. ie. F enters into the race. In the next race, B
drops out and G enters and so on. It is also given that H is the only horse that scored in 2 consecutive races.

Q.1 What was the position of horse F in race 4?


a. 4th b. 2nd c. 3nd d. cannot be determined e.1st

Q.2 What is the ratio of the points scored by “G” and E in race 5?
a. 1:3 b. 3:1 c. 1:5 d. cannot be determined e. 5:1

Q.3 The first 3 rankers of the race 3 in some order?


a .C,E,G b. D,E,F c. C,D,F d. C,E,F e. C,G,F

Q.4 D could have come third in which of the following races?


a. Race 1 b. Race 3 c. Race 4 d. Race 5 e. cannot be determined

Q.5 Suppose if C had 6 points and E had 7 points while the points of the rest of the horses remain unchanged
then E could’ve come second in
a. One race b. Two races c. At most one race d. At most 2 races
e. cannot be determined
Answer:
For Solid DI Set 1
The table of the winners is as follows:
Race1 A C E
Race2 B F D So 1-b, 2-d, 3-a, 4-c
Race3 C E/G G/E
For 5 interchange the places of C and E in race 1. Hence the answer is d, 5-d
Race4 H F D
Race5 H G/E E/G

Solid DI Set - 2
Question
For Questions 1 to 5 read the following instructions:

The following Table gives the number of students in all the different classes of Indian Public school in the years
2050 and 2051 respectively.
2

Class Students in the year Students in the year


2050 2051
VII 120 140
VIII 84 100
IX 72 70
X 64 60
XI 48 56
XII 36 54

It is known that
i. New students join the school only in class VII
ii. No student leaves the school before passing out from class XII
iii. The students, who fail, have to repeat the year.

Q.1 If no student of class XI failed in the year 2020, then what is the pass percentage of class XII for the year
2050?
a. 22.66% b. 16.66% c. 83.33% d. 77.77% e. None of these

Q.2 If the new joiners in the year 2051 were 76, then the number of students failed in class XII in the year
2050?
a. 10 b. 14 c. 8 d. 16 e. 20

Q.3 How many students of class IX failed in the year 2050, if no student of class XI failed in the year 2050?
a. 12 b. 8 c.10 d. 14 e. None of these

Q.4 If number of students of class VII failed in the year 2050 is 64, then what is the total number of students
failed in the year 2050?
a. 150 b. 182 c. 195 d. 164 e. 176

Q.5 The highest pass percentage of class VII can be?


a. 55% b. 60% c. 58% d. 50% e. 65%

Answer:
For Solid DI Set 2

A general formula here is Strength (c, y) = Pass(c-1,y-1) + Fail(c,y-1)


Where c is the class and y is the year.

1. No. of failures in class XII in 2050 is 54-48=6. Hence the pass percentage is 5/6*100.ie 83.33

2. Back track and obtain the failures and passes in that particular year from the formula. Answer is 16.

3. Same procedure, start from the bottom. Answer is 20.

4. This is a supplement of question. no 7. Answer is 182.

5.The highest pass percentage for class VII can be obtained only when class XI has no failures( because, all the
other classes have certainly a min number of failures). You’ll get it as 66. so its 66/120 ie.55%

1-c ,2-d, 3-e, 4-b, 5-a


3

Solid DI Set - 3
Question

For Questions 1 to 5 read the following instructions:

Six people A,B,C,D,E and F are standing in a row(from left to right) in


that order. A is at place 1, B is at place 2 and so on. They are rehearsing
a dance sequence for a dance competition. When the signal is given the
following 3 steps are performed.
Step 1: They break away and form a triangular formation with 4 and 5 in front of 1, 2 and 3 and between 1 and
2 and 2 and 3 respectively, while 6 is in front of 4 and 5.
Step 2: Then 6 goes a step backward and joins between 4 and 5 who form the front row now.
Step 3: After a few dance moves, both these rows merge with the person in front of 1 joining between 1 and 2
and so on.
These 3 steps form a round. These steps are repeatedly performed till the dancers attain their original positions.
This is called one sequence.

Q.1 How many times are step 1, step2 and step 3 are performed before one sequence?
a. 8 b. 6 c. 4 d. 5 e. 10

Q.2 At which place is C after the completion of the penultimate round?


a. 1 b. 3 c. 5 d. 2 e. 6

Q.3 How many members come to their original positions at least once before the sequence ends?
a. 1 b. 2 c. 3 d. 4 e. 5

Q.4 Suppose their initial positions were in the order mentioned before but from right to left, and the steps 1 and
2 remain the same while in step 3 the person in front of 3 joins between 3 and 2 and so on to form a single row.

How many rounds before this sequence ends?


a. 4 b. 6 c.8 d. 10
e. They never come back to their original positions

Q.5 In the above mentioned sequence, how many members come back to their original position at least once
before the sequence ends?
a. 2 b. 3 c. 4 d. 5 e. 6

Answer:
For Solid DI Set 3

A B C D E F is the initial formation.

Step1: A B C
D E
F

Step 2: A B C
DFE

Step 3: A D B F C E

This is completion of Round 1.


After 5 rounds the formation will again be A B C D E F.
4

1-d, 2-d, 3-a

Another formation is F E D C B A

Step 1: C B A
E D
F

Step 2: C B A
EFD

Step 3: C E B F A D

From this formation it takes 6 rounds to complete a sequence.


4-b, 5-e

Solid DI Set - 4
Questions
For Questions 1 to 5 read the following instructions:

12 Hockey teams participated in a tournament. These teams were equally distributed into 2 pools A and B. In
the 1st round, each team played a match against all the other teams in the same pool. Top 3 teams with highest
average (from both the pools) went to the next round, where all the six teams played against each other once.
Again the top 3 teams with highest average qualified to the finals. In the final round, all the 3 teams played
against each other and the team with the highest average was declared the winner.

Scoring: A win earns 4 points, loss earns -2, and a tie will result in 2 each.

Average = Total points/ Number of matches

The following table shows the points tally

Teams A1 A2 A3 A4 A5 A6 B1 B2 B3 B4 B5 B6
Total 28 0 10 0 16 6 -6 26 6 12 4 14
Average 2.33 0 1 0 1.6 1.2 -1.2 2.16 1.2 1.2 0.8 1.16

Also
--The winner of the tournament won both its matches in the finals
--the total points earned by all the teams (Played in the second round) after second round are 100.

Q.1 Which 2 teams do not play against each other in the finals?
a. A1,B2 b. A1,B6 c. A5,B2 d. B2,B6 e. none of these

Q.2 Find the number of matches won by A2 and A4


a. 0 b. 1 c. 2 d. 3 e. cannot be determined

Q.3 How many points did the second runner up earn in the finals?
a. -4 b. 2 c. 4 d. 0 e. cannot be determined

Q.4 After round 2, the highest average of any team can be


a. 2 b. 2.4 c. 2.6 d. 2.8 e. cannot be determined

Q.5 The first runner up of the tournament was


a. A1 b. B1 c. B6 d. B4 e. None of these
5

Answer:
For Solid DI Set 4

The teams playing in second round are A1, A3 A5,B2,B4 and B6.
Final is played between B2, B6 and A1.
Total points earned by the teams which dropped out in second round are 10 + 16 +12 =38. So points earned by
B2, B6 and A1 after round 2 is 100-38 = 62.
Total points earned by these teams after final round = 28 +26+14=68, ie. They together earned 6 points in the 3
matches happened in the final round. A1 won both its matches in final ie. It got 8 points. So the points earned in
these 2 matches =4+4-2-2 =4 points. So the match between B2 and B6 resulted in 2 points. Hence it cannot be a
draw.

Suppose, B2 had lost that match , then it must’ve lost 4 points in the final round , while B6 must’ve gained 2
points which means it’s points after second round must’ve been 12, equal to that of B4, which contradicts the
fact that top 3 with highest average made it to the finals. Hence B2 must’ve won the B2 v/s B6 encounter. That
means B2’s points after second round must’ve been 24. while B6’s must’ve been 18.

Hence 1-c, 2-c, 3-a, 4-b, 5-e

Solid DI Set - 5
Question
For Questions 1 to 5 read the following instructions:

Friday morning, local pediatrician Dr. Johnson N. Johnson had appointments with five infants scheduled at
9:00, 9:30, 10:00, 10:30, and 11:00. Each of the five, including the Ortiz baby, is a different number--1, 2, 3, 4,
or 5--of months old. The following notes are available.

1. Immediately after seeing infant Beth, Dr. Johnson examined the Majors infant, who is 2 months younger than
Beth.
2. Erica isn't the one of the five who is 1 month old.
3. The doctor saw David later in the morning than the 1-month-old.
4. The 9:30 appointment was with the 3-month-old baby.
5. The Luce infant isn't the one who is 5 months of age.
6. Dr. Johnson saw the Nash infant, then examined Alice, who is 2 months older than the Nash baby.
7. The pediatrician's 10:00 appointment was with Chad, who isn't the Majors or the Nash baby.
8. The Prior baby isn't the 1-month-old and wasn't the doctor's 9:00 examinee.

Q.1. How old(in months) is infant Beth?


a. 1 b. 2 c. 3 d. 4 e. 5

Q.2 At what time did infant Erica meet the doctor?


a. 9 b. 9:30 c.10 d. 10:30 e.11

Q.3 Whose baby is infant Chad?


a. Ortiz b. Majors c. Luce d. Nash e. Prior

Q.4 Which of the following is not correct w.r.t infant David


a. He is Nash’s baby b. He is 2 months old
c. His appointment was at 11.00 d. none of these
e. His appointment was at 10:30

Q.5 Who was the last infant to meet the doctor?


a. Alice b. David c. Chad d. Erica e. Beth
6

Answer:
For Solid DI SET 5

By clue 7, Dr. Johnson saw infant Chad at 10:00, the middle appointment of the five. By clue 1, Dr. Johnson
saw Beth and then the Majors baby, while by clue 6, he saw the Nash baby and then Alice. Since Beth is 2
months older than the Majors infant (1) and Alice is 2 months older than the Nash infant (6), Beth can't be the
Nash baby and Alice be the Majors infant (the only way possible to combine the two clues), so four different
babies are named between the two clues. Since Chad is neither the Majors nor the Nash baby (7), either Dr.
Johnson saw Beth at 9:00 and the Nash child at 10:30, or he saw the Nash child at 9:00 and Beth at 10:30.
Trying the latter case, he would have examined Alice at 10:00 (6) and the Majors baby at 11:00 (1). By clue 4,
Alice would be 3 months old, with the Nash baby then 1 month old. However, neither Erica (2) nor David (3)
could be the Nash infant, so this arrangement cannot work. Dr. Johnson's 9:00 patient was Beth, with the
Majors baby his 9:30 (1); and he saw the Nash infant at 10:30 and Alice at 11:00 (6). The Majors baby is 3
months (4) and Beth 5 months (1) old. By clue 3, David is the Nash child and Chad the 1-month-old. Erica is
the Majors baby. By clue 6, David Nash is 2 and Alice 4 months old. Beth is neither the Luce (5) nor Prior (8)
infant and is the Ortiz baby. Chad's surname is Luce and Alice's Prior (8). In sum, Dr. Johnson saw the five
babies as follows:

• 9:00 - Beth Ortiz, 5 months


• 9:30 - Erica Majors, 3 months
• 10:00 - Chad Luce, 1 month
• 10:30 - David Nash, 2 months
• 11:00 - Alice Prior, 4 months

Hence 1-e, 2-b, 3-c, 4-c, 5-a

Solid DI Set - 6
Question

Directions for Q.1 to Q.5: Read the information given below and answer the questions that follow.

Five friends (Avinash, Vijay, Alok, Vivek and Rajiv) went to a hotel and ordered five different soups (Tomato,
Chicken, Vegetable, Corn and Mutton), followed by lunch. After lunch, they ordered five different desserts.
(Mango, Pista, Vanilla, Tutti-Fruti and Casatta).

(i) Avinash ordered for vegetable soup and Mango ice-cream


(ii) Vivek didn't order mutton soup but ordered Vanilla ice-cream
(iii) Vijay is a vegetarian who ordered Tutti-Fruti ice-cream
(iv) The person who ordered corn-soup also ordered Pista ice-cream while
Rajiv ordered Casatta ice-cream

1. Who among the following ordered Tomato soup?


(1) Avinash (2) Vijay (3) Alok (4) Vivek

2. Which among the following soups was ordered by Vivek?


(1) Vegetable (2) Corn (3) Chicken (4) Mutton

3. Which of the following combinations is FALSE?


(1) Rajiv - Chicken soup
(2) Tomato soup - Tutti Fruti ice-cream
(3) Chicken soup - Vivek
(4) Alok - Corn soup

4. Which of the following ice-creams was ordered by Alok?


(1) Vanilla (2) Tutti-Fruti (3) Mango (4) Pista
7

5. Which of the following combinations is TRUE?


(1) Avinash - Tomato soup - Tutti-Fruti ice-cream
(2) Vijay - Corn soup - Mango ice-cream
(3) Alok - Mutton soup - Pista ice-cream
(4) Vivek - Chicken soup - Vanilla ice-cream

Answer:
For Solid DI SET 6
Make a table as below

Avinash Vijay Alok Vivek Rajiv


Tomato X X X X
Chicken X X X X
Veg X X X X
Corn X X X X
Mut X X X X
Mango X X X X
Pista X X X X
Vani X X X X
T.F X X X X
Cass X X X X

1. Ans.(2) 2. Ans.(3) 3. Ans.(1) 4. Ans.(4) 5. Ans.(4)

Solid DI Set - 7
Question

1. If (a) '1 2 3' stands for These are boys',


(b) '1 3 4' stands for 'boys are naughty' and
(c) '1 4 5' stands for 'Naughty boys sing',
which numeral stands for 'naughty'?
(1) 1 (2) 2 (3) 3 (4) 4

2. In a certain code 'BROAD' is written as 'ASNBC'. How is 'PLUGH' written in


that code?
(1) OMTHG (2) QMTHG (3) QMTFG (4) OMTGH

Answer:
For Solid DI SET 7

A1) Correct answer is (4)


1 = boys, 2 = these, 3 = are, 4 = naughty, 5 = sing

A2) Correct answer is (1)


8

p stands for Preceding letter and s for Succeeding letter. Coding is done by taking the preceding and succeeding
letters alternately

Solid DI Set - 8
Question
An Inspector has discovered that 5 people were involved in the murder of Miss Batliwala. Nine suspects are
short listed by him. Maina, Naina and Raina are women. Ooman, sangha, Topaz, Unni Vasu and Wagle are
men. Through intelligent deliberation and analysis, Ghote has also made a checklist of points.
I) There were at least two women involved in the crime.
II) Raina will never collude with Ooman.
III) Sangha and Topaz will always commit crimes together.
IV) Unni and Vasu never work together.

A). If Maina was involved in the crime and Naina is not, which statements
given below are true necessarily.
I) Either Unni or Vassu but not both are involved.
II) Sangha and Topaz will be involved
1. I only 2. II only 3. Both I and II 4. Either I or II

B) Greatest number of combinations are possible if which of the following


are definitely involved in the crime.
1. Sangha and Topaz 2. Vasu 3. Raina 4. Wagle

C) Which of the following statements is definitely true ?


I) If only two women are involved in the crime Sangha and Topaz must
be involved.
II) If Raina is not involved in the crime, Ooman must be involved.
III) If either Maina or Naina is not involved in the crime Sangha and Topaz
must be involved.
1. I only 2. I and III only 3. II only 4. III only

Answer:
For Solid DI SET 8

A) Correct answer is 2.
Women : M, N and R Men ; O, S, T, U , V & W
If M is involved and N is not means that R must be involved so 3 men, of which O is not a part, of must be
involved so analysing given statements : either U or V but both not involved This means that U or V can pair up
only with S & T But S & T have the option of choosing W. So U or V must not necessarily be involved so
statement I is not necessarily true

B) Correct answer is 1.
If V is involved U cannot be involved If R involved O cannot be involved since We require 2 more men of
which U and O are not available so the option is S & T.

C) Correct answer is 4.
M and N can pair with U +W + O or V + W + O. If R is not involved, it can give options like M and N with S +
T +U or S + T + V or S + T + W so is definitely not true.

Solid DI Set - 9
Question

1) Find the word which does not fit in the group.


(1) D A B (2) H E F (3) T S R (4) P 0 N
9

2) Insert the missing word from the brackets.


54 (HIDE) 98 53 (....) 16
(1) FACE (2) 6135 (3) HIDE (4) ECAF

3) Which will come next in the series?


N Q L S J U–
(1) W (2) T (3) I (4) H

Answer:
For Solid DI SET 9

A1) Correct answer is (3)


All other options contain a vowel (A, E, I, 0, U) as the middle term.

A2) Correct answer is (1)


Numbers outside the brackets denote the alphabet, a = 1, b = 2, c = 3. HIDE has numbers 4,5,8 and 9. For
second series, we get EC and AF which are then reversed t

A3) Correct answer is (4)


Alternate letters are one letter away in the alphabetical order. There are two sets one beginning with N-L-J and
hence H, the other beginning with Q-S-U. The former is descending, while the latter is ascending.

Solid DI Set - 10
Question

At the IIM 5 areas of specilisation are available. They are Marketing, finance, production, personnel and
systems. However it is necessary for any student to take up a combnination of two of the above.

Based on common preference, preset combinations called moducles are offered, coded M001, M002, M003,
M004 and M005. Each allows two specialisations such that every specilisation is assigend to two modules.
Further to allow for uncommon prefercences a student is allowed to take up more than one commbination in
such a manner that the specialisation common to the two modules is necessarily dropped.

However, a smart student, by manipulating combinations, may be able to specialise in more than two subjects.

Additional information is as follows :


M001 has not been assigned production. M003 has not been assigned marketing. M004 has not been assigend
finance. By choosing M001 and M003 one may specialise in marketing and systems. By choosing M001 and
M002 one may specialise in Marketing, production finance and personnel. By choosing M001 and M004 one
may specialise in finance and personeel .

1. What is the maximum number of modules any person may take to specialise in at least two areas ?
1. 2 2. 4 3. 5 4. 3

2. Which of the following number of specialisations is impossible ?


1. 2 2. 3 3. 4 4. None of these

3. Personnel is available in modules


1. M002 and M003 2. M001 and M003 3. M002 and M004 4. M003 and M005

4. Finance is available in modules


1. M001 and M003 2. M003 and M004 3. M002 and M003 4. M001 and M004
10

Answer:
For Solid DI Set 10
1) 2 2) 2 3) 3 4) 1

The main thing is that if any specialisation is common to the 2 modules selected it gets dropped and is not a
specialisation. M001 does not have production and M003 does not have marketing. But their combination gives
us marketing and systems so M001 must have marketing since Moo1 and M002 have 4 different subjects three
must be no common specialisation and M002 will not have marketing but will have production M003 will have
systems included because it is not a part of M001 so M001 will have finance or personnel and it will be
common to M003. Since the M001 and M004 combination does not have marketing as a specialisation it means
that they have marketing in common. Since they have finance and personnel in common M001 has finance and
M004 has personnel. So M002 also has personnel. So M003 must also have finance. This leaves M005 with
production and systems.

Solid DI Set - 11
Question

There are two ethnic groups, the Dravids and the Aryans. No marriage is permitted within a group. After
marriage, men become a part of their wives’ group; women remain in their own group. Children belong to the
same group as their parents. Widowers and divorced males revert to the group of their birth. Marriage to more
than one person at a time and marriage to a direct descendant is forbidden.

1. A Dravid male could have


a. an uncle in either group b. a Dravid daughter
c. a Dravid son d. an Aryan son-in-law

2. Which of the following is not permitted under the rules stated?


a. a Dravid man marrying his father’s sister.
b. an Aryan woman marrying her mother’s brother.
c. a widower born Aryan, marrying his brother’s widow.
d. a widower marrying his wife’s sister.

3. If widowers and divorced males had retained the group they had upon marrying, which of the following
would be permissible, assuming that no previous marriage had occurred?
a. a woman marrying her dead sister’s husband.
b. a woman marrying her divorced daughter’s ex-husband.
c. a widower marrying his brother’s daughter
d. a divorced male his ex-wife’s divorced sister in law.

Answer:
For Solid DI SET 11
1) The key to remember in this question is that when it says “Dravid male” they refer to his group when he was
born, and not after he gets married. I will shorten Dravid to D and Aryan to A.
Given that, the answer is trivial. You can eliminate b and c immediately, since a D must be married to an A,
becomes an A, his children (sons and daughters) are A’s. Option d is similarly eliminated since his daughter A,
must marry a D. That leaves option a, which can be quickly verified. An uncle could be maternal or paternal.
His paternal uncle could be a D (if unmarried), or an A (if married). His maternal uncle could be an A (if
unmarried), or a D (if married). Hence option a) is correct.

2) Try and eliminate each option.


Option a) D marrying his father’s sister(A).Possible.
Option b) A marrying her mother’s (A) brother (A). Not possible.
Option c) A widower born A, therefore now A, marrying his brother (A)’s widow (D). Possible.
11

Option d) A widower X, marrying his wife’s (X’) sister (X’). Possible.


Hence answer is b)

3)Once again eliminate each option:


Option a) A woman X marrying her dead sister X’s husband (X (because he retains X after his wife died). X->
X not possible.
Option b) A woman X marrying her divorced daughter X’s ex-husband (X). Not possible.
Option c) A widower X (born X’, but X after marriage, retained after his wife’s death), marrying his brother
(X’)’s daughter (X) (since daughter will get her mother’s group which must be X). Not possible.
Option d) A divorced male X (born X’) marrying his ex-wife’s (X)’s divorced sister-in-law (X’ or X). Sister-
in-law can be the wife’s husband’s sister -> (potentially her husband’s sister (heaven forbid!), who would be an
X’) or her brother’s wife (also an X’). Either way X, marrying an X’ makes it permissible.

Solid DI Set - 12
Question

In 2001 census, the following data regarding two village ‘Rajgarh’ and ‘Sukhgarh’ was found
I. Rajgarh has 2104 more males the Sukhgarh
II. Sukhgarh has 2041 fewer females than Rajgarh
III. Sukhgarh has 687 more males than females
IV. Rajgarh has 750 fewer females than males

1. What is the total number of males in Rajgarh and Sukhgarh?


1. 11,924 2. 10,510 3. 12,510 4. Cannot be determined

2. What is the difference between the number of males in Sukhgarh and the number of females in Rajgarh?
1. 27,961 2. 1,354 3. 2,747 4. Cannot be determined.

Answer:
For Solid DI SET 12

1.1 Let x be number of males in Rajgarh, z the number of males in Sukhgarh, y the number of females in
Rajgarh, and t the number of females in Sukhgarh. Check the options if x + z = 11924. Then x = 7014, z = 4910,
y = 6264 and t = 4223
So, all other equations (ii) , (iii) and (iv) are satisfied.

2. 2 Here y – t = 2041 and z – t = 687 So, y – z = 1354

Solid DI Set - 13
Question

A hedge fund has recently acquired four companies - BOB, MOB, ZOB and DOB. The sales of DOB are half
that of BOB whereas the profits of DOB are double that of BOB. The expenses of ZOB are Rs 3 crores less than
that of DOB whereas the profit of MOB is Rs 1 Crore less than that of ZOB. The expenses of BOB are three
times that of DOB. It is also known that the sales of ZOB are Rs 15 crores or one-fourth that of MOB. The
auditing firm has also reported that sales of DOB are Rs 10 crores more than that of ZOB and the expenses of
BOB are 90% of its own sales.

Q1) The total sales of all the four companies is (Rs crores):
a) 200 b) 150 c) 125 d) 160

Q2) The expenses of BOB exceed that of ZOB by (Rs crores):


a) 34 b) 43 c) 33 d) 62
12

Q3) Which company had the maximum profits?


a) BOB b) MOB c) ZOB d) DOB

Q4) The expenses of MOB exceed the profits of BOB by (Rs crores):
a) 51 b) 52 c) 53 d) 54

Q5) Total profits of the four companies are (Rs. Crores):


a) 15 b) 20 c) 25 d) 40

Q6) The profits of MOB form what percentage of the expenses of ZOB?
a) 16% b) 20% c) 15% d) 23%

Answer:
For Solid DI SET 13

Based on the information given, best way to solve this is to make a table. May seem tedious initially but would
save you time for answering other questions

Sales Expenses Profits


BOB 2X=5 0.9 x 50 = 45 50 – 45 = 5
MOB 60 60 – 2 = 58 B–1=3–1=2
ZOB 15 A – 3 = 12 B = 15 – 12 = 3
DOB X = 25 A = 45/3 = 15 2 x 5 = 10
Total 150 130 20

A1) Correct answer is (b)


A2) Correct answer is (c)
A3) Correct answer is (d)
A4) Correct answer is (c)
A5) Correct answer is (b)
A6) Correct answer is (a)

Solid DI Set - 14
Question

For questions in this test, use the following answer choices

a) If the question can be answered by using one of the statements alone, but cannot be answered using the other
statement alone.
b) If the question can be answered by using either statement alone.
c) If the question can be answered by using both statements together, but cannot be answered by using either
statement alone.
d) If the question cannot be answered based on the information provided

1) Average of ten numbers s 15.34. If the fifth number is incorrect then what would be the average of the
remaining nine numbers?

I. In the given ten numbers the average of the first five numbers is 14.35
and the average of the last six numbers is 15.43.
13

II. In the given ten numbers, the average of the first six numbers is 14.38
and the average of the last six numbers is 15.40.

2) Is the prime number p equal to 37?

I. p = n2 + 1, where n is an integer
II. p2 is greater than 200

Answer:
For Solid DI SET 14

A1) Correct answer is (a)


Only A alone is sufficient. From B, we can get only the sum of the fifth and the sixth numbers.

A2) Correct answer is (d)


Since 142 = 196 and 152 = 225, if follows from (B) that p > 14

In (A) the expression n2 + 1 can represent a prime number less than 37, equal to 37, or greater than 37,
depending on the value of n. For example, if n = 4, the 42 + 1 = 17; if n = 6, then 62 + 1 = 37; if n = 10, then
102 + 1 = 101; and 17, 37, and 101 are all prime numbers. Thus, (A) alone is not sufficient
Both the statements together are not sufficient

Solid DI Set - 15
Question

For questions in this test, use the following answer choices

a) If the question can be answered by using one of the statements alone,


but cannot be answered using the other statement alone.
b) If the question can be answered by using either statement alone.
c) If the question can be answered by using both statements together, but
cannot be answered by using either statement alone.
d) If the question cannot be answered based on the information provided

1) Two schools decide to send their students to a picnic on the same day.
How many students attend the picnic from the first school?

I. 40 students in all attend the picnic


II. If you multiply the number of students from first school with the
number of students from the second school, the answer is 300

2) Radhesh is a class X student and is practicing for his board exam. He


draws a circle of radius 9.6 cm. He also draws a chord, whose length is
I. If you measure the perpendicular distance from the centre of the centre
of the circle to the chord, you get 6 cm
II. If you draw a triangle with two end points being the end points of the
chord and the third point touching the circle, the 3rd angle is 80 degrees

Answer:
For Solid DI SET 15

A1) Correct answer is (d)


14

While at first glance, you may be tempted to answer ©, since there are two linear equations and two variables,
as you try to solve the question you will get two answers 30 and 10. So you can’t say which one of those applies
to the first school

A2) Correct answer is (a).


Statement I allows you get the answer since the perpendicular from the centre of the circle to the chord is going
to bisect the chord. Then using Pythagoras theorem, you can calculate the length of the ½ chord
Statement II doesn’t provide you any concrete information to be useful

Solid DI Set - 16
Question

Mr. Malik went to US on a business trip and rented a car to drive from his hotel to the plant that he was visiting.
Being new to driving in US, he was very cautious and wanted to ensure he drove within the speed limits. The
graph below shows the speed he was driving at different times of the day

1) For what percent of the time was he driving at 40 miles per hour or faster?
a) 20 b) 45 c) 40 d) 50

Q2) How far, in miles, did Mr. Malik drive between 8:30 and 9:00?
a) 0 b) 20 c) 30 d) 40

Q3) The total distance traveled by Mr. Malik is


a) less than 50 miles b) between 50 and 100 miles
c) greater than 100 miles d) Can’t be calculated

Q4) How much time Mr. Malik would have taken if he had traveled the whole distance at the average speed of
40 miles per hr?
a) 2 hrs b) 2.8 hrs c) 2.4 hrs d) 3 hrs

Answer:
For Solid DI SET 16

A1) Correct answer is (d)


A quick glance at the graph will show you that x axis has equally spaced intervals. Out of the 5 intervals that he
drove, for 2.5 of those the graph on y axis is above 40, so 50%

A2) Correct answer is (d)


Since his speed was 40 miles per hr, he would have driven 40 miles
15

A3) Correct answer is (b)


Take the average speed for 30min. intervals and divided by 2 (since each interval is ½ hr)
( 37.5 + 40 + 45 + 40 + 30 )/2 = 192.5 / 2 = 96.125

A4) Correct answer is (c)


Use data from previous question re distance traveled 96.125 / 40 = 2.4 hrs approx.

Solid DI Set - 17
Question

Two shopkeepers go to buy newly designed weights from the trade fair. In a particular shop the salesman asks
their limits of weighing at a time. Both of them tell their limit as 120 kg at a time. The salesman shows them the
price list as:

Any weight less than 10 kg ? Rs. 10 per piece


More than 11 kg but less than 50 kg ? Rs. 20 per piece
More than 51 kg but less than 100 kg ? Rs. 30 per piece.
More than 100 kg ? Rs. 50 per piece.

The salesman gave them two options:

In the 1st option, the shopkeepers might have to put the weights on both sides. He gave them an example:
Suppose you want to weigh 7 kg, so put 10 kg on weight side and put 3 kg on the goods side.

In the second option, the shopkeeper did not need to put the weights on goods side.

1. If the 1st shopkeeper opts for the 1st option, what is the minimum number of weights he would need to buy at
the lowest cost?
1. 8 2. 10 3. 5 4. None of these

2. The 2nd shopkeeper opts for the 2nd option. What is the min. no. of weights he would need to buy at the
lowest cost?
1. 5 2. 7 3. 10 4. None of these

3. One of them needed as much more amount as the other has left after purchase. Suppose both of them left
their home with the same amount. Who borrowed and what amount was borrowed?
1. First shopkeeper borrowed Rs. 30
2. Second shopkeeper borrowed Rs. 30
3. Second shopkeeper borrowed Rs. 15
4. First shopkeeper borrowed Rs. 15

Answer:
For Solid DI SET 17
A1) Correct answer is (3)
In the first option, the minimum no. of weights needed is 5. The weights are 1 kg, 3 kg, 9 kg, 27 kg, 81 kg.

A2) Correct answer is (2)


In the second option , the minimum no. of weights needed is 7. The weights are 1 kg, 2 kg, 4 kg, 8 kg,16 kg, 32
kg, 64 kg.

A3) Correct answer is (3)


First shopkeeper needs 3 x 10 + 1 x 20 + 1 x 30 = Rs. 80
Second shopkeeper needs 4 x 10 + 2 x 20 + 1 x 30 = Rs. 110
Therefore Second shopkeeper needed to borrow Rs. 110-80/2 = Rs. 15
16

Solid DI Set - 18
Question

The following is the list of instructions to be followed


Step1: x=0 A=2 B=3
Step2: If X<5 then do Steps3-6 otherwise Step7
Step3: Y=A+B replace A by B, replace B by Y
Step4: Type Y
Step5: Increase X by 2
Step6: Goto Step2
Step7: Exit

1.When X=4 what value of Y is typed?


1] 12 2] 8 3] 13 4] 5 5] 8

2.Suppose Step5 is replace by 'Increase X by 5' then what would be the last value of Y typed?
1] 5 2] 4 3] 6 4] 9 5] 8

3. After the set of instructions is over at Step 7 what is the value of X


1] 5 2] 4 3] 3 4] 7 5] 8

Answer:
For Solid DI SET 18

1. While X=0, Y=5, A=3, B=5


While X=2, Y=8, A=5, B=8
While X=4, Y=13, A=8, B=13
Hence [3]

2. While X=0, Y=5, A=3, B=5


If X is increased by 5 further values of Y will not be calculated.
Hence [1]

3. The last value of X is 4


Hence [2]

Solid DI Set – 19 Question

Five brothers have among them a pair of twins who are neither the oldest nor the youngest. E is older than C but
younger than B. D is younger than 3 brothers

1) The youngest is
1] A 2] B 3] C 4] D

2). One of the twin pair is


1] B 2] C 3] D 4] E

3). The eldest is


1] A 2] B 3] C 4] E

4) The one who has as many elder brothers as younger brothers is


1] A 2] B 3] E 4] none of these

5) Who is the fourth brother


1] A 2] B 3] D 4] E
17

Answer:
For Solid DI SET 19

The following table explains the answers

1 2 3 4 5
B E A D C

1) [3] 2) [4] 3) [2] 4) [1] 5) [3]

Solid DI Set - 20
Question

Each item has a question followed by two statements each giving some data.

• Mark 1 if the question can be answered by using one of the statements alone; but cannot be answered
by using other statement alone.
• Mark 2 if the question can be answered by using either statement alone.
• Mark 3 if the question can be answered by using both statements together, but cannot be answered by
using either statement alone.
• Mark 4 if the question cannot be answered even by using both the statement together.

1) a and b are two positive numbers. How many of them are old?
I. Multiplication of b with an odd number gives an even number
II. a2 – b is even.
1] 1 2] 2 3] 3 4] 4

2)A, B, C, D have to stand in a queue in descending order of their heights. Who stands first?
I. D was not the last, A was not the first.
II. The first is not C and B was not the tallest.
1] 1 2] 2 3] 3 4] 4

3) Of two boxes of sweets A and B, which costs more?


I. The box A has 32 sweets of one kind and box B has 24 sweets of
another kind.
II. 6 sweets of box B cost more than 8 sweets of box A.
1] 1 2] 2 3] 3 4] 4

Answer:
For Solid DI SET 20

1) From statement I - b is even but nothing can be said about a


From statement II, a2 - b is even only if both are of the same parity (odd
or even),which means a is also even. Therefore none of a and b are odd.
Using both statements we can answer the question. Choice (3)

2) Taking II and I we get A, B, C are not the tallest, hence D has to be the
tallest. Thus D stands first. Choice (3)
3) From statement I, the price cannot be calculated.
18

From statement II, 6 sweets of B>8 sweets of A or 24 sweets of B > 32


sweets of A
Hence, we can say box B is more expensive than box A. Choice (3)

Solid DI Set - 21
Question
Each item has a question followed by two statements each giving some data.

• Mark 1 if the question can be answered by using one of the statements alone; but cannot be answered by
using other statement alone.
• Mark 2 if the question can be answered by using either statement alone.
• Mark 3 if the question can be answered by using both statements together, but cannot be answered by
using either statement alone.
• Mark 4 if the question cannot be answered even by using both the statement together.

1) How many boys are there in the class?


I. The number of boys in the class is 8 more than the number of girls in the class, which is five times the
difference between the number of girls and boys in the class.
II . The number of girls in the class is four less than half the total number of students in the class.
1] 1 2] 2 3] 3 4] 4

2) How many guests attended the party hosted by Rajesh? He invited 60 people for the party.
I. 12 of the invitees expressed their inability to attend the party where as another 12 did not get back to Rajesh
nor did they attend the party later on.
II. 50% of the invitiees confirmed that they would attend the party, but only 80% of them actually attended the
party.
1] 1 2] 2 3] 3 4] 4

Answer:
For Solid DI SET 21
1) From statement I
B=G+8
G = 5 x (B – G) = 5 x 8 = 40
B = 48
From statement II
G + 4 N/2 Choice (1)

2) From statement I only 60 people, 12 have expressed inability and 12 have


not come. Still nothing can be said about the other 24.
From statement II though 80% of 50% did come. No information on the
other 50% is given. Choice (4)

Solid DI Set - 22
Question
Priya has agreed to feed a friend’s dog for a full week, running from Monday to Sunday. A total of seven brands
of dog food are available, of which exactly five, M, N, O, P and Q are protein-enriched, and exactly two, R and
S, are vitamin-enriched. Priya must adhere to the following feeding instructions.
On each of the seven days, a different brand of dog food must be fed.
Protein-enriched dog food cannot be fed for more than two consecutive days.
R must be fed earlier in the week than S.
M must be fed earlier in the week than R.
M must be fed earlier in the week than Q and exactly four of the other brands must be fed between N and Q.
19

1) Which of the following brands must be scheduled to be fed on Wednesday?


[1] M [2] O [3] P [4] R

2). If Q is scheduled to be fed on Sunday, which of the following could also be true?
[1] R and O are scheduled to be fed on consecutive days.
[2] N is scheduled to be fed earlier than M.
[3] Two days intervene between the day O is fed and the day P is fed.
[4] P is scheduled to be fed on Tuesday.
3) Which of the following pair of brands could be scheduled to be fed on consecutive days?
[1] M and O [2] N and P
[3] O and S [4] Q and R

Answer:
For Solid DI SET 22
The possible combinations that meet all the conditions
Mon Tue Wed Thurs Fri Sat Sun
a) N M R O/P S Q P/Q
b) M N R O/P S P/O Q

1. [3] 2. [1] 3. [3]

Solid DI Set - 23
Question
DIRECTIONS for the questions 1 to 3
A quack- Dr. Fool’s Paradise- fools his customers by claiming to predict future using a unique method. He has
three parrots kept in three different cages. Each cage has three cards with a single non-zero digit inscribed on
every card. No two cards have the same number & no cage contains two cards with digits totalling ten. Further,
the total of three cards in the first cage is greater by two than the total in the second cage & by four than that of
third cage. When a customer asks for his prediction, the quack lets out three parrots which randomly picks one
card out of their respective cages. Before the prediction is made, the quack totals the digits on the three picked
cards & charges the customer the same number of rupees as the total of the cards. One day a customer paid
seven rupees for his prediction.
1. What is the lowest payment possible?
(1) Rs. 5 (2) Rs. 7 (3) Rs. 6 (4) Rs. 8

2. What is the maximum possible amount that anyone can pay?


(1) Rs. 22 (2) Rs. 23 (3) Rs. 24 (4) Rs. 25

3. Which of the following payments can never be made by any customer?


(1) Rs. 19 (2) Rs. 16 (3) Rs17 (4) Rs. 23

Answer:
For Solid DI SET 23
The cards are numbered 1,2,3…..9
There fore, the sum of numbers on all cards = 9 * 10 /2 = 45
If sum of numbers of cards in the 1st cage = X , then
Sum of numbers of cards in the 2nd cage = X-2
Then , sum numbers of cards in the 3rd cage = X-4
X + X-2 + X-4 =45 or X =17
There fore the three cages would have cards totaling 17 , 15 and 13 respectively.So, each cage should have one
card from each of the following three sets of cards .
1,2,3 4,5,6 7,8,9
The following combinations are possible if one has to satisfy the requirements.
20

Case 1 : 17 = 9 + 6 +2 9 + 5 + 3 9 + 5 + 3
Case 2 : 15 = 8 + 4 +2 8 + 6 + 1 6 + 7 + 2
Case 3 : 13 = 7 + 5 + 1 7 + 4 + 2 8 + 4 +1
But combinations 2 and 3 are not possible because the only way a person can pay Rs 7 is if three cards are 4 , 2,
1=7
But in combination 2 and 3 , this is not possible.
Therefore the only combination possible is 1.
The lowest possible payment is 3 + 2 + 1 = 6
Ans 1 (3)
Ans2 (3)
The maximum possible payment is 9 + 8 + 7 = Rs 24.
Ans3 (4)
A customer cannot pay Rs 23 as 23 = 9 + 8 + 6 as 9 and 6 are in the same cage.
Option (2) is possible as all three cards mentioned are in different cages. A customer can pay Rs 19 i.e 19 = 9 +
3 + 7. as all three cards are in different cages.

Solid DI Set - 24
Question

DIRECTIONS for the questions 1 to 5


Robert received a large order for stitching school uniforms for Pink Flower school.He has two cutters who will
cut the fabric , five tailors who will do the stitching , and two assistants to stitch the buttons and button holes.
Each of these 9 persons will work for exactly 10 hours a day.Each Pink Flower uniform requires 20 utes for
cutting,1hour for stitching and 15 minutes for buttons and button holes. The Little Flower uniform requires 30
minutes , 1 hour and 30 minutes respectively.
1) Find the maximum number of Little Flower uniforms that Robert can complete in a day.
(1)30 (2) 35 (3) 40 (4) 36

2) If on a day ,Robert decides to complete 20 Little Flower uniforms , then how many Pink Flower uniforms can
he complete that day?
(1) 30 (2) 35 (3) 40 (4) 36

3) If Robert decided to complete 30 Little Flower uniforms only and no other uniform on a particular day, then
how many total man hours will go idle?
(1) 30 (2) 60 (3) 45 (4) 36

4) If Robert hires one more assistant , then what is the maximum number of Pink Flower uniforms that he can
complete in a day ?
(1) 60 (2) 48 (3) 120 (4) 50
5) Robert has the option to hire one more employee of any category.Whom should he hire to get maximum
increase in production capacity , assuming that he needs to stitch only Pink Flower uniforms on that day?
(1) One cutter (2) One tailor
(3) One assistant (4) Either a tailor or a cutter.

Answer:
For Solid DI SET 24

When he has to complete maximum number of Pink Flower uniforms in a day, he will not at al devote to the
Little Flower uniforms .Now , to find the maximum number of Little Flower uniforms , we have to concentrate
on the time resource .It will be the category with the least available time which will then decide the maximum
no. of uniforms which can be prepared.
Cutting :20hours available and30minutes required per uniform.
Thus 20*60/30 = 40 uniforms can be cut.
Stitching 50 hours available and 60 minutes required per uniform .Thus 50*60/60 = 50 uniforms can be
stitched.
21

Buttoning : 20 hours available and 30 minutes required per uniform. Thus ( 20 * 60)/30 = 40 uniforms can be
buttoned .Obviously , the least number of the above three will determine the maximum number of uniforms that
can be completed.
Ans1 (3 ) 40 uniforms of Little Flower can be completed.
Total uniforms of Little Flower to be completed = 20
Cutting time required = 20 * 30 = 600 minutes = 10 hours
Stitching Time required = 20 * 60 = 1200 minutes = 20 hours
Buttoning time required = 20 * 30 = 600 minutes = 10 hours
Cutting time remaining = 20 –10 = 10 hours: Stitching time remaining = 50 –20 = 30 hours;
Buttoning time remaining = 20 – 10 = 10 hours. Since the times for cutting , stitching and buttoning each Pink
Flower uniform are 20 minutes , 60 minutes and 15 minutes respectively.
We can cut ( 10 * 60) /20 = 30 Pink Flower uniforms , we can stitch ( 30 * 60)/ 60 = 30 Pink Flower uniforms
and we can stitch
( 30 * 60 )/60 = 30 Pink flower uniforms , and we can button ( 10 * 60 )/60 = 30 Pink Flower uniforms, and we
can button
( 10 * 60 )/ 15 = 40 Pink Flower uniforms
30 is the maximum number possible ( the bottleneck resource )
Ans2(1)
Ans3(1)
Total man- hours available = 20 + 50 + 20 = 90 per day.
If Robert makes 30 Little Flower uniforms in a day , time utlised
= (30 * 30) minutes + ( 30 * 1) hours + (30 * 30 ) minutes
Total time utilized = 15 + 30 + 15 = 60 hours
Man hours remaining = 90- 60 = 30 hours
Again we have to basically find out the bottleneck resource
Total cutting time available = 20 hours
Number of uniforms that can be stitched = ( 50 * 60) / 60 = 50
Total stitching time available = 50 hours
Number of uniforms that can be stitched = ( 50 * 60)/60 = 50
Total buttoning time available = 30 hours
Number of uniforms that can be buttoned = 30 * 60 /15 = 120
The minimum hour = 50 = The maximum uniforms that can be completed
Ans 4(4)
When he makes only Pink Flower uniforms on a day , as per the current availability
No. of uniforms that can be cut = 20 * 60 /20
Number of uniforms stitched = 50 * 60 / 60 = 50 , and the number of uniforms buttoned = ( 20 * 60 )/15=80.
Thus maximum no. of uniforms completed = 50 and the bottle neck was time for tailoring ( stitching). Hence to
get the maximum increase in the production capacity , he should employ one more tailor. Ans5 ( 2 ).
Solid DI Set - 25
Question
1) A, B, C, D & E are five different integers. When written in the ascending order of values, the difference
between any two adjacent integers is 3. D is the greatest and A is the least. B is greater than E but less than C.
The sum of the five integers is equal to E- 1. A + B is:
A] 15 B] -8 C] -9 D] 16

2) Mark your answer as


(1) If statement (A) alone is sufficient to solve the question but statement (B) alone is not;
(2) If statement (B) alone is sufficient to solve the questions but statement (A) alone is not;
(3) If neither (A) nor (B) is individually sufficient to solve the question but a combination of these is sufficient
to solve the question;
(4) If both the statements (A) & (B) are individually sufficient to solve the question and
(5) If both the statements taken together are not sufficient and more information is required to solve the question
Q. If the average height of three people if 68 inches, is the shortest person more than 60 inches tall ?
(A) The height of the tallest person is 72 inches
(B) One of the persons is 70 inches tall
22

1] 1 2] 2 3] 3 4] 4
Answer:
For Solid DI SET 25
1. Answer is B]
We know that D>C>B>E>A and D+C+B+E+A=E-1. We also know that the numbers are E-3, E, E+3, E+6,
E+9 or E = -4.
B = -1, A = -7, A+B = -8
2. Answer is 1]
Only the first statement is needed. If x, y and z are the heights,
X + y + z = 68 x 3 = 204
If z = 72, x + y = 132
Since x and y are less than 72 (z is the tallest person), the shortest person has to be as tall as 132 – 72 = 60
inches.

Solid DI Set - 26
Question
In a one-day cricket international between India and Australia. Australia managed to score 329 runs in 50 overs.
To win the match India was required to score 330 runs in 50 overs. Tendulkar and Seghwag opened the innings
for India. The first wicket fell at the end of the 9th over when the score was 63. At the end of the 19th over the
second wicket fell at 128. 3rd wicket fell at the score of 187 when Tendulkar was caught behind in the 32nd
over. 4th & 5th wicket fell when score was 261 in 42nd over. Finally, India scored 333 with a four hit by
Harbhajan of the first fall he faced in the 50th over. India won the match by 4 wickets with two balls to spare.
Tendulkar scored approx 37% of the total runs scored by India, but man of the match was given to the player
who scored curtail runs in the pressure situation at the end of the innings. Two more players scored more than
50 runs. Seghwag and Pathan scored in mid 20’s. Dhoni scored more than Pathan and Seghwag but still
Seghwag was not the least scorer. Indian batting order – Tendulkar - Seghwag – Pathan – Dhoni – Yuvraj –
Dravid – Kaif - Harbhajan – Zaheer – Balaji – kumble. Each team had 11 players 5 batsmen, 1 wicketkeeper
and 5 bowlers, where only 10 wickets is to be taken of a team. The game was played in accordance with the
rules laid down by the ICC.

1. What can be said about Sachin’s contribution in the three partnerships he was involved in assuming that there
were no extras conceded by the Australian team?
1] 39, 40 & 44 2] 38, 39, 46 3] 40, 37 & 46 4] 41, 39, 44

2. What can be concluded from theanalysis of the paragraph given above inaccordance with the two statements
give with this question:
Statement 1: Dhoni scored second highest runs and was the 5th wicket to fall
for India in the 42nd over.
Statement 2: Dravid scored a duck in the match.
1] If statement 2 is true then statement 1 is necessarily false.
2] If statement 1 is false then statement 2 is necessarily true.
3] If statement 1 is true then statement 2 is necessarily true.
4] None of the above.

3. What can be said regarding the following two statements?


Statement 1: Kaif was not out at the end of Indian innings with second highest score.
Statement 2: Tendulkar was not involved in the highest partnership.
1] Both statements could be true.
2] At least one of the statements must be true.
3] At most one of the statement is true.
4] None of the above
23

Answer:
For Solid DI SET 26

1. 2 Sachin scored approximately 37% runs scored by India i.e. 333 *37% = 123 runs approx. He was involved
in the partnerships of 63 (Seghwag), 65 (Pathan) and 59 (Dhoni). Seghwag and Pathan scored in mid 20’s
(between 24 – 26) As Pathan scored more than Seghwag, so lets assume they score 25 and 24 respectively. In
first partnership of 63 if Seghwag scored 24 so Sachin cannot score more than 39 runs. In second partnership of
65 if Pathan scored 26 so Sachin cannot score more than 39 runs. There in the first two partnerships Sachin
cannot score more than 78 runs. So he must have score 45 runs in the third partnership with Dhoni to make 123
runs. So option 2.
2. 1 If statement 2 is true, than 1 cannot be true as if Dravid scored a duck than he must be the one to get out as
the 5th wicket in the 42nd over for India. In such case either Dhoni or Yuvraj was the 4th wicket to fall for India
as they were batting after the fall of 3rd wicket (Sachin). Dravid can only come to the crease after the fall of 4th
wicket and he is the second batsman to get out in the 42nd over. So he can only be the 5th wicket to fall without
scoring a run. See the batting order for India. So option 1.
3. 3 Statement 1: Kaif batted at no 7th, which means he came to crease
after the fall of 5th wicket in 42nd over when score was 261. So Kaif was
involved in a partnership of 72 runs for the 6th wicket, where as either or 5th
or 6th (Rahul or Yuvraj) was involved in two partnerships of 74 and 72. So
logically it could not be true.
Statement 2: 1st wicket partnership was of 63, second wicket partnership
was of 128 – 63 = 65, third wicket partnership was of 187- 128 = 61 when
Tendulkar got out), 4th partnership was of 261- 187 = 74, 5th wicket fell at
same over (given), 6th wicket partnership was of 333 – 261 = 72.
So statement 2 is absolutely true.

Solid DI Set - 27
Question

A Quality Assurance inspector is planning to inspect certain vehicles on this Monday and Tuesday at Tata
Motors. He has to inspect three vehicles on each of the two days. On these two days put together, he has to
inspect at least three Indica’s, at least one Safari and at least one Ace. The registration numbers of a vehicle
available for inspection are as follows:
Indica : I 21, I 22, I 24, I 23
Safari : S 11, S 21, S 12, S 13
Ace : A 22 , A 11 , A 12
Each vehicle is given a registration number, which consists of a single – letter code (I, S or A) followed by a
two – digit numerical code (21, 22, etc.)
The following restrictions are placed in a selection of the vehicles to be inspected:
(i) Not more than two vehicles of the same kind can be inspected on one day.
(ii) Vehicles whose registration numbers have a numerical code of 21, 22 and 23 are available for inspection on
Tuesday only.
(iii) No vehicle inspected on Monday can be inspected on Tuesday.
1. Which of the following vehicles must be inspected by the vehicle inspector?
1] S 13 2] A 22 3] I 23 4] I 24

2. Which of the following is possible combination of vehicles that can be inspected on Tuesday?
1] I 21, S 21, I 14 2] S 11, A 12 , I 23 3] I 22, S 13, I 21 4] I 23, S 21, A 22

Answer:
For Solid DI SET 27

1. 4 Among the Indica’s I 21, I 22 and I 23 can be inspected only on the second day. As three Indica’s have to
be inspected in two days and at most two Indica’s can be inspected on Tuesday, I 24 must be inspected on
Monday.
24

2. 3 I 24 must be inspected on the first day. As only one Indica can be inspected on the first day, at least two
Indica’s must be inspected on the second day. Hence option 2 and 4 are not possible combinations. Option 3
does not violate any condition.

Solid DI Set - 28
Question

There are six companies – Rediff, Bazee, Naukri, Contest 2 Win, Shaadi and MCX. In the dotcom companies
market capitalization, four of these six companies have secured the first four positions. Three venture capitalists
– Natwest, Chrysalis and General Atlantic – have invested in these companies their individual speculations
regarding the companies that would top the market capitalization among the first four positions are given in the
following tables.
Venture Speculated Positions
Capitalists 1st 2nd 3rd 4th
Natwest Rediff Bazee Naukri MCX
Chrysalis Bazee Shaadi Rediff Contest 2
Win
General Atlantic MCX Naukri Bazee Contest 2
Win

It is also known that,


(i) No companies ranked in market capitalization in the same position as speculated by Natwest but exactly two
of the four companies speculated by them finished the market capitalization within the first four positions.
(ii) Exactly two companies ranked in market capitalization in the same positions as speculated by Chrysalis,
while only one company speculated by them failed to finish the market capitalization within the first four
positions.
(iii) Exactly two companies speculated by General Atlantic ranked in the market capitalization within the first
four positions while only one company ranked the market capitalization in the same position as they speculated.

1. Which company ranked third in market capitalization?


1] Naukri 2] Shaadi 3] Rediff 4] Contest 2 Win

2. Which of the following is the correct order of the companies,


whichfinishedthemarket capitalization in the first four positions respectively?
1] Rediff, Bazee, Contest 2 Win, Shaadi.
2] Shaadi, MCX, Rediff, Contest 2 Win.
3] Shaadi, Naukri, Rediff, Contest 2 Win.
4] Cannot be determined.
Answer:
For Solid DI SET 28

1. 3 From (ii), three of the four companies speculated by Chrysalis finished in the first four positions. Hence,
exactly one of the other two companies – Naukri and MCX – finished in the top four positions.
From
(i), Exactly one of Rediff and Bazee finished in first four positions. Hence, from
(ii), Both Contest 2 Win and Shaadi must have finished in first four positions. From
(iii), Bazee did not finish in the first four positions.
Hence, Rediff finished in the first four positions.
Now, the companies finished in the first four positions are Contest 2 Win, Rediff, Shaadi and MCX / Naukri.
From (ii), Two of the companies as predicted by Chrysalis finished at the same positions.
\ Rediff cannot finish at 2nd or 4th positions as in this case the above value for Chrysalis can be at most one.
From (i), Rediff cannot finish at first position.
\ It must have finished at third position.
25

2. 2 If Contest 2 Win did not finish at the finish, then it should have finish at first position, in this case no
company according to General Atlantic’s predictions could have finished at the correct position. Hence, Contest
2 Win finished at 4th position; Shaadi finished at first position and MCX finished at 2nd position. The correct
order is
Shaadi MCX Rediff Contest 2 Win

Solid DI Set - 29
Question

There are 11 players on each side in ODI cricket, and one team bats while the others bowl. Then the teams swap
after 50 overs. Kraig Chapal, the coach of the Indian ODI cricket team is a master strategist when it comes to
shuffling the batting order. In a match in which India scored 340 runs, 9 batsmen were out and the last unbeaten
pair of batsmen together scored 40 runs before they ran out of overs. In the batting order the 4th and the 5th
men each scored exactly a 100 runs, while the first two batsmen scored 50 between themselves. The ratio of the
runs scored by the first three batsman, Ganguly, Sehwag and Pathan (though not necessarily) 1, 2 and 3 in the
batting order) was 6 : 4 : 3. Two consecutive batsman scored 0, and the first 9 batsmen in the batting order
scored either 0 or scores that were multiples of 5. Dhoni was the 8th batsmen in the batting order, and the next
person after Dhoni scored a 5. The runs scored by Tendulkar if added to theextras (bonus runs not scored by any
batsmen but given by the bowling team to the batting team) equaled Ganguly’s scores. Sehwag scored one-fifth
of Yuvaraj and equal to Tendulkar. Srinath scored 5 times what Tendulkar scored. Munaf and Mohanty scored
18 and 22 respectively, while together they scored 25 more than Dhoni and Pathan together. Kumble was
India’s most successful bowler of the match, and Harbhajan took 2 wickets less than Kumble.

1.What was the number of extras in the Indian score?


(1) 0 (2) 10 (3) 20 (4) Indeterminate

2. Tendulkar’s number in the batting order was immediately after


(1) Yuvaraj (2) Srinath (3) Yuvaraj or Srinath (4) Indeterminate

3. What is the total of the least 5 scores in the Indian Cricket team (excluding extras)?
(1) 20 (2) 30 (3) 35 (4) 38

4. Who has scored 5 runs?


(1) Kumble (2) Harbhajan (3) Tendulkar (4) Kumble or Harbhajan

Answer:
For Solid DI SET 29

• It is given that the first a batsmen scored multiples of 5.Also the first two scored a total of 50 and the
ratio of scores of the first three batsmen Ganguly, Sehwag and Pathan was 6 : 4 : 3.
i.e. either 6x + 4x = 50 or 6x + 3x = 50 or 4x + 3x = 50
As the only possibility is 6x + 4x = 50, Ganguly scored 30, Sehwag scored 20 and Pathan scored 15.
Hence Pathan was 3rd in the batting order.
• Yuvraj scored 5 times that of sehwag, i.e. 100.
• Tendulkar scored equal to sehwag, i.e. 20
• Srinath scored 5 times Tendulkar, i.e.100
• Hence Yuvaraj and Srinath occupy positives 4 and 5, not necessarily in that order
• Ganguly = Tendulkar + Extras. \ Extras = 10
• Munaf and Mohanty are batsmen 10 and 11, though not necessarily in that order.
• Together they score 40 which is equal to 25 + Dhoni + Pathan. Hence Dhoni = 0
• Batsmen 9 scored 5. As the two who score 0 are consecutive, batsmen 7 scores 0. Hence Tendulkar has
only one slot left, i.e. 6. Now all questions can be answered.
26

Solid DI Set - 30
Question

At 3:00 pm afternoon I’m 11th in a college fee payment counter queue. The queue increased at the rate of 1
person every minute the counter clerk takes 3 minutes to entertain a person. There is one girl after every 2 boys
in the queue. The Tea break of 10 minute is at 3:35. The first person is a boy and the person behind me is a girl.
The ratio of boy to girl gets interchanged after the 17th person. At 3:02 and 3:04 girls join the queue.

1. Assuming that there was delay of 5 minutes at 3:07 pm in the counter clerks working. At what time I’ll be
free to catch up with my friends?
1. 3:43 2. 3:48 3. 3:45 4. 3:40

2. Where will I be at 3:18 hrs, i.e. my position? If the 3rd person in the queue decides to pay the fees of two
persons (including herself) and a boy who was exactly after her, left the counter without paying his fees?
position
1. 4th position 2. 5thposition 3. 6th position 4. None of these
Answer:
For Solid DI SET 30
1. 2 After 2 boys there is a lady. 1 st person is a boy, thus it can be BBG or BGBBGB. And so on, where B
stands for boy and G stands for girl. The person behind me is a girl (12th in a row). There for it is BBGBBG i.e.
3rd 6th 9th 12th & 15th are girls. After 12th person girls ratio interchanges i.e. it becomes 2 girls to boys in a
group … at 3:02 and 3:04.
I am 11th in a queue, i.e. my number shall arrive after 10 people (3 minute per) 10X3 = 30 min. + 10 minutes
tea break + 5 minutes interruption + 3 minute processing time. (30 + 10 + 5 + 3 = 48 min. So I’ll be free by 3:48
pm..
2. 2 Counter clerk takes 3 minutes to entertain on person. So in 18 minutes 18/3 = 6 persons have paid their
fees.
I was 11th so now I am 5th Now one of the girls has paid 2 fees, so it is like being 6th in the queue, but a boy
exactly after her, so we are back to being 5th position..

Solid DI Set - 31
Question
The table below shows the sales and expenditure of three companies JAI foods, HAWAI foods and WAI foods
for the financial year 2004-2005. These companies are located in the different parts of the world. Expenditure
break ups are in percentage. Closely analyse the data and answer the questions.
Particulars Jai foods (Sales Rs 7.5) Hawai foods (Sales Rs 1.9) Wai foods (Sales Rs 6.9)

Operating Profit 7 9 9
Interest paid 10 11 12
Rental 21 19 18
Taxes 8 8 11
Salaries 12 9 12
Raw material cost 25 18 19
Power and Electricity 5 10 7
Labour cost 5 6 4
Transportation 3 2 3
Maintenance 2 5 2
Miscellaneous 2 3 3
Total 100 100 100
27

* Sales to be read as Rs. million / month


* Sales break ups are in percentages (%)

1. What can be concluded about the expenditure in salaries, raw material,


transportation and rental of all the three companies listed above in the table?
1. Percentage wise Wai food is spending more then the other two.
2. Revenue wise Jai food is spending more then Hawai and Wai together.
3. Revenue wise Hawai and Wai are spending less then Jai foods.
4. Percentage wise Hawai spends more than Jai, which in turn spends more
than Wai.

2. If the interest paid by JAI FOODS is reduced by 1%, and everything else remains the same, its operating
profit will:
1. Increase by 1% percent. 2. Not change.
3. Increase by 0.0075 million 4. Increase by 2% percent.

3. If the average monthly salary in WAI FOODS is Rs 9,500, then how many employees does WAI have?
1. Can’t say 2. 18 3. 81 4. 87

Answer:
For Solid DI SET 31
1. 3 Option 1, 2 and 4 are not true, by visual observation and approximation. Revenue wise hawai and wai are
spending less then Jai foods..
2. 3 The reduction of 1% in the expenditure will result in increase of 10% x 1% in operating profit for Jai foods,
so operating profit of Jai foods will increase 0.1% of 7.5 m. Increase of 0.0075 million..
3. 4 Total salary paid by WAI foods = 6.9 x 0.12 x 1,000,000 = 828000. So number of employees = total salary
paid/ average salary = 828000 / 9,500 = 87 employees..

Solid DI Set - 32
Question
Between two persons, A and B, there is one apple and one banana. When I asked them as to who among them
had the apple, they gave two statements each in
reply :
A: B has the apple.
B has the banana.
B: A doesn’t have the banana
A has the apple.
It is known that at least one of A and B tells a least one false statement.

1. Which of the following statements must be true?


(1) Each of A and B tells at least one true statement.
(2) At least one of A and B tells at least one true statement.
(3) Neither A nor B tells at least one true statement.
(4) Both A and B tell at least one true and at least one false statement.

2. If it is also known that each of A and B tells at least one true statement, then which of the following
statements must be true?
(1) All the four statements given by both A and B are true.
(2) Both the statements given by exactly one person out of A and B must
be true.
(3) Each of A and B tells at least one false statement
(4) B has the banana as well as the apple with him.
28

Answer:
For Solid DI SET 32
The given statements are :
A : B has the apple.
B has the banana.
B : A doesn’t have the banana.
A has the apple
Let us study the choice and eliminate the incorrect choice:
1. (1) We can have an arrangement in which A’s statements are true and false in that order, and B’s statement
are both false, which defers the statements given in choice (1)
Bothe A and B cannot be liars as the first statement of A is the same as the second statement given by B, and
one of them must have the banana. Hence, at least one of A and B tells at least one truth.
(3) This means that both A and B are liars which is not possible.
(4) This suggests that both A and B are altermators (i.e. one truth and one lie), which defies the arrangement
mentioned in (1).
Hence, only choice (2) is correct. Choice (2)
2. (1) Defies the initial condition.
(2) Is correct as either A’s both statements are true and B’s statements, are true and false, in that order, or both
of B’s statements are true and A’s statements are false and true, in that order.
(3) Is incorrect as per the arrangement given above.
(4) Is incorrect as in the second arrangement mentioned in (2) Choice ( 2)

Solid DI Set - 33
Question
From one sequence of digits another sequence of digits has to be formed as per the following rules:
Single switch: Interchange of any two adjacent digits.
Double switch: Interchange of any two digits with maximally one digit in between.
Random switch: Interchange of any two digits.
1. Using only single switching what is the minimum number of steps required to change 42153 to 12345?
a) 4 b) 5 c) 6 d) 7

2. Using only double switching what is the minimum number of steps required to change 53124 to 12345?
a) 3 b) 4 c) 5 d) 6

3. Using only random switching what is the minimum number of steps required to change 312546 to 654321?
a) 4 b) 5 c) 6 d) 7

4. Using only single switching for first two steps and then double switching what is the minimum number of
steps required to change 54321 to 12345?
a) 5 b) 6 c) 7 d) 8

5. Using only single switching for first three steps and then random switching what is the minimum number of
steps required to change 621534 to 123456?
a) 5 b) 6 c) 7 d) 8
Answer:
For Solid DI SET 33
1. b Starting with 42153 interchange 3 and 5 to get 42135. Next, interchange 2 and 4 to get 24135. Next,
interchange 4 and 1 to get 21435. Next, interchange 1-2 and 4-3 to get the required sequence 12345. Hence, the
single switch operation has to be invoked 5 times.
2. b Starting with 53124, interchange 1-5, then 5-3, then 2-5, and then 4-5 to get 12345.
3. a Starting with 312546, first interchange 5-1, then 6-3, then 3-1, and 4-2 to get 654321.
4. b Starting with 54321, first interchange 2-3, then 4-5, then 2-5, then 1-5, then 1-4 and then 3-4 to get 12345.
The first two operations being single switches and he rest double.
5. b Starting with 654321, first interchange 2-3, then 2-4, then 3-5, then 1-6 to get the required sequence
123456.
29

Solid DI Set - 34
Question
The following questions consist of a questions and two statements ,I and II.
Choose
(1) If one of the two statements (I or II) alone is sufficient but the other statement alone is not
(2) If each statement alone is sufficient to answer the question asked.
(3) If statements I and II together are sufficient to answer the question but neither statement alone is sufficient.
(4) If even statements I and II together are not sufficient to answer the question
1) P, Q, R are three polygons of having different number of sides. Which has the largest area among them?
I. Their perimeters are the same and Q is a square.
II. R is a pentagon.
1.1 2.2 3.3 4.4

2) What is (ad – bc)?


I. a, b, c, d are 4 consecutive terms of an A.P with common difference = 3.
II. a, b, c, d are 4 consecutive positive even numbers.
1.1 2.2 3.3 4.4

Answer:
For Solid DI SET 34
Ans 1) 4 We cannot find the polygon which is the largest area.
Because we don’t have any information about P.
Ans 2 ) 2 Each statement independently will give the answer.
Assume in statement (I) the values of a, b, c, d as x, x
+ 3. x + 6, x + 9. Find ad – bc …it is equal to –18.
Similarly for statement (II) we get the difference as –8.

Solid DI Set - 35
Question
Srinivas intends to draw one rectangle of integer sides with a pencil which can last for a maximum possible
length of 100 units only. Let R denote the set of all possible distinct rectangles from which Srinivas can choose
to draw one such rectangle.
1. The number of rectangles in set R is
1. 636 2. 601 3. 613 4. 625

2. All the rectangles in R are formed in to groups such that all the rectangles of same perimeter are in the same
group. What is maximum number of groups that is possible?
1. 99 2. 96 3. 49 4. 51

Answer:
For Solid DI SET 35
1. 4 Given 2(l + b) =100. But minimum 2(l + b) will be 4 units.
i.e. 49 possible values that perimeter can assume for each possible l + b = n, we have different ways of choosing
distinct pairs of l & b.

Value of (l+b) Number of Ways


2 1
3 1
4 2
5 2
6 3
49 24
50 25
30

= 2(1 + 3 + ……… 25) – 25 = 625


2. 3 As explained above, 49 distinct parameters are possible or 49 groups are possible.

Solid DI Set - 36
Question
A student is allotted 1950 points to bid for courses totaling 16 credits in IIM. For every extra credit undertaken,
he gets an additional 75 points. He is required to bid a minimum of 75 points per credit for all uncapped
courses. However, in the case of capped courses, he can bid more than 75 points per credit. “A” decides to take
18 credits in which all courses, other than one course which is 3 credits, are uncapped.

1. What is the maximum number of points he can bid for the capped course?
(1) 975 (2) 950 (3) 1125 (4) 1050 (5) None of the above

2. What would be the maximum number of points he could bid for the same course if he decides to take a total
of 16 credits only?
(1) 925 (2) 975 (3) 950 (4) 1125 (5) 1050

Answer:
For Solid DI SET 36
Ans 1) 1
Minimum points to be allotted for uncapped course = 15 * 75 = 1125
Therefore Maximum points available for capped course = 2100 – 1125 =
975

Ans 2) 2
Round about method
Total credits = 16, Total points allotted = 1950
Capped credits = 3, Uncapped credits = 13
Minimum points allotted for uncapped courses = 13 x 75 = 975
Maximum points available for capped course = 1950 – 975 = 975
Short cut method
Maximum points available for capped course = 975
As 150 less points allotted and 150 (2 x 75) used up. Choice (2)

Solid DI Set - 37
Question
Each of the letters of the alphanumeric addition EAT + THAT = APPLE are distinctly different. The addition is
done in the decimal system.
1). What is the sum of the digits of APPLE?
a. 9 b. 10 c. 11 d. 12 e. 15

2). What is the value of L?


a. 3 b. 4 c. 5 d. 6 e. 2

Answer:
For Solid DI SET 37
A1) d A2) a
The hints are:
1. A has to be 1.
2. T has to be 9.
3. So E is 8
4. If three digit number added to 4 digit number to get a
5 digit number then the second digit of APPLE has
to be zero.
31

Using all these we get


(8 19) + (9 2 1 9) = (10 0 38)

Solid DI Set - 38
Question
The following questions consist of a questions and two statements ,I and II.
Choose
(1) If one of the two statements (I or II) alone is sufficient but the other statement alone is not
(2) If each statement alone is sufficient to answer the question asked.
(3) If statements I and II together are sufficient to answer the question but neither statement alone is sufficient.
(4) If even statements I and II together are not sufficient to answer the question

1) A survey was conducted among 500 people and it was found that 58% preferred coffee, 47% liked tea and 9
% drank milk in the morning. How many people preferred nothing between coffee, milk and tea?
I. 21% liked both coffee and tea. No person liked both milk and tea.
II. 4 % liked both coffee and milk.
1.1 2.2 3.3 4.4

2) What is the value of x if it is a natural number?


I. y is divisible by 3 and y – x = 720.
II. y is divisible by 9 and y – x = 720.
1.1 2.2 3.3 4.4

Answer:
For Solid DI SET 38
Ans 1.3
From statement I we know that the number of people
drinking milk and tea is zero, so number of people
drinking all 3 is also zero. From I and II we know that
the number of people drinking exactly 1 drink or 2
drinks. So, we can find out the number of people not
drinking any of the drinks.
Ans 2.4
We cannot get the answer from statement II. Statement
I is implied if statement II is true. Hence we cannot find
the answer.

Solid DI Set - 39
Question
Rahul Sharma has a factory which manufactures Alpha I, Beta I and Gamma I. All these are manufactured by
processing Omega 34. Alpha I requires 1 kg/unit, Beta I requires 2 kg/unit and Gamma I Gamma I requires 2.5
kg per unit of Omega 34, which costs Rs. 2 per kilogram. The total availability of Omega 34 is 350 kilogram.
the processing is done on a machine having production hours of 160 hours in day shift and 182 hours in the
night shift. The time required per unit production is as follows.

Product Day Night


Shift Shift
Alpha I 2 hours 2.5
hours
Beta I 3 hours 4.0
hours
Gamma 1 hour 1.5
I hours
32

The machine costs Rs. 1 per hour. Selling price of Alpha I Beta I and Gamma I are Rs. 8 per Unit, Rs. 12 per
Unit and Rs. 3.50 per Unit respectively. At least 5o units of Alpha I have to be produced and at the most 150
units of Gamma I can be produced.

1. Which of the following is possible?


a) 75 units of Alpha 1, 4 units of Beta I in day shift, manufacture 40 units
of Beta I and 40 units of Gamma I and 10 units of Alpha I in night shift.
b) 75 units of Alpha I, 4 units of Beta I, in the day shit. Manufacture 40
units of Beta I and 40 units of Gamma I and 5 units of Alpha I in night
shift.
c) 74 units of Alpha I, 4 units of Beta I in day shift, 25 units of Beta I, and
10 units of Gamma I and 30 units of Alpha I in night shift.
d) 74 units of Alpha I, 5 units of Beta I in day shift, 40 units of Beta I, 10
units of Gamma I and 30 units of Alpha I in night shift.

2. What percentage of the available raw material is utilised if 100 units of Alpha I, 40 units of Beta I, and 10
units of Gamma I are produced?
a) 50% b) 58% c) 68% d) 78%

3. If Rahul Sharma spends initially 150 machine hours in day shift to manufacture Gamma I, 10 hours to
manufacture Alpha I, and in the night shift manufactures 53 units of Gamma I and spends the rest of the night
shift to manufacture Alpha 1. What will be his profit/loss?
a) loss of Rs. 65 b) profit Rs. 65
c) No profit to loss d) This manufacturing pattern is not possible

4. If the minimum possible manufacturing requirement for Alpha I is met and the remaining raw material is
utilised for the manufacture of the other 2 products then
a) 10 units of Beta I can be manufactured
b) 0 units of Beta I can be manufactured
c) 5 units of Beta I can be manufactured
d) Any of the above

5. If in the shift, 65 units of Alpha I, 4 units of Beta I, 15 units of Gamma I and in the night shift. 20 units of
Beta 1,40 units of Gamma I and 20 units of Alpha I are manufactured then which of the following is true?
I. 5 hours of machine time is unutilised
II. Total cost of production is Rs. 54
III. More than 20% of the available raw material is unutilised
a) I only b) II only c) I and III d) I and II

Answer:
For Solid DI SET 39
1. c 74 units of Alpha I and 4 units of Beta I in day shift will require (74 x 2 + 4 x 3) = 160 hours of day shift
which are available as per the condition given in the question. 25 units of Beta I, 10 units of Gama I and 30
units of Alpha in night shift will require (25 x 4 + 10 x 1.5 + 30 x 2.5) = 190 hours which again satisfies the
condition given in the question.
2. b Amount of raw material i.e. Omega 34 utilised will be (100 x 1 + 40 x 2 + 10 x 2.5) = 205 kg. Total amount
of Omega 34 available is 350 Kg. So % of total = 205/350 x 100 = 58%.
3. d Since in the question it is mentioned that the maximum no of units of Gamma I have to be 150 but
according to the question total no of Gamma I units being manufactured are 203.
4. d Minimum possible manufacturing requirement of Alpha I is 50 units. So amount of raw material used will
be 50 kg. Remaining raw material is 300 kg. so any of the first three options are possible.
5. c In the day shift time utlised (65 x 2 + 4 x 3 + 15 x 1) = 157 hours. So 3 hours unutilised in day shift. In the
night shift time utilised (20 x 4 + 40 x 1.5 + 20 x 2.5) = 190 hours. So 2 hours unutilised in night shift.
So total unutilised time is 5 hours.
Total raw material consumed ((65 + 20) x 1 + (20 + 4) x 2 + (55 x 2.5)) =
33

270.5 kg.
So amount of raw material unutilised will be (350 - 270.5)/350 x 100 = 22.7% which is more than 20%. Hence
c).

Solid DI Set - 40
Question
Toyota motor company’s teams A, B, C, D, E and F are racing new concept cars competing in a season of 6
races. Points awarded for rank 1, 2, 3, 4, 5 and 6 are 10, 8, 5, 3, 1 and 0 respectively. Points scored by five
teams in the first 4 races are given in the table below.

Race Team Team Team Team Team


A B C D E
1 1 10 0 3 8
2 0 1 3 10 8
3 10 3 5 0 1
4 10 5 8 1 0

The Championship is won by the team which scores the highest.


Mark 1: If the question can be answered by using one of the statements alone, but cannot be answered using
the other, statement alone.
Mark 2: If the question can be answered by using other statements alone.
Mark 3: If the question can be answered by using both statements together, but cannot be answered by using
either statement alone.
Mark 4: If the question cannot be answered.
1. If Team C wins the next two races, who wins the championship?
I. Team F finishes second in the fifth race.
II. Team F does not finish second in the sixth race.
1.1 2.2 3.3 4.4

2. If Team E wins the next race and finishes second in the last race, who wins the championship?
I. Team D finishes second in one of the races.
II. Team F does not win any race.
1.1 2.2 3.3 4.4

Answer:
For Solid DI SET 40
1. 3 From the table, Team F scores 5, 5, 8 and 3 in the Race 1, 2, 3 and 4
respectively.Team C has 36 points after the 6 races:
Statement I and II are individually insufficient as the positions of the other players in the last two races
are not known.
Combining I and II: Maximum number of points Team F can get = 8 + 5 = 13.
His maximum possible score at the end of 6 races = 21 + 13 = 34.
Team A cannot finish second in the fifth race as Team F is second.
Maximum possible score of Team A is 21 + 13 = 34.
No other player can score more than this, as their scores in the first four
races are less than Team F and Team A.
Therefore, Team C wins the championship with 36 points.

2. 4 End score of Team E = 17 + 10 + 8 = 35.


From statement I, Team D’s maximum possible score = 14 + 8 + 10 = 32.
Team F and Team A’s maximum possible score = 21 + 5 + 10 = 32.
Team F and Team A’s maximum possible score = 21 + 5 + 10 = 36.
Thus, nothing can be said about the winner.
Therefore,Statement I is insufficient to answer the question.
34

Statement II is insufficient to answer the question by itself.


Considering both the statements together nothing can be said about the
winner.
Therefore, Question cannot be answered.

Solid DI Set -41


Question
Twenty-one participants from four countries (Africa, America, Australia & Europe) attended a United Nations
conference. Each participant was an expert in one of the four fields, labour, health, population studies& refugee
relocation. The following five facts about the participants are given.
(a) The number of labour experts in the camp was exactly half the number of experts in each of the three other
categories.
(b) Africa did not send any labour expert. Otherwise, every continent, including Africa, sent at least one expert
for each category.
(c) None of the continents sent more than three experts in any category.
(d) If there had been one less Australian expert, then the Americans wouls have had twice as many experts as
each of the other continents.
(e) Mike and Alfanso are leading experts of population studies who attended the conference. They are from
Australia.

1.Which of the following numbers cannot be determined from the information given?
(1) Number of labour experts from America.
(2) Number of health experts from Europe.
(3) Number of health experts from Australia.
(4) Number of experts in refugee relocation from Africa.

2. Which of the following combinations is NOT possible?


(1) 2 experts in population studies from America & 2 health experts from
Africa attended the conference.
(2) 2 experts in population studies from America & 1 health expert from
Africa attended the conference.
(3) 3 experts in refugee relocation from America & 1 health expert from
Africa attended the conference.
(4) Africa & America each had 1 expert in population studies attending
the conference.

3. If Ramos is the lone American expert in population studies, which one of the following is NOT true about the
numbers of experts in the conference from the four continents.
(1) There is one expert in health from Africa.
(2) There is one expert in refugee relocation from Africa.
(3) There are two experts in health from America.
(4) There are three experts in refugee relocation from America.

4. Alex, an American expert in refugee relocation, was the first keynote speaker in the conference. What can be
inferred about the number of American experts in refugee relocation in the conference, excluding Alex?
(I) At least one (II) At most two
(1) Only I and not II
(2) Only II and not I
(3) Both I and II
(4) Neither I nor II
35

Answer:
For Solid DI SET 41

Labour Health PS RR TOTAL


Africa X B
America 2B
Australia Mike, Alfanso B+1
Europe B
Total A 2A 2A 2A 21

Now solving for A and B , we get 7A = 21 and 5B+1 = 21


=> A = 3 and B= 4
=> Now we will complete the table and get the answer

Labour Health PS RR TOTAL


Africa X 4
America 1 8
Australia 1 1 Mike, Alfanso 1 5
Europe 1 1 1 1 4
Total 3 6 6 6 21

Ans1 (4 ) Ans2 ( 4 ) Ans3 ( 3 ) Ans4 ( 3 )

Solid DI Set - 42
Question
Divide a circle into 8 equal parts; number them clockwise from 1 to 8. Assume that you are at 1 & your initial
score is 1. If you move a step clockwise, you just add the number in the next slot to your current score. If you
move a step anticlockwise, you add the number in that slot but subtract 2 from the total to get your new score. If
you move to the number diagonally across, you add that number to your score but subtract 3 from the total. You
can move to a slot only once.

1. Your maximum score after the second move can be


(1) 12 (2) 11 (3) 10 (4) 8

2. Assuming you move a step clockwise in your first move, then the digit that you would have left untouched
after the third move will be
(1) 7 (2) 4 (3) 8 (4) 5

3. If you have made three moves, without seeking to maximize your score, then you most likely to get a score
of
(1) 12 (2) 9 (3) 8 (4) 10

Answer:
For Solid DI SET 42
Ans1 (1)
Moving in anticlockwise direction in 1st step will give the score 1 + 8 –2 = 7 . second step in anticlockwise
direction will give the score = 7 + 7 –2 = 12 .

Ans2 (3)
Only digit 8 cannot be reached in the third move.

Ans3(4).
The most common score in the three moves is 10 irrespective of the direction of the moves.
36

Solid DI Set - 43
Question
Sally and Sue have a strong desire to date Sam, they all live on the same street yet neither Sally nor Sue knows
where Sam lives. The houses on this street are numbered 1 to 99.

Sally asks Sam. “Is your house number a perfect square?” He answers. Then Sally asks. “Is it greater than 50?”
He answers again. Sally thinks she now knows the address of Sam’s house and decides to visit. When she gets
there, she finds out she is wrong. This is not surprising, considering Sam answered only the second question
truthfully.

Sue, unaware of Sally’s conversation, asks Sam two questions. Sue asks, “Is your house number a perfect
cube?” He answers. She then asks, “Is it greater than 25?” He answers again. Sue thinks she knows where Sam
lives and decides to pay him a visit. She too is mistaken as Sam once again answered only the second question
truthfully.

Sam’s house number is less than the numbers of the houses where Sue and Sally live, and that the sum of all
three of their numbers is a perfect square multiplied by two.

1) What is Sally’s house number?


1. 64 2. 81 3. 25 4. 24 5. Can’t say

2) What is Sue’s house number?


1. 64 2. 81 3. 55 4. 54 5. Can’t say

3) What is Sam’s house number?


1. 45 2. 55 3. 65 4. 72 5. 75
Answer:
For Solid DI SET 43
1) 2 2) 1 3) 2
Since Sally thinks she has enough information, we deduce that Sam answered his house number was a perfect
square greater than 50 (answering Yes to both). There are two of these {64, 81} and Sally must live in one of
them in order to have decided she knew where Sam lives. Sam answered only the second question truthfully, so
his house number is greater than 50, but not a perfect square.
Since Sam answered Sue’s second question truthfully, he had to have answered yes to “Is it greater than 25?”.
Sue was able to deduce Sam’s number, so he also must have said it was a perfect cube. Cubes greater than 25:
{27,64}. Sue must live in one of these houses to deduce Sam’s number.
Since Sam’s number is greater than 50 and is less than Sue’s number, she must live in 64. Since Sue and Sally
are not roommates (we’re told there are three numbers), Sally must live in 81.
Given fact: the sum of their numbers is a perfect square multiplied by two.
Sue + Sally + Sam = 2p (for p an integer)
Or, 64+81+Sam = 2p2
Applying the constraint that Sam’s number is greater than 50 and less than 64, it looks like Sam = 55 (p=10).

In summary,
Sam = 55 Sue = 64 Sally = 81

Solid DI Set - 44
Question
Mr Odd collects all the even numbers between 1 and 100 (both included) and adds them up. Mr Even does the
same for all odd numbers between 10 and 100 (both included). Mr. Prime collects all prime numbers between
50 and 65 (both included) and adds them up. Mr Square collects all the perfect squares of positive integers
between 1 and 100 (both included) and adds them up.
37

1. The final sum obtained is what percentage of the sum of first 100 positive integers?
a) 90% b) 100% c) 110% d) 125%

2. The minimum value is obtained if we subtract the scores of:


a) Mr Even and Mr Square b) Mr Odd and Mr Prime
c) Mr Square and Mr Prime d) Mr Odd and Mr Even

Answer:
For Solid DI SET 44

1. c Mr. Odd -> 2 + 4 + 6 +……..+ 100 = 50/2 [4 + (50 - 1) 2] = 25 x (4 + 98) = 2550


Mr Even -> 11 + 13 + 15 +…….. + 99 = 45/2 [22 + (45 - 1) 2] = 2475.
Mr Prime -> 53 + 59 + 61 = 173
Mr Square -> 1 +4 + 9 + 16 + 25 + 36 + 49 + 64 + 81 + 100 = 385,
Sum of 1 to 100 = 100/2 x (100 + 1)= 50 x 101 = 5050.
Odd + Even + Prime + Square = 2550 + 2475 + 173 + 385 = 5583
Hence,Required Percentage = 5583/5050 x 100% = 110%.

2. d From the above.

Solid DI Set - 45
Question
DIRECTIONS for questions 1 to 3:
Among the marriages conducted for poor people by the “Association for Troubled Lowers” five grooms –
Ranjha, Majnu, Romeo, Farhaad ;and Devdas – get married to the brides Heer, Laila, Juliet, Siri and Paaro, not
necessarily in the same order. Each man marries exactly one woman and vice versa. Ten days after the
marriages took place, the marriage registrar lost the records of these marriage, but remembered only the
following information in connection with these five marriage.

(a) Either Laila married romeo or Ranjha marride Heer.


(b) Farhaad married either Paaro or Heer.
(c) Devdas marries Juliet only in case Paaro marries Farhaad.
(d) Ranjha and Romeo married Laila and Siri, not necessarily in the same order.
(e) Majnu does not marry Juliet.

1. Which of the following is a correct married couple?


(1) Heer – Ranjha (2) Farhaad – Paaro (3) Romeo – Juliet (4) Paaro – Majnu

2. If the condition (a) is replaced by “Heer married Romeo only if Laila married Ranjha” than who married Siri?
(1) Ranjha (2) Romeo (3) Majnu (4) Cannot be determined

3. Who among the following married Ranjha?


(1) Heer (2) Parro (3) Siri (4) Cannot be determined

Answer:
For Solid DI SET 45

For solving these questions, the symbol ‘_’ means ‘married’ and ‘x’ means ‘did not marry’. Let us take down
the information as given below.
Either Laila - Romeo or Ranjha - Heer
Farhad - (Paaro or Heer)
Devdas - Juliet, only if Parro - Farhad
Manju X Juliet
38

We get the following initial arrangements :

Heer Laila Juliet Siri Paaro


Ranjha X • (d) X • X
(d)
Manju - X X (e) X X
Romeo X X • X
(d)
Farhad X•(d) X X X • (d)
Devdas X X - X X

From the above, we conclude that Devdas marries Juliet, and hence Paaro marries Farhad ©. This means that
Manju marries Heer. Now, Ranjha is not married to Heer. Hence Romeo must be married to Laila; hence,
Ranjha must be married to Siri (d)
We get the following pairs of married couples.
Ranjha – Siri
Manju – Heer
Romeo – Laila
Farhad – Paaro
Devdas – Juliet

1. Choice (2)

2. Choice (4) We know that Ranjha and Romeo must marry Siri and Laila (not necessarily in that order). But
now from the replaced condition we get only the following additional implication “if Laila did not marry
Ranjha then Heer did not marry Romeo” which still is not enough to determine who among Romeo and Ranjha
marries Siri.

3. Choice (3)

Solid DI Set - 46
Question
DIRECTIONS for questions 1 and 2 : These questions are based on the data given below.

In order to save the world from the hand of evil forces, eight superheroes are sitting around a circular table to
plan their strategy. It was known that Batman and Robin neither sat on adjacent seats nor directly opposite to
each other. Superman sits besides Flash Gordon and opposite to Heman while Mandrake sat directly opposite to
Phantom who sits adjacent to Superman.

1. The eighth hero, Spiderman, must be sitting next to


(1) Batman (2) Robin (3) Phantom (4) Heman

2. Which of the following statements must be true?


(1) Batman sits to the left of Heman.
(2) Robin sits to the left of Flash Gordon
(3) Spiderman sits to the right of Phantom
(4) None of these

Answer:
For Solid DI SET 46
It is known that :

(a) Batman neither sits adjacent nor sits opposite to Robin.


39

(b) Superman sits between flash Gordon and Phantom (this will give rise to two arrangements)

(c) Heman sits opposite to superman and adjacent to Mandrake, who sat directly opposite to Phantom.

Hence, we get the following arrangements.

Case – I Flash sits to the left of Superman

Case – II Phantom sits to the left of Superman

1. Choice (3) As can be observed in the above possibilities, the Spiderman must be seated adjacent to the
Phantom.

2. Choice (4) The Position of Batman and Robin are not fixed, hence choices (1) and (2) cannot be correct. As
in case II, Spiderman sits to the left of the Phantom; hence even choice (3) is incorrect.

Solid DI Set - 47
Question
There are five identical looking boxes having different objects in it and every box has a label indicating their
contents. The following is the description of the contents and the label of each box:
Contents Label
Two Pins PP
Two Balls BB
Two Clips CC
One Pin and One Clip PC
One Ball and One Clip BC

Somebody has mischievously interchanged these labels in such a way that no box contains the label correctly
explaining its contents.

1. If the first box opened contained label PP and the second box opened contained label PC and out of the
combined four items, at least one item was a Ball, which of the following will be definitely true?
1. Other three items will not contain two Balls.
2. Other three items will not contain any Clip.
3. Other three items will contain at least one Clip.
4. None of the above.

2. If the box PP contained two Clips, the box CC contained two Pins and the box BB contained at least one Ball,
which of the following will definitely not be true?
1. The box BC contains one Pin and one Clip.
2. The box BB contains one Ball and one Clip.
40

3. The box BC contains two Balls.


4. The box PC contains two Balls.

3. If the information is available that box PC does not contain either any Pin or any Clip and box PP does not
contain any Pin and box CC contains one Clip and one Ball, which of the following will definitely be true if
only one of the remaining boxes is opened?
1. It will have one Pin and one Clip.
2. It will have at least one Clip.
3. It will have two Pins.
4. It will have at least one Pin.

4. If the first box, containing the label BC was opened and it was found that one item is a Ball, which of the
following would be definitely true?
1. The other item may either be a Ball or a Clip.
2. The other box with BB label will contain a Ball and a Clip.
3. The other item will not be a Ball.
4. None of the above.

Answer:
For Solid DI SET 47
1. 4 : If the first box label was PP then it could be any one of the other four and if the second box was PC, it
could be any one of the other four. The union of the two indicates that the two boxes opened could be any two
of the five and hence none of the statements can be DEFINITELY true.

2. 3 : From the given data, PP is CC, BB is BC and CC is PP. That leaves BB and PC to be allocated to PC and
BC. Hence PC is BB and BC is PC. Thus the only statement that is DEFINITELY NOT true is that BC contains
two balls.

3. 4 : From the given data, CC is BC, PC is BB and PP is CC. That leaves behind PC and PP, hence statement 3
is definitely true.

4. 4 : If one item is a Ball, the other one can be either a Ball or a Pin or a Clip. Hence none of the statements are
true.

Solid DI Set - 48
Question

The table shown below is a board on which a counter is moving from cell to cell. Initially, the value of the
counter is the value of the cell it is in. Follow the rules given below to answer the questions.

Cell No. 1 2 3 4 5
A 2 6 7 3 1
B 3 -3 -2 2 1
C -4 -2 -1 6 7
D 5 -5 3 1 2
E -3 -4 -5 -2 1

Rule 1: If the counter moves from cell 1 to cell 2, its value will be the value of cell 1 multiplied by the value of
cell 2. For example, if the counter is moving from cell (A, 1) to cell (D, 5), its value will be 2 x 2 = 4. Next
time if it moves from (D, 5) to (D, 3), its value will be 4 x 3 = 12.
Rule 2: If the counter moves from cell 1 to cell 2 and the value of cell 1 is greater than that of cell 2, then the
counter value is the value of cell 2, else the counter value will be the value of cell 1.
41

1. Rule 1: If the counter moves from (B, 3) to (D, 4) to (A, 1), its value will be:
a) 4 b) 8 c) –2 d) –4

2. Rule 1: If the counter moves from (A, 2), to (B, 2) to (C, 2) to (D, 3) to (E, 4), its value will be
a) 1 b) – 226 c) 0 d) -216

3. If the counter moves from (A, 1) to (C, 3) to (E, 5) to (D, 4), its value according to rule 2 is
a) 1 b) – 4 c) 2 d) – 1

4. If rule 1 supersedes rule 2 for the value of counter > 0, but not for the value of the counter < 0, and the
counter moves from (B, 3) to (D, 1) to (C, 3) to (B, 4), what will be its value?
a) 2 b) – 1 c) – 2 d) 1

Answer:
For Solid DI SET 48
1. d Value of counter = (-2) x (1) x (2) = -4

2. d Value of counter = (6) x (-3) x (-2) x (3) x (-2) = -216 220.

3. a Value of counter = Value (4, 4) = 1

4. a (B, 3) to (D, 1) = 5, (D, 1) to (C, 3) = -5, (C, 3) to (B< 4) = 2

Solid DI Set - 49
Question

There exists a 10 digit number such that the 1st digit from left represents the number of 0's in the number, the
2nd digit from left represents the number of 1's occurring in the number and so on until the 10th digit represents
the number of 9's in the number.

1. The sum of the digits of the number is:


1. 8 2. 9 3. 10 4. 19

2. The number of 0s in the number is:


1. 9 2. 8 3. 7 4. 6

3. The number of 1s occurring in the number is:


1. 0 2. 1 3. 2 4. 3

Answer:
For Solid DI SET 49
Since each digit represents the number of that particular digits representing in that number, the total sum is 10,
as only 10 digits exist in the number. That is the sum of digits is equal to the total digits by the property given.
All the counting is done from the left side of the number. To find the number of 0's note that you cannot have 9,
8 or 7. For the place which gives the number of 7 clearly it can at maximum be one since the sum of digits is
equal to 10, suppose the value is 1 then clearly the 7 which occurs can only be in the first digit otherwise again
the sum will exceed 10. Suppose 7 is the first digit and 1 is the 8th digit then the ones digit must have at least 2
since there is already one 1 occuring in 8th digit, suppose the second digit is 2, then the 3rd digit has to be 1,
since we have one 2 occuring in the number.
42

But if this happens then the sum of digit exceeds 10 thus there can be no 0 in 8th, 9th or 10th digit. Suppose the
7th digit is 1, i.e. there is one 6 in the number, then the 6 can only be in the 1st digit otherwise the sum of the
digits will exceed 10, so if the 6 is in 1st digit, then, at least one 1 is present, (because the 7th digit already has
1), now the number of 1 has to be greater than 1 as you cannot have the second digit as 1 (then two 1's will be
present in the number).

Thus the 2nd digit is 2 and the third digit which represents the number of 2's has to be 1 as the second digit is 2,
thus the number is 6210001000. And the remaining solutions follow.
1. Choice 3
2. Choice 4
3. Choice 3

Solid DI Set - 50
Question
Subhart, a mathematics wizard, develops a coded language. He converts all words in some number series in the
following manner:

All vowels (and no other consonant) are represented by prime numbers.


For all consonants, whose positions in the alphabet series are prime numbers, they are represented by a number
which is twice the number giving their positions in the alphabet series. For example: The letter “C” being the
3rd alphabet. Will be denoted by 3 ´ 2 = 6.
If the position of any consonant in the alphabet series is a number, half of which is a prime number, it is
represented by adding a 5 at the end of that number. For example, The letter “Z” being the 26th alphabet, will
be represented by 265.
All remaining consonants will be represented by a number giving their position in the alphabet series.

1. The word coded as “22 17 12 17 38 8 145 11 22 23 225” is-


a) PATAMXNEPUR b) FEGTSHNJKID c) KALASHNIKOV d) TUTUPHENKO

2. The word coded as “38 145 11 16 7 18” is:


a) WEAKER b) STRIKE c) SNIPER d) TAURUS

3. The world “CALCUTTA” will be coded as:


a) 6 29 8 9 16 18 18 29 b) 20 17 10 20 295 4 4 12
c) 6 17 12 6 29 20 20 17 d) 12 11 15 12 29 20 20 31

Answer:
For Solid DI SET 50
Write down all the letters as per the given rules.

1. c We get K = 22, L = 17, S = 38, hence (c).

2. c S = 38, N = 145, P = 16, hence SNIPER.

3. c C = 6, L = 12, C = 6, hence (c).

Solid DI Set - 51
Question
These questions are based on the following pie chart and the table, which gives the details of expenses of an
Excellent Management Training Institute in India (EMTI)
43

EXPENSES OF EMTI FOR 1995 – 96

Table shows INCREASE IN EXPENSES FROM 1994–95 TO 1995-96

Faculty 25% 1. If it cost EMTI Rs.4.4 lakhs for printing in 1995-96, salaries in
Advertising 20% 1994-95 are closest to (in Rs.lakhs)
Material Preparation 10%
Printing 25% (1) 6.0 (2) 4.8
Administrative expenses 7.5% (3) 3.0 (4) Insufficient data
Salaries 5%
Others 7.5% 2. If a pie-chart were to be drawn for the expenses in 1994-95 also,
which expense would show the maximum drop in the angle from
1994-95 to 1995-96?

(1) Others (2) Administrative expenses (3) Salaries (4) Insufficient data

3. If the profits were 20% of total turnover in 1995-96, and total expenses in 1995-96 are the same as the
turnover of 1994-95, profit as a percentage of turnover in 1994-95 is how many percentage points less than that
in 1995-96?

(1) 6.4 (2) 7.6


(3) 12.6 (4) Insufficient data

4. If the total expenses of EMTI in 1995-96 are Rs.16 lakhs, the total expenses in 1994-95 are closest to (in Rs.
lacs).

(1) 12 (2) 13
(3) 14 (4) 15

5. If the total expenses of EMTI in 1995-96 was Rs.22 lacs and the profit that year stood at 10% of the total
expenses, the turnover of EMTI in 1994-95 was closest to which of the following?

(1) Rs.19 lacs (2) Rs.20 lacs


(3) Rs.21 lacs (4) Insufficient data

You might also like